Sei sulla pagina 1di 610

USMLE

THE

STEP 2CK
CRASH
COURSE
THE USMLE GUYS
C 2022 T e USMLE P e a a C a (T e USMLE G )

1
The USMLE Guys Step 2 CK Crash Course usmleguys.com

Tab C

Cardiolog 7
C a A e D ea e (CAD) 7
Hea Fa e (HF) 11
Va a D ea e 14
Ca d a e 17
Pe ca d a D ea e 20
S c e 23
D de a 25
H e e 27
Tac a a 32
B ad a a 37
Ca d ac A e 41
A c D ea e 44
M ca d a I fa c 48

Endocrinolog 50
H e d 50
T dN d e ,T d Ca ce , T dS ,a dH d 54
Pa a d D ea e 57
D ea e f e Ad e a G a d 60
D ea e f e P a Ga d 65
D abe e Me a dC ca 72
Ce a D abe e I d , Ne e c D abe e I d ,a dP a P d a 76
M e E d c e Ne a a 79
P a O a a I ff c e c , P c cO a a S d e (PCOS), a d P a
H ad Ma e 83

Gastroenterolog 85
D a aa dE a ea M D de 85
Ga e a ea Ref D ea e (GERD) a d E a 88
Ca cE a ea I ,E a ea R e, a d E a ea Ca ce 91
U e a d L e GI B eed 94
Ma -We Tea a d E a ea Va ce 98
I ab e B e S d ea dI fa a B e D ea e (U ce a e C a dC
D ea e) 100
Ma ab S d e 105
D e c ,D e c , Me e e c I c e a, I c e c C 109
C Ca ce , C cP , Fa a Ade a P ,a dL c S d e 113
D -I d ced L e I ,Ac c He a , a d N -A c c Fa L e D ea e 116

2
Get The Step 2 CK Drills Book: usmledrills.com
The USMLE Guys Step 2 CK Crash Course usmleguys.com

V a He a 119
L e C a dC ca 122
Ac e a d C c Pa c ea a dC ca 126
Pa c ea c Ca ce , C a ca c a, a d He a ce a Ca c a 129
D ea e f e Ga b adde a d Ac e C a 132
P a Sc e C a a dP a B a C a 136

Hematolog -Oncolog 139


Mc c cA e a 139
Mac c c A e a 142
N c cA e a 145
T aa e a 149
S c e Ce A e a 151
Paee D de 153
B dTa f a dTa f Reac 156
I e ed T b a 162
B eed D de 165
MGUS, M eM e a, Wa de Mac b e a, a d P c e a Ve a 168
Le e a a d T L S d e 171
L a 175

Infectious Disease 178


Ac e I fec D a ea (Wa e ) 178
Ac e I fec D a ea (I f a a ) 183
Se a T a ed I fec 187
I fec f e Fe a e Re d c e T ac 191
HIV 194
Ba c ,H a ,a dP e c P e a (PCP) 197
C cc d d c a dC c cc 200
Me 203
M c bac e T be c a d M c bac e A C e (MAC) 207
C C d, I f e a, P a ,a dC ca 210
R 213
U a T ac I fec 216
T c -B e D ea e 218
M -B e D ea e 221
Pa a c I fec 227

MSK, Dermatolog , Rheumatolog 231


C S de , E b , a d W C a 231
M c eea I e f e K ee 235
L e Bac Pa 243
O e , O e ec ,a dB eT 247

3
Get The Step 2 CK Drills Book: usmledrills.com
The USMLE Guys Step 2 CK Crash Course usmleguys.com

O e a ,R e a dA ,a dA S d 250
Se cA a dO e e 254
G a dP e d (Ca c P aeD daeC a De ) 256
Ta a a ' A e a d G a Ce A e 259
S e c Sc e a d Ra a d P e e 261
S e cL E e a ,A dS d e, S e S d e 264
P a d De a 267
P a a R e a ca a d F b a a 270

Nephrolog 273
E ec eD de - P a 273
E ec eD de - S d 276
Ne a 280
U a I c e ce 283
Re a , B adde , a d P a c Ca ce a d BPH 285
Ac e K d e I (AKI) 287
C c K d e D ea e (CKD) 289
Ac e I e a Ne (AIN), P e e a D ea e, a d Ac e T b a Nec (ATN) 292
Ne cS d e 295
G e e 297
Ac d Ba e D de 300
Re a T b a Ac d (RTA) 303

Neurolog 305
La be -Ea S d ea dM a e aGa 305
G a -Ba S d e, B , a d C e fe d Ja b D ea e 308
M e Sc e (MS) a d A c La e a Sc e (ALS) 312
Se e a Hea L a d Ve 315
De e a 319
C a a He a a a d Ce eb a He a 323
T a e I c e c A ac (TIA) a d S e 326
Se e 330
Te a d C ea 333
Headac e 337

Obstetrics and G necolog 341


N a P c C a e f P e a c a d E e c e Rec e da 341
Pe aa V 344
N a Lab a d B eec P e e a 346
B ea feed : C I e a dC ca 349
H da d f M e , Ge a a T b a c Ne a a, a d H e e e G a da 352
H e e eD de fPe a c 356
Pe a a I fec 360

4
Get The Step 2 CK Drills Book: usmledrills.com
The USMLE Guys Step 2 CK Crash Course usmleguys.com

Te a e c D a d T e Ad e e Fe a Effec 367
P ace a P e a, P ace a Acc e a S ec , Va a P e a, a d P ace a Ab 369
S a e Ab , T ea e ed Ab , Se c Ab , a d Ec cPe a c 373
Te f A e d a dD S d e 377
C a , P e e Lab , a d P e e P e ab R e f Me b a e 379
Fe a M 383
Ge a a D abe e Me 387
La e Te P e a c C ca 389
C ace 392
Me a C c e a d Me a e 395
P a C ca 398
V a a d Va a D ea e 401
Be a d Ma a B ea Le 405
D e ea a d U e e Ab a e 411
O a a T ,O a a C R e, M e c e 415
E d e a a d O a a Ca ce 418
Ce ca Ca ce 421

Pediatrics 424
Ne a a Ca e 424
C ca fPe a 428
Ne a a G c cca a d C a d a I fec 431
C e a Ma f a 434
Ge e c D de 437
Me ab c D de 440
I def c e c e 444
C e a Hea D ea e 447
C d d a d Ad e ce G a d De e e 454
Ped a c P e e a e Ca e 459
Ped a c GI D ea e 462
Ped a c U c D ea e 469
Ka a a D ea e a d Re e S d e 472
M c a D a d J e e Id a c A 475
Ped a c M c e e a D ea e 479
Ped a c T 482
Ped a c A a I fec a dA a e 485
C d Ab e 491

Ps chiatr 494
Ma De e eD de , Pe e De e eD de , Ad e D de ,
C ca ed G ef, P a B e /De e /P c 494
Ge e a ed A e D de , Pa c D de , S c a A e D de , P ba 498

5
Get The Step 2 CK Drills Book: usmledrills.com
The USMLE Guys Step 2 CK Crash Course usmleguys.com

B a 501
P c cD de 504
P a a cS e D de a d Ac e S e D de 507
A e Def c H e ac D de a d Ob e e-C eD de 510
Ea D de a d B d D cD de 513
D ca ea dS a cD de 516
A S ec D de a d C d c D de 520
Defe e Mec a a d Pe a D de 522
Ac /S b a ce U e D de 527
S ee D de 531
A ed P c a c D de 535
Se S d e, Ne e c Ma a S d e, a d Ma a H e e a 538
P c a c Med ca S de Effec a d Safe C ce 541

Pulmonolog 543
P a E b 543
L Ca ce 548
L a d Med a a T 552
I e a L D ea e 554
S e 558
COPD 560
A a 563
Ac e Re a D e S d e (ARDS) a d Mec a ca Ve a 566
S ee A ea 568
P e a Eff 571
P a H e e 573
P e a 575
A ed Re a C d 578

Surger 581
S c E e a dD ba ce f Te e a e Re a 581
Ta a 585
S e cI , He a c I ,P a C ,B Ca d ac I 588
B a dW d Ca e 591
Pe e a A e D ea e, Ac e L b I c e a, a d C a e S d e 593
Ve a a d G He a 597
P e a e I e , S a B e Ob c ,A e dc 599
P ea eC ca 602
T c e a dP 605

6
Get The Step 2 CK Drills Book: usmledrills.com
The USMLE Guys Step 2 CK Crash Course usmleguys.com

Ca diolog
C a A D a (CAD)

CAD e a c eeb c a e f ec a a e e c a ead


ff c e e de e e ca d a e.

R Fac f C a A e D ea e (CAD):
● D abe e e
● T bacc e
● H e e
● Obe
● H e de a
● F de ee e a e e a ec a a e d ea e ( e <55, e <65)
● A e ab e 45 e a d ab e 55 e

S ab e A a:
● S b e a d c f ca ed b ca d a c e a, ca de c bed a
ee / e e/b a e a a /
● B b e e a d e e ed b e ce e a 10 e
○ T e b d ffe e ce be ee ab e a d ab e a a, ab e a a
e e ed e
● A c a ed ca c de e f b ea (SOB), d a e , a ea
● P ca e a a be a a c de ac ca d a, e e , e

● ECG a e ab e ce f c e a

Ac e C a S d e (ACS):
● Refe a e ec ed c f ed ac e ca d a c e a/ fa c
● T e ea e ee d f ACS:
○ U ab e A a (UA)
○ N -ST e e a ca d a fa c (NSTEMI)
○ ST-e e a ca d a fa c (STEMI)
● T e e ee e f ACS d ffe f ea e ba ed e e e ce f e e a ed
b a e f ca d ac ( .e. )a e a ECG f d

Da dCa f ca :
Classification of Chest Pain Characteristics

1. S b e a c e d c f
2. B b e e e a e
3. Re e ed b e ae
T ca a a Mee 3/3 c a ac e c
A ca a a Mee 2/3 c a ac e c
N -a a c e a Mee 0-1/3 c a ac e c

Ca d 7
Get The Step 2 CK Drills Book: usmledrills.com
The USMLE Guys Step 2 CK Crash Course usmleguys.com

U ab e A a:
● S f a ACS a d NO e e a
T
○ e b d ffe e ce be ee NSTEMI a d UA
● Re e be e e a ed a be de ec ed f 3 af e c e a,
a e a a f a e e e ec ed ACS ca e UA a d
NSTEMI a a f ae e e a ed. Y eed a a d ed a ab e
a e a d ee f e a e e a ed
● ECG c a e ( c a ST- e e de e ,T a e e ) a c e d
c e a a be e e ab e .

NSTEMI:
● T e e e MUST be e e a ed
● ECG c a e be e e
● ECG f d NSTEMI ca c de e a d ST-de e 0.5
c ead a d/ T e >1 c ead
e R a e R/S a >1

NSTEMI ECG

STEMI:
● Ne ST- e e ee a a e J- ( e c be ee e e a f e
QRS c e a d e a f e ST e e )
● T ee a f d t o contiguous leads : 1 ee a a ead e
a ead V2-V3.
● F ead V2-V3 e e e a d ca STEMI a e a f :
○ 2 e 40 ea
○ 2.5 e <40 ea
○ 1.5 e e a de fa e

Ca d 8
Get The Step 2 CK Drills Book: usmledrills.com
The USMLE Guys Step 2 CK Crash Course usmleguys.com

STEMI ECG

ACS Q c Refe e ce:


ACS Troponin Ele ated ECG

UA NO P be(f e e a NSTEMI, a a e )

NSTEMI YES Ye (ST-de e , T- a e e )

STEMI YES ST- e e ee a

W - f ACS:
Step 1: Perform ECG on patients presenting with ACS s mptoms
If ST e e a :E e e eed f ca d ac ca e e a
If N ST E e a e e 2

Step 2: Measure troponins


If e e a ed, a e e e e eed f ca d ac ca e e a .
If ee a e e 3

Step 3: Repeat troponin level after 3-6 hours from s mptom onset
C de b e a e a ECG a d ea e e . If b e a
e e a e e e a ed ECG c a e a e e e , e eed f ca d ac ca e e a .
If e a e, e e 4.

Step 4: Consider Stress Test

Ca d 9
Get The Step 2 CK Drills Book: usmledrills.com
The USMLE Guys Step 2 CK Crash Course usmleguys.com

ACS T ea e :
Re e be e e c MONABASH
M eO e N ae A Be a B c e Ace I b Sa He a

● E e e ac a d ca d ece e a , ace b , a , be a
b c e, e
● M e f eeded d e a e e a d ae f c e a
● He a f NSTEMI a d UA

*N e c d e ef a e de PCI-c e ea e f 1 ea d
e e (DES) a d 1 ba e e a e

Content Re ie Questions:

W c f ef ECG f d a db a e aec e eda f


ab e a a?
A. Ee a ed a d ST e e de e V1 a d V2
B. Ee a ed a dT a e e V5 a d V6
C. N a a dT a e e V5 a d V6
D. Ee a ed a d ST e e de e V1 a d V2

W a ef d a d be de ed e a a e e e e ED
c e ACS?

A. T
B. C e X- a
C. D-d e
D. ECG

Ca d 10
Get The Step 2 CK Drills Book: usmledrills.com
The USMLE Guys Step 2 CK Crash Course usmleguys.com

H a Fa (HF)

Fa a Hea Fa e D a c C e a:
● T eda f ea fa e e e :
○ 2 a c e a OR 1 a a d 2 c e a
○ C e a ca be a b ed a e ed ca c d

Major Minor

Pa a c a d ea B a e a e ede a

E e a ed a e e e D ea e e

O ea He a e a

S3 a P e a eff

P a ae N c a C

Ca d e a CXR Tac ca d a

P a ede a CXR

We f 4.5 ea e 5 da
e e ea e f e ed HF

C R Fac f HF:
● C a ea d ea e
● D abe e
● T bacc e
● H e e
● Obe
● Va a ea d ea e

S cD f c :
● C a ac e ed b e ec f ac (EF) a d d a f e ea ( be )

Da cD f c :
● C a ac e ed b a e e a ed e ec f ac (EF) a d e f e ea
(f be )

R -S ded Hea Fa e:
● I a ed ab f e de f e ea b d e
● C ca e c de ad a ced ef - ded ea fa e, PE, ARDS, e c e MI,
ca d , a e e , - ded a e d ea e, ca d a e

Ca d 11
Get The Step 2 CK Drills Book: usmledrills.com
The USMLE Guys Step 2 CK Crash Course usmleguys.com

Lef -S ded Hea Fa e:


● I a ed ab f e ef de f e ea b d ec c a e
a d eb d
● C ca e c de c a a e d ea e (CAD), MI, -e HTN

Ec ca d a HF U ed A e :
● A a a d e c a e
● Lef a d e c a cf c
● Da c LV f c
● Re a a ab a e
● Va a d ea e

C Ca e f CHF E ace ba :
Causes

N c a ce ed A e a

I c ea ed a a e M ca d a fa c

I c ea ed f d a e E ec e ab a

W e e a fa e S e (e a ca )

U c ed HTN Med ca

I fec H e d

Ac e Dec e a ed Hea Fa eW -U :
● 12- ead ECG
● C e ad a
● Lab a e (BNP, , ec e e , CBC, ABG f c e ee
e a d e )
● Ec ca d a

Ma a e e CHF E ace ba :
REMEMBER LMNOP: La (f e de), M e, N a e , O e ,P

Ma a e e f HF Red ced EF:


● I a ea :
○ D e c ea
○ A e e b c e
○ Be a b c e
● Sec da ea :
○ M e a c c d ece a a (MRA)
○ I ab ad e
○ H d aa e ae
○ D

Ca d 12
Get The Step 2 CK Drills Book: usmledrills.com
The USMLE Guys Step 2 CK Crash Course usmleguys.com

*L fe e d f ca c d e a 3 a f a da , ce a ,ac
ed c /ce a ,a da ce f be

Content Re ie Questions:

W c f e e ed ca d e a e e a ea f e ea e f ea
fa e?

A. Ca c c a e b c e
B. D e c ea
C. A e e b c e
D. Be a b c e

Ca d 13
Get The Step 2 CK Drills Book: usmledrills.com
The USMLE Guys Step 2 CK Crash Course usmleguys.com

Va a D a

I e f Hea M :
Grade and Work-Up Characteristics

G ade I Fa e be , a a dbe ca d

G ade II S f b ea ea d

G ade III L de a G ade II, a ab e ec d a

G ade IV L d a a ab e ec d a

G ade V M ca be ea d e c e aced e e c e . Pa ab e
ec d a e e
G ade VI M ca be ea d e e c e ff e c e . Pa ab e ec d a
e e
N e: A c ade III e a da c be ed
a ec ca d a

Val ular Common Causes Findings/S mptoms Murmur Treatment


Disease
M a Va e C ec e e M fe a a c M d- cc c , If a c,
P a e (MVP) d de Pa a f ed b a a e be a b c e
Pa c a ac c
A e e a e D ea
e ce a e f ea E ec e e a ce Hea d be a ea e
a D e
C e a
M a Va e Se e e MVP P . Ede a H c ed a d M a a e
Re a D ea b e ace e
Pa a M c e A-f b H c
d f c ( a
ca ed b a MI) Hea d be a ea e
Rad a e ea a
R e a c fe e
M a Se R e a c fe e P a ede a O e a Pe c a e
(MS) D ea Hea d d da e M a Ba
O ea Va
He Hea d be a ea e Va e
A-f b/ b e b R b Re ace e
D a a
H a e e
I fec e e d ca d

Ca d 14
Get The Step 2 CK Drills Book: usmledrills.com
The USMLE Guys Step 2 CK Crash Course usmleguys.com

T c d M ec da JVD H c a L d e c
Re a ca e He a e e a e ef d
A c e e a b de Ad e e
Pa a ce e Pe e a ede a a a
af e MI
D ec a e T c d a e
Dece e a e a
Eb e ' a a e ace e
R e a c a e d ea e
Ca c d d e

T c d R e a c ea d ea e JVD O e a Pe c a e
Se Ca c d d e He a e a L fe e c ba
Ca d ac A c e da c a
Pe e a ede a a e
Hea d a e e ef e ace e
e a b de
A c R e a c ea d ea e W de e e e (de 1) Ea + A c a e
Re a E d ca d M e' ,M e e c ed, d a c, e
(AR) A c d a ) b ,a d a
C e a bc d be a ed
a c a e P ec d a e a : a ca e
Ca c f c a e d ea e ed aced dec e ce d
A c d ec a e a a d fe 2) A F
CHF :
C e a - c ed, d
aeda c
b e, ea d a
a e
3) M d- c

A cSe C e a defec Hea fa e, c e, S4 d e a a c A c Va e


(AS) R e a c fe e a da aa eca c Re ace e
Ca c f ca a a f e d a ef d S c c e ce d -
a dec e ce d ea d a
Ea d ea e f d ba e ad a
c de d ea e ca d a d a e
e e , e c e,
a dc e a

Ca d 15
Get The Step 2 CK Drills Book: usmledrills.com
The USMLE Guys Step 2 CK Crash Course usmleguys.com

Ma e e &M :
Squatting Standing Hand Grip Valsal a Inspiration

-MVP -MVP -A c e -MVP -T c d


e e e dec ea e e e ae e
-H e c -H e c -M a -H e c e d
ca d a ca d a e a ca d a a
bec e bec e c ea e bec e
fe de de
-O e -O e -O e
bec e de bec e fe bec e fe

Content Re ie Questions:

Y ea a d c a a ab e ec d a e e e c e
f e ef e a b de f e a e .W a ade d ca f a ?

A. G ade II
B. G ade III
C. G ade IV
D. G ade V

Ac ca e f c c d e e f e ca c f ca d a ?

A. A c e
B. M a e
C. M a e a
D. T c d e

A a e c e e a e f b ea a d a - c ed a d b
c ea d be a e a e ad a e a a. U a ed a d
f 30 ec d , e c ea e e .W c e e da ?

A. M a a e e a
B. A c e
C. M a e
D. T c d e

Ca d 16
Get The Step 2 CK Drills Book: usmledrills.com
Ca a

Cardiom opath Characteristics Cause Findings/ Echo Results Treatment


S mptoms
Re c e N d a ed Fa a - f a e: E ec e e a ce N -d a ed a d 1) T ea de d ea e
Ca d a e ce Sa c e c e - e ed 2) L d e c e e e e
a ed a D ea e ce c e
e c a f a a e a e e 3) Be a b c e ca c
I f a e: Pe e a ede a c a e b c e ae
N - e ed A d , ac d , a d c ea e f e

Get The Step 2 CK Drills Book: usmledrills.com


ca d ac ce Ga c e d ea e, H e Pa a 4) Ca d ac a a a
d e, a d fa
R d e c a f a He a e e a
a
S a ed ea e : A c e
N a ef He c a , Fab
e c a c d ea e, a d c e
f c a ed ea e
D a ed D a f e I c e a CHF : Lef e c e 1) ACE b , be a b c e , a d
Ca d a b e ce D ed e e c e d a d e c
The USMLE Guys Step 2 CK Crash Course

I fec :V a,c a a , ca ac dec ea ed a 2) F b a ce c e ca


D ed e c e ca e, e ee e
c ac f e O ea 3) De ce e a f a ae
b e ce Ge e c D ed ad a e ( ace a e ICD)
Pa a c 4) Ca d ac a a
W e ed c S b a ce e e: c a d ea
f c Ac , ed ca , Lef a a
c ca e Pe e a ede a e a e e

C e ca
e : Ta b
usmleguys.com

Ca d 17
H e c Lef e c a Ge e c ca e: 60-70% f Pe c e LV e F e BB CCB
Ca d a e e e c e
ac ee e e S ca e Sec d e BB a d CCB OR BB
LV f D ea f e a dd a de OR CCB a d
b c a a e d a de
C e a
M a e a LVOT b c AICD ace e
Pa a
Da c N a c E OH ab a
d f c M : Ha f c a d EF
c e ce d - Hea a a
M ca d a dec e ce d c

Get The Step 2 CK Drills Book: usmledrills.com


c e a
Hea d be a e
a e a d ef e a
b de
Rad a e ea a

Content Re ie Questions:

W c f ef de d be ed a e eda f e c e ca d a ?
The USMLE Guys Step 2 CK Crash Course

A. C ee e
B. C e X- a
C. ECG
D. Ec ca d a

W a e e ca e f e c ca d a ?

A. HTN
B. Sa c ee e e a
C. Ac
D. HIV
usmleguys.com

Ca d 18
W c f e e be c a ac e e a be ea d a a e e c ca d a a f ca ef
e c a f ac b c ?

A. Ha c e ce d -dec e ce d c ea d a e ef e a b de
B. L - c ed, d aeda c b e ea d a e a e
C. H c a e ef e a b de
D. O e a b da c ea d be a e a e

Get The Step 2 CK Drills Book: usmledrills.com


The USMLE Guys Step 2 CK Crash Course
usmleguys.com

Ca d 19
The USMLE Guys Step 2 CK Crash Course usmleguys.com

P a a D a
Ca e f Pe ca d :
Cause E ample

I fec V a , bac e a , f a, a a c
MI P - ca d a fa c d e (D e e S d e)
f 7-90 da af e MI
Ca ce L ,e a ea , b ea , e e a, a, ea a

Me ab c Ue a, e e eed c eae ea e e
Ta a e ca d S e , ad a , a a
A e SLE, c e de a, RA, a e d a

F d /S f Pe ca d :
Findings/S mptoms Characteristics

C e a -P e c( e b ea )
-S a / abb /b
-I ed b ea f ad
ECG (be a e ) -Ne de ead ST e e a
-Ne de ead PR de e
Pe ca d a f c b -Sc a c d ea d e e ef e a b de

T ea e f Pe ca d :
Cause Treatment

V a/d a c NSAID + c c c e

Ue a Da

S e c d ea e T ea de d ea e

V a / d a c ca e a e G c c c d (a e effec ed e)
c a d ca NSAID
*N e a a ea e , a a e e ca d a e ec e d a ca
ac a d e e a ed b a e e e (CRP)

Ca d 20
Get The Step 2 CK Drills Book: usmledrills.com
The USMLE Guys Step 2 CK Crash Course usmleguys.com

Pe ca d a Eff :
● Ca e : Sa e a e ca d
● F d / :
○ N e e d a c ab f ca d ac a ade
● ECG:
○ L a e QRS c e e
○ E ec ca a e a
● Da : Ec ca d a
● T ea e : Pe ca d a d e ca d ce e

Ca d ac Ta ade:
● S :C e a , e c e, c ed ea
● F d :
○ H e , c ea ed JVP, c ac e , ac ca d a, d a ea d ,
e e a ede a
○ P a ad (dec ea e c BP f ea e a 10 H
a )
● ECG: Sa e a e ca d a eff ( a e QRS c e e a d/ e ec ca
a e a )
● Da :
○ D NOT eed ECG a e eda , a e eda ba ed c ca
f d a d e ea e f a e e d a ca ab e
○ A ea c a be c a e ( a ded)
○ D a , e e c a a da a e a e ef a d c b
a ad
○ IVC e a
● T ea e : Pe ca d ce e

C c e Pe ca d :
● Ca e:
○ Sa e a e ca d , e US c ae
■ 1) V a / d a c
■ 2) F ca d ac e
■ 3) F ad a ea
○ O de e US, TB a c ca e
● F d / :
○ S f f d e ad
○ D ea
○ Pe ca d a c
○ K a '
○ P a ad
○ I c ea ed JVP
● Da :
○ Ec ca d a
○ I c ea ed e ca d a c e ca c f ca
○ M de a e e a e e fb a a
● T ea e : Pe ca d ec

Ca d 21
Get The Step 2 CK Drills Book: usmledrills.com
The USMLE Guys Step 2 CK Crash Course usmleguys.com

Content Re ie Questions:

A a e c e e ED abb c e a a e dee a a d
e e ed ea f a d. W a e be a e e ab eda ?

A. Ec ca d a
B. C e X- a
C. T
D. ECG

W c f e e d NOT e ec f d a a e ca d ac a ade?

A. B ad ca d a
B. H e
C. P a ad
D. I c ea ed JVP

A a e a ed ca fH d a a ad a ea
ea a c e e a a c ef c a fd ea. O ec ca d a
c ea ed e ca d a c e ca c f ca ed. W a e e da f
a e ?

A. H e c ca d a
B. C c e e ca d
C. Ca d ac a ade
D. A d

Ca d 22
Get The Step 2 CK Drills Book: usmledrills.com
The USMLE Guys Step 2 CK Crash Course usmleguys.com

Ref e S c e:
Vaso agal Situational Carotid Sinus S ncope

M c ca e f Ca ed b ca d b a d/ Ca ed b ca d b
c e a de e e e a d/ a de e e e

Ca ed b ca d b Of e e f a be a e Risk factors: O d a e, a e,
a d/ a de e a a e, c , e ec e ec
e e defeca , a , a ad a ea ,
a e c e c d ea e
Of e e f e f S mptoms: L eaded e ,
a .e. e a b e- f , S mptoms: L eaded e /
d e e e e a f be a c d, e c e, c e, fa
da e , a ea a de a e e ca e
S mptoms: L eaded e ,
b e- f , Diagnosis f e c H f ec a a f ed
e a f be a c b a ca e a b c e
c d, d a e , a ea a d a ECG
a Re d cbe c e
ca d a a e
Diagnosis f e c
c b a ca
e a a d a ECG

Ca be c f ed e
ab e e f a
d b ( a eeded f
da )

Orthostatic H potension

D cb d e e f 20 H ea e OR d a cb d e e 10 H
ea e e a d f e ea ed .
● I ed a e a c e cc 10-20 ec d
● De a ed a c e cc af e e e a e

Ca e :
● De e ed a a c a e, ed ca ,a , a a da c ff c e c ,
E OH e

Ca d ac A a :
● AV b c , ca d ac a e , a e c a ac a a , e c a
ac a a
● Of e c e
● Da e ECG

Ca d 23
Get The Step 2 CK Drills Book: usmledrills.com
The USMLE Guys Step 2 CK Crash Course usmleguys.com

Ca d ac Ca d a D ea e:
● S c a c d c a ead c e: H e c ca d a , ca d ac
a ade, a c e , ac e ca d a fa c , a e b , ac e
a c d ec
● H ae f a f a e e ec d a d c e
● O e ec f c ec f c ca d a d ea e

Ce eb a c a D ea e:
● Ve a e e eb ba a a e c e a bc a a ea d e ca ead
c e
● L f ca e f f ca e ca def c e e
● Da e a e a a d ca d a d

Content Re ie Questions:

A a e e c ea de d e f d a ea da da f de e
ea . Y ec a c e a e ca e a d a c ec a c a .
W a d da cb d e ec a e eeded c f da e
e a e f ea ed a d ?

A. 5 H
B. 10 H
C. 15 H
D. 20 H

W c f ef ca e f c e ca be c f ed a ab e e ?

A. O a c e
B. Va a a c e
C. Ca d c e
D. A c e

S c ef c f ef ca e d a e e e 1- ea a ae
f a c a e de?

A. Va a a c e
B. O a c e
C. A c e
D. S a a c e

Ca d 24
Get The Step 2 CK Drills Book: usmledrills.com
The USMLE Guys Step 2 CK Crash Course usmleguys.com

D a

LDL:
● L -de e , ed c e e a a d e bef
a e ce f d ed e a f ea e e

HDL:
● H -de e , e e c e e f a e e a d a e bac e e
e e e a e ce

T ce de :
● L df d eb d, ed a e e f ce be ee ea b c b e
a e ce

E e be:
● Sec d c LDL lo ering ed ca ed af e a
● Mec a e inhibition NPC1L1 protein c e decreased
cholesterol absorption from the intestines
● Ma de effec : D a ea

B e Ac d Se e a :
● I c de : C e a e, c e ,a dc e e ea
● B d b e ac d e e e, pre enting reabsorption of bile acids a d ca
e a e GI ac
● Decreases LDL
● S de effec : D a ea

N ac :
● Increased HDL, decreased LDL
● Ac a e NIACR1 a d NIACR2 c decreases release of free fatt acids,
decreases LDL s nthesis
● S de effec : F , , a e e a , a d a ea
● Pretreat ith aspirin a d f

Fb ae :
● I c de : Ge f b a d fe f b a e
● S a e PPAR a a, c ea e e a e c ca ab e TG e
● Decreases trigl cerides, increases HDL
● S de effec : M , e e a ed LFT , e fe e a fa

Sa :
● Decreases LDL, decreases trigl cerides
● C e e inhibitors of HMG CoA reductase, e a e- e c e e
b e
● S de effec : M , e e a ed LFT

H I e Sa T ea :
● A a a 40 80
● R a a 20 40

Ca d 25
Get The Step 2 CK Drills Book: usmledrills.com
The USMLE Guys Step 2 CK Crash Course usmleguys.com

M de a e-I e Sa T ea :
● A a a 10 20
● R a a 5 10
● L a a 40
● P a a a 40
● S a a 40

W e e a ea ?
● Ad a e 20-75 LDL 190 /d
● D abe e e 2 a e 40-75 a ASCVD f 7.5% ea e
● Pa e a e c e c d ea e a e 75 a d e
○ C a ea d ea e (MI, a a, c a a e e >50%)
○ Ce eb a c a d ea e (TIA, CVA, ca d a e e >50%)
○ Pe e a a e d ea e (c a d ca )
○ A c a e c e c d ea e (a e )
● A e 40-75 ea a d LDL 70 /d a d <190 /d d abe e , ASCVD
20% ( )

W e de a e e a ea ?
● D abe e e 2 a e 40-75 a ASCVD c e f LESS a 7.5%
● Pa e a e c e c d ea e a e OLDER a 75
○ C a ea d ea e (MI, a a, c a a e e >50%)
○ Ce eb a c a d ea e (TIA, CVA, ca d a e e >50%)
○ Pe e a a e d ea e (c a d ca )
○ A c a e c e c d ea e (a e )
● A e 40-75 ea a d LDL 70 /d a d <190 /d d abe e , 5% <7.5%
(b de e )

Content Re ie Questions:

W c f e e ed ca a a e a de effec fc a e f ?

A. A a a
B. N ac
C. Fe f b a e
D. C e a e

H ac - d ced c a e f ea ed?

A. Ce a f ea e
B. Pe ea e e d
C. Pe ea e a
D. Pe ea e d e da e

W c f ef a ea e a aef a a e d abe e e 2, a e 45
a ASCVD c e f 7%?

A. A a a 40
B. R a a 40
C. R a a 20
D. A a a 20

Ca d 26
Get The Step 2 CK Drills Book: usmledrills.com
The USMLE Guys Step 2 CK Crash Course usmleguys.com

H
Diagnosis of H pertension

Classification mmHg

N a b d e e S c <120 H
Da c <80 H
E e a ed b d e e S c 120 129 H
Da c <80 H
H e e Sa e1 S c 130 139 H OR D a c 80 89
H
H e e Sa e2 S c a ea 140 H OR D a c a ea 90
H

Af e a ff ce e e a ed ff ce b d e e, de ece e eda f
e e e a e :
● Mea eb d e e f 130/ 80 H
● A b a b d e e (ABPM) ea da eb d e e f
130/ 80 H
● T ee ff ce-ba ed b d e e ea e e aced e a e d f ee
a ea f 130/ 80 H

E ce eed
c f a ead :
● A a e e e e e e e c ( 180/ 120 H AND NO
f ac e e d- a da a e) OR e e e c ( 180/ 120 H AND
f ac e e d- a da a e)
● A a e e e a a c ee b d e e 160/ 100 H a d
a e d- a da a e

H e e eE ce a a :
● Ne ca a fe a f e e ee e e c
● Headac e, c f ,a a a ea, ( - ca e ca )
● G a e BP 10-20% ef
● BP a ed ced b a a f 25% b e d f f da f ea e

Sec da H e e (C Ca e ):
Cause Findings

P a d e d ea e I c ea ed e c ea e c ce a , ab a
a
Re a a e e :T e a e e , ef ac
ea fa e, e e a ed e c ea e af e ACE- b ,
fa a ede a, a e c d e e, abd a
b

Ca d 27
Get The Step 2 CK Drills Book: usmledrills.com
The USMLE Guys Step 2 CK Crash Course usmleguys.com

P a ad e H ae ad e a a a ,
e a e a, d -e a e e , ad e a
c de a a
S ee a ea Da e e ce a d fa e, c f ,
de e , ca d ac a a

Sec da H e e (U c Ca e ):
Cause Findings

H d H TSH
S f d ( a ,c d e a ce, fa e, e c)
H e aa d H e ca ce a
C aca f ea a H e e e e e e e
De a ed b ac a -fe a e
L a e a b d e e e e e e e
T -a d-f ac e ea d e e e c e
Rb c ca ed b a f d a ed e c a
a e e
P e c c a Pa a ee a b d e e ac ca d a
T ad f a e eadac e, a a ,da e
D -e a e e
Ad e a c de a a d be e a a ed f
e c c a
C ' d e C d fac e , ce a be , a ce
ea e
T ecc e
D -e a e e
Ad e a c de a a

L fe e M d f ca :
Modification Details

Dea a e c Ma 2.3 f d /da


We A f dea b d e , a ab 1 H dec ea e BP
f e e 1 ed c b d e
P a e e a Ma a a a a ef fe f a d e e ab e

DASH d e Ve e ab e a d f ead f a be e a e , -fa da


d c , e a , ,f ead f ed ea
E ec e 3-4 e e ee f 40 e , ae b c e e c e
effec e f e BP
Ac ed c Me 2 d e a d e 1d e da

Ca d 28
Get The Step 2 CK Drills Book: usmledrills.com
The USMLE Guys Step 2 CK Crash Course usmleguys.com

W d ece e a e e e ed ca ?
● Pa e - f- ff ce da eb d e e 135 H c 85 H
da c
● Pa e a e a e ff ce b d e e 140 H c 90 H da c
(f - f- ff ce ead a a ab e)
● Pa e a - f- ff ce ff ce ead b d e e f 130 H c
80 H da c a ea f ef :
○ E ab ed ca d a c a d ea e, ea fa e, ca d d ea e, e
e, e e a a e a d ea e
○ T e 2 d abe e e
○ CKD
○ A e 65
○ A e a ed 10- ea f a e c e c ca d a c a d ea e f 10%

I a D T ea f H e e :
● T a de- e d e c (b ac a e , a e e 60)
● ACE b /ARB ( -b ac a e e a 50 ea d)
● Ca c c a e b c e (b ac a e , a e e 60)

Medication Specific Indications Side Effects

ACE b HF a a c LV d f c , A ede a, c , e a e a, ac e
STEMI, NSTEMI, cd f c , e a fa e a e b aea e a
d abe e , e a CKD a e e
ARB Sa e a ACE b H e a e a, ac e d e
e a c a e e , e ae f
c a da ede a c a ed ACE
b
T a de d e c Hea fa e c c d e d ea e, H a e a, a e a,
e c d e a e e a, e ce a,
e ce a, e de a,
e ca ce a, e a d f c
Ca c c a e N ab e d ca Pe e a ede a, a e a a
b c e
Bb c e F a ac e MI, ea fa e, B ad ca d a, e , AV b c ,
a a c ef e c a b c a , ce a, e a
d f c ,a a fb a /f e , d f c
a a, e e a e ,
e d
A ab c e BPH D e
M ea c c d HF EF ae a c, LVEF G ec a a, e ae a
ece 35%, MI LVEF 40%
a a ae a c a e d abe e

Ca d 29
Get The Step 2 CK Drills Book: usmledrills.com
The USMLE Guys Step 2 CK Crash Course usmleguys.com

Va d a Hea fa e ed ced e ec H d a a e: Ref e ac ca d a, d


f ac de e a ed ca d ced
ea ISDN: Da e d BP
c c a e f de e a e5
ISDN ed e e ea a a b
CAD N de: C a de c f e a
d e e e, e ec a
e a ff c e c a e

A e e e Pe a c :
● Me d a
● Labe a
● N fed e
● H d aa e
● T a de d e c ( a ede a)
● C d e ( ec d e, eb d e e b e)

Beta Blocker To icit

S :
● B ad ca d a e ( c )
● Ca d e c c
● B c a , e a de e
● H ce a
● Me a a c a e, de , e e, c a

T ea e :
● A a , b ea ,c c a
● S a c ce a ea ed IV de e
● Se e be da e e
● IV f d a d a e
● IV ca
● IV ca c a
● Va e
● H d e a d c e f
● L de ea

C anide To icit

Ca e :
● Sodium nitroprusside
● F e a d c , , a
● C e ca e e: Me a e ac , e e d c , a

S :
● C e - ed , a e ed e a a ,c a, a ea ,a a, ac ea,
e ab c ac d

Ca d 30
Get The Step 2 CK Drills Book: usmledrills.com
The USMLE Guys Step 2 CK Crash Course usmleguys.com

T ea e :H d c ba a d fa e

Content Re ie Questions:

A a e e e b c , e e , a d -a d-f ac e ea d e
e e c e .W c f e ef d a a be e e ?

A. H e ae a
B. H e ce a
C. De a ed b ac a -fe a e
D. Pe e a ede a

A 55 ea d Af ca A e ca ae e PMH e e f af - af e a
c c a BP f 148/88 H . He e e e f ea eb d
e e be 143/84 H .W c f ef ed ca d ca ed f a ea ?

A. Me d a
B. ACE b
C. Ca c c a e b c e
D. Be a b c e

W c f ef NOT a be de effec f a de d e c ?

A. H e ae a
B. H a e a
C. H a e e a
D. H e ce a

Ca d 31
Get The Step 2 CK Drills Book: usmledrills.com
The USMLE Guys Step 2 CK Crash Course usmleguys.com

Ta a a
A a Fb a :
● Lac fd c P a e
● S c de a a , fa e, eaded e
● I e a e a ( a e RR e a )
● Ec cf c a e : I a e af b

A a Fb a E a a :
● ECG
● Ec ca d a
● TSH, f ee T4, CBC, e c ea e, HbA1c

D ea e C A c a ed Af b:
● H e e , a a d ea e e ec a a e a d e a , CAD,
COPD, b c e ee a ea, d abe e , e d , a ed /d ,
ac

A a Fb a T ea e :
● U ab e a e : S c ed ca d e
● S ab e a e : Ra e c
● Ra e c : Be a b c e , -d d d e ca c c a e b c e , d
● R c : Ca d e (f a e dd e d aec ,c e
a e , ef e c a cd f c d e af b)

CHA2DS2-VASc:
Condition Points
C e e ea fa e( ef e c a
C 1
cd f c )
H H e e 1
A2 A e 75 ea 2
D D abe e e 1
S2 P e TIA b e b 2
V Va c a d ea e 1
A A e 65 74 ea 1
Sc Se ca e ( .e. fe a e e ) 1

CHA2DS2-VASc Sc e:
Score Risk Anticoagulation Therap
0 L N a c a a ea
1 M de a e Oa a c a a a d be c de ed
2 or more H Oa a c a a ec e ded

Ca d 32
Get The Step 2 CK Drills Book: usmledrills.com
The USMLE Guys Step 2 CK Crash Course usmleguys.com

ECG A a F b a :

A a F e:
● Ra d, e a a a de a a ( a a e )
● M a e a ea e c d ( a a a ea d ea e, -ca d ac e ,
a d a d ea e )
● S c de a a , fa e, eaded e ( e af b)
● Ra e c : N -d d d e ca c c a e b c e a be a b c e e
e d
● C ea ed ab a a a e f cce

ECG A a F e:

Pe a eA a C ac (PAC):
● P a e cc ea e a ca d ac c c e a a a P a e
● Ca be be a ea ea e e a a c a f c a
ca d ac d ea e
● Ca be a ec af b
● O eed be ea ed be a b c e f a c( a a )
● 24- H e a f e f e e c f PAC
● Ec ca d a a e ca d ac c e/f c

Pe a eA a C ac ECG:

Ca d 33
Get The Step 2 CK Drills Book: usmledrills.com
The USMLE Guys Step 2 CK Crash Course usmleguys.com

Pa a S a e c a Tac ca d a (PSVT):
● S dde e, e a, a c e ac ca d a
● I c de AVNRT, AVRT, a a ac ca d a, a d c a ac ca d a
● S : Pa a , SOB, c e, eaded e , d a e ,c e a

Pa a S a e c a Tac ca d a (PSVT) T ea e :
● U ab e a e : S c ed ca d e
● S ab e: Va a a e e ( a a a, ca d a a e) ade e e ea dde P
a e
● If e , e AVNRT e AVRT, ee e P a e f fa a
f e a a ac ca d a
● If a e d e d ade e, Va a a, a ca d a a e e be a b c e
CCB

Pa a S a e c a Tac ca d a (PSVT) ECG:

W de C e Tac ca d a:
● Ve c a ac ca d a (V- ac ): A a ae de f e a c d c
e a d be e AV de
● SVT abe a c : I e acce a a d ec ac a e e c a
ca d
● If f bea e e = AV d ca = V- ac
● Unstable de c e ac ca d a (V- ac SVT abe a c ): S c ed
ca d e
● S ab e V- ac : A da e ( ca a de)
● S ab e a d e f V- ac SVT abe a c : Ra e c ( a a) a e e
a d ea

Ve c a Tac ca d a:

Ca d 34
Get The Step 2 CK Drills Book: usmledrills.com
The USMLE Guys Step 2 CK Crash Course usmleguys.com

W ff-Pa -W e (WPW) S d e:
● Acce a a :I e d ec f a a
e ce AV de
● Delta a e, PR e a , ST a d T a e c a e
● Af b c WPW a e a d ca de e a e
e c a fb a
● A d a AV b c a e (BB, CCB, d ,
ade e)
● He d a ca ab e a e : S c ed
ca d e
● He d a ca ab e a e : P ca a de
ad f e e c ab a
WPW ECG

T ade de P e :
● C a de QRS c e ee ECG ( f e
QRS c e a ea )
● Pa a , eaded e , c e
● U ab e a e : IV a e a d def b a
● S ab e a e : IV a e , ea de ca e f a e e a

QT P a :
● H a e e a, a e a, ca ce a
● Je e a d La e-N e e d e (a a ece e, e e a deaf e )
● R a -Wa d d e (a a d a )
● ABCDE d ( e c)
○ ca IA a A c ( d e, ca a de, a d d a de) d e
QT e a
○ ca III a A c ( a a d b de)
○ a B c ( ac de, f e)
○ a C c
○ a De e a ,D e c
○ a iE e c

T ade de P e (ECG):

Ca d 35
Get The Step 2 CK Drills Book: usmledrills.com
The USMLE Guys Step 2 CK Crash Course usmleguys.com

Content Re ie Questions:

A 66 ea d fe a e f e e a a a d f d be af b
ECG. W c f e f ec e da a a e?

A. N a aee a c a a ea
B. A
C. Wa fa
D. A a d a fa

A 59 ea d a e a e e a a a a d eaded e . H ea ae
150 a d ECG a a fb a RVR. H b d e e 65/40. W a e
a ae e e ?

A. S c ed ca d e
B. IV e
C. IV e a a
D. IV ade e

W ee e de a a e f W ff-Pa -W e (WPW) d e ee ECG?

A. QRS c e
B. ST e e
C. T a e
D. P a e

A e de a e be ea ed f a UTI a a a e bec e c ba e aff. T e


ae e a e d a d2 a e e e e ce c e. W a e e ee
ECG?

A. A a fb a
B. A a f e
C. S e ec ca a e a
D. T ade de P e

A a e f d a eT ade de P e ECG. W a e a ae a
ed ca f ea e f c d ?

A. IV e
B. IV ade e
C. IV a e
D. IV d e

Ca d 36
Get The Step 2 CK Drills Book: usmledrills.com
The USMLE Guys Step 2 CK Crash Course usmleguys.com

Ba a a
S B ad ca d a:
● N a f d a ee , ad a d e de , e ec a e a ee
● S HR <60
● N ea e e ed f a a c
● If a c a d e d a ca ab e, c ec f MI ( e c a, fe a )
a d ea f e e
● If MI, c ec f e c d ( d , fec , ed ca )
● If e d a ca ab e: IV a e 0.5 e e 3-5 e 3
● If e e a e, ace e a ace a e

Pe a e Ve c a C ac (PVC ):
● U a a a c, cca a a a
● Occa a cc a a ea
d d a ,b a a c a ed ce a
ca d ac a d -ca d ac d ea e a e
● O eed add a e f a e ec ed
a e ca d ac d ea e

PVC on ECG

F De ee AV B c :
● De a ed a f ef ea a e
e ce e fc d c
● PR e a >0.2 ec d
● S d ab (e ec e , TSH) a d
e a a fc e ed
● U a be ,b a c a ed f a ea d
d a ed ca d a e

First Degree AV Block

Second Degree AV Block

M b T e I:
● PR e a e e e e a d e a -c d c ed P a e cc .
T e f ed b a e PR e a c a ed e a P a e bef e e
b c ,a d ce e ea

M b T e II:
● PR e a e a e a e bef e a P a e cc c fa c d c e
e ce .

M b T e I Sec d De ee AV B c :
● Y e a e a e e (d e a a e) d e a e
e ea b c
● S d ab (e ec e , TSH) a d e a a fc e ed

Ca d 37
Get The Step 2 CK Drills Book: usmledrills.com
The USMLE Guys Step 2 CK Crash Course usmleguys.com

● I c ea PR e a a d e ed RR e a eac c d c ed P a e f e
c ce
● G ed bea e a e cc c a f 3:2, 4:3, 5:4
● A c a ed ca d ac d ea e ( f a e), fe a MI, e d ea e, AV de
b c ed
● Ca d a a e ( c ea e a a ) e M b eI
● A e e M b eI
● T ea e : If a d e e b e ca e, ea e ece a (
e !!)
● T ea e f ab e a e :
○ A e
○ If ab e, e a c a e ac
○ If BP IV d a e HF IV d b a ea d e a e ac a d
e ce ab e e e b e ca e a d ea
○ If e e be e ace e f e a e ace a e

M b T e II Sec d De ee AV B c :
● C a PR e a , e P a e fa c d c e e ce
● A c a ed ca d ac d ea e ( f a e), fe a MI, e d ea e, AV de
b c ed
● S d ab (e ec e , TSH) a d e a a fc e ed
● S :A a c fa e, d ea, c e a , e c e c e,
dde ca d ac a e
● E ec ea da e e M b e II
● Ca d a a e e M b e II
● T ea e f ab e a e :
○ A e
○ If ab e, e a c a e ac
○ If BP IV d a e HF e IV d b a ea d e a e ac
a d e ce ab e f e e b e ca e a d ea
○ If e e b e ca e ace e a e ace a e
● S ab e a e a e a c a e ac ad aced e e e e b e ca e
ae e a ed. If e e b e, e a e ace a e

2:1 AV B c :
● D ff c de f M b eI M b e II ec d de ee AV b c
● PR e a ea e a 0.3 ec d e QRS c e a , e M b eI
● W e a e e e fe e c d c ed P a e , ea eb c
definitel M b eI( e e a e e M b e II)
● If ca d a a e e b c ,M b e I; f e b c ,M b e II

Ca d 38
Get The Step 2 CK Drills Book: usmledrills.com
The USMLE Guys Step 2 CK Crash Course usmleguys.com

ECG Sec d De ee AV b c :

T d De ee AV B c :
● P a e a e ea QRS
c e e
● S d ab (e ec e ,
TSH) a d e a a fc e ed
● A c a ed ca d ac d ea e
( f a e), fe a MI, e
d ea e, AV de b c ed ,
c e a:A c a ed ae a

● S : Fa e, d ea, c e a , e c e c e, dde ca d ac a e
● H fe c a a a a e
● T ea e f ab e a e :
○ A e
○ If ab e, e a c a e ac
○ If BP IV d a e HF IV d b a ea d e a e ac
○ O ce ab e f e e b e ca e a d ea
○ If e e b e ca e, ace e a e ace a e
● S ab e a e a e a c a e ac ad aced e e e e b e ca e
ae e a ed. If e e be= e a e ace a e

Ca d 39
Get The Step 2 CK Drills Book: usmledrills.com
The USMLE Guys Step 2 CK Crash Course usmleguys.com

Content Re ie Questions:

O ECG a a a c a e f d a e a PR e a f 0.25 ec d e
ab a e ed a d d a ed f de ee AV b c . W a e a ae
e e a a e e ?

A. A e
B. Te a ace a e ace e
C. Ade e
D. TSH a d e ec e

W c e f AV b c c a ac e ed b a PR e a e e c ea e
a -c d c ed P a e cc ?

A. F de ee AV b c
B. Sec d de ee AV b c M b eI
C. Sec d de ee AV b c M b e II
D. T d de ee AV b c

W c f ef c d e e b ad ca d a AV b c ?

A. A e a MI
B. I fe a MI
C. P a e b
D. H e c c

Ca d 40
Get The Step 2 CK Drills Book: usmledrills.com
The USMLE Guys Step 2 CK Crash Course usmleguys.com

Ca a A
I a Se :
Initiate CPR
● P de e
● A ac a d def b a
● A e

A e/P e e E ec ca Ac (PEA):
● PEA: P e e ce f a ea b e e ea c ac e e e ec ca
ac e ca d ac f c ac ea d ce e(
e d- a e f )
● A e: C e e ac f e ec ca a d ec a ca ea ac ( e d- a
ef ). Ca d ac f a e
● A e a d PEA a e - c ab e

A e/P e e E ec ca Ac (PEA):
● I a e
● CPR f 2 e
● Ob a IV/IO acce
● E e e e e 3-5 e
● Sec e a a
● Rea e f c a e c ab e
● If c ab e c e CPR a d e e e a ab e, a d ea
e e b e ca e f a e/PEA, c e c ec e e 2 e f a c ab e

A e/PEA Re e b e Ca e (5 H a d T ):
● H :
○ H a
○ H e a
○ H e ae a
○ H d e (ac d )
○ H e a
● T :
○ T ( ed ca )
○ Ta ade
○ T b (c a a )
○ Te e a
○ Ta a

A e/P e e E ec ca Ac (PEA):
● CPR f 2 e
● Ob a IV/IO acce
● E e e e e 3-5 e
● Sec e A a
● Rea e f c ab e ( .e. e e V Tac , V F b)
● *S c ab e e e e c , e ea CPR f 2 e a de e ee e
3-5 e a d ea e f c ab e
● S c ab e e e a a e c c e CPR f 2 e b
amiodarone a d ea e e b e ca e

Ca d 41
Get The Step 2 CK Drills Book: usmledrills.com
The USMLE Guys Step 2 CK Crash Course usmleguys.com

● F f ee e c ab e e e ea
e b e ab e a ea e ,e d a da ea d e
e ea e a e e dec a e e dead

Ve c a Fb a /P e e Ve c a Tac ca d a:
● I a e ( ee ab e)
● Shock
● CPR f 2 e , b a IV/IO acce
● If c ab e c a a CPR f 2 e ,e e e e e 3-5 e ,
ad a ced a a c ec , f c ab e e c CPR f 2 e , e
a da e, ea e e b e ca e ( e e b ee ee a
e ea : c ec , f c ab e e c ,e e e e e 3-5 e c ec
, f c ab e e c CPR f 2 e , ea da e e ea )
● If c ab e c e CPR a d e e ea dc e c ec e
a d e e 2 e

Ve c a Fb a :
● Ve c a a a f e c a c ac
● N ca d ac , b d e e
● S : S c e/ca d ac a e
● ECG e a QRS d a a ab e a da de

Ve c a Fb a ECG:

Ca d 42
Get The Step 2 CK Drills Book: usmledrills.com
The USMLE Guys Step 2 CK Crash Course usmleguys.com

Content Re ie Questions:

Y ae e a ed c e a a daef a e a a c de b e. W e
a e e a e , e aef d a e ea dae b ea .W a e be
a e ?

A. P ace a e (ECG ad , BP c ff, e )


B. S a a d IV/IO
C. S c
D. S a CPR

Y a e ca ed a c de b e a d a a a e CPR, de e , a ac
a d def b a , a d af e 2 e f CPR a e e .T e aef d be
e c a fb a .W a e e be e a a e e ?

A. CPR
B. Def b a
C. E e e
D. A da e

Y a e ca ed a c de b e a d a a a e CPR, de e , a ac
a d def b a , a d af e 2 e f CPR a e e .T e a e f d be
a e. CPR e a ed, IV acce b a ed, a d e a e cce f ba ed.
W a e e be e a a e e f a e ?

A. IV e e e
B. IV a da e
C. IV d ca e
D. IV a e

Ca d 43
Get The Step 2 CK Drills Book: usmledrills.com
The USMLE Guys Step 2 CK Crash Course usmleguys.com

A D a
Ma fa S d e (a c d de e f a d e c a ac e c ):
● FBN1 a a d a a (e c de ec ec e e e fb -1)
● Ca d ac d ea e e a ca e f bd a d a :A c d ea e
(a e a d aa ,a c e a , a d d ec )a d a a e a e
● MSK d ea e: A ac dac , ec def , df a ,d c e e a,
c ,a d
● O e a c a ed d ea e: Ec a e , d a ec a a, a e e a ,
ae

Aortic Dissection

S a f d C a f ca :
● T pe A e e ascending aorta ( a a ea c a c a d de ce d
ac c a a)
● T pe B a e d ec e a ce d a a.

Ca e :
● De e e a e (HTN, c ac c ca e, e f )
● Ge e ca ed a ed (Ma fa , L e -D e , a c a E e -Da d e, T e
d e)
● Ta a c (b f ce , a e c)

C ca T ad:
● S a / fe- e ea / a a e eea d ca ed e c e /abd e
a d/ bac ee a f a e
● P ca e a f d : Ab e ce f a a e e ca d e a d/ >20
H cb d e e d ffe e ce be ee a
● Med a a a d/ a c de CXR

A cD ec F d :
Finding Anatomic Cause

F ca e c def c ( e) D ed ca d BP
H a e e C e ed ef ec e a ea e e
H e d e C e ed e ce ca a e c a
Ac e a a e a S a c d c e a
S c e, e Ca d ac a ade, a c a e e a ,
ca d a fa c , e a ,
e e e

A c D ec I a :
H :Ta e a ea ec ca d a (TEE)
H : CT a a

Ca d 44
Get The Step 2 CK Drills Book: usmledrills.com
The USMLE Guys Step 2 CK Crash Course usmleguys.com

Ma a e e :
● 2 a e b e IV , a e a e
● Ma a HR e a 60 a d c BP be ee 100-120
● T pe A: Be a b c e a d e
● T pe B: Be a b c e
● O e a d a f eeded f BP c ,a d af e HR c ed a d BP
e be a b c e a e
● G e IV d f a a e a
● F e ca e e a e e a d d e ef

T ac c A c A e (TAA):
● Def ed a 50% c ea e da ee c a ed a a da ee f a e e f
ea a
● U a a a c
● A ce d a ca e : Ma fa , L e -D e , a c a E e -Da d e,
T e d e
● De ce d a a e ca a ea e :A e ce , de a e, ,
a d e e

Location of Aneur sm Anatomical Position

A ce d a ca e F ea c a e e b ac ce a c
A cac a e I e e b ac ce a c e e
De ce d a ca e D a e ef bc a a a e

Location S mptoms/Findings

A ce d a e Hea fa e 2/2 a c e a f a c d aa a d
a a d

MI 2/2 c e fac a a e
Ta e ea d D a a 2/2 e a ea c e
de ce d ac
a e H a e e 2/2 ef ec e a ea e ec e

He da a a c aa 2/2 e c e ec e

C , e ,d ea, e 2/2 c e f
e ac e b c a ee

T b e b e e a ca a d e 2/2
c e f e ce a e e e a ca a
De ce d a a T ea e a e de e ad ace e, ca bac
a e a

Ca d 45
Get The Step 2 CK Drills Book: usmledrills.com
The USMLE Guys Step 2 CK Crash Course usmleguys.com

T ac c A c A e (TAA):
● M e c ca e: Se e e c e a a d e
● De ce d TAA e ca ca e a -e a ea f a: He a e e
● Chest X-ra findings: W de ed ed a a e e, e a ed a c b, ac ea
d aced f d e
● Best test: E e CT MR a a

Abd a A cA e (AAA):
● Da ed abdominal ultrasound e da ee ea e a 3 c
● Risk factors: Ad a ced a e, Ca ca a ace, fa f AAA, bacc e,
e e ce f e a e e e a e ,a e ce
● Negative risk factor: D abe e e
● Surgical/endo ascular repair indicated if:
○ A a c AAA 5.5 c
○ Ra d e a d AAA (d a e e 5 e a - e d f e >10
e e ea )
○ AAA a c a ed e ea a e a a e a c e ea
a e d ea e a e a c a a
○ He d a ca ab e a e AAA e e e
( e , f a /bac a , a e a ), a e e eca c f
e a d ece e a bed de a dc f e e ce f AAA
** R - AAA 65
75 - AAA
AAA .

A ac Occ e D ea e (Le c e S d e):


● Occ a e b f ca f ea a ec ac a e e
○ B c , , a d, e ca e , c a d ca
○ E ec e d f c
○ Ab e d ed fe a e

Content Re ie Questions:

A 30 ea d a e e e e ED ac a f ea c e a ad a e
bac . O e a e a e ed a e 22 H cb d e e d ffe e ce b d
e e be ee a a d e e a MSK ab a e c d a ac dac a d ec
def .W a e e e ed a e ?

A. CFTR e e
B. HTT e e
C. FBN1 e e
D. HEXA e e

Ca d 46
Get The Step 2 CK Drills Book: usmledrills.com
The USMLE Guys Step 2 CK Crash Course usmleguys.com

A a e e e e ED a / fe- e a a e e e a d ca ed ec e .
T e a e a a 25 H cb d e e d ffe e ce b d e e be ee a
a d CXR ed a a de a ed. T e a e e d a ca ab e. W a
e e be e c f eda ?

A. CT a a
B. Ta e a ea ec ca d a
C. MR a a
D. Ta ac c ec ca d a

W c f ef a d ca f abd a a ca e e ?

A. A a c AAA 6.2 c d a e e
B. K AAA ada ee a e a d 6 e
C. K AAA ada ee a e a d 9 e a ea
D. He d a ca ab e a e AAA e e e
e ,fa a , a e a

Ca d 47
Get The Step 2 CK Drills Book: usmledrills.com
The USMLE Guys Step 2 CK Crash Course usmleguys.com

M a a I a

Region of the ST segment ele ations on ECG Coronar Arter In ol ed


M ocardium

A e e a V1-V2 LAD
A e a ca V3-V4 LAD
A e aea V5-V6 LAD LCX
La e a I, aVL LCX
I fe II, III, aVF RCA ( a )
LCX ( e fe e )
P e P e ead ECG V7, V8, V9 LCX
e -ae a ( f ST de e e e V1 V3
c de d e ead ECG f d
ST e e a e e)
R e c e MI V4R, V5R, V6R (c ec - ded ead f RCA
ab a e f e ST e e T a e
ead II, III, a d aVF)

Complications Following Acute MI

Lef Ve c a F ee Wa R e:
● H - ee
● S f e ca d a a ade
● Ec ca d a a e ca d a eff , a e e
● Ma a e f d , e a d a e , e ca d ce e , ed a e ca d ac
e

I e e c a Se R e:
● H -7 da
● H e a d ac ca d a
● Ne : Ha , d, a d c. Hea d a e e ef a d e a
b de
● R eda ed ec ca d a ( ef e c a )
● I ca e a d a e f a e ca d e c c , ed a e
ca e a

Pa a M ceR e:
● H -7 da
● H e a d e ee a ede a
● M d-, a e-, c ; de ead ad a ; e e e ac e MR
● Ec ca d a e ea f a e e f e a a e
● T ea af e ad ed c d de, ae ,a dd e c
( e e e a d a e f e e) a d e e e c ca
e e

Ca d 48
Get The Step 2 CK Drills Book: usmledrills.com
The USMLE Guys Step 2 CK Crash Course usmleguys.com

Ve c a A e :
● Wee -
● e e ST e e e e a af e ece MI
● C f ed ec ca d a d e c a fa f ef
e ce
● M a e a
● T ea af e ad ed c (ACE b )a da c a a f b ec
f ca LV d f c

De e S d e:
● 7-90 da af e a MI
● Pe cc e a a d fe e
● Pe ca d e ca d a eff
● Le c , e e a ed ESR, a d CRP
● D ff e ST- e e e e a ECG

Ve c a A a :
● 2 e c c (a e af e 48 ,a e ea ea a d ca )
● S dde ca d ac dea d ac e MI a e f e c a a a (VF)
● Re f ca d a c e a, ec , e ef , ea , a d ca f a

Content Re ie Questions:

A fe a MI e ee ST e e a c f ef ead ?

A. II, III, aVF


B. V1-V3
C. V4-V6
D. V7-V9

A a e c e e ED c b e a c e a b b e e a d e
ST e e e e a a e ee ead I, aVL. W c a e e ed?

A. LAD
B. RCA
C. LCX
D. A a

W c f ef c ca e cc 2 f MI?

A. De e d e
B. Pa a ce e
C. I e e c a e e
D. Lef e c a f ee a e

Ca d 49
Get The Step 2 CK Drills Book: usmledrills.com
The USMLE Guys Step 2 CK Crash Course usmleguys.com

Endocrinolog
H

S /S :
● Pa a ,a e , e , e , ea e a ce, a e ea, a
e a , a c e ea e , e ef e a, e e d e ac ,a a fb a

Ca e fH e d :
● G a e ' d ea e, c ade aa d c d a e , TSH- d c a
ade a , bac e d

Lab :
● Be a e e TSH:
○ If e TSH a, f e - ece a ( e e a e a
c ca f e d d fd f c e
a a d/ aa ec ed c ca e ef e f ee T4)
○ If e TSH e e a ed, e f f ee T4 de e e e e e f
d
○ If e TSH , ef f ee T4 a d f ee T3 de e e e e e f
e d

Graves Disease

TSH ( )- ece a b d e (TRAb) ac a e e ece ca a ae


e e a ed d e d c a d ec e a d f e ead e f a .

S :
● T e f e d e a c a ced f c , e b a
ede a, e

Lab :
● P e ece a b de
● L TSH, e e a ed f ee T4 a d T3

I a :
● Rad ac e d e a e ca d ff e c ea ed a e

Da :
● P e ece a b d e OR ad ac e d e a e ca
diffuse increased a e

T ea e :
● Be a b c e (a e )f e a a c e ef
● U e f a de ( e a e, PTU f e a ) e f e (1-2
ea f e), a -e ea , a e ea e de ad d e
ab a dec
● Rad d e ab a OR dec f def e ea e

E d c 50
Get The Step 2 CK Drills Book: usmledrills.com
The USMLE Guys Step 2 CK Crash Course usmleguys.com

To ic Adenoma and To ic Multinodular Goiter

S /S :
● S f e d a d b a ab e d d e ca e a

Lab :
● L TSH e e a ed f ee T4 a d/ T3

I a :
● If ec ed d d e fe ca e a , a d d be e f ed
(c f d e e e ce a d c a ac e a f d e)
● T c ade a: Rad ac e d e a e ca c ea ed a ea e
ca fa e a ab e d d e, f e e f ad ac e d e
a e d d e
● T c d a e : Rad ac e d e a e ca c ea ed a e
e e f ca a ea

Da :
● Made ad ac e d e a e ca c e d de ( ee a )

T ea e :
● Be a b c e (a e )f e a a c e ef
● U e f a de ( e a e, PTU f e a )a a e ea e de
ad d e ab a dec
○ Pa e e e e , e a e d ea e
● Rad d e ab a dec

Subacute Th roiditis

Ca ed b a fec (a e - a fa a d ea e).

S :
● Neck pain; tender diffusel enlarged th roid; a d ca f
e d f 2-8 ee f ed b e d , e d ,a d
e ba e e df c ; f e ece f e e a fec
( a fec )

Lab :
● L TSH e e a ed f ee T4 a d T3 a (2-8 ee f e d ) e
TSH a d e e a e f ee T4 a d T3 ( d )
● E e a ed b
● E e a ed WBC c
● E e a ed ESR a d CRP

I a :
● Rad ac e d e a e ca a e

Da :
● U a ade c ca ba ed a e
● E e a ed ESR a d/ CRP AND ad ac e d e a e ca a e
d e e d a e ca c f da

E d c 51
Get The Step 2 CK Drills Book: usmledrills.com
The USMLE Guys Step 2 CK Crash Course usmleguys.com

T ea e :
● T ea a NSAID ed e
● If ece a , be a b c e (a e )f a c e ef e a e a f
e d
● If ece a , e e e a e a f d

TSH-Producing Pituitar Adenoma

S :
● S f e d a d de a e e ( e)
● V a f e d defec , eadac e
● Fe a e a e ca e e e ce a ac ea a d/ a e ea/ e ea

Lab :
● N a e TSH; e e a ed e a , f ee T4 a d T3
● E e a ed a a b f c e e

I a :
● Rad ac e d e a e ca a ad d e a e, d ff e
e e de a e e
● MRI f e a ad a ac ade a c ade a

Da :
● S f e d a e d ea e
● Lab a e TSH, e f ee T4 a d T3, e e a ed a a
b
● Ade a ee MRI f e a ad

T ea e :
● Oc e de e ee d e a e da e ec f e a
ade a

Content Re ie Questions:

A a e e d e e , ea e a ce a d f d a e e ef e a ca
e a .W a e be a e de e f e a e a e d ?

A. F ee T3
B. F ee T4
C. TSH
D. T a T3

A 38- ea - d a e e d e e ,a e ,a d a a a d a a ab e d d e
fe ca e a . Lab TSH e e a ed f ee T4. Of e f , c
e e be e a a e e f a e ?

A. T dec
B. F e eed e a a f d e
C. Me a e
D. T d a d

E d c 52
Get The Step 2 CK Drills Book: usmledrills.com
The USMLE Guys Step 2 CK Crash Course usmleguys.com

A 60- ea - d fe a e e e a e , e , a d ea e a ce. S e ed
a e e e a e a a d ab e c a ced f c . Lab
TSH, e e a ed f ee T4 a d T3 e e , a d e ef ece a b de .
W a e e f d a e ad ac e d e a e ca ?

A. D ff e c ea ed a e
B. D ff e dec ea ed a e
C. N d a (f ca ) c ea ed a e
D. N d a (f ca ) dec ea ed a e

E d c 53
Get The Step 2 CK Drills Book: usmledrills.com
The USMLE Guys Step 2 CK Crash Course usmleguys.com

T N ,T Ca ,T S ,a H
T dN d e :
● T d d e ae ec ed ba ed ca e a ee a a c de a f d
a
○ Ne be e de TSH a d e f ultrasound f e d
○ If TSH a e e a ed:
■ Pe f f e eed e a a (FNA) f d e f a c e
be a a c ( d ec c, c ca c f ca ,
e a/ f a e a , a c e e d be d da
e e be ca ce )a d e1c e
■ If a c c e a f FNA e (c c, dc e ),
a e d be ed d a d
○ If TSH e be e ef radioacti e iodine uptake scan and
measure FT4 and T3
○ If ad ac e d e a e ca f c a d e:
■ Pe f f e eed e a a (FNA) f d e f a c e
be a a c ( d ec c, c ca c f ca ,
e a/ f a e a , a c e e d be d da
e e be ca ce )
■ If a c c e a f FNA e (c c, dc e )
a e d be ed d a d
○ If ad ac e d e a e ca f c a d ea d a e a FT4
a d/ T3 e e a , ea f c ade a/ c d a e
○ If ad ac e d e a e ca f c a d ea d a e a a
FT4 a d T3, b e e ( bc ca e d )

Th roid Cancer

S :
● Ra d d a , a e e , f ed d e a ac ed ad ace e , aea
ce ca ade a

FNA f d e ca a c e ca ce .

F Ma T e f T d Ca ce :
● Pa a d ca ce : M c d ca ce , c e a e a c ea
c a c c e " a A e e e a ea a ce
● F c a d ca ce : Sec d c d ca ce , a c a ed
RAS a
● A a a c d ca ce : Ra d e a ec a , e
● Med a d ca ce : Ne e d c e ade f a af c a (C ce )
e d, e e ca c ,a c a ed RET e e a a d MEN2
c d

Th roid Storm

R Fac :
● U ea ed e d , a a, e ,b , fec , ce a d ec a
d (a da e, c a )

E d c 54
Get The Step 2 CK Drills Book: usmledrills.com
The USMLE Guys Step 2 CK Crash Course usmleguys.com

S /S
● Fe e ( 104-106), ac ca d a a a, ea fa e, e ,
AMS/c a, e

Lab :
● L TSH a d e e a ed f ee T4 a d/ T3, e ce a, e ca ce a, be
ab a f WBC c ( ), e e e a ed ALT a d/ AST

Da :
● Se e e c a fe e , AMS/c a, a d/ ca d ac d f c a d ab
c e e d ( TSH a d e e a ed FT4 a d/ T3)

T ea e :
● Be a b c e a de (PTU) d e d a ed ad c a
a e c c c d b e ac d e e a (c e a e)

H poth roidism

S /S :
● C d e a ce, fa e, de e , e a , ed c , a /c a e
a ,b e a ,c a , ff fac e , e a ed e, a e e , ede a,
/a e ea e a a, dec ea ed b d , e ec e d f c

Ca e fH d :
● Ha d , dec , ad d e ab a , ec ad a , d e
def c e c , S ee a d e

Lab :
● Be a e e TSH
○ If e TSH a, f e - ece a ( e f e a e a
c ca f e d d fd f c e
a a d/ aa ec ed c ca e ef e f ee T4)
○ If e TSH e e a ed, e f f ee T4 de e e e e e f
d
○ If e TSH , ef f ee T4 a d f ee T3 de e e e e e f
e d

Hashimoto s Th roiditis

De c f e d a d a e ce ca e e d fa e
( e a be e e ).

S /S :
● S f d b e e ce a a , d ff e e ( ca
e de )

Lab :
● H e TSH, e f ee T4

E d c 55
Get The Step 2 CK Drills Book: usmledrills.com
The USMLE Guys Step 2 CK Crash Course usmleguys.com

P ea b de ef :
● T b (T )
● T d e da e (TPO)
● TSH ece (ae )

Da :
● C ca a d ab f d c e Ha ( a d f
d , e TSH, e f ee T4, e TPO, a d/ T
a b de )

T ea e :
● Le e( d e e ace e ) f fe
● Ha e ce a a :G c c c d e e f d

Content Re ie Questions:

A 65- ea - d a e a f G a e d ea e ec e e PACU f
a ee e ace e . T e a e bec e ac ca d c e 140 . T e a e e ea e
e 105.2 F a d b d e e 90/60. T d ec ed. W c f e
f d ca ed a a f e a a e e ?

A. Labe a
B. PTU
C. I ed a e a dec
D. C e a e

A 44- ea - d fe a e f d a e e ea ec ed d d e ba ed ca
e a .W a e e be e a a e e f e ed?

A. U a d f e d
B. F e eed e a a f e d e
C. T a dec
D. Rad ac e d e a e ca

Ha d ca a c a ed c f ef f d ?
A. L e TSH
B. T b a b de
C. T d e da e (TPO) a b d e
D. D ff e e

E d c 56
Get The Step 2 CK Drills Book: usmledrills.com
The USMLE Guys Step 2 CK Crash Course usmleguys.com

Pa a D a
Primar H perparath roidism

S /S :
● T ca a a c d e ca ce a
● B e , e , (abd a) a ,a d c a c e e ae ae e e a :
O e fb ac ca ca b e a , e a , abd a a /c a ,
de e a d/ c

Lab :
● E e a ed a a d e (PTH), e ca ce a, ae a,
a e e a, e e a ed 1,25-d d a D

Da :
● H e ca ce a a d e e a ed PTH (def e)
● L b e e a de , e ca ce a b e e a ed PTH ( ca ce c
a e aa d )
● If a e a e ca ce a a d PTH e dd e- e f a a e f PTH,
ef 24- a ca c e ce
○ If >200 e Ca e da OR Ca/C a >0.02 d a a
e aa d

T ea e :
● Pa a dec f a c a e

Familial H pocalciuric H percalcemia

A a d a a f e ca c - e ece e e ca c a d
e ca c e e .

S /S :
● T ca a a c( a f e ca ce a)

Lab :
● H e ca ce a, a PTH, ca c a, e a e e a
● 24- a ca c e ce <200 /da
● Ca/C a <0.01

Da :
● H e ca ce a, PTH a d e e a ed, 24- a ca c e ce <200
/da , a d/ Ca/C c ea a ce <0.01 a d ca e f fa a ca c c
e ca ce a

T ea e :
● U a ea a ce a e (be d ea e c e), f e ca ce ae e e
c de c aca ce

E d c 57
Get The Step 2 CK Drills Book: usmledrills.com
The USMLE Guys Step 2 CK Crash Course usmleguys.com

Secondar H perparath roidism

Occ e a e a d ea e a e ca def c e c ca c ab , ade ae


a e, e ab .

Ca e :
● Chronic kidne disease (CKD), ca c a ab , ed ( d e c ,
b a e ), ca c a e

S :
● U a a e a de e d d ea e a e ca ec da
e aa d

CKD Lab F d :
● L 1,25 d d a D, ca ce a, e ae a, e e a ed PTH

T ea e :
● Dea ae e c f ed b a e b de (fde a e fa ) f
a e e ae a
● V a D e e f e a D def c e c

H poparath roidism

Ca e fH aa d :
● T dec / aa dec ,a e de c f aa d a d ,
e e cd de f e aa d

S /S :
● Te a , T ea ' ,C e ' , e e , ed QT e a,a a ,
ea fa e, a e /de e

Lab :
● H ca ce a, a PTH, a 1,25-d d a D,
e ca c a, e ae a

Da :
● H ca ce a a a ae a PTH a d e ae a

T ea e :
● Ac e a c d ea e (f ca e a f aa d a d ) ea
IV ca c a ca c
● C c aa d (a e d ea e, f a e d ea e, ad a ) ea
a ca c a D

E d c 58
Get The Step 2 CK Drills Book: usmledrills.com
The USMLE Guys Step 2 CK Crash Course usmleguys.com

Content Re ie Questions:

W c f ef ab c e eda f a e aa d ?

A. 24- a ca c e ce <200 /da


B. 24- a ca c e ce Ca/C a >0.02
C. E e a ed a a d e
D. H ae a

A 21- ea - d a e a ed ca de e a ea ca ee e
ed a e d e ca ce a a ab a ab f d . F e - a
a PTH e e a d 24- a ca c e ce <200 /da . W a e e
da ?

A. P a e aa d
B. Sec da e aa d
C. Fa a ca c c e ca ce a
D. E ce e ca c a e

A a e a e aa d e e e c f ef
f d ?

A. Te a
B. Ne a
C. P ed QT e a
D. P eT ea '

E d c 59
Get The Step 2 CK Drills Book: usmledrills.com
The USMLE Guys Step 2 CK Crash Course usmleguys.com

D a A a Ga
Primar Adrenal Insufficienc (Addison s Disease)

U a e e fa e de c f e ad e a a d ; ca a e f ad e a
a d fa c , fec , ea a c a .

S /S :
● H e ,a e a, e , e e a , abd a a ,
a ea/ , a a /a a a

Lab :
● H a e a, e ae a, e ca ce a, ad e ,e a, a e a

Secondar Adrenal Insufficienc

D ea e f e a a d c e d ed ACTH ec e ( a ade a,
S ee a d e, a e ).

S :
● We , a ea/ , e (e e ), f
a / aa c ( eadac e, a f e d defec )

Sec da Ad e a I ff c e c Ha :
● Le f ca e , e ae a, e e a

Lab :
● H a e a

Da :
● H -d e ACTH a e (250 c c )
○ If ba a c a d ACTH e e a ed, e a ad e a ff c e c
○ If ba a c a d ACTH , e ec da e a ad e a
ff c e c

T ea e :
● P a ad e a ff c e c : G c c c d e ace e
( d c e/ ed e/de a e a e) PLUS e a c c d e ace e
(f d c e) PLUS a d e e ace e (DHEA)
● Sec da ad e a ff c e c : G c c c d e ace e (H d c e)

Adrenal Crisis

L fe- ea e e e e c ee a a e a , ec da , e a ad e a
ff c e c c e a ad e a e a ae c e a e c a a a, e ,
fec .

S :
● H potension, fe e , a ea/ , abd a a

E d c 60
Get The Step 2 CK Drills Book: usmledrills.com
The USMLE Guys Step 2 CK Crash Course usmleguys.com

Lab :
● Se c , ACTH, a d e e, e , CBC a d c e e

Da :
● C f ed ACTH a e AFTER ea e

T ea e :
● La e b e IV acce
● B 2-3 L NS
● H d c e ad d ef ed b a e a ce d e
● Ide f a d ea fec ca e(fa cab e)

Congenital Adrenal H perplasia (CAH)

21 H d a e Def c e c :
● M c >90% f CAH
● Dec ea ed c , dec ea ed a d e e ( a e a, e ae a)
● I c ea ed a d e , a b e a a (c e a ) fe a e
● I c ea ed 17 OH- e e e

11 H d a e Def c e c :
● I c ea ed 11 de c c e e( ea c c d)
● H e e
● I c ea ed a d e , a b e a a (c e a ) fe a e
● Dec ea ed c , dec ea ed a d e e

17 H d a e Def c e c :
● Dec ea ed c a da d e
● I c ea ed ea c c d
● H e e , ae a
● P e c fe a e ( e a fa a d a a e ea)

Cushing s S ndrome (H percortisolism)

S :
● R ded face, ae, ecc e , be , c e e a ce, d a fa ad, ede a,
e e , e e a,

Be I a Te :
● 24- a f ee c e ce ( e ea f 2 a e )
P ○ ef C S d e f a c e ce >3 e e
f a
● OR La e- a a c ( e ea f 2 a e )
○ P ef C S d e f e e a ed a e a a a d e c
e e
● OR O e 1 de a e a e e e
○ P ef C S d e fc >1.8 c /dL (fa e e
c )

I a Te ( ) P ef C S d e:
● Ne be e measure ACTH

E d c 61
Get The Step 2 CK Drills Book: usmledrills.com
The USMLE Guys Step 2 CK Crash Course usmleguys.com

● If ACTH is lo ( e a 5 / ) e e a e a af f ACTH de e de
C S d e a d adrenal MRI/CT d be e f ed
○ MRI/CT e ad e a / e a a
● If ACTH a /e e a ed ( ea e a 20 / ) e e a e a af f ACTH
de e de C S d e a d pituitar MRI d be e f ed
○ If a a a <6 ee , e f fe e a
a (IPSS). Ce a e e a IPSS d ca e Cushing s disease, ac f
ee a d ca e ectopic ACTH secretion
○ If pituitar mass >6 mm ee , e f CRH stimulation test a d
de amethasone suppression test. A ae e a d a
d ca e Cushing s disease

Primar H peraldosteronism

E ce e e ea e f a d e e f ae a ad e e d c
a e a /b a e a ad e a e a a.

S :
● H e e ( fe e a ee-d e e )a d ae a (c ca
ae a c )

W e W -U f P beP a H e ad e Da :
● H e e a d ae a
● Y a ea d e e e
● Pa e e a ee-d e e e e
● H e e ad e a c de a a a
● Se e e HTN ( ea e a 150 c, 100 d a c)

Be I a Te W e S ec P a H e ad e :
● Pa aad e e c ce a a a e ac OR a a e
c ce a
○ P e e a a a d e e c ce a e e a ed a d a a e
ac a a e c ce a
● C f da a d ad a d ea e e f ead e e
e ce (c f ed a a e e e a ed a d e e e e )

W e A B c e ca Te C e P a H e ad e :
● Pe f ad e a CT ca
● If CT ca b a e a ab a e , ab a e , a aea
ab a a e e 35, e f ad e a e a ( de e ead e e
e e ad e a e b d)

T ea e :
● U a e a ad e a ade a a e a ad e a e a a ea ed aea
ad e a ec
● B a e a ad e a e a a ea ed ac e

E d c 62
Get The Step 2 CK Drills Book: usmledrills.com
The USMLE Guys Step 2 CK Crash Course usmleguys.com

Pheochromoc toma

S :
● T ad f eadac e, ea , a d ac ca d a a a a ed
e e

Be I a Te :
● 24- e f ac a ed e a e e a d ca ec a e
○ Te e f ea e e, e a e e, e e e, e e e,
d a ef d be a a e e e a ed

Ne Se :
● Ne e af e e 24- e f ac a ed e a e e a d ca ec a e
CT/MRI f e abd e a d e
● If e c c a de f ed a b c e ca e a d a , e e c e
ef ed af e ca e ec f e e c c a
○ VHL d e, MEN2, NF1 e e c d de a c ea ed c de ce f
P e c c a

T ea e :
● P e a a-ad e e c b c ade ( e be a e) f ed b
be a-ad e e c b c ade ( a ) af e ade a e a a-ad e e c b c ade
● S ca e ec f e e c c a

Content Re ie Questions:

11 d a e def c e c e e e a ed e e f c f ef e ?

A. 11 de c c e e
B. 17 OH- e e e
C. 17 de c c e e
D. 11 OH- e e e

Pa e a ad e a ff c e c (Add d ea e) ca e e a b
c f ef ab ab a e ?

A. H e ae a
B. H e ca ce a
C. H ad e
D. H e a e a

A 45- ea - d a e a ed ca f Add d ea e e e e ED
1-da f fe e , a ea, a d .T e a e b d e e a a 88/60,
e e a e 101.5, ea a e 108, S O2 97%. W c f ef e e e a
ca e f a e ?

A. P e c c a
B. Ad e a c
C. C d ea e
D. E ce ee e e d e

E d c 63
Get The Step 2 CK Drills Book: usmledrills.com
The USMLE Guys Step 2 CK Crash Course usmleguys.com

A 41- ea - d a e fa f MEN2 e e e ED BP 160/104 HR 100


SPO2 98% a d T 98.6. T e e eadac e, ea ,a d a a .W c f e
f e be a e e ec a a e a a e c c a?

A. CT/MRI f e abd e a d e
B. P a a a d e e c ce a
C. P a a e ac
D. 24- e f ac a ed e a e e a d ca ec a e

A 39- ea - d a e e e ec c ae, ecc e ,a d e e .Y


ec C d ea d ef e a a e 24- a f ee c e ce ,
c aeb ef a c e ce ea e a 3 e e f a.
W a e e be e a e e ca e f a e C d e?

A. Mea e ACTH e e
B. P a MRI
C. MRI f e abd e a d e
D. Mea e a a a d e e c ce a

W c f ef a d ca - a a e e e f a
e ad e ?

A. H e e a d ae a
B. Pa e e a -d e e e e
C. H e e ad e a c de a a a
D. Se ee e e ( ea e a 150 c, 100 d a c)

E d c 64
Get The Step 2 CK Drills Book: usmledrills.com
The USMLE Guys Step 2 CK Crash Course usmleguys.com

D a P a Ga
H popituitarism

Ca e :
● P a ade a , f a e d ea e ( e a a , e c a , ac d ),
ad a , fec , e, a e , S ee a d e, e e c a

A e P a H e Affec ed:
● F c e- a e (FSH), e e (LH), c c (ACTH),
d- a e (TSH), ac , e

Deficienc of ACTH (Secondar Adrenal Insufficienc )

S :
● We , a ea/ , e (e e ), f
a / aa c ( eadac e, a f e d defec )

Sec da Ad e a I ff c e c (C a ed P a Ad e a I ff c e c ) Ha :
● Le f ca e , e a e a, e e a

Lab :
● H a e a

Da :
● H -d e ACTH a e (250 c c )
○ If ba a c a d ACTH , e ec da e a ad e a
ff c e c

T ea e :
● Sec da ad e a ff c e c : G c c c d e ace e ( d c e)

Deficienc of TSH (Central H poth roidism)

S :
● C d e a ce, fa e, de e , e a , ed c , a /c a e
a ,b e a ,c a , ff fac e , e a ed e, a e e , ede a,
/a e ea e a a, dec ea ed b d , e ec e d f c

Lab :
● Se TSH: N a
● Se f ee T4: L - a
● Se T3: L a

Da :
● S f d df c ab a eda ( e
f ee T4 a , e TSH , a ae a)

T ea e
● L fe e e

E d c 65
Get The Step 2 CK Drills Book: usmledrills.com
The USMLE Guys Step 2 CK Crash Course usmleguys.com

Deficienc of Gonadotropins (Secondar H pogonadism)

Sec da ad : Def c e ec e ff c e- a e (FSH) a d


e e (LH).

S :
● Ma e : Dec ea ed b d , dec ea ed ce a , ec a a, fe
● Fe a e : H f a e , e ea/a e ea, fe , a a a

Lab :
● Ma e : LH a d/ FSH a ae a , e e e
● Fe a e : LH a d/ FSH a ae a , e e ad

Da :
● Ma e : Mea e LH a d e e e be ee 8 10 AM. If e e e a d LH
a ae a, e a e a ec da ad
● Fe a e : Pa e e ea a e ea LH a d/ FSH
a ae a a d e e ad OR fe a e e ea
a e ea a d ab e ce f a a b eed af e e e ec a e e e
( ed e e e 10 da f 10 da ), e a e a ec da
ad

T ea e :
● Ma e : Te e e e ace e ea
● Fe a e : I a f be e ad a e(fa cab e) f ed b e e
a d e ea

Deficienc of Growth Hormone (GH)

S :
● C d e : Dec ea ed e
● Ad : Dec ea ed ea b d a , e e a, d de a, e e a ed f
ca d a c a d ea e

Lab /D a :
● S f a / a a c d ea e, a a (FSH, LH, ACTH, TSH,
GH, ac ), a d a - e fac -1 (IGF-1) e e (c a ed a
ca e e f a e a d e )
● If IGF-1 a , a ac e a e ( e ece a ) ca be
ef ed, a d f GH e e e a def c e c f e c f ed

T ea e :
● G e e ace e ea ( ec b a a GH a ac a );
ea e fe f e e ad

Deficienc of Prolactin

S :
● Lac f ac a f de e

E d c 66
Get The Step 2 CK Drills Book: usmledrills.com
The USMLE Guys Step 2 CK Crash Course usmleguys.com

Da :
● L ac e e ( ea ed a a ca a a a e a
e da e a ) a d dec ea ed d c

T ea e :
● N ea e c e a a ab e

Pituitar Apople

He a e e a a d.

S :
● S dde e e e eadac e, d a/ , be a a

I a :
● MRI f e a a d e a e

Da :
● S a d a c e a a e

T ea e :
● S ca dec e f e a a d(f e c )

Sheehan S ndrome

I fa c f e a a df e c c ( a e e e eb d
a f ).

S :
● I ab ac a e ( de e ), a e ea/ e ea, f e a a ,
e , d (a f a )
● S ca cc ed a e ea af e a da a e

Lab :
● W - f a (def c e c e f ACTH, TSH, LH, FSH, IGF-1, ac ) ee
ab e de f e ec f c f d a d d a def c e c e . ACTH a e ed
a d ea ed ed a e d e fe- ea e c ca , e e a f
4-6 ee e a ae

I a :
● MRI a a a a ed e a (e e a ): T a e
e de e ( ee - )

Da :
● H , ab , a d a c e da

T ea e :
● T ea d d a e def c e c ( ee e def c e c e a d e ea e e
ea e a f ec e)

E d c 67
Get The Step 2 CK Drills Book: usmledrills.com
The USMLE Guys Step 2 CK Crash Course usmleguys.com

H perprolactinemia (Prolactinoma)

S /S :
● P e e a a a e :O e ea/a e ea, dec ea ed b e e a de ,
fe , a ac ea
● P e a a a e : Headac e a f e d defec (b e a fed )
● Ma e a e : Dec ea ed e d e, e ce, ec a a, a ac ea

Lab :
● E e a ed ac e e , beee a e a e (IGF-1, ACTH,
LH, FSH, TSH, a d f ee T4)

I a :
● Pe f MRI f e a f e ac e a e e ,a d f a ade a
de f ed e f b c e ca e f e a e

T ea e :
● If a c c ac a (<10 ) a ac ac a ee a :
○ I a ea e d a ea c a cabe e
○ If ea e fa e d a ea ( ac e e e a e e a ed e
e f a ac ade a c ea e ) ceed a e da e

Acromegal - E cessive Growth Hormone (GH)

S /S :
● Headac e, a f e d defec (b e a e a a), e a ed a a d a d fee ,
c e , e ca a , ee a ea, a e a e e , d abe e , ef
e c a e , e e , ca d a , ca a e d e, c

Be I a Te :
● I - e fac -1 (IGF-1) e e , e e a ed IGF-1 e e / f
ac e a c f eda . Pe f MRI f e a
● If IGF-1 e e a, e f a c e e a ce e . If GH ade ae
e ed, e f MRI f e a

I a :
● T MRI c f GH ec e a ade a
● If a ade a ee , e f c e a d abd a CT a d ea e
e- e ea e a d GH e e a e f ec c ca e f ac e a

Da :
● S / f ac e a e e a ed IGF-1 e e OR IGF-1 e e a a d
a c e e a ce e GH ade a e e ed

T ea e :
● Ta e da e f e be a e
● F e ab e de e , ed ca ea a a
a a (d a ea cabe e)

E d c 68
Get The Step 2 CK Drills Book: usmledrills.com
The USMLE Guys Step 2 CK Crash Course usmleguys.com

Cushing s Disease (ACTH-Producing Pituitar Adenoma)

S :
● R ded face, ae, ecc e , be , c e e a ce, d a fa ad, ede a,
e e , e e a,

Be I a Te :
● 24- a f ee c e ce ( e ea f 2 a e )
P ○ ef C d e f a c e ce >3 e e
f a
● OR La e- a a c ( e ea f 2 a e )
○ P ef C d e f e e a ed a e a a a d e c
e e
● OR O e 1 de a e a e e e
○ P ef C d e fc >1.8 c /dL (fa e e
c )

I a Te ( ) P ef C S d e, Ne Be S e Mea e ACTH:
● If ACTH a /e e a ed ( ea e a 20 / ) a e a af f
ACTH-de e de C d ea d a MRI d be e f ed
○ If a a , a <6 ee , e f fe e a
a (IPSS). Ce a e e a IPSS d ca e C d ea e, ac f
ee a d ca e ec c ACTH ec e
○ If a a >6 ee , e f CRH a e a d de a e a e
e e .A ae e a d a d ca e C
d ea e

T ea e :
● Ta e da c ade ec
● I a e ab e de e , fa e , ed ca ea
cabe e

TSH-Producing Pituitar Adenoma

S :
● S f e d a d de a e e ( e)
● V a f e d defec , eadac e
● Fe a e a e ca e e e ce a ac ea a d/ a e ea/ e ea

Lab :
● N a e TSH, e e a ed e a a d f ee T4 a d T3
● E e a ed a a b f c e e

I a :
● Rad ac e d e a e ca a ad d e a e, d ff e
e e de a e e
● MRI f e a ad a ac ade a c ade a

Da :
● S f e d a e d ea e

E d c 69
Get The Step 2 CK Drills Book: usmledrills.com
The USMLE Guys Step 2 CK Crash Course usmleguys.com

● Lab a e TSH, e f ee T4 a d T3, e e a ed a a


b
● Ade a ee MRI f e a ad

T ea e :
● Oc e de e ee d e a e da e ec f e a
ade a

Gonadotroph Adenoma

S :
● Headac e, a f e d def c ; ae fe , e a e a e , e a
d f c

I a :
● MRI f e a e e ce f a ade a

Lab :
● P e a a e : E e a ed FSH a d/ a a b PLUS a e ed LH
● Me : E e a ed FSH c ce a , e e a ed a a b ( ec f c), a
e ec ed e e f FSH a a b TRH

Da :
● S , a ,a db c e ca ab c e ad ade a

T ea e :
● If a e e ca ( a def c / eadac e) f
a f c a d ea a MRI
● If/ e e c de e , e f a e da e

Content Re ie Questions:

A 36- ea - d a e e e e ce a e e e a e a e e 12 f b d.
S ce de e e e e a , a e a, a d a bee ab e ac a e f e2
ce de e . G e e a e , a e e f d MRI f e a
a da 3 f de e ?

A. MRI f e a a c ade a
B. MRI f e a a ac ade a
C. MRI f e a a d e a e
D. S a a a a ed e a (e e a )

W c f ef ab f d d e ec f d a a e Sec da Ad e a
I ff c e c ?

A. H e ae a
B. H e e a
C. H a e a
D. H ec

E d c 70
Get The Step 2 CK Drills Book: usmledrills.com
The USMLE Guys Step 2 CK Crash Course usmleguys.com

Sec da ad ae a c de c ab ab a e ?
A. L e LH, e e e
B. H e LH, e e e
C. L e LH, e e e
D. H e LH, e e e

A 56- ea - d a e a ed ca f d abe e a d e e e e e
c c e ab e a ed a , a d , a d fee a e a c e .T e a e
ca de f e e f ced e e c a e b e a e bee e
ef a ea . W a e be a e e a e e ca e f e e ?

A. I - e fac -1 (IGF-1) e e
B. TSH
C. G e e e
D. ACTH e e

A e e a a a e e ac e a ca e e e ce a b c f ef
?

A. A e ea
B. I c ea ed b e e a de
C. I fe
D. Ga ac ea

A 42- ea - d a e e e e , a a , e , eadac e, a d a fed


defec c d b e a e a a. Lab e TSH, e e a ed e a
a d f ee T4 a d T3, a d e e a ed a a b f c e e . A MRI f e
a ad e e e ce f a ac ade a. W a e e
f d a e a e ad ac e d e a e ca ?

A. Ze ad d e a e a e d
B. L ad d e a e, d ff e e e de a e e
C. H ad d e a e e f ca a ea
D. H ad d e a e, d ff e e e de a e e

E d c 71
Get The Step 2 CK Drills Book: usmledrills.com
The USMLE Guys Step 2 CK Crash Course usmleguys.com

D ab M a C a
Sc ee :
● Pa e e e a ed BMI ( 23 A a A e ca a d 25 a e ) a ea
f e be c d :
○ F -de ee e a e d abe e
○ Da ed ca d a c a d ea e, HTN, c c a d e
○ HDL c e e <35 /dL ce de >250 /dL
○ P ca ac
○ H ace ( - ea dH a c a e )
● P ed abe c (A1C 5.7%-6.4%) ( e ea ea )
● Pa e e a a d abe e d be c ee ed fe e e 3 ea
● Be a a e 45, a a e be c ee , e ea ed e e 3 ea
● Pa e a e c ee ed A1C a d/ fa a a c e (d a ade e
c ec e A1C 6.5% fc ec e fa a a c e e e a e 126)
● 75- a - a c e e a ce e ed e a c a e f
e a a d abe e ( e f fa 92 OR e 180 OR 153)

S :
● P d a, a, c a, b , e

C ca :
● D abe c e a , d abe c e ac d , e a e ce c a e, d abe c
f ce , d abe c e a

Da :
● C ec e A1C 6.5% OR f c ec e fa a a c e e e a e 126 OR
e a c e e a ce e

T ea e :
● L fe e d f ca a d ed ca e a ( a ea a ef ),
a e ' a A1C a e f 7.0%

Med ca :
● L fe e d f ca : Dec ea e A1C 1.0-2.0%, d ff c a a
● Me f : Dec ea e A1C 1.0-2.0%, f fe- ea e ac c ac d (d e
e a e fa e)
● I : Dec ea e A1C 1.5-3.5%, e e da ec , ce c , f
ce a, e a
● S f ea (G de): Dec ea e A1C 1.0-2.0%, e a , f ce a
● GLP-1 ece a (D a de): Dec ea e A1C 0.5-1.5%, e e ec (da
ee ), f e e GI de effec
● G de: Dec ea e A1C 0.5-1.5%, e a , ce a
● T a d ed e (P a e): Dec ea e A1C 0.5-1.4%, f ea fa e, f d
ee , e a , b e f ac e

D abe c Ne a :
● Ne e da a e ca ed b e e b d a e e e ed e d

E d c 72
Get The Step 2 CK Drills Book: usmledrills.com
The USMLE Guys Step 2 CK Crash Course usmleguys.com

Effec :
● Ca d ac: Tac ca d a, a c e , e ca d a fa c
● Pe ea a c e ed f c :S c a e (d , ca ), ede a, a ,
, f /fee a
● GI: Ga ae , a ed e a ea
● GU: E ec e d f c , b adde d f c

T ea e :
● U a e e b e, e e a d/ e ce c c , a
BMI a a e d e a d e e c e, ea HTN a d HLD, f ca e
● Ne a c a ea ed SNRI ( e afa e, d e e), c c c
d (a e), ce a a e e c d ( e aba , aba e )

Diabetic Foot Ulcer Classification

G ad :
● G ade 0: S e f c a ce a e a a c e f e ( de
e ed)
● G ade 1: F - c e ce , e d , ca e, b e ed
● G ade 2: Te d ca a ed ( a ab e b e ed)
● G ade 3: Pa ab e b e ed

Sa :
● Sa e A: N fec c e a
● Sa e B: I fec
● Sa e C: I c e a
● Sa e D: I fec a d c e a

T ea e :
● A a c e e ef e a a c ee f ce
● Mec a ca ff ad ca a e
● S ca deb de e f e ce ( e e ec c e)
● W dde
● A b c ( fec )
● Re a c a a ( e e a a e d ea e a d a a )
● A a

Diabetic Retinopath

Sc ee :
● Pa e da ed DM d a e d a ed f d e a a e e 2-3 ea

S /S :
● P fe a e d abe c e a : Ne a c a a ( e b d e e ) c ca
e ca a c a e a e
● Mac a ede a: Re a c e a d ede a ca ca a ea a e ( e
e da e ee ), a d ead be f a ac

E d c 73
Get The Step 2 CK Drills Book: usmledrills.com
The USMLE Guys Step 2 CK Crash Course usmleguys.com

Diabetic Ketoacidosis

S :
● De e a d (<24 ) a af e ec a e e ( fec a
ea )
● P a, d a, dec ea ed ,d c a, a ea/ , abd a
a , e a , c a, ac ca d a, e , e e a , b ea f
d

Lab :
● H e ce a ( a e e a ed 350-500 /dL, b ca be e ), dec ea ed
b ca b a e, a a e ab c ac d ( a ca >20), e e e e
b de a d e e e , e e a ed a a a (b ca <320 / ),
d a e a e d a e a, ae a, e e a ed BUN a d c ea e,
b e e e a ed a a e a d a e, e de a, e c

H perosmolar H pergl cemic State

S :
● De e ad a ( e e e a da ) a af e ec a e e ( fec , a
ea )
● P a, d a, dec ea ed ,d c a, e a ,c a, f ca
e c def c , e e , ac ca d a, e

Lab :
● G c efe e >1000 /dL b a 600-800 /dL; d dec ea ed
a b ca b a e; a a a e e a ed ( ca >320 / ); -,
- e a e a; e d a e a; a e a; e e a ed BUN a d
c ea e; e de a; e c

Diabetic Ketoacidosis (DKA) vs H perosmolar H pergl cemic State (HHS)

DKA:
● H e ce a ca c e 350-500 /dL
● A a e ab c ac d
● Ke e a
● M e e e e abd a a
● Pa a a <320 /
● B ca b a e <18

HHS:
● N e e a
● M e fe e e e ca (c a)
● N a a a
● Pa a a >320 /
● G c efe e >1000 /dL
● B ca b a e >18

T ea e :
● A e e IVF e ( a .9% NS, e f dc ce ba ed d a d
c e e e ) e ac f d def c e 24

E d c 74
Get The Step 2 CK Drills Book: usmledrills.com
The USMLE Guys Step 2 CK Crash Course usmleguys.com

● P a e ace e IV a c de
● IV , c a ed bc a e af e e ac d e ed a d a e ca
ea
● IV d b ca b ae e a e a H <6.9

Content Re ie Questions:

A 68- ea - d a e a f e 2 d abe e e e f a a a e a .U
e a a f f , ce e e e ce f a ce c f - c e a
a e f , e d , ca e, b e ed. W a ade ce d e a e
a e?

A. G ade 0
B. G ade 1
C. G ade 2
D. G ade 3

W c f ef ab f d e c a ac e c f e a e ce c ae
c a ed d abe c e ac d ?

A. A a e ab c ac d
B. Ke e a
C. B ca b a e <18
D. P a a a >320 /

W c f ef ed ca a a de effec f ac c ac d ?

A. Me f
B. I
C. G de
D. P a e

E d c 75
Get The Step 2 CK Drills Book: usmledrills.com
The USMLE Guys Step 2 CK Crash Course usmleguys.com

C a D ab I ,N D ab I ,a
P a P a
Central Diabetes Insipidus

I a ed e ea e f a d e c e (ADH) f e a a d.

Ca e :
● Id a c( c ), e e , e e c a , e a a c/ f a e
a , c e a

S :
● P a, d a, c a

Lab :
● N a e d , a ec f c a a 1.005 a d a
a <200 O / , a a a a >287 O /

Da :
● See a e de a e

T ea e :
● L d a d e d e a d/ a a a de e

Nephrogenic Diabetes Insipidus

Re a ce ADH ec ec b e f e d e .

Ca e :
● C c ea , e e c a , d e d ea e, e e e ca ce a,
ae a

S :
● P a, d a, c a

Lab :
● N a e d , a ec f c a a 1.005 a d a a
<200 O / , a a a a >287 O /

Da :
● See a e de a e

T ea e :
● L d a d e de
● D b e- d
● T a de d e c ( d c a de)
● A de
● I d e ac

E d c 76
Get The Step 2 CK Drills Book: usmledrills.com
The USMLE Guys Step 2 CK Crash Course usmleguys.com

Primar Pol dipsia

I c ea ed , e a ae a e ae c ( c a c e e ,
aa c e d ce e ).

S :
● P a, d a, c a

Lab :
● H a e a, e a <100 / , a a a - a a e
(a d 275 O / )

Da :
● See a e de a e

T ea e :
● N a a ab e e a ( e fea b e, a e acce ae a e
a aea )

Water Restriction/Deprivation Test

Te ed d ffe e ae a d a, ce a d abe e d ,a d e e c
d abe e d .

Se :
● Re c a e a e a e d f 145 ea e a d a a a f
295 ea e ( e e c a e ADH e ea e)
○ G e e c a e ba e e a f e c a ed e ac e e
e
● Af e a e e c f e a >700, a e a a d a
● If e a 700 a d e d e a 145 ea e a d a a a
e a 295 ea e e de e
○ If e a d be af e de e -ce a DI
○ If e a e 15-45% f e a e ( .e. a a e)
e e c d abe e d

Content Re ie Questions:

Pa e c f ef c d ae e e e a e a?

A. Ce a d abe e d
B. Ne e c d abe e d
C. P a d a
D. De d a

E d c 77
Get The Step 2 CK Drills Book: usmledrills.com
The USMLE Guys Step 2 CK Crash Course usmleguys.com

W c f ef e /a a ca be ed d ffe e a e be ee ce a d abe e
d a d e e c d abe e d ?

A. 24- a f ee c e ce
B. Wa e de a e
C. P a a a d e e c ce a
D. H -d e ACTH a e

A 40- ea - d a e e e ec c ec ed d abe e d .A ae
de a e ef ed. T e a e e c ed f d ae f e ea
e d e ceed 145 a d a a a e ceed 295. Af e ea e
a f e ea , e a a 150. E e de e ad e ed a d
e e a e e e a e ' e a a e a d b ed 450. W a e
e da ?

A. Ce a d abe e d
B. Ne e c d abe e d
C. D abe e e
D. P a d a

E d c 78
Get The Step 2 CK Drills Book: usmledrills.com
The USMLE Guys Step 2 CK Crash Course usmleguys.com

M E N a a
Multiple Endocrine Neoplasia T pe 1 (MEN1)

A a d a e e cd de .

T A c a ed MEN1 (3 P : Pa a d a d ,A e P a ,a d
E e Pa c ea c e d c e ):
● Pa a d ( e) ead e aa d
● P a ade a
● Ca c d
● C a e (a fb a ,c a e a )
● Pancreatic and GI tumors
○ Ga a (Z e -E d e)
○ I a
○ N f c
○ G ca a ( a e)
○ VIP a ( a e)

Carcinoid Tumor Characteristics

S /S :
● C cf , d a ea, abd a a ,b e b c , e a e a , - ded
a a ea d ea e, b c a

Be I a Da c Te :
● 24- a e ce f 5- d d eace c ac d (5-HIAA)
● If e 24- a e ce 5-HIAA, CT MRI f e abd e a d e
de f ce

Gastrinoma-Zollinger-Ellison S ndrome (ZES)

Ga ec e (d de a a c ea c) e e d c e .

S :
● Pe c ce d ea e (PUD), d a ea, e
○ PUD c ca ( e GI b eed/ e f a ), PUD e a ed ca
ea e , ce d a e d de

W -U :
● E e a ed e a e e a e ( ca a) a c H
● If e ea ed e a c ce a a d a c Haee ca , ec e
a e ca d ffe e a e e a e a a d ZES
○ Sec e a e ZES c ea e e a a d b a
a c G ce dec ea e e a e e
● U e e d c e a cf d a d e ce , ce d a
d de
● Af e e e d c , ca a CT MRI a a ece
c a (SRS)

E d c 79
Get The Step 2 CK Drills Book: usmledrills.com
The USMLE Guys Step 2 CK Crash Course usmleguys.com

Insulinoma

S /S :
● Fa a da ce a ( e ,a e , eaded e , ea ,
ac ca d a, c f , fc c e )

W -U :
● S e ed 72 fa ce a e e a ed a a ,C e de e e ,
a d e e c e a a
● T a abd a a a /CT ca e e ( )

Glucagonoma

S /S
● Nec c a e e a, e , e ce a, c cda ea, e
b

W -U :
● E e a ed a a ca e e a d c e e e ,a e a
● L ca e CT MRI e a a - ece c a (SRS)

VIPoma

S /S :
● La e e ae d a ea ( f e d e ), f e de , a ea/ ,
ce ea e / cec a

Lab :
● H c d a, e ce a, e ca ce a, dec ea ed a d B12

W -U :
● E e a ed e a ac e e a e de (VIP) e e
● CT MRI a d f ed b a a ece c a ( f eeded)

Da :
● P e e ce f 2 d f MEN1 (3 P ) OR e e ce f 1 d f MEN1
a e fa e be d a ed MEN1 OR e e c a a a
e e MEN1 a

T ea e :
● S a c e aa d ea ed aa dec
● P a ade a ea ed ed ca a a e e a d/ a e da
ade ec ( ee a ec e f ec f c )
● Z e -E d e ea
● Re ec f a

Multiple Endocrine Neoplasia T pe 2A (MEN2A)

A a d a e e c d ea e a f e RET - c e e.

E d c 80
Get The Step 2 CK Drills Book: usmledrills.com
The USMLE Guys Step 2 CK Crash Course usmleguys.com

T A c a ed MEN2A (3 C Med a d ca ce (Ca c ),


P e c c a (Ca ec a e ), P a e aa d (e e a ed Ca c )):
● Med a d ca ce
● P e c c a
● P a e aa d

Da :
● 1 e f e c a ca c ca fea e f MEN2A e e ca a e
RET a
● 1 e f e c a ca c ca fea e f MEN2A f -de ee e a e
MEN2A d a
● 2 f e c a ca c ca fea e f MEN2A fa e
RET a

T ea e :
● Med a d ca ce ea ed dec (af e e a a f
e c c a)
● S a c e aa d ea ed aa dec
● P e c c a ea ed ad e a ec

Multiple Endocrine Neoplasia T pe 2B (MEN2B)

A a d a e e c d ea e a f e RET - c e e.

C d /T A c a ed MEN2B:
● Med a d ca ce
● P e c c a
● MSK ab a e
○ Ma fa d ab
○ K c
○ L d
● M c a e a
● I e a a e a
● M e a ed c ea e e

Da :
● 1 e f e c a ca c ca fea e f MEN2B e e ca a e
RET a
● 1 e f e c a ca c ca fea e f MEN2B f -de ee e a e
MEN2B d a
● Ma f e c a ca c ca fea e f MEN2B fa e
RET a

T ea e :
Med a d ca ce ea ed dec (af e e a a f
e c c a)
P e c c a ea ed ad e a ec

E d c 81
Get The Step 2 CK Drills Book: usmledrills.com
The USMLE Guys Step 2 CK Crash Course usmleguys.com

Content Re ie Questions:

W c f ef f d c e Z e -E d e (ZES)?

A. Dec ea e e a e e f ec e a e
B. U ce d a e d de e e d c
C. P e a cf d e e d c
D. E e a ed e a e e

Ca cf d f ee d c e e a a e 2B c de a b c f ef
d de ?

A. Med a d ca ce
B. P e c c a
C. P a e aa d
D. M e a ed c ea e e

A 50- ea - d a e e e ec c c a fc cf , d a ea,
abd a a . A 24- a e ce f 5- d d eace c ac d (5-HIAA)
ef ed c f d be e e a ed. Ba ed e e a d e a
f d , c f ef d f e e ca e f e a e .

A. Ca c d
B. VIP a
C. G ca a
D. I a

E d c 82
Get The Step 2 CK Drills Book: usmledrills.com
The USMLE Guys Step 2 CK Crash Course usmleguys.com

P a O a a I ,P O a a S (PCOS),
a P a H a Ma
Primar Ovarian Insufficienc

S :
● O e ea/a e ea, e e def c e c ( a a d e , fa e ,
dec ea ed b e de ) a e de 40

Lab :
● FSH e e a ed ( e e a a a e e e )a d e e ad

Da :
● A e de 40 e e a ed FSH a d e ea/a e ea

T ea e :
● P e a de ad ea a e 50

Pol c stic Ovarian S ndrome (PCOS)

S :
● O e ea/a e ea, e a , ea d e ( , ac e)

Lab :
● E e a ed ea e e (LH) e e a d LH efe e c , a
f c e- a e (FSH) e e , e e a ed LH/FSH a , e e a ed e
a d e e e , d e e a ed ac e e

I a (U a d):
● 12 f c e eac a , 2-9 d a e e , a d/ c ea ed a a e >10 L

C ca :
● I e a ce/ e 2 d abe e , d de a, ee a ea, ac c fa e
d ea e, de e ,a e

Da (R e da C e a):
● Re e 2 e f ef :
○ O e ea/a e ea
○ C ca a d/ b c e ca f ea d e
○ P c c a e a d ( ee a e ad ec f c )

T ea e :
● We - e e / be e a e
● C b ed e e - e a c ace e

Primar H pogonadism in Males

S :
● Fa e a e be ( ac f b d a , dee e ce), a e e , dec ea ed
e d e, fe , f e a a , dec ea ed c e a , dec ea ed b e ea
de , ec a a

E d c 83
Get The Step 2 CK Drills Book: usmledrills.com
The USMLE Guys Step 2 CK Crash Course usmleguys.com

Lab :
● L e e e a d/ e c , e e a ed e e (LH) a d/
f c e- a e (FSH)

Da :
● S f ad e aae e e e e e e ea ed
be ee 8-10 AM. E e a ed LH a d FSH e e c f a ad

T ea e :
● Te e e e ace e ea

Content Re ie Questions:

A 31- ea - d fe a e e e ec c c a f e ea, e a ,
ac e, a d . Lab e e a ed ea e e e e ,
f c e- a e e e , a d e e a ed e a d e e e .W c f ef
ae ea e ee e a ?

A. 18 f c e eac a
B. F c e ed be ee 2-9 da ee
C. O a a e 18 L
D. S e 4c c

L e e e, e e a ed e e, a d e e a ed f c e- a eae
c e c f ef da e ?

A. P a ad
B. Sec da ad
C. Be a c e
D. E e e e e e

P c c a a d e ca c a ac e ed b c f ef b c e ca
f d ?

A. Dec ea ed LH/FSH a
B. N a LH/FSH a
C. E e a ed LH/FSH a
D. U de ec ab e LH a d FSH

E d c 84
Get The Step 2 CK Drills Book: usmledrills.com
The USMLE Guys Step 2 CK Crash Course usmleguys.com

Ga oen e olog
D a aa E a a M D
D a a: O a ea E a ea D a a
Orophar ngeal D sphagia Esophageal D sphagia

D a a cc a a D a a cc ec d af e
a
Pa e a e ce ca e a Pa e a e a c
e ca f a ca be d e e a e
ca f
O e c a :
A a
C /c e ea
Na a ea e a
Se a f e d a f d
e a e a
D

Ne Be S e Ide f Ca e f O a ea D a a:
● Ne c a d ea e: V de f c a de a ea a e
● Pa e e c : F be ce d c ce a a f a
THEN de f c a de a ea a e (f f d e d c )
● I a e a e de e f be ce d c ce a a f a ,
de f c a de a ea a e , de f a ca e f
a ea d a a, e e e ef a a a ea a c

Ne Be S e Ide f Ca e fE a ea D a a:
● H f c e c e, ad a , ca ce e e f
e a ea ca ce : F Barium s allo THEN e e d c
● N e f e ab e ed ed ca :I a da c e da upper
endoscop
● If c a ab a e af e e e d c , e :
○ If d a a d a e ef ba a (f a ead ef ed)
○ If d a a d a d d ef e a ea a e

Ac a a a:
● Lac f e a ed a - d f ee a a d e e a ea
ce d f c ( c ee ea a )
● S :
○ D a a f d a d d , e a f d e ed f d, ea b ,
c e a , a d/ d ff c be c

Ga e e 85
Get The Step 2 CK Drills Book: usmledrills.com
The USMLE Guys Step 2 CK Crash Course usmleguys.com

Ac a a a D a :
● Made a esophageal manometr ( f e ae c c e e ba a
ef ed)
○ If ba a ef ed c e ac a a a c de d a ed
e a e d a beak-like narro ing, a e a d a - d f
ee a , de a ed e a ea e
● F e a ea a e , e e d c ef ed ca e f ac a a a
e ee a a c a a c ed

D de fH ec ac e E a ea M :
● T a ec ac e e a ea d de : D a e a ea a a d
ec ac e ( ac a e)e a
● S :
○ D a a, -ca d ac c e a , ea b , a d/ e a
● E a ea a e f d :
○ D a e a ea a :Pe a ec ac ed a e a
○ H e c ac e ( ac a e)e a :H e ec ac ( a
e e ce) ee a
● Da :
○ E c f e c d
○ I a e c de a e a e a e d c
○ Da e ade e a ea a e
● T ea e :
○ Pe e ( a e bef e ea )
○ If e e fa , e ca c c a e b c e (d a e )
○ If ca c c a e b c e fa , e c c c a de e a ( a e)

Content Re ie Questions:

A 60- ea - d fe a e e e ec c c a fd a a. T e a e e
d ff c a a a d e ce ca e e a ed ee e ce
e .S e ae e a ced ec e ea b e .
W c f ef e be ec f c d a c e da de f e ca e f
d a a?

A. E a ea a e
B. V de f c
C. F be ce d c ce a a f a
D. S ca

A 44- ea - d fe a e e e ec c a 2- fd a a f d a d
d , e a f d e ed f d, a d ea b . If ec ac a a a a e ca e
f a e d a a, a ee a ba a e ?

A. D a ed e a e d a bea - e a
B. H e ec ac ee a
C. C ee ea a f e e e a ea ce
D. S c e e a 1/3 f e e a

Ga e e 86
Get The Step 2 CK Drills Book: usmledrills.com
The USMLE Guys Step 2 CK Crash Course usmleguys.com

W c f ef ed ca d be a a ae de f ea e a a e e
f a a e c f ed ec ac e ( ac a e)e a ?

A. Pe e bef e ea
B. D a e
C. I a e
D. B

Ga e e 87
Get The Step 2 CK Drills Book: usmledrills.com
The USMLE Guys Step 2 CK Crash Course usmleguys.com

Ga a a R D a (GERD) a E a
Gastroesophageal Reflu Disease (GERD)

S :
● Hea b ( a d a ), e a ,d a a, c e a , a d/ c cc ,
a e e ee

Da :
● If a e a eca c c a ea b a d/ e a ,c ca
da ca be ade f e -
● If a a ae e e , e e d c a a ed a e eda

Aa Fea e :
● Ne e fd e a a e 60 ea
● H f GI ca ce f -de ee e a e
● L f a e e/ e a ed e
● P e e ce f b d e GI ac ( e a e e , e e a, e a c e a, cc b d
) def c e c a e a
● D a a
● Od a a
● Pe e

T ea e :
● M d/ e e GERD ( e a ce a ee ):
○ L fe e d f ca
■ We (f e e )
■ Ee a f e ead f e bed
■ A da ce f d e a e
○ A ac d
○ H a e 2 ece a a (fa d e)
○ S c a fa e
● Se e e/f e e GERD ( ce a ee e):
○ P b ( e a e)

GERD C ca :
● E ee a
● Ba e ' e a
● Pe c c e
● La a ea ef a d/ a ea a d ac ea e
● Ref - d ced a a

Ba e ' E a :
● Occ e c c GERD ead ac a e f ed a e a f a f ed
a e e ea a cc a e e
● I c ea ed f de e e a ea ade ca c a
● A a e d ece e b (PPI) ea e (e e fa a c)

Ga e e 88
Get The Step 2 CK Drills Book: usmledrills.com
The USMLE Guys Step 2 CK Crash Course usmleguys.com

Medication-Induced Esophagitis

Med C Re b e f Med ca -I d ced E a :


● NSAID , a b c ( e ac c e), a c de, ,b ae

R Fac :
● Ab a e a ea a a , a e ( de ), e ( a e), a e (
d e a / )a da ff d e ed e ed ca (a
ed ca e/ ae)

S :
● U a dde e f d a a, d a a, e e a a
ca e e 1 f e f ed ca

Da :
● H f a ed ca e e a fd a a,
ea b , a d/ d a a
● U e e d c e e ed f e ee e a e e e
af e 1 ee f d c ca a e ed ca

T ea e :
● Ce a f ca a e ed ca (f b e) c f df

Eosinophilic Esophagitis

C c fa a f ee a e ce ( ed ae e )
ca e a ea d f c .

C d e/a e ed a ed a e a ac c e a e cd de
(a a, a e c ,f d a e e ).

S :
● D a a, f d ac , e abd a a , GERD

W -U a d D a :
● C ca a d c e e ce a f
e a e cd de ( ) a d/ fa fe ce a a a e
e d c b e
● Da def e e a e a c ca fe ce a ,
e - ed a e a ea b ,a de c f e c d c a
ca e e a ea e a

T ea e :
● A e e a d a da ce f de f ed f d a d e e a a e e
● P b ca c c c d
● E a ea d a f a e de e e a ea c e ( f
ea / e, a ed ca e a fa e bef e a e )

Ga e e 89
Get The Step 2 CK Drills Book: usmledrills.com
The USMLE Guys Step 2 CK Crash Course usmleguys.com

Content Re ie Questions:

A 66- ea - d a e e e e d e a, d a a, d a a, a d e e a f
e a 2 ee . O CBC e a e H b 9.8. W c f e f da c e
da e a ae de f e ca e f a e ?

A. Ba a
B. E a ea a e
C. U e e d c
D. C c

W c f ef ed ca e ca e ed ca d ced e a ?

A. A
B. Fa d e
C. O e a e
D. Ta

A 20- ea - d a e a a ed ca fa e c a d ef da e e
e e ec c c a fd a a, f d ac ,a d e abd a a
a a bee bee e e e a 3 ee . He e f a e e d c
b . A ed a ce f c ce e c e a e e
da ?

A. Ba
B. Ne
C. E
D. La e a ce

Ga e e 90
Get The Step 2 CK Drills Book: usmledrills.com
The USMLE Guys Step 2 CK Crash Course usmleguys.com

Ca E a a I ,E a a R ,a E a a
Ca
Caustic Esophageal Injur

U a e e a e f a a c a da c ea e c ea d c.

S :
● H e a a ,d a a/ d a a, a ca ed e a ea , e e a
e a c e , e a d e
● Re e a bac a :P bee a ea e f a ed a
● S ca abd e ( d , eb d): E a ea a c ef a ca
e

U eP ca E a F d :
● O a ede a, e , ec a e d e ba e
● Ce f e ec e a f bc a e e e a e e

Lab :
● If da a e a a , ab ca be ea a
● If da a e a a e e e a ed WBC, CRP, ac a e, e a d d e
ab a e ,a d b c e a a be ee

I a :
● CXR: e f ed a e e a
● A f ef a d ca e e a ea e f a :
○ Med a a de e ed a
○ S bc a e e e a e ec
○ P e a eff
○ H d e a

CT: Pe f ed ALL Pa e W e S ab e
● E a ea a b
● Pe e a ea fa b

T ea e :
● S a c a e d be a ed f e ca e a d .
C a d ca ed ee e c NG c e ed ae a ( a e e e
a d ead ef a )
● T e / f ef a CT a a ec d
de e e e c e (e a a ec )

C ca :
● E a ea c e ,f af a ,e a ea a ce ca c a

Effort Rupture of the Esophagus (Boerhaave S ndrome)

U a ca ed b a ; cca a e f ,c d-b , e e.

R e e cc a ef e a e a a ec f ed a e a ead
e e f a cc e e ed a a ca .

Ga e e 91
Get The Step 2 CK Drills Book: usmledrills.com
The USMLE Guys Step 2 CK Crash Course usmleguys.com

S :
● Re e a c e a ,d ea, d a a, fe e , ac ea, ac ca d a,
e , a d/ e

P ca E a :
● Ce f e ec

I a :
● CXR:
Med a
○ a de e ed a
S bc a e
○ e e a e ec
P e a eff

H d
○ e a
● C a e a a :U e ae- bec a a af f ,a d f e a e e
e ba
○ A a a f ca a da ae e f ef a ( ea f
c a ef a b e)
● CT:
○ E a ea a ede a
○ Pe -e a ea f d ( a c a b bb e )
○ Med a a de
○ A a df d e e a ace

Da :
● C ca c a e a a CT e de ce f e a ea e

T ea e :
● NPO ( b )
● TPN ( a a e ea )
● B ad ec a b c
● P b
● Da a e ff d c ec
● Deb de e f ec c e
● S ca e e ca e-b -ca e ba

Esophageal Cancer

S a Ce Ca c a:
● R fac :S ,ac e, HPV fec ,c d /
● L ca : d-e a

Ade ca c a:
● R fac : Ba e ' e a , GERD, , BMI
● L ca :M e a a c c

S :
● D a a, e

Da :
● U e e d c b b fa ea a c ec f e a ea
ca ce

Ga e e 92
Get The Step 2 CK Drills Book: usmledrills.com
The USMLE Guys Step 2 CK Crash Course usmleguys.com

Content Re ie Questions:

W c f ef da e e a aef c ce de f fa
e a ea e f a e e a e d a ca ab e a e ?

A. C a e a a a af
B. C a e a a ba
C. U e e d c
D. E d c c a d (EUS)

W c f ef c e ad a c e e a ea ef a ?

A. Med a a de
B. P e ed a
C. Ab a c e f ea -e a ea ece
D. S bc a e e e a e ec

W c f ef da c e / e e d ff ce a e eda f
a ce ca c a f ee a ?

A. CT ca de f ca ed d-e a
B. U e e d c b c e a ce
ca c a
C. P e feca cc b d e
D. MRI de f ca ed a ee a a c c

Ga e e 93
Get The Step 2 CK Drills Book: usmledrills.com
The USMLE Guys Step 2 CK Crash Course usmleguys.com

U a L GI B
Upper GI Bleeding

C U e GI B eed Ca e :
● Ma -We ea , e a ea a ce , e c ce d ea e, a e e
af a , a c a -e e c f a,

U e GI B eed Pe e a :
● He a e e (b ed): Ac e de a e- e e e b eed ab e a e fTe
● He a e e (c ffee d): S a e e/ ed b eed ab e a e fTe
● Me e a: L e e GI f b eed, ca be a a e e
● He a c e a: U a f e GI b eed, be massi e e GI b eed

Med ca c P ed e U e GI B eed:
● NSAID , a aee ,a c a a , SSRI , CCB , a d a d e ea a

S :
● S f e a( e f ac ca d a a c e e
e a d e a c c )

Lab :
● CBC, c a (PT/PTT, INR), c e e ,ab , e e e
● F e d a ca ab e ea dc , e d a ca ab e f
e e e b eed e a d c ee
● Ma a e 24 f e b a e dec ea e f b eed
● Ac e e GI b eed e e a ed b d ea e (BUN)- -c ea e > 36:1

Ma a e e :
● Kee a e NPO
● S e e a e
● E ab a e b e IV a a e-b e, e- e ce a c d
● I ed a e ea e f e a d, b f cc a d
● G e IV b
● If ec ed e a ea a ce b eed e c e de a d cef a e

U e GI B eed Ma a e e : PRBC T a f Pa a e e
● Se e e, b eed : T a f e b d d c 1:1:1 a f ac ed ed b d
ce (PRBC ), f e f e a a (FFP), a d a e e
● He d a c ab f a c a db (e ): T a f e 2-4 PRBC
● He d a ca ab e a d e b <9 /dL a e a - f ad e e
ca d a c a c e (CAD): T a f e 2-4 PRBC
● He d a ca ab e a d e b <8 /dL a e - f ad e e
ca d a c a c e :Ta f e2 PRBC

U e GI B eed Ma a e e : P a a a d P a e e T a f Pa a e e
● G efe f e a a e :
○ Ab a c a a ab f a f
○ Af e f f PRBC a f ( e a de f ab )
● G e aee e :
○ CBC a e e <50,000

Ga e e 94
Get The Step 2 CK Drills Book: usmledrills.com
The USMLE Guys Step 2 CK Crash Course usmleguys.com

○ If a e e a aee e a (de e d a e )
○ Af e a f f f PRBC

U e GI B eed Ma a e e : E d c
● If e d a ca ab e, f ab e
● W e e d a ca ab e, e f e e d c a d de f ca e f e GI
b eed a d a e d ea e ec f c ea e
● If ca e f e GI b eed a a e e e d c , ef c c

Lower GI Bleeding

L e GI B eed : Ca e
● I fa a b e d ea e, ce , d e c ,a d a a, c e cc ,
fec c ,c ca ce

L e GI B eed Pe e a :
● He a c e a (f e b d e a ): T ca e e a f e GI b eed, a ee
a e e GI b eed
● Me e a (da a ): U a f e GI b eed, a e - ded c c
b eed ( e GI b eed)

Med ca c P ed e L e GI B eed:
NSAID , a aee ,a c a a

S :
● S f e a( e f ac ca d a a c e e
e a d e a c c )

Lab :
● CBC, c a (PT/PTT, INR), c e e ,ab , e e e
● F e d a ca ab e ea dc , e d a ca ab e f
e e e b eed e a d c ee
● Ma a e 24 f e b a e dec ea e f b eed

L e GI B eed Ma a e e :
● Kee a e NPO
● S e e a e
● E ab a eb e IV a a e-b e, e- e ce a c d
● I ed a e ea e f e a d, b f cc a d
● S ec f c e d f a f e ab ed f e GI b eed eed f
a f d d a ed ba ed ab a d c bdc d

He a c e a Ma a e e : He d a ca U ab e Pa e
● If e d a ca ab e, f ab e, e e f e e d c e
e d a ca ab e
● If ce f d e e d c a d b eed c e , e ef
a a ca e ce f b eed
● If ce f d a a OR a e ac e b eed af e e
e d c a da ce a f d NEXT e c c

Ga e e 95
Get The Step 2 CK Drills Book: usmledrills.com
The USMLE Guys Step 2 CK Crash Course usmleguys.com

He a c e a Ma a e e : He d a ca S ab e Pa e
● If a a e e a c e a e d a ca ab e, e be e e
c c
● If ce de f ed a d b eed e e ec e a e def c e c a e a,
ef e e d c
● If ce de f ed c c b b eed a a ed a d e f- ,
e ec a a a e e a ae

Minimal Bright Red Blood Per Rectum

Ca e :
● He d , ,a a f e , ec a ce , c ,c ec a ca ce

E a :
● E e a ec f ea
● D a ec a e a a
● A c c c

C c f M a B Red B d Pe Rec :
● Aa a a c c e a de f a e:
○ S fc ec a ca ce (c a e ca be c e c f ,
e a ed e )
○ Feca cc b d e
○ C c e f e GI b eed ( e e a, a c e )
○ Fa f fa a e ed a c ca ce

C c Rec e da b A ef M a B Red B d Pe Rec :


● A e de 40: If e ce de f ed ba ed , f e - eeded
● A e 40-49: Pe f d c c c
● A e 50 ea : Pe f c c
○ If c ee c c ef ed 2 ea f ca ce a d
ade a e a a fc , eed e ea

Content Re ie Questions:

A 64- ea - d a e a e e e ae e e e ee e e c .H
ea a e 124 a d b d e e 82/66. W c f ef e e be e
a a e e ?

A. T a a a e a c e c (TIPS)
B. U e e d c
C. A a
D. P ace e f a e b e IV

Ga e e 96
Get The Step 2 CK Drills Book: usmledrills.com
The USMLE Guys Step 2 CK Crash Course usmleguys.com

A 59- ea - d a e a ed ca e e e ED 4 f
e a c e a. He e d a ca ab e a d e e e a e a a a
ca a e a e b e IVS a e aced. A ea d c ee e a d ab e
a .W a e e be e de f e ce f e GI b eed?

A. A a
B. U e e d c
C. C c
D. La a

A 24- ea - d a e a ed ca c e ec ca d e a 2-da
f a b ed b d e ec . He de e a e .W a e
a ae e e a a e e ?

A. D a ec a e a a d a c
B. Abd a ad a
C. S d c
D. C c

Ga e e 97
Get The Step 2 CK Drills Book: usmledrills.com
The USMLE Guys Step 2 CK Crash Course usmleguys.com

Ma -W T a a E a a Va
Mallor -Weiss Tear

L d a c a ea f ed a e a a d a ac .

P ec a Fac :
● V ( ea a c e ead fe e ), c ,b abd a
a a, a ( e f )

S :
● Fa ed b d c ffee d e ae e ( fe eceded b b d e e
ec ), e a c a , a d f e e e b eed e a c c

Lab :
● CBC, c a (PT/PTT, INR), c e e ,ab , e e e ( a e ab de ed f
ca e f e GI b eed)

U e E d c :
● S e d a ea e c a( e ea ) ac e b eed

T ea e :
● S ab e a e f e d a c ab :
○ Ga acce a e b e IV a a e-b e, e- e ce a
c d
○ S ab e a e b f cc a da d a f eb d d c f
eeded
○ Kee a e NPO a d e e e a e
○ G e IV b
● E d c c e e f ac e b eed :
○ T e a c a a
○ He c
○ E d c c ba d a

Esophageal Varices

E a ed e ee a e bef dec e e a e .

Sc ee :
● E d c c c ee f e a ea a ce a e c

P a :
● S a a d ed a ce : N - e ec e be a-b c e ( ad )
● La e a ce : E a ea a cea a (EVL)

E a ea Va ce Ma a e e :
● Kee a e NPO
● S e e a e
● E ab a e b e IV a a e-b e, e- e ce a c d
● I ed a e ea e f e b f cc a da db d d c
a f a eeded

Ga e e 98
Get The Step 2 CK Drills Book: usmledrills.com
The USMLE Guys Step 2 CK Crash Course usmleguys.com

● G e IV b
● G e a ac e a e ( e e c e de a e a a )
● G ea b c a (f e 3d e ea ce a )
● U e e d c e d c c a cea a (EVL) e f ed f a cea ce
de f ed
● Ta a a e a c e c (TIPS) f c ed b eed af e EVL f
f eb eed af e EVL

Content Re ie Questions:

A 55- ea - d fe a e PMH fac ab e e e e a a e e


f e a 3 , e b ed e a e e e a 30 e .T e a e
e d a ca ab e f ac a df db a d e e d c ef ed
eea e d a ea e c a de f ed. W a e e da ?

A. Pe c ce d ea e
B. Ma -We ea
C. E a ea a ce
D. E a

Ac c a e a a c ee e d c ef ed ee ed - ed e a ea
a ce a e de f ed. W c f e f ed ca d be a a e ea e f
e a ea a ce a ?

A. Nad
B. A d e
C. E ed e
D. P e a e

I a a e c f ed ac e a cea e a e a de ee d c c a cea
a c ed b eed , a e e be e e ?

A. T a a e a c e e b a (TACE)
B. Pe c a e a e a c b a d a a e (PTBD)
C. Pe c a e a e a cc a a (PTC)
D. T a a a e a c e c (TIPS)

Ga e e 99
Get The Step 2 CK Drills Book: usmledrills.com
The USMLE Guys Step 2 CK Crash Course usmleguys.com

I ab B S a I a a B D a (U a
C a C D a )
Irritable Bowel S ndrome

S :
● C c abd a a (c a ,fe e e a e a ed defeca )
● C a , d a ea, b c a a d d a ea (a e a )
● Of e abd a b a ,fa e ce, a d/ be c

I ab e B e S d e W -U :
● P ca E a :
○ T ca a ( be d e de e a a f abd e )
● Lab :
○ CBC f a ec ed f IBS a d feca ca ec /feca ac fe , a da
a e , e c e f ce ac d ea e f e d a ea

I ab e B e S d eAa Fea e :
● O e e 50 ea
● Ab a ab
● GI b eed
● N c a d a ea
● P e e e abd a a
● U e a e
● Fa f GI d de (c ca ce , IBD)

Da :
● R e IV c e a ed. Pa e a e2 e f ef a ae
a c a ed 1 e ee e de f abd a a e a 3 :
○ Pa e a ed defeca (e e e )
○ C a e fe e c
○ C a e f /a ea a ce

Ulcerative Colitis (UC)

S :
● B d da ea, c c d ff e abd a a , e e , fe e , e

E a e a e e :
● MSK: A f a e ,a d
● E e :U e ,e ce
● B a :P a ce c a
● S :E e a d , de a a e

Lab :
● E e a ed feca ca ec / ac fe
● E e a ed e a a e aa e( a ce c a )
● A e a
● E e a ed ESR
● H ab e a

Ga e e 100
Get The Step 2 CK Drills Book: usmledrills.com
The USMLE Guys Step 2 CK Crash Course usmleguys.com

I a :( eeded a eda f UC)


● Ba e e a: S e ed c , f a a, a ed e f ec
● CT/MRI/U a d: N ec f c c e f eb e a
● Abd a Rad a : "T b " d e ede a

C c :
● M c a fa a a e rectum a d a a
constant/unbroken ( a c a) c c fe e a a e eac a e
de a ca ed e ee ed a c a be

B :
● C ab ce e ,c ba c ,c a , c a a d b c a fa a

Da :
● Da ea >4 ee AND ac e fa a e d c AND b f d
c e UC

T ea e :
● M d de a e d ea e:
○ F e e aa e( , a)
○ Sec d e c c c d (e , , e e a, a ), a d/ a
5-a a c c ac d (5-ASA) a e
● Se e e d ea e:
○ H a a e ca e a d IV c c c d
○ If e e IV c c c d , add e e f ab c c e
○ If e e ed ca a a e e , c ec e d e

C ca :
● F a c
● T c e ac
● He a c c
● C c ef a

To ic Megacolon

N b c ec cd aa e c c .

Ca e :
● I fa a b e d ea e (UC a d C d ea e) fec c (C. ,
, , E. )

S /S :
● Abd a a , fe e , ac ca d a, e , abd a d e ,da ea

Lab :
● E e a ed ESR a d CRP, e e a ed WBC , ab e a, a e ec e
ab a e

I a :
● Abd e c CT a a d IV c a ( ec a c a c a d ca ed):
Me ac da ed e a d a e e >6 c

Ga e e 101
Get The Step 2 CK Drills Book: usmledrills.com
The USMLE Guys Step 2 CK Crash Course usmleguys.com

● Abd a ad a ( ed e ): P e e ce f e a -f d
e e ,c c d a e e >6 c , c a a a

C c a d Ba E e a:
● A dd e f ef a f ec

Da :
● I a de a e ac a ea 3 f e f :
Fe e , HR >120 bea /
○ , WBC >10,500/ c L, a e a
● A d a ea 1 f e f :
○ H e , de d a , a e ed e e fc c e , ab a
e ec e

T ea e :
● S e ca e (f d , b ad- ec a b c ,c eeb e e )
● If IBD ca e:
○ IV c c c da d f ab
○ S e (c ec a d e ) f a e fa e d ed ca ea
● If fec ca e:
○ A b c ea
○ S e (c ec d e e c c a a e) f a e fa
e d ed ca e a

Crohn s Disease

Ta a ( c aa d b c a) e e f d ea e a eea e GI
ac (f a ).

S :
● Ca abd a a ,b d -b d da ea, b e a a ab , e
ec f c ( a e f a ce , d a a, d a a,
de e d e f e e )

E a e a I e e :
● MSK: A f ce a ( ac ,a d )
● E e :E ce , e ,
● B a :P a ce c a
● S :E e a d , de a a e

Lab :
● E e a ed feca ca ec / ac fe
● E e a ed WBC
● A e a( def c e c fc c d ea e)
● E e a ed CRP
● E ec e ab a e
● V a B12/ a D def c e c ( a ab )

I a :
● MRI a b e e e a (MRE) a da f c ce
● F d c de b e e e , c ea ed b e a e a ce e a d c e ,
a d c a e a c a ac e c f a b e fa a

Ga e e 102
Get The Step 2 CK Drills Book: usmledrills.com
The USMLE Guys Step 2 CK Crash Course usmleguys.com

● C ca c a f a a d/ c e a be e e

C c :
● S a ea a b e be ee a ea f d ea e (d ea e eed be
c )
● C bb e e a ea a ce f e a e ( a c a e d a
c a c a e a d ce a )
● Fe e ec a c a b e ab a ec a a

B :
● P e e ce f a a eda

Da :
● S c e C d ea e e e a ,e d c cf d ,
ca f d a a fa a a e e a a e

T ea e :
● L a d dC d ea e f e a d/ - ded c : B de de
● L a d dC d ea e f ef - ded a d ef - ded c c d ea e:
P ed e
● M de a e/ e e e C d ea e f a: U e ab ed ab
● M de a e/ e e e C d ea e f a: A -TNF a e (ada ab)
d a (a a e)

Content Re ie Questions:

W c f ef ca f d c e eda f ce a e
c ?

A. P e e ce f a a e b c a
B. P e e ce f c ab ce e
C. V a f e d de a c a
D. Nec eb a d e e ce f b

W c f ef da c da e d be e f ed a e e
a ec ed c e ac da ?

A. C c b
B. Abd e c CT a a d IV c a
C. Ba e e a
D. Ga af e e a

Ga e e 103
Get The Step 2 CK Drills Book: usmledrills.com
The USMLE Guys Step 2 CK Crash Course usmleguys.com

A 24- ea - d a e e 1 f abd a a a dc c d a ea a b d .
Lab a e c e def c e c a e a a e a a e e a ed feca ca ec . T e
a e de e a c c c a c bb e e a ea a ce f e e a e
a da a ec d ea e. G e e ef d , a ca ab a a
be ee c c e e e da e e e a ?

A. N -ca ea a a
B. C ab ce e
C. V a f e d de a c a
D. La e a d a c e fe ace a c

Ga e e 104
Get The Step 2 CK Drills Book: usmledrills.com
The USMLE Guys Step 2 CK Crash Course usmleguys.com

Ma ab S
Carboh drate Malabsorption

S :
● Wa e da ea, f a e ce

Ca e :
● Ce ac d ea e (da a e face f e e e )
● P d c / ec e defec a c ea c a a e
● Dec ea ed d acc a da e e a e e

Lab :
● E e a ed b ea d e

Protein Malabsorption

S :
● M cea , e e a ed ede a, a e ea

Ca e :
● Dec ea ed e a ab e face a ea
● Pa c ea c ff c e c
● Dec ea ed/d f c a a c ea c b ca b aea d ea e

Lab :
● H e e a, ab e a

Fat Malabsorption

S :
● S ea ea, a e e ,da ea fa e ce, a e

Ca e :
● D ed b e ac d a a
○ Dec ea ed e c
○ Dec ea ed ec e c e a
○ Bac e a dec a f b e ac d a e a bac e a e
○ D ed e e e a c c c a fb e a a b e d ea e a
b e e ec
● Pa c ea c ff c e c
○ I ff c e a e, c a e, a d b ca b a e d e c c a c ea
a c ea ec

Lab :
● F ac a fa e c e >7%

Vitamin and Mineral Malabsorption

S /S :
● A e a: I ( c c c), V B12 ( ac c c), f a e ( ac c c)

Ga e e 105
Get The Step 2 CK Drills Book: usmledrills.com
The USMLE Guys Step 2 CK Crash Course usmleguys.com

● Pa e e a, f ba a d e e: V B12
● S f b eed ( f , e a a): V K
● Te a , C e ,a dT ea ; a e e a: Ca c a d a D
● G :B a ,
● N eb d e ,f c a e ea :V A

Ca e :
● F fa be a (A, D, E, K) b ed b a e ca e f fa a ab
● I ea d ea e/ e ec ead dec ea ed B12 ab

Lab :
● I :L e b , e a c , fe , e , a a , ea ce a
e (MCV), a d a -b d ca ac
● V B12: H ea ce a e (MCV), c e e, a d e a c ac d
(MMA); V B12 e e
● F a e: H ea ce a e (MCV) a d c e e; e a d RBC f a e
● V K: Dec ea ed K de e de fac (II, VII, IX, X, e C a d S), dec ea ed
e K
● V A: Dec ea ed e e
● V D/ca c :L e ca c , c ea ed a a e aa e

Small Intestine Bacterial Overgrowth

Bac e a ca f d c bec e e a e ed e d de ,
ed de , a a ca defec .

S :
● B a , ae da ea, f a e ce, abd a a , e

Lab :
● L B12 e e , e e a ed feca fa e e

Da :
● P e ca b d a e b ea e
● Je a a aec e >103 c f (CFU / L)

T ea e :
● A b c ( fa )
● If def c e c e e , e e e (V B12, fa be a )

E ocrine Pancreatic Insufficienc

C c a c ea , a c ea c e ec , a c ea c d c b c a e c e
f c .

S :
● S ea ea (c c), e ,b a , abd a a (c a , - a d a ),
f a e ce, f def c e fa be a (A, D, E, K) a d/ a B12

I a (U/S, CT, MRI):


● Pa c ea c ca c f ca

Ga e e 106
Get The Step 2 CK Drills Book: usmledrills.com
The USMLE Guys Step 2 CK Crash Course usmleguys.com

● Pa c ea c d c d a a
● Pa c ea c a

Da :
● C ca d a f e c c a c ea a d fe c e
a c ea c ff c e c
● If ca e c ea b a feca e a a e-1 ( d be <200 c / e c e
a c ea c ff c e c

T ea e :
● E e a c ea c e e

Lactose Intolerance

Lac a e def c e c e GI d ba ce f ac e e d e ab b ea
d ac e.

S :
● B a ,fa e ce, d a ea, abd a a (c a ), a ea

Lab :
● S c a >125 O /
● S H <6

Da :
● P e ac e d e b ea e

T ea e :
● Re c f ac e c a d c
● Lac a e e e

Content Re ie Questions:

A 36- ea - d fe a e a e a ed ca fC d ea e e I ea
e ec e e e c ca c a f fa e, a e e a f b a e a e , a d
f ba a d e e e fee . W c f e f a def c e c e
e e bef e ?

A. V a A
B. V a B12
C. V a C
D. V a D

W c f ef f d c e a e e bac e a e ?

A. P e ca b d a e b ea e
B. E e a ed feca fa e e
C. Ceca a aec e >103 c f (CFU / L)
D. B a , f a e ce, abd a a

Ga e e 107
Get The Step 2 CK Drills Book: usmledrills.com
The USMLE Guys Step 2 CK Crash Course usmleguys.com

W c f ef f d c e e a ab ?

A. M c e a
B. A e ea
C. Ge e a ed ede a
D. H e a b e a

Ga e e 108
Get The Step 2 CK Drills Book: usmledrills.com
The USMLE Guys Step 2 CK Crash Course usmleguys.com

D ,D ,M I a, I C
Diverticulosis

Ab a c e eb e eeb d e e e e ae ec c a ce a e f e
c .

S :
● U a a a c, b be a e e a c e a, e e ,ca

Lab ( f b eed ):
● CBC, c a (PT/PTT, INR), c e e ,ab , e e e

Da :
● Made a c c
● I a :
○ Ta ed ed b d ce ca a d a a ca de f ac e b eed
● C c :
○ V a ed b d e e ac e b eed

T ea e :
● S ab e a e
● If ac e b eed, ca ec fe e ed c c
● If a a ed de f b eed, e b a f b eed e e
● Se e a c ec f e c e b eed f e d c ca d
a a c e a fa e

Diverticulitis

I fec fa a f e a d e c a.

S :
● C a ef e ad a (LLQ) abd a a (RLQ a A a a ),
fe e , a ea/

C ca :
● Ab ce f a , ef a ,f a, b c

Lab :
● Ca be a e e a ed CRP a d WBC

I a (CT a a d IV c a ):
● L ca ed b e a c e
● Fa a d
● C cd e c a

Da :
● C ca a d e a

Ga e e 109
Get The Step 2 CK Drills Book: usmledrills.com
The USMLE Guys Step 2 CK Crash Course usmleguys.com

T ea e :
● O a e : a a b c (c f ac e da e)
● I a e : NPO d d e , IV a b c ( e ac - a bac a ), IV f d
○ Pe f a :S e
○ Ab ce 4c :Da a e
○ Ob c f a: S ca e ec f e ed b e

Mesenteric Ischemia

H ef f e a e e .

R Fac f E b a :
● Ca d ac a a (A-f b), a a d ea e, e c a a ca e ,a c
a e ce , fec e e d ca d , ece MI

R Fac f T b :
● Pe e a a e d ea e, d a e, d abe e , , e e , e de a

R Fac f N cc e Me e e c I c e a:
● Ca d e c c e ae f ef

S /S :
● F e e e ca e a e b : Se e e abd a a ( e b ca ) f
e a , , d a ea ( e e b d ), be ce fe b
( ee fac )
● F e e e ca e a b :C c a d a abd a a , be
e
● F cc e e e e c c e a f e ca e f e f
(c e e ea fa e, e ,a a ) e e e abd a a
( e b ca )

Lab :
● E e a ed WBC a d ac a e e e ( e ab c ac d a a )

I a (H -Re CT A a ):
● Ab e a f e e e (c ff ) e e b cc e
● N cc ee b de ae e e f defec

C ca :
● B e ec
● Pe f a
● Pe

Da :
● C ca / c e ed de ( ece a ), ca e f cc e
d ea e a ca de a def eda

Me e e c I c e a Ma a e e :
● S ab e a e :
○ S e e a e
○ NPO, a a c c

Ga e e 110
Get The Step 2 CK Drills Book: usmledrills.com
The USMLE Guys Step 2 CK Crash Course usmleguys.com

○ IV f d > a e
○ A b c ea
○ E c b ad- ec a b c
○ P b
● T ea ba ed ca e f e e e c c e a:
○ E b :S ca e b ec
○ T b : Me e e c b a
○ Se e a b e e ec e c ca ( ec )a e

Ischemic Colitis

Red c ef f ec , ca a e a d ced a e ed
e ( e cfe e a d ec d c ) c ac ff c e c aea b df .

R Fac :
● He da ( e d da ), ca d a fa c ,a ac e

S :
● Ca abd a a ( ef ded), e a c e a, e e

Lab :
● E e a ed ac a e, LDH, CPK, a d a a e e e aec e e e e d ea e
(da a ed b e )

I a (CT Abd e IV C a ):
● B e a ede a a d b
● P e a c (a b e a )

Da :
● Made def e c c (ede a f eb e , f ab e a d e e a
c a be ee a ea f a )

T ea e :
● S e ca e a d b e e
● E c b ad- ec a b c
● F e ec :C c e ec

Content Re ie Questions:

A 82- ea - d a e e e e ED f a e a ed ca
a dc a fc a ef e ad a abd a a .V a c de SPO2 96%
HR 126 BP 88/60 T 101.8. W a e be e e a a e e f a e ?

A. CT abd e a a d IV c a
B. Ob a IV acce
C. E d ac ea ba
D. C c

Ga e e 111
Get The Step 2 CK Drills Book: usmledrills.com
The USMLE Guys Step 2 CK Crash Course usmleguys.com

A 68- ea - d a e a ed ca fa a fb a e e e ED e ee
e b ca abd a a f e ca e a .H -e CT
a a a ab e a f e e e e e ca e .W c f e
f e a aeda ba ed e e c ca fea e a d CT f d ?

A. I c e c c
B. Ac e e e e c c e a
C. D e c
D. D e c

W c f ef ed a e e def eda f c e cc ?

A. MRI abd e
B. CT abd e IV c a
C. C c
D. C ca d a

Ga e e 112
Get The Step 2 CK Drills Book: usmledrills.com
The USMLE Guys Step 2 CK Crash Course usmleguys.com

C Ca ,C P , Fa a A a P ,a
L S

C Ca ce Sc ee :A ea eR
● I ae c c a a e 50 (45 f Af ca A e ca a e ) a d c e
c ee a e 75 ( a e efe e ce a d fe e ec a c c de a e eaf e )
● C c e e 10 ea ( FIT f cc b d ea , a e DNA e e
ee ea , CT c a e e f e ea a d a be f ed b
c c f e)

C Ca ce Sc ee :P e Fa H fC Ca ce
● I ae c c a a e 40 ( 10 ea bef e e a e a fa e be
da , c e e ea e )
● C c e e 5 ea

C Ca ce Sc ee :L c S d e
● I ae c c a a e 20-25 ea ( 2-5 ea bef e ea e e fa
e be d a ed c ca ce , c e e ea e )
● C c e e 1-2 ea

L c S d e (He ed a N C ec a Ca ce ):
● A a d a d de affec DNA a c e a (MMR) e e
● I c ea ed f ce a ca ce (e ec a c ec a ca ce a d e d e a ca ce )
bef e a e 50
● T a e c ae e ab a d f a f MMR e
c e

C Ca ce Sc ee : Fa a Ade a P (FAP)
● I ae c c ( fe be d c f ed b c c f e)
a a e 10-12 ea
● E e ea c c ( fe be d c )

Fa a Ade a P (FAP):
● A a d a d de e f a ed APC e e
● P e e ce f e be ( e a 100) f ade a c ec a be
20 30 f affec ed d d a
● C a d d de a ca ce c ,a a c a ed f c a a d a a
d ca ce

C cP :
● I fa a ( - e a c) a e ade f fa a ce a e a
a a de e a e
● Ha a a ( e e a be b a bec e a a ) a e ade f
d a ed e
● Se e e a ed e c de a ca e e f c d e a c
( - e a c) c ae e c f ec a d e e e a ed
(f e e d a c)
○ If e a c f 10 de f ed: Re ea c c 3-5 ea
○ If e e e a ed 10 , d a a a : Re ea
c c 3 ea

Ga e e 113
Get The Step 2 CK Drills Book: usmledrills.com
The USMLE Guys Step 2 CK Crash Course usmleguys.com

○ If 3-4 e e e a ed <10 f d, c c e ea ed 3-5


ea .
○ If 5-10 e e e a ed <10 f d, c c e ea ed 3 ea
● Ade a ( c e a c ) a e ca e ed a b a,
, b . Af ca A e ca , e , BMI a fac
○ 3-4 b a ade a <10 : Re ea c c 3-5 ea
○ 5-10 b a ade a <10 : Re ea c c 3 ea
○ Ade a 10 , b ade a , ade a - ade
d a a: Re ea c c 3 ea
○ 11 e ade a e e : Re ea c c 1 ea

Colon Cancer

S /S :
● Ma a a c (ea a e); b c ; GI b eed ( e a c e a e e a);
abd a a ;c a e fe e c ,c e c , ca be ; a ab e ec a
abd a a

C S e f Me a a :
● Li er a d ,a e a e e a d e a de ( ac a c a ,
e b ca d e )

C c :
● G d a da d ( e be e ) e a e e e fc ca ce

Da :
● H ca f d b a ed f e a e f e GI ac ( a c c GI
e )c e c ca ce

T ea e :
● T e ca ed d ea e ea ca e ec a dc e ea
● T e e a a c d ea e ea a a ec e ea

Content Re ie Questions:

A 39- ea - d fe a e e e e a e e e a e a d
abd a a a d e feca cc b d e . T e a e e d e a fa f
e fa e be c ec a ca ce d a ed be ee e a e f 40-55 ea . A
c c ef ed, a d a b ed c de c e
f a f MMR e .W c f ef e a a ce a e
a f de e ?

A. E d e a ca ce
B. O e a c a
C. Ac e e d e e a
D. H d ' a

Ga e e 114
Get The Step 2 CK Drills Book: usmledrills.com
The USMLE Guys Step 2 CK Crash Course usmleguys.com

A 50- ea - d a e de e c ee c c a d f d a e4 b a ade a
<10 .W a e e e d f e ec e ded a be ee c ee
c c a d e c c ?

A. 1 ea
B. 3 ea
C. 5 ea
D. 10 ea

A 52- ea - d fe a e de e e f c ee c c a d f d a ea e e
e a ed 12 e? W a e e e d f e ec e ded bef e
a e de e e ea c c ?

A. 1 ea
B. 3 ea
C. 5 ea
D. 10 ea

Ga e e 115
Get The Step 2 CK Drills Book: usmledrills.com
The USMLE Guys Step 2 CK Crash Course usmleguys.com

D -I L I ,A H a ,a N -A
Fa L D a
Drug-Induced Liver Injur (DILI)

I c He a :
● D e-de e de , ae e d ,d ac d ec ca e ce / e dea
● E a e: Ace a e

Id c a c Reac :
● M f ca e f DILI
● N d e-de e de , a ab e ae e d
● I e ed a ed da a e. E a e :A c -c a a a e, e
● N - e ed a ed ( e e c ca e f a f e ab ). E a e :A da e
a d, e c a e, a ae

S /S f Ac e DILI:
● A a c e e ab a e
● Fe e , e ad a a , a d ce, ,a e a, a ea/ , ae
, da e, e a e a
● Ac e e fa e

S /S fC c DILI
● Ja d ce, a c e , a a e e a, c , e a c e ce a a

Lab :
● D - d ced e ( e a ): ALT e e a ed 3 e e f a e
● D - d ced e (c e a ): A a e a a e e e a ed 2 e e
f a e
● D - d ced e ( ed): F d fb DILI e a a d DILI c e a

Da :
● Da fe c : C ca c e DILI eb d e
f e ca e f e
● If d - d ced e ca c e a , a e ed e b a
b c a d ca ed
● L e b c f da fc c e d ea e ec ed, f eda
ce a e e f DILI

T ea e :
● I ed a e ca a e ed ca
● N-ace c e e f ace a e c
● L-ca ef a ae c
● Ac e e fa e ea ed e a a

Alcoholic Hepatitis

H :
● Hea ac ef >20 ea ( ca da )

Ga e e 116
Get The Step 2 CK Drills Book: usmledrills.com
The USMLE Guys Step 2 CK Crash Course usmleguys.com

S /S :
● RUQ e de e , a d ce, fe e , e a e a ,a e a, a c e

Lab :
● E e a ed a a a e ( a e a 300 IU/L) AST e e ce a (
e) a ALT e e , e e a ed INR, e e a ed e b b , e e a ed GGT, e e a ed
WBC ( e ed a ce)

Da :
● Made ba ed c ca e e a a d e ab ce a e a a db a
b c a e ed ab a d a abd a a d

T ea e :
● Ac ce a a d ea e f da a
● S e ca e
● P a a a GI b eed
● P ed e e e e ca e
● L e a a f e e e fa e ( f a e e a a ed a d f d
a e a fac e a e af e a a )

Non-Alcoholic Fatt Liver Disease (NAFLD)

He a c ea f ea ac e.

A c a ed D de :
● Obe , d abe e e ( e a ce), d de a, e e

S /S :
● U a a a c, be e a e a fc f d ea e a
e ed

Lab :
● U a d- de a e e e a ed AST a d ALT (AST a e a ALT) a d
a a e a a e e e a ed 2-3 e f a

Da :
● U a d, CT, MRI ca a be ed a eda
○ U a d ed, de a e ec c e e (b e)
● R e a e a ,a e e a ,a d e c a ab :
○ A e e a : Ga a b e e,a c ea a b d , a -
cea b d ,a - e / d e c a a b d -1
○ V a e a : He a A I G, e a B face a e , face a b d a d
c ea b d ,a - e a C a b d
○ He c a e , fe ,a d a b d ca ac (TIBC)

T ea e :
● T ea a a c a ed e ab c d de
● We f e e a e
● Ac ce a f a e c e ac

Ga e e 117
Get The Step 2 CK Drills Book: usmledrills.com
The USMLE Guys Step 2 CK Crash Course usmleguys.com

Content Re ie Questions:

W c f ef ed ca a aef e ea e f ace a e c
a a e e e a ed ALT e e e a a ea, , e a ,a d
a ?

A. L-ca e
B. N-ace c e e
C. G ca
D. A da e

Ac c e a c a ac e ed b c f ef ea be ee AST a d
ALT?

A. AST:ALT a 2
B. AST:ALT a ea 1
C. ALT:AST a 2
D. E e a ed ALT a AST

W c f ef a df d c e -a c c fa e
d ea e?

A. P e e ce f e c ec cf d
B. B e f e e ed e
C. N d a e a c c
D. H e ec c e a c e e

Ga e e 118
Get The Step 2 CK Drills Book: usmledrills.com
The USMLE Guys Step 2 CK Crash Course usmleguys.com

V a H a
Hepatitis A

Ta :C a a ed f d/ a e e - - e (c ec ac , e a ec e,
IV d e)

I c ba T e: 28 da

Pe e : He a A acc e

S :
● Fe e , abd a a , a ea, a e a, e da ea d ae ,a d
a d ce

Lab :
● E e a ed ( f e d a ca ) e a a fe a e a d e b b

C ca :
● A e e a , ea e a
● F a e a c fa e ( a e, a cc e e e
d ea e a d de a e)

Da :
● P e e I Ma -HAV a b de a e fac a d

T ea e :
● S e ca e
● L e a a f e e f a e a c fa e

Hepatitis B

Ta : M e - -c d, e a ec e, IV d e, a f a d a a (ae
e f HBV), ea ca e e eed e c

I c ba T e: 1-4

Pe e : He a B acc a , -e e a

S f Ac e He a B:
● R e ad a a , a d ce, a e a, a ea

S /S fC c He a B:
● Fe e , a ,a , a d ce, e ce a a ,a c e , e e a ede a,
e e a

A c a ed D de fC c He a B:
● P a e d a, e ba e a ,c , e a ce a ca c a

Lab D Ac e P a e:
● E e a ed e a a fe a e (1000 2000 /L)

Ga e e 119
Get The Step 2 CK Drills Book: usmledrills.com
The USMLE Guys Step 2 CK Crash Course usmleguys.com

● ALT > AST


● Se b b e e ca a
● E e a ed ALT e e e a 6 a c c e a

Lab C c He a B:
● E e a ed e a a fe a e ( d- de a e) a ba e e ( b a)
● E ace ba e e e e e a ed ALT ( 50 e e f a)a d
e e a ed a a-fe e d e ace ba
● Dec ea ed WBCS, a e e , a d a b , e e a ed PT a d b b e
e c

T ea e :
● S e ca e
● If a e ac e e fa e, a e c , e ca e f ALT e e ce
e e f a e HBV DNA e e af e 6 , a a a
e a (e eca , e f )

Da P a e fC c He a B I fec :P e Ma e
● I e ea :
○ HB A , HBeA , a a -HBc, HBV DNA, a / d e e a ed ALT
● I e-ac e, HBeA - e:
○ HB A , HBeA , a a -HBc, HBV DNA, e e a ed ALT
● I e-ac e, HBeA - e a e:
○ HB A , a a -HBc, a -HBe, HBV DNA, e e a ed ALT
● I ac e c c HBV:
○ HB A , a a -HBc, a -HBe, b e HBV DNA, a / d e e a ed ALT
● Occ HBV:
○ T a a -HBc, a -HB , a -HBe, b e HBV DNA, a ALT

Da P a e f Ac e He a B I fec :P e Ma e
● Ea a e:
○ HB A , HBeA , I Ma -HBc, be a a -HBc, HBV DNA, e e a ed ALT
● W d a e:
○ I M a -HBc, be a a -HBc, HBV DNA, e e a ed ALT
● Rec e a e:
○ T a a -HBc, a -HB , a -Hbe, b e HBV DNA, a ALT

Hepatitis C

Ta : M e - -c d, IV d e, a f a d a a a (ae
e ), ea ca e e eed e c , e a ec e

I c ba T e: 2 ee -6

S :
● Abd a a , e , a ea, d a ea, a e a, a a, a a a

A c a ed D de fC c He C:
● E e a ed c b e a, a ed de , e b a fe a e
e e , d abe e e , a c a ea a da, c , e a ce a
ca c a

Ga e e 120
Get The Step 2 CK Drills Book: usmledrills.com
The USMLE Guys Step 2 CK Crash Course usmleguys.com

Lab :
● Se ALT ca a f a e e e e eee a

Da :
● Pa e fac a d c e HCV
● P e ef e HCV RNA a b d e c f ed HCV RNA e

T ea e :
● A a e a ba ed a e e
● T e e c d a e ce ea a a f
e a ce a ca c a c ee a da e e d c c ee f e a ea
a ce

Content Re ie Question:

A 60- ea - d a e e e e a fe e , a , a d ce, a d a c e .
He a e a a d e e a ae ed e a .C c e a B ec ed. W c
f ef a e c e ada f e e ea a e fc c
He a B?

A. P e HB A , HBeA , a a -HBc, HBV DNA. N a ALT e e .


B. P e HB A , HBeA , a a -HBc, HBV DNA. E e a ed ALT e e .
C. P e HB A , a a -HBc, a -Hbe, HBV DNA. E e a ed ALT e e .
D. P e a a -HBc, a -HB , a -Hbe, HBV DNA. N a ALT e e .

A a e ec ed f a ac e e a B fec a a e c e bac e
f a a -HBc, a -HB , a -Hbe, HBV DNA. T e a e ALT e a a e.
T e ef d ae c e c f ef a e f ac e e a B
fec ?

A. Ea a e
B. W d a e
C. Rec e a e
D. La e a e

W c f ef e a ae c f eda f e a C
fec a a ca e f e c ?

A. H d f e a c ec e b d fb e aa d
ae c a d e f e e ea e a c e
B. N d a e a c c a d
C. P e ef e HCV RNA a b d e c f ed HCV RNA e
D. H e f e ca da e be a d a f e e e e f e
be CT

Ga e e 121
Get The Step 2 CK Drills Book: usmledrills.com
The USMLE Guys Step 2 CK Crash Course usmleguys.com

L C a C a
Liver Cirrhosis

Ca e :
● Ac c e d ea e, c c a e a (B a d C), ac c fa e d ea e,
e c a , a b a c , a ce c a , a a-1
a def c e c , a e e a

S /S :
● Ja d ce, , da e, a c e , e a c e ce a a , a e , a a
e e a, de a a a, ec a a, e ce, d a ea, e e e
ede a, ea b , e a e a , e e a , ca ed ae

Lab :
● B b a e -c e a ed c a d e e a e a d ea e
e e e e
● H ab e a a d c ea ed b e( e f e ec e
e a c e e )
● AST a d ALT a de a e e e a ed
● A a e aa e d e e a ed (ca be e e e e e a ed f a ce
c a a b a c e ca e f c )a d d e e a ed
a a- a a e da e (GGT) (ca be e e e a c c e d ea e)
● P be a e a a d e e a ed c ea e( e a e a d e)
● T b c e a, e e a( e c e e a )

I a :
● U a d
○ Pe e ce f e d e
○ I c ea ed ec e c
○ R be a
○ H e f e ca da e/ ef be
○ U a d-ba ed e a a a e e e ff e b ea a a
a e eed f a a

C C ca :
● He a c e ce a a
● A c e
● He a e a d e
● E a ea a ce
● S a e bac e a e
● He a ce a ca c a
● P a e b

Da :
● Def eda ade e b
● Fe e c ca , ab f d , a d ad a cf d ff c e a e
eda

Ga e e 122
Get The Step 2 CK Drills Book: usmledrills.com
The USMLE Guys Step 2 CK Crash Course usmleguys.com

T ea e :
● T ea de ca e f c (a a a e a ,ac ce a , e c.)
● Ad ed ca (a d e a c ed , a e d e e dec ea ed e f c )
● T ea c c ca
● L e a a f e dec e a ed c a d a e ca d da e

Hepatic Encephalopath

P ec a Ca e :
● I c ea ed a a: GI b eed, fec , e ce e a e
● Med ca / b a ce : Be da e e , a c c ,ac
● De d a
● Me ab c a d e ec e ab a e :H a e a, e ab c a a
● Va c a cc :P a e a c e b
● He a ce a ca c a

G ad S e :
● G ade I: A e ed ee a d/ c e a e c a ed a e
ba e e
● G ade II: N e ed e, e a c, a e ,d a a, be a a c a e
● G ade III: N e ed ace, e e (e d ),
a a e be a
● G ade IV: C a ( e e )

T ea e :
● L e a a e e ac e ac (add fa f e e )
● C ec a ec a ca e
● I a e e ca e f G ade III a d IV (a d a G ade II
ade a e e ca e a e )

Ascites

F d e e ea ca .

S /S :
● Abd a d e ( e e), abd a d c f ,fa d e ec
abd a e a , e a , ea a e ,d ea

I a :
● U a d, CT, MRI ca c f e e ce f a c e

Pa ace e :
● D a a e fa c cf df da c ea e c e

A c cF dA a :
● Ce c a d d ffe e a : C ec ed e c 250/ d ca e e a d
eed f a b c
● Se - -a c e a b ad e (SAAG) f 1.1 /dL d ca e e e ce f a
e e ( ee c , ea fa e)
● T a e c ce a :Ta da e f d f a e <2.5 /dL ( .e. c a d
e c a c e ), e da e f d f a e 2.5 /dL ( .e. ca d ac a c e )

Ga e e 123
Get The Step 2 CK Drills Book: usmledrills.com
The USMLE Guys Step 2 CK Crash Course usmleguys.com

● C :
○ C ea / e (c )
○ C d / b d ( fec )
○ M ( c ce de - e ca e .e. a a c ,c , a c
d )
○ B d / (c , a a c )
○ B ( e e e a ed b b ca ed b ed a b adde e f a ed
d de a ce )

T ea e :
● Ac ce a (bac fe f c a f eeded)
● S ed a ed ce e a e f (ACE/ARBS, NSAIDS, BB)
● Na e c 2 da
● C b a ea f e de a d ac e

Spontaneous Bacterial Peritonitis (SBP)

S :
● D ff e c abd a a , fe e , AMS, d a ea, e , e a,
aa c e

Da :
● A c cf dc ec ed e c 250/
● A c cf d e bac e a c e

T ea e :
● E c ea e d- e e a ce a (cef a e)
● P a f a e f SBP ( e - fa e a e) c
GI b eed (cef a e, e e - fa e a e ce b eed
c ed)

Hepatorenal S ndrome

P ec a Fac :
● I fec , GI b eed ( a ead e )

C a ac e c :
● E e a ed e c ea e
● N e a, a e ed e
● Na ec e e a 10 E /L

Da :
● P e e ce f e d ea e a e e
● P e e ce f AKI
● N e a b ab e ca e f AKI

T ea e :
● T ea ca e f e d ea e
● N e e e ab c ca a e
● M d d e, c e de, a d a b -c ca a e
● Da f e fa e d (+/- TIPS ced e e e e)

Ga e e 124
Get The Step 2 CK Drills Book: usmledrills.com
The USMLE Guys Step 2 CK Crash Course usmleguys.com

Content Re ie Question:

W c f ef f d c e e c abd a a d?

A. H e f e ca da e be
B. H e f e ef be
C. H e f e be
D. P e e ce f e d e

A a e c c e a C fec a dc e e e a
fe e a d a e ed e a a . O ca e a e a e ed a e f ca
a c e d ff e abd a a a a . A a ace e ef ed a d e a c c
f d c ec ed e c 400/ .W a e e be e a a e e ?

A. U a d-ba ed e a a
B. CT abd e c a
C. E c a b c ea e cef a e
D. T a a a e a c e c (TIPS) ced e

A a e c c e a C fec a dc e e e a
fe e a d a e ed e a a . O ca e a e a e ed a e f ca
a c e d ff e abd a a a a . A a ace e ef ed a d e a c c
f d c ec ed e c 400/ .W a e e da ?

A. C c e a C fec e ace ba
B. S a e bac e a e
C. He a c e ce a a ec da c c e a C fec a dc
D. He a ce a ca c a

Ga e e 125
Get The Step 2 CK Drills Book: usmledrills.com
The USMLE Guys Step 2 CK Crash Course usmleguys.com

A a C Pa a a C a

Acute Pancreatitis

Ca e :
● Ga e ,ac , e ce de a (>1000 /dL), f e d c c
e ade c a a c ea a (ERCP), ed ca ( a a e, d da e),
a a

S :
● Se e e c a e a c abd a a ( cca a RUQ a d a e LUQ) c
de e ac e , a ad a e e bac , e e ed b /be d e f a d,
a ea/

Lab :
● E e a ed e a a ea d a e (>3 e f a)

I a :
● U a d: E a ed a d ec c a c ea
● CT (c a -e a ced): E a ed a c ea
● MRI (MR T1): B ed a c ea c a , e a ed a c ea

Da : Made e 2 f3f c e aae e


● Ac e e e a d e eee a c a c a ad a e e bac
● E e a ed e a a e a d a e f >3 e f a
● I a c e ac e a c ea ( a d, CT, MRI)

T ea e :
● Pa c IV ae
● F d e ace e ea
● C ec e ec e a d e ab c ab a e

Severe Pancreatitis

S f fa e f e e a (ac e e a fa e, c , a d/ e a fa e) f
>48- e d.

R Fac a d C ca P ed c f Se e e Pa c ea :
● A e >75, a c a ca e, be
● P e e ce f eb d e de e ad e a
● Lab c d e e a ed e a c ( e c ce a ), CRP >150 /L, e e a ed BUN
a d c ea e
● I a e a eff , a c ea c ec , e a a c ea c f a a

Add a S /S Se e e Pa c ea :
● Fe e , ac ea/ e a, e ,C e ' ( e b ca ecc
f a e ea e a e), G e -T e (f a ecc f
e e ea e a e)

Ga e e 126
Get The Step 2 CK Drills Book: usmledrills.com
The USMLE Guys Step 2 CK Crash Course usmleguys.com

Se e e Pa c ea C ca :
● Pe a c ea c f d c ec
● Pa c ea c e d c
● Nec a c ea
● P - e - e e e c e b
● E ace ba fa e c bd

Chronic Pancreatitis

Ca e :
● Rec e ac e a c ea ,ac , e ce de a, bacc , c cfb ,
a e a c ea ,c c b c f e a a c ea c d c ( e ,
,c )

S /S :
● E a c a , a ad a e e bac , e af e ea a d e d ,
ea ea (f a ), e

Lab :
● Se a a ea d a e a be a d e e a ed

I a :
● U a d: I c ea ed a c ea c ec e c
● Abd a X- a : Pa c ea c ca c f ca
● C - ec a CT/ MRI a e c e a ce c a a c ea a (MRCP):
Pa c ea c ca c f ca , a c ea a ,d aa f e a c ea c d c , e a
a c ea c

Da :
● S a d fc c a c ea f d c - ec a CT
MRCP c e c c a c ea

T ea e :
● Ab a f ac a d bacc e
● L -fa a ea
● Pa c ea c e e e e
● A e e NSAID f a c , e ea e d
a a e a f fa e c a ac c

Ga e e 127
Get The Step 2 CK Drills Book: usmledrills.com
The USMLE Guys Step 2 CK Crash Course usmleguys.com

Content Re ie Questions:

A 44- ea - d fe a e a fac ab e e e e ED c a f e ee
c a e a c abd a a f e a 1 da . S e e e a ade be e e
a d be d e f ada d e e e a ea. Lab ab a e c de
e e a ed e a a ea d a e c ae5 e e e f a.
C a -e a ced CT f e abd e a e a ed a c ea . W c f e f
e e be e a a e e ?

A. Pa c ea c b a dc e
B. E c ea e cef a e
C. Pa c IV d
D. Pa c ea c e e e e a

A 61- ea - d ae a e fac ab e e e e ED e ee
e a c a c ad a e e bac a d e af e ea .T e a e a e d e
d a ea c a ea ea a d d ff c f . Se a a e a d a e e e a e 1.5
e e f a.C - ec a CT f e abd e e de ce f a c ea c
ca c f ca .W a e e da ?

A. Ac e a c ea
B. C c a c ea
C. Pa c ea c ca ce
D. C a ca c a

W c f ef f d c e eda f e e e a c ea ?

A. L e a c
B. CRP >150 /L
C. Pa c ea c ec CT
D. E e a ed c ea e

Ga e e 128
Get The Step 2 CK Drills Book: usmledrills.com
The USMLE Guys Step 2 CK Crash Course usmleguys.com

Pa a Ca ,C a a a, a H a a
Ca a
Pancreatic Cancer

R Fac :
● T bacc e, be ,de fa a d/ ea , fa , e ed a c c
a c ea

S /S :
● A e a ( ac f e e a d e ), e , cac e a, a e a
● E a c a ( a ) ad a e bac ade e e, a ,
a d a da
● He a e a , a d ce, da e
● Na ea/ , d a ea/ ea ea

Lab :
● E e a ed ca b daea e 19-9 (CA 19-9)
● If a d ce e a c a : A e f c e a (e e a ed a a fe a e ,
a a e a a e, a d b b )

I a :
● U a d ( f a d ce e e ): Ma e a c ea
● CT ( f a d ce): Ma e a c ea

Da :
● E d c c a d (EUS): G ded ec a e b f e a c ea c a
ca e ec f e a ba e, de e d a f d

T ea e :
● S ca e ec f a e ea a e f ed b ad a c e ea
● Pa a e ea e f e ec ab e ec e a c ea c ca ce

Cholangiocarcinoma

R Fac :
● Primar sclerosing cholangitis, c , a e a ,L c d e, c c
fb

S /S :
● E a e a cc a ca c a ( b c b a ee): He a e a , a d ce, da
e, a e , RUQ a (d , ac , c a ), fe e , ea , e
● I a e a cd ca c a ca c a :A a c RUQ a (d ), e

Lab :
● E a e a cc a ca c a :
○ E e a ed a b b , d ec b b ,a da a e aa e
○ ALT/AST, PT/INR a ea d ea e a d e e a ed/ ed a d ea e
e e
● I a e a cc a ca c a: E e a ed a a e a a e a d GGT
● P e a e CEA a d CA 19-9

Ga e e 129
Get The Step 2 CK Drills Book: usmledrills.com
The USMLE Guys Step 2 CK Crash Course usmleguys.com

I a :
● Ma e c e a ce c a a c ea a (MRCP)
● I a e a ca de a e a cd ca d a d a e a e a cc a ca c a
( a be a e a cd ca d aa e a c a ca c a)
● P e e ce fa a d ca d aa a e a cc a ca c a

Da :
● Made def e e a e, b d e f eed f eb ac
a a ce fe a e a e ca e ec ba ed a
e a e e ec ab e ca e , a a e ea e e a e

T ea e :
● F e e ec ab e d ea e, e f ed b ad a c e ea

Hepatocellular Carcinoma (HCC)

R Fac :
● C f a ca e, c c a e a ,ac , ac c fa e d ea e,
bacc , af a

S /S :
● Ma a a c RUQ a , RUQ a ab e a , e , ea a e

Pa a e a cS d e :
● E c , e ca ce a, ce a

Lab :
● P e a a-fe e (AFP)
● P be e ca ce a, ce a, e c
● De e d c c e c d (c ,c c a e a ,ac c e a ,
e c.) ab ab a e a a be e e
○ E e a ed BUN a d c ea e
○ P ed PT e e a ed INR
○ H ab e a
○ P e e B a d/ C e

Da :
● Da ca be ade c a -e a ced a d, CT, MRI e ae
c de ed beca e f c ,c c e a B, e da f
HCC
● B e e ed (ca e-b -ca e dec ) e bab e HCC ee a ,
b fac a d e eeded beca e ef d c a e
a a e e
○ R fc ca ( e e e , e a , e )
○ R f eed HCC a e eed e ac
○ P b a d a d e fa e e a e

T ea e :
● Re ec f ca ed HCC

Ga e e 130
Get The Step 2 CK Drills Book: usmledrills.com
The USMLE Guys Step 2 CK Crash Course usmleguys.com

● L e a a a f e c e d f c e a e e ec ,b
e de ce f a c a a a d de e e d a ea a e
● L c e a e -d ec ed e a (ab a /e b a )f e d a f
f e ec / a a
● S e c ea f e d a f f e ec / a a / c e a
e -d ec ed e a

Content Re ie Questions:

A ea a-fe e (AFP) a e c e a c a ed c f e
f d f ca ce ?

A. Pa c ea c ca ce
B. C a ca c a
C. He a ce a ca c a
D. O e a c a

Ma e c e a ce c a a c ea a (MRCP) de a e e e ce f a a
d ca d aa a c a ed c d fc a ca c a?

A. I a e a c c a ca c a
B. D a e a e a c c a ca c a
C. Pe a c a ca c a
D. Nec c c a ca c a

W c f ef a fac f e de e e fc a ca c a?

A. P a ce c a
B. V a e a
C. C
D. W d ea e

Ga e e 131
Get The Step 2 CK Drills Book: usmledrills.com
The USMLE Guys Step 2 CK Crash Course usmleguys.com

D a Ga b a a A C a

I c de a (A a c) Ga e :
● M d d a a e
● P e e ce ed a a c de a f d a
● N ea e ece a

Biliar Colic

S :
● Se e e, c a ,d a e RUQ; f e ad a e e de ;
a 4-6 ; e e acc a ed b d a e , a ea/
● S f e de e af e ea a ea fa
● Lab a e a
● S fe e ec
● T ea ed e a a a e c /NSAIDS f ed b e ec e a a c c
c ec ec

Acute Cholec stitis

Occ f c cd c b c .

S /S :
● Se e e a d c a RUQ a a e a 6 , b e ad a e
de , fe e , ac ca d a, a ea/ , eM ' , a a d
a ad

Lab :
● E e a ed WBC c a ef f
● T ca ee a a a e aa e e a b b

I a :
● Abd a a d a b adde a c e ede a
● P e a cM '

C ca :
● Pe f a f e a b adde , a e c ec c ec e e cf a
a e e ,e e a c ec

Da :A f eF
● P e e ce f a e a b adde a c e ede a ee a
● P e a cM '
● C e c a (HIDA ca ) a b adde f defec ( e f ed f abd a
a a e ca )

T ea e :
● IVF e a , a a a e e NSAID d
● E c a b c ( e ac - a bac a e a e e ) e a b adde
e ed
● C ec ec 3 da f e f a e

Ga e e 132
Get The Step 2 CK Drills Book: usmledrills.com
The USMLE Guys Step 2 CK Crash Course usmleguys.com

● If c ca ab e eae e ,a b c a dda e a b adde a


ec a e c ec

Acalculous Cholec stitis

● Of e d a d cc a ed a d c ca a e
● C ca f c ec ( ef a f e a b adde , a e c ec )
ae ec

S :
● Fe e , a ab e RUQ a , a d ce, e

Lab :
● E e a ed WBC c ,c a ed b b ,a a e a a e, a d ALT/AST

I a :
● Abdominal ultrasound sho ing:
○ Pe c ec cf d( a )
○ Ga b adde a c e
○ P e a cM '
○ S b e a ede a
● Abd a CT
○ Ga b adde a e a ce e
○ Ede a f e a b adde
● HIDA
○ Ga b adde f defec

Da :
● P e e ce f fac (c ca e )
● S / (fe e ; abd a a ; e c ; e e a ed c a ed b b ,
a a e a a e, a d ALT/AST) c e d ea e
● I a f d c e d ea e ( a b adde a c e , e c ec c
f d, e a cM ' )
● E c fa e a eda e

T ea e :
● IVF e a , a a a e e
● E c b ad ec a b c ( e ac - a bac a e a e e ) e
a b adde e ed d ea e a e e e f a b adde be d a ed a e
a e ed
● C ec ec f a e c ca ab e eae e da e
a b adde a e c a e c ec

Emph sematous Cholec stitis

Sec da fec f e a b adde a a -f bac e a (C d , E. c ).

R Fac :
● D abe e , a e e , a e 50-70

Ga e e 133
Get The Step 2 CK Drills Book: usmledrills.com
The USMLE Guys Step 2 CK Crash Course usmleguys.com

S /S :
● Fe e , RUQ a , a ea/ ,ce a a f RUQ

Lab :
● E e a ed c a ed b b (d e e f c d )
● E e a ed WBC c a ef f

I a (Abd a CT):
● CT a -f d e e e a b adde e a d bea e e a b a
ee
● A e a f e a b adde

Da :
● S a d fe e a c ec a e a b adde
ee abd a CT

T ea e :
● E ca b c ( e ac -a bac a ) a d c ec ec

Acute (Ascending) Cholangitis

Bac e a fec e f b a b c (f e, c e, a a c ).

S :
● C ac ' ad: Fe e ( a c ), RUQ a , a d a d ce
● I e e e ca e : AMS, e , e a d e , a

Lab :
● E e a ed WBC c ef f , e e a ed CRP
● E e a ed a a e a a e, a a- a a e da e (GGT), a d c a ed
b b
● AST/ALT f e e e a ed, cca a e ee e e a ed f a e a a ac e
e a c e ec

I a :
● Abd a a d:
○ B a d a
○ B ed c e
● Abd a CT:
○ B a d a
○ B a e

Da :
● Pa e a e e f ef :
Lab c
○ e fec / fa a (e e a ed WBC c CRP)
Fe e
○ c
● AND a , a eb
○ Lab f d c e c e a (e e a ed b b 2 /dL OR e e a ed
a a e a a e, GGT, ALT, AST >1.5 e f a)
○ I a b a d a a ca e f b c

Ga e e 134
Get The Step 2 CK Drills Book: usmledrills.com
The USMLE Guys Step 2 CK Crash Course usmleguys.com

T ea e :
● IVF e a a d a a a e e
● E c a b c ( e ac - a bac a ca ba e e )
● ERCP f b a d a a e ( e c a e a e a cb a da a e ca
da a ea f ERCP fa ca be e f ed)

Content Re ie Questions:

A 59- ea - d fe a e e e e ED fe e , c , abd a a ,a d a d ce. O


e a e a ca ed e RUQ. Lab c de e e a ed WBC c ef f , e e a ed
CRP. Abd a a d b a d a a d e e e ce f a e eb ed c.
Ga b adde a e a.W a e e da ?

A. P ce a a b adde
B. B a c c
C. Ac e c ec
D. Ac e (a ce d ) c a

A 41- ea - d fe a e e e e ED e e e, c a ,d a e RUQ
ad a e de c a bee f
e a 3 .S e e e
de de f c a e a 6 . Lab a e a a d abd a a d
a e ae ed a b adde a c e .W a e e be e
a a e e f a e ?

A. IV e ac f a c
B. ERCP e e a
C. IV a b c ea e e a e e
D. E e e c c ec ec

W c f ef e f ec e a a d be ff c e f eda f ac e
c ec ?

A. P e e ce f a e a b adde a c e ede a abd a


a d
B. P e a cM '
C. C e c a (HIDA ca ) a b adde f defec
D. P e e ce f a e ec b ed c

Ga e e 135
Get The Step 2 CK Drills Book: usmledrills.com
The USMLE Guys Step 2 CK Crash Course usmleguys.com

P a S C a a P a B a C a
Primar Sclerosing Cholangitis

C c d ea e c a ac e ed b fa a a dfb f e a e a ca de a e a c
b e d c ead c e a a de e a e fa e.

A c a ed ce a ec .

S :
● I a a a c, e a d ce, e a e a , e e a ,e c a
(f )
● E d c bac e a c a : Ja d ce, fe e ( c ), RUQ abd a a

Lab :
● E e a ed a a e a a e, e e a ed b b , ALT a d ALT a e a 300
IU/L
● P-ANCA, e a a b e a, e e a ed I M e e , a e c ea e
DR 52a

I a :
● E d c c e ade c a a c ea a (ERCP) a e c e a ce
c a a c ea a (MRCP)
○ N e a ea f c a d e e a d a f e a e a c a d/
e a e a cb ed c

L e B :
● O ed e ec ed a d c PSC
● O a e :Fb c a b e d c , e aced b c ec e e a
c ce c a e ( a ea a ce)

C ca :
● Fa - be a def c e c e
● C a
● C a ca c a
● C ( ead e a ce a ca c a)

Da :
● L e e ab a e e a ea f c a d e e a d a f e
a e a c a d/ e a e a c b e d c ee ERCP/MRCP

T ea e :
● L e a a
● E d c cd a a d/ e fad a c e (f a c e ef,
c a e)
● P ac c a b c f e ec e e d c bac e a c a
● Sc ee f e a b a ca ce , e a ce a ca c a(f e a ec ),
a dc ca ce

Ga e e 136
Get The Step 2 CK Drills Book: usmledrills.com
The USMLE Guys Step 2 CK Crash Course usmleguys.com

Primar Biliar Cholangitis (Primar Biliar Cirrhosis)

I e ed a ed de c f e b a b ed c ead c e a a de e a
c .

P a affec e .

A c a ed S e ' d e, e a da , Ha ' d ( e
d d ea e), a d c a e c e de a.

S /S :
● I a a a c, e fa e, , a a a d/ a ea a ,
e c a , e e a , a d ce, RUQ a , e a e a , e e a ,
c

Lab :
● N a / d e e a ed AST a d ALT
● E e a ed b b e e (d ec a d d ec )
● E e a ed a a e a a e (ca be e e e) a d a a- a
a e da e (GGT)
● H e de a
● Positi e antimitochondrial antibodies (AMA), a c ea a b d e (ANA)
● E e a ed I M e e , ce a , b e ac d , a ae
● Lab c e c ad a ced d ea e

I a :
● U ed e c de e a e a cb a b c ( a d, MRCP, ERCP)
● Pe a a e MRI ( ded a e d a e
ba c e )

L e B :
● N a e de c ec a
● De c f e b a b ed c

C ca :
● C ( a ead e a ce a ca c a)
● Def c e c e f fa - be a
● O e

Da :
● E c de e a e a c b a b c a d e e d ea e
● P e e ce f f ef c e a:
○ A a e a a e 1.5 e e f a ( e)
○ P ea c d a a b d e (AMA)
○ N a e de c ec a a d de c f e b a b ed c
e b

T ea e :
● U de c c ac d (UDCA)
● L -e ( e b c e ca a d f c e , b e, a A
a d D e e , TSH, b e e a de e )

Ga e e 137
Get The Step 2 CK Drills Book: usmledrills.com
The USMLE Guys Step 2 CK Crash Course usmleguys.com

● If a e de e c add a ec e ded c ee f c ca ( e
e d c f e a ea a ce , e a df e a ce a ca c a)
● He A a d B acc a
● Ac ce a

Content Re ie Questions:

P a ce c a a c a ed c f ef d ea e ae ?

A. S e ' d e
B. Sa c d
C. U ce a e c
D. M a c a e d ea e

A 55- ea - d fe a e e e e ED a d ce, ,a d e a e a . He ab
e e a ed a a e a a e a d e e a ed b b e e a d e f P-ANCA.
Y ec a ce c a a a beda .W a e e be e
de f e ca e f e a e ?

A. B d c e
B. Ma e c e a ce c a a c ea a (MRCP)
C. L e b
D. N f e - eeded a e eda

T e e e ce f c f ef a e /a b de a f eda cc e af
a b a c a ?

A. Ca b daea c a ed a e 19-9 (CA19-9)


B. A c d a a b d e (AMA)
C. c-a e c a c a b d e (c-ANCA)
D. A a-fe e (AFP)

Ga e e 138
Get The Step 2 CK Drills Book: usmledrills.com
The USMLE Guys Step 2 CK Crash Course usmleguys.com

Hematolog -Oncolog

M A a
Microc tic Anemia

Ca e :
● I def c e c , c c fa a , aa e a, de b a ca e a, ead

Iron Deficienc Anemia

Ca e :
● Red ced a e, b d ( e a , GI d ea e), ed ced ab
(ce ac d ea e, H. P )

S /S :
● Fa e, ca, a ,e ec e e a ce, e e a ed ea e ,a c

Lab :
● CBC RBC c , dec ea ed e b a d e a c
● L ea c c a e, ea c c a e b , ab e
e c c ec
● L e e e, c ea ed e a fe , c ea ed TIBC, fe e e

Da :
● Ca be ade f a f ef c e aae e:
○ A e a a e e f e e a
○ Se fe <30 / L
○ T a fe a a ( c e /TIBC 100) <19%
○ Ab e ce f a ab e b e a b

T ea e :
● Se e e fe- ea e a e a ea ed ed b d ce a f
● Oa IV e e a d- de a e ca e

Anemia of Chronic Disease

Ca e :
● I fa a d ea e , a a c ,c c fec

S /S :
● Fa e, e e a ed ea e , a , ac ca d a, d a e , eadac e

Lab /D a :
● MCV ca be c c c c c
● L e c c ec
● L e , TIBC, a / c ea ed e fe , a fe a a
● E e a ed ESR a d CRP de e d fa a ca e

He a -O c 139
Get The Step 2 CK Drills Book: usmledrills.com
The USMLE Guys Step 2 CK Crash Course usmleguys.com

T ea e :
● T ea de fa a d de
● Se e e a d fe- ea e a e a ea ed ed b d ce a f
● I e e e e b def c e c a e a a d a e a f
c c d ea e

Sideroblastic Anemia

Ca e :
● N e e b e: C e a a , e d a c d e , e fe a e
e a
● Re e b e: A c e, e a, ed ( a d, e d), c e def c e c

S /S :
● Fa e, e e a ed ea e , a ,d ea e e , e a e a

Lab :
● Va ab e, b fe c c c a e a (e ec a c e a ca e )b ca be
ac c c
● Pe ea b d ea a RBC ab a e c a de c e
● L / a e c c ec
● I c ea ed e a fe a a , e fe , e
● L e c e a d ce a e e fc e def c e c ca e
● P e e e c e c e a ca e

Da :
● R de b a b e a b

T ea e :
● B d a f f a ca e c d d a c e f
e ad ( ea c ea - defe a e defe a )
● C de a e ce (e d a a d c e c a e a e- a e
e e )

Lead Poisoning

Ac e E eS /S :
● He e: A e a
● Ne : Headac e, d ff c c ce a , de e
● MSK: A a a, a a
● GI: Abd a a ,c a
● S e c: Fa e, e a ee

C cE eS /S :
● He e: M c c c a e a
● Ne : Dec ea ed c ,a e , de e ,d a e a , e
● Re a : Lead e a a d ead- e a ed e c

Lab :
● E e a ed b d ead e e ( 5 c /dL)
● CBC c c ca e a

He a -O c 140
Get The Step 2 CK Drills Book: usmledrills.com
The USMLE Guys Step 2 CK Crash Course usmleguys.com

● B d ea a e f ba c
● P b e e e a ed c ea e

Da :
● B d ead e e ed c f da ( 5 c /dL)

T ea e :
● B d ead e e 5-40 c /dL: E e ed c ( e a f e e f e 30
c /d ), e ea b d ead e e
● B d ead e e >40 c /dL: Re a f a e f e e, bec ea
e a (DMSA EDTA) f f ead- e a ed c
● B d ead e e >80 c /dL: Re a f a e f e e, c e a ea
(DMSA EDTA)

Content Re ie Questions:

W c f ef f d b d ea c e ada f ead ?

A. Mac c c ed b d ce
B. Ba c ed b d ce
C. Dac c e
D. H e -J (HJ) b d e

W c f ef de a ec e ada f def c e c a e a?

A. L e e e, c ea ed e a fe , c ea ed TIBC, dec ea ed fe
e e
B. L e e e , dec ea ed e a fe , c ea ed TIBC, dec ea ed fe
e e
C. L e e e, c ea ed e a fe , dec ea ed TIBC, dec ea ed fe
e e
D. L e e e, c ea ed e a fe , c ea ed TIBC, c ea ed fe
e e

A 36- ea - d fe a e a e e e ec c ac a f fa e. T e a e ae
e ab e e e c e d e e f b ea a d ac ed e e e a ed ea e .
Lab a c c ca e aa d de e e e , c ea ed e
a fe , c ea ed TIBC, a d fe e e.W c f ef f d f e e
ae c e e da ?

A. Med a b d
B. A c
C. O a a e a a
D. L c e a

He a -O c 141
Get The Step 2 CK Drills Book: usmledrills.com
The USMLE Guys Step 2 CK Crash Course usmleguys.com

Ma A a
Macroc tic Anemia

Ca e :
● Re c c ; def c e c f f a e, a B12, c e; e d ea e; ce a
ed ca a e fe e e d e e ab (a , e ,
d d e, de, d ea, c e ea )

Reticuloc tosis

I a e ed b d ce e a ed RNA.

Ca e :
● Rec e f b d , e e ed a (B12, f a e) , ec e fb e
a f a a c e ea ,e e /e d e e e ,
e ca e a

Vitamin B12 Deficienc

Ca e :
● Pe c a e a, ba a c e , a ab d e , e a de ( B12
e e a )

S :
● Fa e, - eaded e , a , c e , a a ,d ea, ,
e c a e e a(e >a ), e e ea e , a ab a e

Lab :
● CBC ac c c ed b d ce , be d e e a b c e a,
e b a d e a c
● L e c c ec
● Pe ea b d ea e e e ed e
● L e a B12
● If e a B12 b de e , ea e e a c ac d a d c e e
c b be e e a ed a B12 def c e c
● P ea a b de c fac f e ee e c a e a

Da :
● A f ef c e aae e:
○ L e a B12
○ E e a ed e e a ca d c e e e e e B12 e e e
a a e
○ Re e B12 e e a ( c ea ed e c c e c , c ea ed
e b , f e e e ed e b d ea )

T ea e :
● Pa e e a B12 f e a c (a e a, e ca a fe a )a d a
a B12 f a a c d d a
● O e- e e e d c f d d a e c a e ad e e e a ed GI
ca ce

He a -O c 142
Get The Step 2 CK Drills Book: usmledrills.com
The USMLE Guys Step 2 CK Crash Course usmleguys.com

Folate Deficienc

Ca e :
● D e ( ac ff , e e ab e , f f ed ce ea ), ba a c e ,C d ea e, ce ac
d ea e, e ce eac e

S /S :
● Fa e, - eaded e , a ,c e dec e, a a ,d ea, a ce ,
e a be defec ( fe )

Lab :
● CBC ac c c ed b d ce , be d e e a b c e a,
e b a d e a c
● L e c c ec
● Pe ea b d ea e e e ed e
● L e f ae e e
● If e f a e b de e , ea e e a c ac d a d c e e
● I f a e def c e c e a c ac d e e a a d c e e c ea ed

Da :
● A f ef c e aae e:
○ L e f ae e e
○ N a e a c ac d e e a d e e a ed c e e f e f ae
e - a a e
○ Re e f ae e e a ( c ea ed e c c e c , c ea ed
e b , f e e e ed e b d ea )

T ea e :
● O a f c ac d e e a

Content Re ie Questions:

A 42- ea - d a e e e ec c ac a f fa e. Lab a MCV f 104,


e b f 9.1, a e c c ec ,a d e ea b d ea
e e e ed e .W c f ef e dc f eda f
a B12 def c e c ?

A. E e a ed e a c ac d a d e e a ed c e e
B. E e a ed e a c ac d a d dec ea ed c e e
C. E e a ed e a c ac d a d a c e e
D. N a e a c ac d a d e e a ed c e e

W c f ef c e a a e da ed f a e def c e c ?

A. E e a ed c e e
B. T b c e a
C. H e c c ec
D. H e e e ed e b d ea

He a -O c 143
Get The Step 2 CK Drills Book: usmledrills.com
The USMLE Guys Step 2 CK Crash Course usmleguys.com

A 51- ea - d fe a e e e ec c ac a f ea e e a d fa e. U
e a e f d a e ea a b de c fac . W c f e
f def c e c e e be de f ed a e ?

A. F a e
B. B6
C. B12
D. C e

He a -O c 144
Get The Step 2 CK Drills Book: usmledrills.com
The USMLE Guys Step 2 CK Crash Course usmleguys.com

N A a
Normoc tic Anemia

Ca e :
● Ea a e f c c ca e a( def c e c ) ac c c a e a (f a e, B12,
c e def c e c e ), a a ea ed c c c ac c c a e a, ea b d
● A e a fc c d ea e ( e ca e ac c c), CKD, ea fa e, e d a c
d e, a a c a e a, d
● He ( c e ce , e ed a e c ,a e e c a e a,
c a a c e c a e a, a a c a e b a, a f
eac , G6PD)

Hereditar Spheroc tosis (HS)

Ca ed b a e e e bef RBS e ba ea dc eea f a .

E ace ba Fac :
● I fec ead e a a a a( a B19), e e a , def c e c
e e a RBC d c ( , f a e, B12)

S :
● S fa e a (fa e, a , a a ), e a e , a d ce,
e e a

Lab :
● E e a ed ac a e de d e a e, a d c a ed b b , dec ea ed a b ,
dec ea ed e b e e
● E e a ed ea c c a e b c ce a
● H e c c ec
● Ne a e C b e
● Pe ea b d ea e c e
● EMA b d e dec ea ed f e ce ce f c e
● P e cfa e

Da :
● Lab c e e ed a e c , fa f HS, a d/ e c e
e e ea b d ea AND ec f a e (EMA b d , c
fa )

T ea e :
● P ea e c a e a f e e e ce fa a e
eb b e a
● E dcb d a f a eeded ( a fec )
● F ae e e a f e e ee e
● S e ec f e e ee e
● C ec ec f a c e ed a e

He a -O c 145
Get The Step 2 CK Drills Book: usmledrills.com
The USMLE Guys Step 2 CK Crash Course usmleguys.com

Warm Autoimmune Hemol tic Anemia

Ca e :
● Med (a b c ,c e , NSAIDS), a fec ,a ed de ,
fe a e d de

S :
● Fa e, d ea e e , a a , a d ce, da e, e e a

Lab :
● Dec ea ed e b a d a b , e e a ed ac a e de d e a ea d d ec
b b
● E e a ed e c c ec
● B d ea e c e c e c e
● A eC b e f I G a d/ C3d

C ca :
● E e a ed f e b e b

Da :
● P eC b e , ab c e e ca e a, a d e c f e
ca e( e c a f eac , PNH)

T ea e :
● Ta f a d e d a c f e fe- ea e a e a
● T ea de d de ( d ca a e d , ea a ed de ca ce )
● G c c c d ab
● F c ac d e e ac e
● Ve b e b a ( ec a e e a )

Cold Agglutinin Disease

Ca e :
● L fe a ed de , a ed de , fec (M. , EBV)

S ( e e c de e):
● Ac c a , Ra a d e e , ed e c a , fa e, d ea
e e , a a , a d ce, e e a

Lab :
● Dec ea ed e b a d a b , e e a ed ac a e de d e a ea d d ec
b b
● E e a ed e c c e c
● Pe ea b d ea RBC a a
● A eC b e f I M a d/ C3d
● P ec da e

Da :
● Lab c e e , eC b e f C3d a d c d a e
( 64 a 4 C) c f eda

He a -O c 146
Get The Step 2 CK Drills Book: usmledrills.com
The USMLE Guys Step 2 CK Crash Course usmleguys.com

T ea e :
● A dc d
● Se e e a e a ea ed RBC a f PLUS a a ee IVIG f eeded
● T ea de d de
● R ab a d be da e

Paro smal Nocturnal Hemoglobinuria

S /S :
● Fa e, d ea, b a ca ( a-abd a e , ce eb a e ),
ed/b ac (c a-c ed) e, a d ce, e ec e d f c ,c e a , ce
d a, a e e , e a ff c e c

Lab :
● Dec ea ed e b a d a b , e e a ed ac a e de d e a e a d d ec
b b
● Pa c e a (dec ea ed ed b d ce , e b d ce , a d a e e )
● He b a( ef e eb e a e f RBC e)
● E e a ed BUN a d c ea e(f e a ff c e c )
● E e a ed e f c e ( f e a c/ a b )
● Ne a e C b e
● F c e dec ea ed GPI-a c ed e e e a b d ce
● B e a b e d e a a

I a :
● U ed de ec a-abd a (US, CT MRI) ce eb a e (CT MRI) b

Da :
● Lab c e PNH f c e dec ea ed GPI-a c ed
e (CD59 a d CD55) e ea b d ce

T ea e :
● S a c a e ea ed c e e b (ec ab, a ab)
● RBC a f f e eea e a
● S e e a f c ac d a d f a e ac e e

Glucose-6-Phosphate Deh drogenase (G6PD) Deficienc

M a e e G6PD, c ec RBC f da e .

Ca e f O da e I :
● Med ( a e, f e , f a , f ea ), fa a bea , fec

S :
● Pa , a d ce, abd a a , da e

Lab :
● Dec ea ed e b a d a b , e e a ed ac a e de d e a ea d d ec
b b
● E e a ed e c c e c
● Ne a e C b e

He a -O c 147
Get The Step 2 CK Drills Book: usmledrills.com
The USMLE Guys Step 2 CK Crash Course usmleguys.com

● Pe ea b d ea He b de , c e c e , b e ce
● Sc ee a dc f a e f G6PD ac

Da :
● P ec f a a a e G6PD a a a e G6PD ac e e

T ea e :
● Ne a a a d ce ea ed ea e e e ca e e c a e a f
● Re a a d a da ce f b a ce ca da e
● RBC a f f e eea e a

Content Re ie Question:

W c f ef ab e c e ada f e ed a e c ?

A. Pe ea b d ea e c e
B. EMA b d e dec ea ed f e ce ce f c e
C. P e cfa e
D. F c e dec ea ed GPI-a c ed e e ea b d ce

A 36- ea - d a e e e e a c a f ed/b ac e. T e a e ae
f e a 2 da e a ced a e f a d a fe e e d fa e. T e
a e ad ed a d a e e e - ef ed e b a, dec ea ed
e e b a d a b , a d e e a ed ac a e de d e a e a d d ec b b .
L e f c e a e e e a ed a d CT abd e a e b ed. A
C b e e a e. Af e e e a da f c e e dec ea ed
GPI-a c ed e e e a b d ce . W a e e da ?

A. Pa a c a e b a
B. Wa a e e ca e a
C. C d a d ea e
D. Ma a a

A 28- ea - d fe a e e e ec c da e. S e a e f e a 24 e
e a bee e da e c .T be a af e be da ed a ac
fec a d be ea ed f a . Lab dec ea ed e b a d
a b , e e a ed ac a e de d e a e a d d ec b b ,a d a e e a ed e c c e
c .AC b e ef ed a d e a e. Pe ea b d ea e
He b d e a d b e ce . W a e a aeda f e e e f d e
e a e ?

A. C
B. Wa a e e ca e a
C. G c e-6- a e de d e a e (G6PD) def c e c
D. Pa a c a e b a (PNH)

He a -O c 148
Get The Step 2 CK Drills Book: usmledrills.com
The USMLE Guys Step 2 CK Crash Course usmleguys.com

T aa a
He b Va a :
● He b A (HbA): M c ad f f e b , ade fa e a e
c a a a b b a d be a b b ( 2 2)
● He b A2 (HbA2): Va a f e b A ee e e ca
a a e , ade f a a b b a d de a b b
( 2 2)
● He b F (HbF): P d ced a 6 ee f e a c a d e bef
a e f e e' b d fe a b d, ade f a a b
b a d a a b b ( 2 2)
● He b Ba (Hb Ba ): Ab a e f e b a c ff a a
b
● He b H (HbH): Ca ed b a f ee a a b e e ead
dec ea ed a a- b c a f e b a df a f 4 e a e

A a T a a e a:
● D ea e f a /a f e4a a b e e , e e e
e e affec ed
● A 4a a b e e a ed ead d fe a ( e fe a de e) a d
e b Ba ( e a e f a a b )
● 3a a b e e a ed ead e b H d ea e (HbH) be a c a
e a e a d a ab e ( de a e c c c a e a) c a
e ee dc a f
● 2a a b e e a ca e a a a a e a e d
c c ca e a
● 1a a b e e a e (ca e a e) a e aa d c c
RBC

Be a T a a e a:
● Be a a a e a a : B be a b e e d c be a b ( 0/ 0), e
e e d c be a b a d e e a ed d ce e be a b ( 0/ +)
e c c ca e a a e ce a d e fe- a f
● Be a a a e a e ed a: B be a b e e d c e be a b ( +/ +)
e de a e c c c a e a
● Be a a a e a : Ge e a a e (ca e a e) e d
c c ca e a ef c be a b e ea d e e ee e
d c a d ed d c f be a b ( / + / 0)

T aa e aS /S :
● A e aa dc c e : S e e a ab a e , e e a , e ,
b b a e ,b a ac f a a
● I e ad: He a e a , ea fa e, ad , d , d abe e

Lab :
● M c c c, c ca e a CBC
● RBC ab a e be a a a e a a a d HbH d ea e ( a e ce , ea d
ce , ce f a e )
● H RBC c
● L e c c ec
● Se e e a ed, e fe e e a ed

He a -O c 149
Get The Step 2 CK Drills Book: usmledrills.com
The USMLE Guys Step 2 CK Crash Course usmleguys.com

● I ca e f e : E e a ed LDH, d ec b b , a b , e a e
C b e

He bA a H -Pe f a ce L dC a a E ec e :
● A a aa e a d fe a HbH d ea e Hb Ba ( fe a a
aa e a 3-8% a e f Hb Ba )
● Be a a a e a c ea ed HbF a d HbA2

Da :
● C f ed b e e e

T ea e :
● F aa e a a e ed a, f c ac d e e a
● B d a f e a e e b e e fa 9.5-10.5 /dL ( ed ce
e a ed a e a e )a dc ea ea ea e ce (defe a )
● L a e ce (f e be a a a e a e a f )

Content Re ie Questions:

W c f ef c ca e be ee a fe a a 4
a a b f e b ?

A. O a ce e
B. Ga c
C. H d fe a
D. Ne a be defec

W c f ef c d e a c ea ed HbA2 e b
e ec e ?

A. A a a a e a
B. Be a a a e a
C. I def c e c a e a
D. S c e ce a e a

He b Ba (e a e f a a b ) de f ed e b e ec e a
a fa e b c ee . W c f e f e e ca e f e b
ab a ?

A. F a e def c e c
B. S c e ce a e a
C. A a a a e a
D. Be a a a e a

He a -O c 150
Get The Step 2 CK Drills Book: usmledrills.com
The USMLE Guys Step 2 CK Crash Course usmleguys.com

S C A a

S c e Ce A e a:
● A a ece e a f e be a b e e e c e a ed
e c e
● I c ea ed a - cc affec e a e

Ac e S /S :
● He e: H e e cc ,a a cc , e c e e a c
● Pa : Ac e c e d e, a - cc e a e de
● MSK: Dac , a a c a ec
● Ne :I c e c e, e a c e
● E e: Re a a e cc
● Re a : Re a fa c
● Ca d : M ca d a fa c
● ID: Se , e , e a

Acute Chest S ndrome

Ca e :
● I fec , a - cc , a e ec a , b , b e b , fa e b

S /S :
● Fe e , c e a , e a, ee , ae ,c , e a d e

I a :
● Ne a f ae c e ad a

Da :
● C e ad a e a f ae e e f ef :
Fe e 38.5 C, c e a ,c ,ee , a e , >2 e ce dec ea e
S O2 f ba e e, ac ea, e f acce ce f e a

T ea e :
● Pa c ae ea d
● IV f d a 1.5 a e a ce a e
● E c a e a f
● E c a b c (cef a e a c )
● VTE a ec a e e a f da a

C cS /S f S c e Ce A e a:
● MSK: O e ec , ce , e
● P :P a e e , e c e d ea e
● ID: O e e
● He e: C e a ed e ca e a
● E e: P fe a e e a

Lab :
● M d de a e a e a( c c ac c c de e d e c c ec )

He a -O c 151
Get The Step 2 CK Drills Book: usmledrills.com
The USMLE Guys Step 2 CK Crash Course usmleguys.com

● S f e : E e a ed ac a e de d e a e, c a ed b b ,a d
a b
● E e a ed e c c e
● Pe ea b d ea : S c ed RBC , H e -J b de
● He b e ec e e HbF a d e HbS ( HbA)

Da :
● Made e ef a ce dc a a , e e ec e ,
e ec c f c

T ea e :
● T ea e d ea ( c ea e e e f fe a e b )
● Va - cc e a ea ed IV f d da a d a a a e e d
● A b c a ( e c ) a e5
● B d a f a d ca ed: S a c a e a, a fa e, ac e e,
ac e c e d e
● He a e c e ce a a a (c a e)

Content Re ie Question:

W c f ef ea e da e a aef a a e c e ce
a e a e e e a ab a ab a e b c a f
e ee a e e b a d bac ?

A. N a ed ca
B. Na e
C. Ke ac
D. H d e

A 42- ea - d fe a e PMH f c e ce d ea e e e e a e c ca
c e a .V a HR 105 T 38.5 BP 144/74 SPO2 90%. EKG a d ae a,
b c e ad a e a f ae e e be a d e a
ee ea d e .W a e e da e e e a ?

A. Ac e c e d e
B. NSTEMI
C. P e a
D. Re c e d ea e

W c f ef d f e b ae c ea ed a e c e ce a e a
a d ea?

A. He b A
B. He b A2
C. He b F
D. He b Ba

He a -O c 152
Get The Step 2 CK Drills Book: usmledrills.com
The USMLE Guys Step 2 CK Crash Course usmleguys.com

Pa D
Thromboc topenia

Ca e :
● I e b c e a (ITP), b c c a a (TMA), e e ,
d - d ced e b c e a ( e a ), fec (HIV, HCV), a c ,
a B12 f a e def c e c , a ed de , d e a ed a a c a
c a a (DIC), a d d e (APS), a a c a e a, e d a c
d de , a a c a e b a (PNH)

Thromboc tosis

Ca e :
● P - e ec , fa a d de ( e a c, a a c ), a a,
fec , e a c a a c e , fa a b c

Immune Thromboc topenia (ITP)

P beI c Ca e :
● A ec d , fec

S :
● Pe ec ae/ a, e a , fa e, a e e e e b eed (GI ce eb a )

Lab /D a :
● T b c e a a e a e e a
● Da fe c

T ea e :
● F c ca b eed (da e a a c ca e d a cc e) ea
aee a f , c c c d , a d IVIG
● F e e e b eed (ca e b dec ea e b 2 /dL) ea c c c d
● F b eed aee e a 50,000 b eed aee e a
30,000, ea c c c d

Thrombotic Thromboc topenic Purpura (TTP)

T b c c a a e f def c e W eb a d fac -c ea ea e
ADAMTS13.

S :
● Fa e, abd a a ,d ea, e ec ae/ a, ea e , b eed , e ca
ab a e (c a, e, e e, eadac e, d e ), fe e , e a ff c e c

Lab :
● Pe ea b d ea c c e ( e e ce )
● Dec ea ed e b a d a b , e e a ed ac a e de d e a ea d d ec
b b
● T b c e a
● Ne a e C b e

He a -O c 153
Get The Step 2 CK Drills Book: usmledrills.com
The USMLE Guys Step 2 CK Crash Course usmleguys.com

Da :
● Pe eda ade ba ed ab f d ea e ea a
def eda a e e e ADAMTS13 def c e c a d ADAMTS13 a a b d

T ea e :
● Pa ae c a e

Heparin-Induced Thromboc topenia (HIT)

A a b d a a c e e f e a a d a e e fac 4 (PF4) a 5-10 da af e


a e a e a 1 da f e a e e.

S /S :
● T b ( ec , b a e e, a fa c , DVT/PE), b eed (GI
ac ), a a a

Lab :
● T b c e a 50% a e dec ea e aee c
● ELISA a a e f PF4- e a a b d
● P e e e ea e a a ( d a da d)

Da :
● Pe eda ade a c ca a fe a f e a ea d
b c e a, 50% dec ea e aee c ( a e ea e def e
da ), a d c f ed e ELISA e e ea e a a

T ea e :
● S e a a d a - e a a c a a ( a d ec b
b -a a ba )

Content Re ie Questions:

A a ed a e be a c a a ed e a f a ece e
b e b . O da 7 f a a ee ad aee c f 320,000
120,000. He a - d ced b c e a ec ed. W a e e be e
a a e e ?

A. C e e a aee d be 100,000
B. S e a a d cea e a c a a
C. S e a a d e a e aee e ba e e
D. S e a a d ed a e a a a ba

He a -O c 154
Get The Step 2 CK Drills Book: usmledrills.com
The USMLE Guys Step 2 CK Crash Course usmleguys.com

A 50- ea - d fe a e a e e e e a abd a a . Pe ec ae a d
aae ed a e e . Lab b c e a, dec ea ed e b a d
a b , e e a ed ac a e de d e a e a d d ec b b .C b ' e e a e.
Pe ea b d ea e c c e .W a e e da ?

A. Pa B19 fec
B. Pa a c a e b a
C. T b c b c e c a
D. I e b c e a

W c f ef ab a e c e I eT b c e a?

A. T b c e a, e e a, a e b
B. T b c e a, a WBC c , a e b
C. T b c e a, e e a, dec ea ed e b
D. T b c e a, a WBC c , dec ea ed e b

He a -O c 155
Get The Step 2 CK Drills Book: usmledrills.com
The USMLE Guys Step 2 CK Crash Course usmleguys.com

B Ta a Ta R a

B dTa f He b T e d:
Condition Hemoglobin Transfusion
Threshold

S a ca e a 10 /dL
Ac e MI 8 10 /dL
P e-e c a a e d ea e, -ca d ac e , 8 /dL
e d a ca ab e a e GI b eed
O c a e ece c e ea e c / ad a 7 8 /dL
ea
Ca d ac e 7.5 /dL
He d a ca ab e e e ca e 7 /dL

Febrile Non-Hemol tic Transfusion Reaction

Ca e:
● Re ea e f c e f d eb d ce f a f

S :
● Fe e /c 1-6 f a f a f

T ea e :
● S a f
● Ace a e a d/ a a e
● C de e ed c ff eb d d c

Urticarial Transfusion Reaction

Ca e:
● D e e e ac a e a b de

S :
● H e NO e fa e c eac ( ee , e ,a ede a)

T ea e :
● S a f
● A a e (d e da e)
● Re e a f f e ea d fa a a e e

Primar H potensive Transfusion Reaction

Ca e:
● Mec a , b e a ac e b ad a ACE- e
a f e f e ca e eac

He a -O c 156
Get The Step 2 CK Drills Book: usmledrills.com
The USMLE Guys Step 2 CK Crash Course usmleguys.com

S /S :
● P ( ) af e a f a f a 30 H d f BP c e
ba e e ce a f ed

Da :
● Da fe c ( e a a a , TRALI, ac e e c eac ,
e )

T ea e :
● N ec f c ea e ece a (c d a d e e ed af e a f )
● C de a d ACE- a f

Transfusion-Associated Circulator Overload (TACO)

R Fac :
● Ca d ac d ea e ( ea fa e), e a d f c , a e be f b d d c
a f ed, a d a e f a f , b d e , e de a d c d e

Ca e:
● V e e ad

S /S :
● Re a d e (d ea, ea, a), e e
● J a e d e , S3 a , a ae

I a :
● CXR d ff eb aea f ae ( a ede a)

Lab :
● E e a ed BNP

T ea e :
● S e e a e , a e e e e e a ec a ca
e a a eeded
● D e c e a (f e de)

Transfusion-Related Acute Lung Injur (TRALI)

Ca e:
● Pa e e d e e a e ac a ed b a f ed d c e a
a c a e

S :
● Fe e /c
● Re a d e e -6 af e a f a f e e a,
c a , e

I a :
● CXR d ff eb aea a f ae

He a -O c 157
Get The Step 2 CK Drills Book: usmledrills.com
The USMLE Guys Step 2 CK Crash Course usmleguys.com

Lab :
● Ta e e e a
● A - e a -HLA a b de

Da :
● Ac e e f e a, b a e a f a e c e ad a 6 f
a f a f WITHOUT e de ce f c c a e ad e-e ARDS
bef e a f

T ea e :
● S a f
● Mec a ca e a
● He d a c
● Pa e ca ece e a f ef e, b f a ed
● B d ba d be f ed a d b d ca ed ca TRALI ( e e
e f d b d f e ec e )

Acute Hemol tic Transfusion Reaction

Ca e:
● C e ca e ead c a bed b d ece ed b a e

S :
● Fe e /c ,fa a , f IV e , e , e b a, e a fa e,
cc e af e a f a f

Lab :
● He b e a, e b a
● L a b , e e a ed LDH a d b b ( e ab )
● P eC b e ( e a ce e ed)
● P ed PT, ed aPTT, fb e , b c e a (DIC)

Da :
● F a f a e de e ab c e c d ( ee ab e) a d
c e ca e de f ed a eda

T ea e :
● S a f
● P de e d a c da
● C ac b d ba ( e e a e a e f ece c ec b d d c)

Anaph lactic Transfusion Reaction

Ca e:
● A e ac e e a f ed d c ( e e I A-def c e a e
f a -I A a b d e a f ed d I A) e dde a e e ea e
f a ea d a eb a ce a d ba

S :
● H e ,a ede a, , ca a, e a d e a d/ ee
ec d - e af e a a f

He a -O c 158
Get The Step 2 CK Drills Book: usmledrills.com
The USMLE Guys Step 2 CK Crash Course usmleguys.com

Lab :
● Af e ea e a d a a a e e, ca e d ff a e ea e a a e
I A a d a -I A e e ( dea a e- a f a e e b a ed f
a a ab e)

Da :
● Ba ed a d e a d e f f a f ,a d a d
e e ea

T ea e :
● S a f
● E e e a c a
● IV f d e c a
● Sa e e e f e a e f ece a
● Ma a ea a ( e , ec a ca e e f ece a )

Transfusion-Associated Sepsis

Ca e:
● B d d c c a a

S :
● Fe e , c , ac ca d a, / e e ( e/fa f >30 H ) cc 0-5
af e a a f

Lab :
● P eb dc e ( a e da e )f ea f ee a e ece ed
a f a d/ b d d c ba
● Ne a e C b e , a a-f ee e b , e a e f ee e b a
● Re ea a d c f a aec ac a d

Da :
● Da c f ed e a a a db dc e

T ea e :
● I a a a e a e a e f ac e e c a f eac a d
a f -a c a ed e d e a e e a
● S a f
● He d a c e ca e
● C f c e ca e , a e b d ba d e e a c a a ed b d
d c
● Sa e c b ad- ec a b c ( a c c be a- ac a )

He a -O c 159
Get The Step 2 CK Drills Book: usmledrills.com
The USMLE Guys Step 2 CK Crash Course usmleguys.com

Content Re ie Questions:

A 64- ea - d ae PMH f ee e ace e e be ed ea e .


T e a e e fa e a d ea e b e e a e c ed a d e a
e c a .P ab e a da a a de f a e b a e f 6.1.
W c f ef e e be e a a e e ?

A. I e e
B. IV ea
C. E e
D. RBC a f

A 52- ea - d fe a e ece a RBC a f e 1 e a f e


de e a fe e ? W a e e be e a a e e ?

A. S e a f
B. T ea ace a e
C. T ea e ca b c
D. T ea d e da e

A 37- ea - d a e ece ab d a f e e e e de e e f e
b a d a da .T e a f ed, a d e a e e d e da e
c e .N e c a c ca a fe a ae ed. W a
e e da ba ed e ef d ?

A. Feb e - e c a f eac
B. U ca a a f eac
C. A a ac c a f eac
D. T a f -a c a ed c c a e ad

A 70- ea - d a e PMH f ea fa e ece ab d a f .U c e


f e a f e a e c a f ea a d e a ee e be a
e d e , S3 a ,a d a a e . A a BNP ad a cee a f
ba e e, a d c e ad a b aea a f a e. W a e e
da ?

A. T a f -a c a ed c c a e ad (TACO)
B. T a f - e a ed ac e (TRALI)
C. A a ac c a f eac
D. T a f -a c a ed e

A 49- ea - d fe a e ece a RBC a f bec e c a c a d a ea


e a d e 45 e af e e a f e a f . V a a e a e ed a d e
e feb e a 101.9, e c S O2 a 83%, a d e BP a d ed 95/54. C e
ad a d ff e b a e a a f ae .W a e e da ?

A. T a f -a c a ed c c a e ad (TACO)
B. T a f - e a ed ac e (TRALI)
C. A a ac c a f eac
D. T a f -a c a ed e

He a -O c 160
Get The Step 2 CK Drills Book: usmledrills.com
The USMLE Guys Step 2 CK Crash Course usmleguys.com

A 32- ea - d fe a e ece a aee a f a a ed a e f e d a c


ab 2 e af e e a f e a f .T e a e a e f e a d
e, e e 66/40, a d e a d e .W a e be a e af e
e a f ?

A. Ad e e e e
B. E ca b c
C. E e e d a
D. Read e a e a e ba f aee (d ffe e d )

He a -O c 161
Get The Step 2 CK Drills Book: usmledrills.com
The USMLE Guys Step 2 CK Crash Course usmleguys.com

I T b a
Factor V Leiden

M a e e ee c d fac V e , ca a ed fac V e be
e e e a c a a ac a ed e C (aPC).

S /S :
● Ve b e b (VTE), ec e fe a

C de Te f Fac V Le de :
● U ed VTE a a e <50 ea , VTE a c ca , ec e VTE

Lab /D a :
● PCR, f c a aPC e a ce a a , d ec e ca a ca a be ed
de f fac V Le de

T ea e :
● D ec a a c a a (dab a a , a aba ) a fa f VTE

Protein C Deficienc

Ac a ed e C e bef ac a c a a fac Va a d VIIIa.

S /S :
● Ve b e b (VTE), a fa - d ced ec , ec e fe a

C de Te f P e C Def c e c :
● U ed VTE a a e <50 ea , VTE a c ca , ec e VTE,
fa f VTE, a d/ a fa - d ced ec

Lab /D a :
● P e C ac e e ca be a e ed af c a a a (aPTT-ba ed a a ,
fac Xa-ba ed a a , a e e ac a e e Ca dc e c b ae
ea e e a c ac )
● D ef f a e c e a ac a fa a a e, ece
a ece ed a c a a

T ea e :
● T ca a a e e f VTE d ec a a c a a (dab a a , a aba )
a fa
● Ma a e e f a fa - d ced ec a fa , IV a
K, e a e c f ac a ed e a f a c a a ,a d e Cc ce a e (
fe f e a a f a a ab e) e e e C ac

Protein S Deficienc

P e S a c fac f ac a ed e C. P e S def c e c a a a d a
c d c e e e Cf ac a c a a fac Va a d VIIIa.

He a -O c 162
Get The Step 2 CK Drills Book: usmledrills.com
The USMLE Guys Step 2 CK Crash Course usmleguys.com

S /S :
● Ve b e b (VTE)

C de Te f P e S Def c e c :
● U ed VTE a a e <50 ea , VTE a c ca , ec e VTE,
fa f VTE, a d/ fa e S def c e c

Lab /D a :
● F ee e S e e e
● D ef f a e c e a ac a fa a a e, ece
a ece ed a fa d ec a a c a a

T ea e :
● T ca a a e e f VTE d ec a a c a a (dab a a , a aba )
a fa

Prothrombin G20210A Mutation

P b G20210A a ca e a c ea ed e b c ce a .

S /S :
● Ve b e b (VTE)

Lab /D a :
● PCR de f G20210A a a
● Ca be e f ed a a e (ac ec , fa a e a c a a e
e fe e)

T ea e :
● T ca a a e e f VTE d ec a a c a a (dab a a , a aba )
a fa

Antithrombin Deficienc

A b a a c a a a b b (fac IIa), fac Xa, a d fac IXa.

S /S :
● Ve b e b (VTE), e a e a ce

C de Te f A b Def c e c :
● K fa fa b def c e c VTE, VTE a c ca ,
c de VTE ( e ce a e ca e e e e ), ec ed e a
e a ce

Lab /D a :
● F c a a a f a aa b ac ( be d ed)

T ea e :
● VTE ea e a e ec f c, b fe a b e ace e e f ed
b LMW e a f ed b a fa f ea e ( e ca e d ec a
a c a a ed) a d a c a a a def e

He a -O c 163
Get The Step 2 CK Drills Book: usmledrills.com
The USMLE Guys Step 2 CK Crash Course usmleguys.com

● If e a fa f VTE, a e d be e ac c
a c a a - , a ,a dd e a c

Content Re ie Questions:

W c f ef e ed b a ca e e b e b d e
e a e a ce?

A. P e C def c e c
B. P e S def c e c
C. Fac V Le de
D. A b def c e c

A a e a ece de f ed DVT b d ed e a ea a a e a e c INR


a e a fa . 5 da af e a ea e e a e de e de a ca ed a ea f
a a d ec affec ea a d e .W c c d e c b e
e a c e e ce f a fa e?

A. P b G20210A a
B. A b def c e c
C. P e C def c e c
D. Be a a a e a

A a e a fa f fac V Le de e e e a e
e e a a d e a d a e a dee e b a d.
G e a e ' fa , c f ef e a a e ea e
da ?

A. Mec a ca bec
B. A
C. Dab a a
D. C d e

He a -O c 164
Get The Step 2 CK Drills Book: usmledrills.com
The USMLE Guys Step 2 CK Crash Course usmleguys.com

B D
Disseminated Intravascular Coagulation (DIC)

Ab a ac a fc a a a dfb .

Ca e :
● Bac e a c e , ca ce ce , a c a e d e a a, b e ca
c ca

S /S :
● B d f IV e c e b a e , e ec ae/ecc e , e
a e a b /e b , a /fa e (AKI/ e a fa e, a
e a e, e d f c )

Lab Ac e DIC:
● Dec ea ed a e e c , a afb e , a a fac V, a a fac VIII
● P ed PT, aPTT, b e
● E e a ed D-d e a d f b de ada d c
● Pe ea b d ea a c c e a d e e ce

Lab C c DIC:
● Paee e e dec ea ed, a, c ea ed
● Pa afb e , a a fac V, a a fac VIII ca a
● PT, aPTT, b e ca a ( ed)
● E e a ed D-d e a d f b de ada d c
● Pe ea b d ea a c c e a d e e ce

Da :
● C ca a d ab a f d c e DIC a de f ab e ca e
( a a c , fec , a c a e d e , b e ca c ca )

T ea e :
● T ea de ca e (c e ea ,a b c )
● S e ca e
● P a a c a a a e e /c a a fac (FFP) e e a
a a ed
○ If a afb e e e <100 /dL, ec ec a e
○ If a e e c <10,000/ c L, e aee a f
○ A c a a f e a e a b e b

Hemophilia A and B

X- ed d de :
He a A: Def c e c fc a a fac VIII
He a B: Def c e c fc a a fac IX

S /S :
● He a , e ca a , e a a ,e a , b eed , e a a,
GI b eed , ac a a e a e

He a -O c 165
Get The Step 2 CK Drills Book: usmledrills.com
The USMLE Guys Step 2 CK Crash Course usmleguys.com

Lab :
● P ed ac a ed a a b a e (aPTT) c c ec ed
de
● Paee c a d b e (PT) ea a a a
● Dec ea ed fac ac e e ( e aA fac VIII ac , e aB
fac IX ac )

Da :
● He aA fac VIII ac e e
● He aB fac IX ac e e

T ea e :
● F e e e/ fe- ea e / b eed ea fac c ce ae( e a
A fac VIII c ce a e, e a B fac IX c ce a e)

Vitamin K Deficienc

V a K a fa - be a c ac a a c fac ec a a ca cade.

Ca e :
● Ne b ( a e e ), fa a ab (c cfb , b a d ea e, ce ac
d ea e, f a a b e d ea e, a e e bac e a e ), f e e e
f a b c , e fa e

S /S :
● M c a b eed , e e a e , ecc e , e a a, ee a

Lab :
● P ed b e (PT), e a a a ed a (INR), a d a a a
b a e (PTT) e e e e def c e c
● Dec ea ed a K-de e de fac ( e C; b ; fac VII, IX, X)

Da :
● C ca a d ab f d c e a K def c e c

T ea e :
● S e e a a K( bc a e IV f d e a ab )

von Willebrand Disease

Pa a W eb a d fac (VWF) e a e e ad e c a e a d aee


a e a ( d f c a VWF a e a ).

S :
● M c c a e b eed (e a f d a , b eed de a ced e ,
ed/ ea e e , GI b eed ), ea b , e a a, e a

Lab :
● P ed aPTT ( e e a ), be d b c e a, a d be
dec ea ed e b d e b d

He a -O c 166
Get The Step 2 CK Drills Book: usmledrills.com
The USMLE Guys Step 2 CK Crash Course usmleguys.com

● VWF a e e e (ELISA), VWF ac e e (f c a a a ), fac VIII ac e e


(f c a a a )

Da :
● C ca a d ab a f d (VWF a e a d ac e e , fac VIII ac e e)
c e W eb a d d ea e

T ea e :
● Se e e b eed eed f a e , ea VWF c ce ae
● M b eed e , ea DDAVP

Content Re ie Questions:

A a ed a e be ea ed f e b ad ec a b c a b d
f IV e. Lab dec ea ed a e e c , dec ea ed a a f b e ,
ed PT a d aPTT, a d e e a ed D-d e . W c f ef e e
da ?

A. W eb a d d ea e
B. Dee e b
C. I e b c e a
D. D e a ed a a c a c a a

A 32- ea - d a e e e ec c a a f ef ee. T e a e a a f
ec e e a a e .A - ef ed a d e a e
f d a e ed ac a ed a a b a e (aPTT) c c ec ed
d e a d dec ea ed fac VIII ac .A e ab a e a.W c f e
f e e da ?

A. He aA
B. He aB
C. V a K def c e c
D. Fac V Le de

A 38- ea - d a e e e ec c a fe a .T e a e ae a
ece e a bee a ec e e-b eed a 1.5-2 a d ed ab a
c e ec e ca ced e. Af e ca ef - e da ed
W eb a d d ea e. W c f e f e a a e ea e a
e ?

A. Ida c ab
B. S e e a a K
C. W eb a d fac c ce ae
D. DDAVP ( a e )

He a -O c 167
Get The Step 2 CK Drills Book: usmledrills.com
The USMLE Guys Step 2 CK Crash Course usmleguys.com

MGUS, M M a, Wa Ma b a, a
P aV a
Monoclonal Gammopath of Undetermined Significance (MGUS)

Pe a a c a a a ce d de .

S :
● A a c a e

Lab :
● Se M e c ce a <3 /dL

Da :
● Lac f CRAB: Ca c ee a , Re a ff c e c /fa e, A e a, a d B e c
e ( e eae e a-def a fe a ), b e a a a ce <10%,
e M e c ce a <3 /dL

T ea e :
● M f e

Multiple M eloma

Ma a fe a f a a ce c d ce c a b .

S /S :
● A e a (fa e, d ea, a , a a ), c ea ed f fec ,b e a ,
e , a ca f ac e , e a ff c e c /fa e, e c (c f ,
b ed , e c , ea fa e)

Lab :
● M e de f ed b e a d e e e ec e , f a , a d f ee
c a a a
● E e a ed c ea e
● H e ca ce a
● E e a ed a e e c ce a
● U a a e, a , a a ed ca ( e a ca e a )
● Pe ea b d ea ea f a
● B e a b e e a ed c a b e a a a ce ( 10%)

I a :
● W eb d d e CT, MRI, eea e c ed- c e

Da :
● B e a c a a a ce 10% b c f ed a ac a PLUS
e e ce a / e da a e (CRAB: Ca c ee a , Re a
ff c e c /fa e, A e a, a d B e c e )

T ea e :
● E be a e ea ed a e a e c ce a a a
● C e ea f a a e be a e e

He a -O c 168
Get The Step 2 CK Drills Book: usmledrills.com
The USMLE Guys Step 2 CK Crash Course usmleguys.com

Waldenstrom Macroglobulinemia

C a a ac c ce ca ab a c ea e e c a I Ma d
f a eb e a d ea e.

S /S :
● Fa e, e , ea , a e a, fb df / e, e
c de ce f fec , e c (c f , b ed , e c ,
ea fa e), e a e a , e e a , ade a

Lab :
● Dec ea ed e b a d e a c
● Pe ea b d ea ea f a
● E e a ed e c e e
● Se e e ec e a d e f a c a I M
e
● B e a b f a b da d a ac d ce

Da :
● Ab a c ea e e c a I M
● B e a b 10% f a b c e a ac d
a a ce d ffe e a (f d b e abec ae)

T ea e :
● M f a a c a e
● S a c a e ea ed ab a d c e ea

Pol c themia Vera

C c e fe a e e a e e e a ed ed b d ce a .

S /S :
● P ,e e a a, e e , e e a , b , a e
c a e ,e a c a ( e c ce d ea e, a d de a e )

Lab :
● E e a ed RBC , a e e , a d WBC
● Dec ea ed e e e c ce a
● B e a e ce a , ab e ce f a ab e
● JAK2 a de f ed a a ea a

Da :
● I c ea ed ed b d ce a
○ W e H b >16.0 /dL, e H b >16.5 /dL
○ OR e e a c >48%, e e a c >49%
● B e a b e ce a f a 3 ce e
● JAK2 V617F JAK2 e 12 a OR dec ea ed e e e e e

T ea e :
● T ea e c eb
● L -d e a

He a -O c 169
Get The Step 2 CK Drills Book: usmledrills.com
The USMLE Guys Step 2 CK Crash Course usmleguys.com

● H d ea f a e e 60 f b

Content Re ie Questions:

A 75- ea - d a e e e ec c ac a f b e a . Lab ab a e
c de e ca ce a, e e a ed c ea e, a d a a e, a , a a ed ca ae
ee . O e ea b d ea ea f a ed. S e e a e e
c ed- c e e a e eb ae a d e . W c f e f f d
e be de f ed a a e e e a ?

A. M e de f ed b e e e ec e , f a , a d f ee
c a a a
B. G e a c e ce f a d e b
C. B fb c e a e c cc
D. D a ec a e a a ab e f e a ed, a d ae e a b de

A 41- ea - d a e e e e a b a e a d .T e f e
a d e e a ,b e e e ab a e e a e ae ed.
T e a e a e d e f b d a d abd a e a e e a ed.
Lab e e a ed RBC , a e e , a d WBC a d f e - dec ea ed e
e e c ce a a db e a b e ce a , ab e ce f
a ab e .W c f ef e e ae e be a ed a a ea a ?

A. BCR
B. ABL
C. HBA1
D. JAK2

W c f ef e ea b d ea f d e be ee a a e
Wa de ac b e a?

A. Sc c e
B. B e ce
C. R ea f a
D. Ta e ce

He a -O c 170
Get The Step 2 CK Drills Book: usmledrills.com
The USMLE Guys Step 2 CK Crash Course usmleguys.com

L aa T L S
Acute L mphoblastic Leukemia

M c c d d ca ce ( ea a e 2-5).

S :
● Fe e , e ec ae, a/ea b , a , e a e a , e e a ,
ade a ,b e a , e

Lab /D a :
● Le c , b e dec ea ed e b a d aee e e
● B d ea ba
● F c e c e e ec e
ba ( a e f d ea e - CD19, c a c CD79a, c a c
CD22, TdT - a d ab e ce f e d ea e)
● B e a b ed a ba

T ea e :
● C e ea

Chronic L mphoc tic Leukemia

M c e e a US ad .

S /S :
● M a e a a c; e ce ca ade a ; fe fe e ,
ea , e , fa e, e e a , e a e a

Lab :
● Le c ( c a d e e a), be d dec ea e aee
a d e b
● H a a b e a
● Pe ea b d ea c a d d e ce
● F c e e ca a (CD5, CD19, CD20, CD23, a aa d
a bda b c a )
● B e a b eeded f d a

Da :
● Pe ea b dB c ec 5000/ c L f 3 , e ea b d
ea a e-a ea a c e .C a B c e c f ed
f c e de a CD5, CD19, CD20, CD23, a a, a bda
b c a

T ea e :
● Ea a ea a c CLL: D ea e
● C e e a , a e ed d ea ,a d ea ba ed d d a e e c
a f ca ( ea e c a e)

He a -O c 171
Get The Step 2 CK Drills Book: usmledrills.com
The USMLE Guys Step 2 CK Crash Course usmleguys.com

Acute M eloid Leukemia

S /S :
● Fa e, c ea ed fec , a , e ec ae/ecc e

Lab :
● WBC ca be dec ea ed, a, f d e e a ed
● N c c, c ca e a
● N a /dec ea ed e c c e c
● T b c e a
● Pe ea b d ea e ba Auer rods
● M e e da e eac e a ca e , ed de f e dba
● F c e CD13, CD33, CD34, CD117, a d HLA-DR
● B e a b e ce a a e/ d ffe e a ed ce
e a a ce e (b a a e 20% f a ce f d eb e a )

Da :
● Ba f a e 20% f a ce f d eb e a b
● B a ce e de ce f be f e d (A e d , e
e e da e, e c a e d ca e d )

T ea e :
● C e ea ( d c ea c a ab e a d da bc )

Chronic M eloid Leukemia

M e fe a e e a a c a ed e BCR-ABL1 f e e. Ca ed b a
P ade a c ed e ec ca a ca be ee c e 9 a d 22:
(9;22)( 34; 11).

S /S :
● Of e a a a c, e e e fa e e , e ec ae, e ce e
ea , abd a f e , a d/ e e a

Lab :
● Le c ( a ab e 100,000/ c L), b c , a d dec ea ed
e b
● Pe ea b d ea a e e ce e c e c a ed a e
e a e c e , ab e ba a, a d e a
● Le c ea a e a a e (LAP)
● B e a b a c c e a a
● Ka ca be ed de f P ade ac e
● F e ce ce b d a (FISH) a a ca be ed de f e BCR-ABL1
f e e
● Re e e a c e a e c a eac (RT-PCR) ca be ed de f
BCR-ABL1 f RNA

Da :
● C f ed a de f P ade ac e f e ce ce
b d a (FISH) a a de f e BCR-ABL1 f e e OR e e e
a c e a e c a eac (RT-PCR) de f BCR-ABL1 f RNA

He a -O c 172
Get The Step 2 CK Drills Book: usmledrills.com
The USMLE Guys Step 2 CK Crash Course usmleguys.com

T ea e :
● T e a e b ( a b, b, da a b, b b)

Tumor L sis S ndrome

Ra d f ce e e e ea e f a e a f a , c ac d, a d
a e.

C ca Ma fe a :
● Ac e d e ,a a , ea , e e , e a , a ea/ ,da ea

Lab /D a :
● Lab a d e def ed a a 25% c a e ba e e 2 e f
ef ab a e a a e ece c c e a : I c ea e c ac d,
a , ; dec ea e ca c
● C ca d e def ed a ab a d e e f
ef : Se e, e e a ed e c ea e c ce a 1.5 e e f
a ea e , ca d ac a a/ dde ca d ac dea

Pe e :
● De e d ( ab ab a e e e bef e c cd a d b de )
ca a e f a ca d ac , da , a b ca e, a d
f e e ab

T ea e :
● Ad ICU c ca d ac
● Ra b ca e, d e c, IV f d ( e e c ac d c a f b e ), a d d a
( e ce a, de e d e e )
● Re c a e a e, ea a e b de ( e e a e ) d a
( e a e a, de e d e e )
● Ca c c ae( ea ca ce a, ab e ca d ac e b a e e ae a
>7.0)
● IV a d de e, a b e , d b ca b a e, d a ( e ae a
de e d e e )

Content Re ie Questions:

A a e ac e b a c e e a ad ed e a ece e ea e . 3
da af e a c c ea e a e a a ea a d a d bec e
e a c. Lab a d a c c ea e e c ac d e e a d c ea eee a 2 e e
e f a.W c f ef a a a e ea e da f
a e c d ?

A. Ra b ca e
B. IV f d a a e e
C. D a
D. Ke ac

He a -O c 173
Get The Step 2 CK Drills Book: usmledrills.com
The USMLE Guys Step 2 CK Crash Course usmleguys.com

A 52- ea - d a e e e a a d e a ea f e ec ae ecc e . Lab


a f d e e a ed WBC c , b c e a, a d c c a e a. O
e ea b d ea e ba A e d ae b e. W a e e
da ?

A. Ac e ba c e e a
B. C c ba c e e a
C. Ac e e d e e a
D. C c e d e e a

A a a c a e de f ed e ab a a a WBC c f
120,000/ c L. He efe ed e a / c eef e - b a ed. A
b e a b a c c e a aa da e e c a ef ed
c de f e e e e ce f a P ade ac e (9;22)( 34; 11). W a e
e da ?

A. Ac e ba c e e a
B. C c ba c e e a
C. Ac e e d e e a
D. C c e d e e a

He a -O c 174
Get The Step 2 CK Drills Book: usmledrills.com
The USMLE Guys Step 2 CK Crash Course usmleguys.com

L a
Hodgkin L mphoma

S /S :
● L ade a , fe e , ea , e , ,d ea, e e a a ,
c

Lab :
● Dec ea ed a e b
● Dec ea ed, a, c ea ed aee c
● L WBC c ( e a e a a d/ e a)
● N a e e a ed ESR
● E e a ed LDH
● I e ca a CD30

I a :
● C e ad a de f ed a a a ,f ed b c e CT, a d/ PET
de f a e f b f e ea ade a ea e c ab e
de de f ed

B :
● E c a/ c a e ea de b

Pa :
● H d /Reed-S e be ce (" ' e e ) a ea ce e a a ed
c ea be

Da :
● H d /Reed-S e be ce ded b fa a f ae
de
● I e e CD30

T ea e :
● C e ea ad a ea f ea a ea dc b a c e ea f
ad a ced a e

Non-Hodgkin L mphoma

S /S :
● L ade a , fe e , ea , e , e a e e a

Lab :
● Dec ea ed e b
● N a e e a ed ca c
● E e a ed LDH

I a :
● U a d a d CT ed de f de ab e f da cb

He a -O c 175
Get The Step 2 CK Drills Book: usmledrills.com
The USMLE Guys Step 2 CK Crash Course usmleguys.com

B :
● E c a/ c a e ea de b

Pa /D a :
● H ca a e f e e ( d a /f c a , d ff e)
● I e e (f c e de f d ffe e da e )
● C e e c de ( a e f e ce ce b d a )
● M ec a a a (PCR, DNA e e c )

T ea e :
● C e ea (R-CHOP: R ab, c c a de, d bc ,
c e, ed e)

Staging Non-Hodgkin and Hodgkin L mphoma

Ea S a e:
● S a e I: S e de e ee a a c a e a
de e e
● S a e II: T e de e e a e de f e d a a OR a
de e c e a a c a e e

Ad a ced S a e:
● S a e III: L de e b de f eda a ed
● S a e IV: N -c e a a c e e ( ea a )

Content Re ie Questions:

A a e -H d a f d PET ca a e e e f ce ca ,
ac a c a , a d a de a e a a c e e de f ed.
W a e a ae a ef a e ba ed e ef d ?

A. S a eI
B. S a e II
C. S a e III
D. S a e IV

W c f ef ce e a ea a ca eda fH d
a?

A. M e d b a ce A e d
B. Reed-S e be ce
C. C d c e
D. P e e ce f e

He a -O c 176
Get The Step 2 CK Drills Book: usmledrills.com
The USMLE Guys Step 2 CK Crash Course usmleguys.com

A 39- ea - d fe a e e e e a ac a f fe e . S e a e f e a 3
da e a ad fe e , ea , a d e a 10- d e a e e e
a . Pa e ac a c a ade a ed e a a dc f ed CT
ca be e ed de . W c f e f d fb
a ae e a aef beH d a?

A. L e c e b
B. B e a b
C. E c a ac a c a de b
D. F e eed e a a f ac a c a de

He a -O c 177
Get The Step 2 CK Drills Book: usmledrills.com
The USMLE Guys Step 2 CK Crash Course usmleguys.com

Infectious Disease
A I Da a (Wa )
Norovirus

Pea T a :W e

Ta :
● C a a ed f d ae,f e , feca - a e, a b ed e f

I c ba T e:
● 24-48

S :
● Na ea; b d , b ( e ); ae b d da ea;
abd a a ; a a ; fe e ; a d eadac e

O b ea :
● Sc /da ca e , c e , e a a

Da :
● C ca d a

T ea e :
● S e ca e

Rotavirus, Enteric Adenovirus, and Astrovirus

Pea T a :W e a d

Ta :
● C a a ed f d ae

I c ba T e:
● 10-72

Pa e P a M Affec ed:
● C de a d c ed

S :
● Wa e da ea, , abd a ca , fe e , a e a, eadac e, a a

Da :
● C ca d a

T ea e :
● S e ca e

I fec D ea e 178
Get The Step 2 CK Drills Book: usmledrills.com
The USMLE Guys Step 2 CK Crash Course usmleguys.com

Enteroto igenic E. coli

Ta :
● Feca c a a ff d ae

R Fac :
● Ta e e ce- ed a ea , c e

I c ba T e:
● 1-3 da

S :
● Wa e da ea, a ea, a , d a 1-5 da

Da :
● DNA be de f e e

T ea e :
● S e ca e

Clostridium Perfringens

Ta :
● T - ed a ed, f db e ( ea , ) fe e e c e ed
e a a /ca e
● S e ca a dc e ea e a d ca fe a e e f d e
ed

I c ba T e:
● 6-24

S :
● Wa e da ea, abd a ca
● S e e 24-48

Da :
● Def eda a e c e

T ea e :
● S e ca e

Clostridioides Difficile

R Fac :
● Rece a b c e (c da c ,f e , ce a ), ece
a a , a e >65, b e

S M d/M de a e D ea e:
● 3e de f a e d a ea 24 , a a dca f e e abd e ,
a ea, fe e , a e a

I fec D ea e 179
Get The Step 2 CK Drills Book: usmledrills.com
The USMLE Guys Step 2 CK Crash Course usmleguys.com

S Se e e D ea e:
● F a c e c c , e , e ac , WBC c >15,000 ce / L,
e e a ed c ea e 1.5 /dL, e a, ac c ac d , ab e a

C. d ff c e ec e ce c f ea e .

Da :
● N c e c ac d a f ca e (NAAT) f C. B e e OR e f C.

T ea e :
● N e e e d ea e ea ORAL a c c f da c
● Se e e d ea e ea a a c c a d de e e f e d ca ed
● Rec e d ea e ea f e de a a c c , ea da d
b e e ec e ce feca c b a a a a ( f a a ab e a ea e
fac )

Giardia Lamblia

Ta :
● C a a ed ae f d, feca - a a

A -R I d d a :
● Ca e e de e ,c de da ca e ce e , ece e

I c ba T e:
● 7-14 da

S :
● Wa e da ea/ ea ea, abd a ca /b a ,fa e ce, a ea,

Da :
● N c e c ac d de ec a a ,a e de ec a a , c c

T ea e :
● T da e a a de

Cr ptosporidium Parvum

Ta :
● C a a ed ae, a e , e e ab e , f , a e ed ,
e - - e ,a a e e

I c ba :
● 2-28 da

R f Se e e D ea e I c ed I d d a :
● HIV fec , a a a ec e , e def c e c e

S :
● Wa e da ea, c a abd a a , a ea, a e a, fe e

I fec D ea e 180
Get The Step 2 CK Drills Book: usmledrills.com
The USMLE Guys Step 2 CK Crash Course usmleguys.com

Da :
● D ec f e ce a b d e , PCR e ,e e a a (EIA )

T ea e :
● I e ef c (ART ea HIV, ed c e d e
a a a , e c.) a d a a de

Listeria Monoc togenes

Pea T a :S e

Ta :
● C a a ed c d c ea , d , f c ee e , ,f

R Fac :
● Pe a e , e ae , e ed, e de ( a ca e e
a e d ea e e e )

I c ba T e:
● 6 10 da ( ca 24 )

S f Feb e Ga e e :
● Fe e , a e d a ea, a ea/ , eadac e, a a /a a a , a
2 da e

Da f Feb e Ga e e :
● C ca d a , ca c e e ec e ed a

T ea e :
● S e ca e

Content Re ie Questions:

Af - e ec e ce f C d de d ff c e fec d be a ae ea c
f ef e e ?

A. O a a c c
B. IV a c c
C. O a c da c
D. IV c da c

A 29- ea - d a e e e ec c f ae da ea a d ea ea,
abd a b a , a d f a e ce. He e ca eR c M a 8 da .
W c f ef a e e ca e f ?

A. C d ef e
B. G a d a a b a
C. C d de d ff c e
D. L e a c e e

I fec D ea e 181
Get The Step 2 CK Drills Book: usmledrills.com
The USMLE Guys Step 2 CK Crash Course usmleguys.com

W c f ef f d ca a c a ed a f e a
c e e ?

A. C d c ea
B. P
C. S f c ee e
D. H e

I fec D ea e 182
Get The Step 2 CK Drills Book: usmledrills.com
The USMLE Guys Step 2 CK Crash Course usmleguys.com

A I Da a (I a a )
Nont phoidal Salmonella

Ta :
● C a a ed f d (e , , ,fe d ce, ea , f ) c ac
a a (e e , e )

I c ba T e:
● 8-72

S :
● D a ea, fe e , a ea/ , abd a ca

Da :
● Def eda a c e da a ade c ca

T ea e :
● M d de a e d ea e a c ca e
● Se e e d ea e ea ed c f ac e f ac

Camp lobacter

Ta :
● C a a ed ,c a a ed ae ce, c ac a a ca e
a e ( e e )

I c ba T e:
● 1-7 da ( a 3 da )

S :
● D a ea (f e e c >10 e de a da , b d da 2-3 f ),
ca e b ca abd a a , f de e ae e eac ea
G a -Ba d e (GBS)

Da :
● S c e( e a e eac ea GBS a c e be
e a e)

T ea e :
● M d- de a e d ea e a c ca e
● Se e e d ea e ea ed a c

Shigella

Ta :
● Pe - - e ,c a a ed ae f d(a e e ab e )

I c ba e:
● 1-7 da

I fec D ea e 183
Get The Step 2 CK Drills Book: usmledrills.com
The USMLE Guys Step 2 CK Crash Course usmleguys.com

S :
● Fe e , f e e a eb d / c c a da ea, abd a ca ,
e e

C ca :
● Reac e a , c e ac , e a b c ,c c ef a , ec a
a e, bac e e a, e c e c d e

Da :
● S c e

T ea e :
● A c cef a e (de e d a c ba ce b e )

Enterohemorrhagic E. coli or Shiga To in-Producing Escherichia coli (STEC)

Ta :
● Ground beef, e - - e

I c ba :
● 1-8 da

S :
● B d da ea; abd a a ;c ca f e c e c d e (HUS)
ee a a , a c a E. O157:H7

Da :
● Rec a ab f a - d c E , E. O157:H7
S b -MacC e a a

T ea e :
● AVOID a b c e, c ea ed f HUS a b c e
● S e ca e cc a d

Entamoeba Histol tica

Ta :
● T a e /f de e c e c c a a ed f d a e , feca - a
c ac fec ed d d a

I c ba e:
● 1-3 ee

S :
● B d da ea, abd a a , fe e , e
● C ca c de c e ac ,f a c b e ec / ef a ,
e

Da :
● S c c , a e e , PCR

I fec D ea e 184
Get The Step 2 CK Drills Book: usmledrills.com
The USMLE Guys Step 2 CK Crash Course usmleguys.com

T ea e :
● Me da e a c

Yersinia enterocolitica and Yersinia pseudotuberculosis

Ta :
● Ra pork products, a e ed

I c ba :
● 4-6 da

S :
● D a ea, abd a a ( e d a e dc ), fe e a ea/ , a ,
bac e e a

Da :
● C ef , a ea e da e , b d, e ea f d, e e e c
de

T ea e :
● S e ca e da d de a e ca e
● C f ac f ca e e ee

Vibrio Parahaemol ticus

Ta :
● Ra seafood and shellfish ( e , ,f )

I c ba :
● 1-3 da

S :
● D a ea ( f e b d ), abd a ca , a ea/ , fe e
● Ma a ca e d fec f e de ed c a a ed ae a d
bac e e a e ca e f e

Da :
● P e c e PCR e

T ea e :
● S a c ca e e e e d de a e ca e
● Se e e ca e ea d c c e

I fec D ea e 185
Get The Step 2 CK Drills Book: usmledrills.com
The USMLE Guys Step 2 CK Crash Course usmleguys.com

Content Re ie Questions:

A a e e e a 3-da f d a ea. T e d a ea fe e 8-10 e de


e da a d c a ac e ed a a eb d a dc a c .W c f e
f a e c e e e a ?

A. S e a
B. G a d a a b a
C. C d ef e
D. N

A a e e e e e a da f b d d a ea a d abd a a .T e a ea
f ea a a b e a a e a a 5 da a . Pa e a e e f c e
S b -MacC e (SMAC) a a a d E. O157:H7 de f ed a e a e
e bef e a e fec d a ea. W a e a a e ea e f a
a e e ef d ?

A. A c cef a e
B. T e - fa e a e
C. C f ac
D. IV ee a cc a d

A a e e e 1 da f b d d a ea, abd a ca ,a d a ea. S e e 2


da a a c ed a e .W c f ef a e e
e bef e c d ?

A. S e a
B. Ca bac e
C. C d ef e
D. V b a a ae c

I fec D ea e 186
Get The Step 2 CK Drills Book: usmledrills.com
The USMLE Guys Step 2 CK Crash Course usmleguys.com

S a Ta I
Chlam dia

O a :C

S Fe a e Pa e :
● Ce c : M a a c, c e e d ce ca d c a e, b eed ,a d
ede a
● Ue :Fe e c a dd a
● I fec a ead e c fa a d ea e (PID)

S Ma e Pa e :
● Ue :M c d a e d c a e, d a
● E dd : U a e a e c a a , ede a, d ce e

E a e a S :
● P c :A a c( a fe a e) bea ec a a , d c a e,
e e , b eed
● C c :N - e e e a ec

Da :
● N c e c ac d a f ca e (NAAT) f e f -ca c e e a ab
a e a d NAAT f a a a ab fe a e e a
● NAAT f a ec a ab f c f c a da ec ed
● NAAT f a c c a ab f c c f c a da ec ed

T ea e :
● A c

Gonorrhea

O a :N

S Fe a e Pa e :
● Ce c : M a a c, b a a c e d c a e, f ab e
ce ca c a
● Ue :U a a , e c fe e c
● I fec a ead e c fa a d ea e (PID)

S Ma e :
● Ue :C e c e d c a e, d a
● E dd :U a e a e c a ede a a d a

E a e a S :
● C c : P ed a fa , e d c a e, e b a ede a ead
c ea ce a beb d e
● P a : Pa , e da e , a d/ ce ca ade
● P c : P ed ae e a e e e . A ec a a , f e ,
b eed , c e d c a e, a d/ e e

I fec D ea e 187
Get The Step 2 CK Drills Book: usmledrills.com
The USMLE Guys Step 2 CK Crash Course usmleguys.com

Da :
● N c e c ac d a f ca e (NAAT) f e f -ca c e e a ab
a e a d NAAT f a a a ab fe a e e a
● NAAT f a ec a a ea ab f c / a a d ea ec ed
● Ga a a dc e fc c a d c a e ec ed c cca c c

T ea e :
● Cef a ea da c

Disseminated Gonococcal Infection

U a Pe e a E e:
● P e a ( a a e c; ee , , a e )
○ U a e c
● Te , de a ,a d a a a e a
○ U a e c (fe e , c )

W -U :
● B dc e
● S a f da a a dc e
● U e a , ec a , ,a d a ea c e c ac d a f ca e (NAAT )

Da fD e a ed G c cca I fec :
● P e N. e e ce b d, a f d,

T ea e :
● Cef a e a c d c c ea d da a e f e c

Genital Herpes Simple Virus (HSV)

Pa e : HSV-1 a d HSV-2 ( ec )

S :
● I a e e a :M e a f a d c e a ce / e c e a
e e a ba e, a ade a ( a f ), d a, e c
(fe e , eadac e, a a )
● Rec e ce: Le e eea d e d a f , e fe e c
. Ma e d a ( e ca ed d ) bef e
f a f ce / e c e

Da A f eF :
● Viral culture: Sa e f e c a f dc ec ed a d a ed ec e, e
de f ed a a b d a
● HSV PCR: De ec f HSV DNA
● D ec f e ce ce a b d
● Se c e : De ec fa b de HSV

T ea e :
● Va ac c ac c fa c c ( -c a e)

I fec D ea e 188
Get The Step 2 CK Drills Book: usmledrills.com
The USMLE Guys Step 2 CK Crash Course usmleguys.com

Human Papillomaviruses (HPV)

Pa e :H a a a e

P beC d Re f HPV I fec :


● N e a a
● Ge a a (HPV 6 a d HPV 11)
○ S e e a e e
○ M e e e d e e (f a , d e- a ed, b a ed, 1 e ea c ,
-c ed, e, b , , e e ed)
● Ca ce c d ce ca , a, a a, a ea , a a , e e

HPV Vacc e:
● Ma e a d fe a e a e 9-26 ea d ( dea f e a e c e)
● C e ad ed a d HPV acc a d e a c beca e f ed afe
f a
● Vacc a ed fe a e f ce ca ca ce c ee ec e da

S philis

O a :T

Ea S :
● P a
○ C a cef a
● Sec da
○ De e ee af e c a c e f a
○ S e c (fe e , a a , eadac e, a , e )
○ Ade a ( a e de bbe e a ed de e ce ca ,
a a , a , a d fe a de )
○ Ra (palm and sole e e , d ff e, ac a a a,
a de e e )
○ C d aa aa( a ea a c a c e)
○ E a c a
○ A ec a
○ He a (e e a ed a a e a a e)

La e S :
● Te a
○ 15-30 ea af e ea ed fec
○ A ce d ac c a a d a a a da c a e e a
○ Ne (affec CNS e , e, e e a a e , a d abe
d a )
○ G a

T ea e :
● Pe c e d a da d ea e f a a e f

I fec D ea e 189
Get The Step 2 CK Drills Book: usmledrills.com
The USMLE Guys Step 2 CK Crash Course usmleguys.com

Content Re ie Questions:

A a e e e e a f e ee a d ef a e .S a f d
a a c f e e ce f N. e a f d. W c f ef
a a a e ea e ?

A. IV cef a e
B. O a a c
C. I a-a c a de a e a e ec f e c
D. J da a e f e c

W c f ef a f e a ?

A. C d aa aa
B. A ce d ac c a ad aa
C. Tabe d a
D. C a e a

W c b c e a ae a eda fc a d a?

A. E e- ed be a a (ELISA)
B. D ec f e ce ce a b d
C. I a a
D. N c e c ac d a f ca e (NAAT)

I fec D ea e 190
Get The Step 2 CK Drills Book: usmledrills.com
The USMLE Guys Step 2 CK Crash Course usmleguys.com

I F a R Ta
Trichomonas

Pa e :T

S :
● S ea a c; ee - e ,f e d c a e; f d ; ;b
a ; a fe e c a dd a; d ae a; c a b eed ; cae
e a e f ce / a a

Da :
● M e c ad e
● Va a H >4.5
● N c e c ac d a f ca e f T.

T ea e :
● Me da e

Bacterial Vaginosis

Pa e : Fac a e a ae be e ac L ec e

S :
● M a a c; a d ff- e( a ) a a d c a e, ea a f d .
T ca a e a d fa a

Da a e A e C e a (a ea 3 f ef c e a):
● C a ac e c a a d c a e( , a - e, e e )
● Va a H >4.5
● P e ff - a e e
● C e ce e e

T ea e :
● Me da e c da c

Candida Vulvovaginitis

Pa e :O e fC C

R Fac :
● A b c e, d abe e e , c ed ae

S :
● V a ;b ; fa a /e e a; e, c ,c a e c ee e- e
d c a e; d

Da :
● Candida on et mount ( e d ae)
● Va a H a a (4.0-4.5)

I fec D ea e 191
Get The Step 2 CK Drills Book: usmledrills.com
The USMLE Guys Step 2 CK Crash Course usmleguys.com

T ea e :
● F c a e

Pelvic Inflammator Disease (PID)

I fec f e e e a ac ( e , fa a be , a d/ a e ).

Pa e :N ,C ,a dM
c

S :
● Fe e , c , e abd a a , a a d c a e, e e a b eed

W -U :
● Pe a c e
● Mc c f a a d c a e
● NAAT f C. ,N ,a dM
● HIV e
● Se c e f

Da :
● Pe eda ade ba ed a e f be e a ac ea d
e e e c e abd a a a d e e e ce f ce ca ,
e e, ad e a e de e b a a e ce a

T ea e :
● Cef a ea dd c c e f ea ed a a a e
● Sec d- e e a ce a (cef )a dd c c e f a e

Perihepatitis (Fit -Hugh Curtis S ndrome)

C ca f PID ee a e e e e ce :
● I fa a f e e ca e
● I fa a f e e ea face f e a e e ad a

S :
● RUQ e c abd a a de ( efe ed) e c a

N ab e Lab :
● A a fe a e ca a d e e a ed

La a c :
● "V " ad e ( ac fb e da e)

Tubo-Ovarian Abscess (TOA)

P bec ca f PID. Sa e a PID ac e abd e a d f e


f TOA ed.

Pa e :U a c ba

I fec D ea e 192
Get The Step 2 CK Drills Book: usmledrills.com
The USMLE Guys Step 2 CK Crash Course usmleguys.com

Da :
● C ca d a ed e a a e ee d a cc e af PID fa a
ad e a a a ( a US)
● Def e da ed e ca ced e a c f TOA e e ce

T ea e :
● A b c ea a e cef e a cef d c c e
● If e e , e , fa a f ee ec ed ed TOA
f e e ed a e ca e e a d ab ce d a a e

Content Re ie Questions:

W c f ef c e a c e ada f bac e a a ?

A. T , a - e, a a d c a e
B. Va a H f 4.0
C. P e ff-a e e
D. C e ce e e

W c f ef ed ca effec e e ea e fa a a c a
fec ?

A. Me da e
B. C da c
C. D c c e
D. Cef a e

W a e be ee a a a d c a e e a fe a e a e
e e c c a ;b a a ;a d e, c , c a e c ee e- e
d c a e?

A. P e d ae
B. T c ad
C. C e ce
D. A eb c e

I fec D ea e 193
Get The Step 2 CK Drills Book: usmledrills.com
The USMLE Guys Step 2 CK Crash Course usmleguys.com

HIV
HIV Sc ee :
● T e fac d be c ee ed a ea a a , dea e e 3-6
● R fac c de:
○ Me a e e e a e HIV a e e HIV a

○ Se e ( e a e e f d )
○ I ec -d e
○ Pe a e e a e HIV a , a e HIV e,
a eb e a, ec d
● F a e fac :
○ S e HIV c ee ef ed d d a a ed 13 75
○ Sc ee f e a a e ,d EACH e a c

Rec e ded Sc ee A a :
● 4 e ea c b a HIV-1/2 a a (de ec HIV 24 a e a d HIV
a b de )
○ P e e c f ed HIV-1/HIV-2 a b d d ffe e a a a

Ac e S a c HIV I fec :
● Ma a e a a ca dd e e e ce ac e a c fec
● T e e e e ce ac e a c fec a de e 2-4
ee f e e
S a e e e a a d c de: Fe e , ade a , a , a ,
eadac e, a a/a a a, d a ea AND e c b c a ac e c painful
mucocutaneous ulcers (painful, found in the mouth, anus, penis, or esophagus
ith sharpl demarcated borders and hite bases encircled b er thema)

Da f Ea HIV I fec :
● Da e e fb HIV c ee a a a ad e
● Ea HIV fec ca c de a ad f >100,000 c e / L
● CD4 c fe a e d a d e f ea e a, CD4 ce c
eb d

C c HIV I fec :
● Pe d f e c e a d ea fec a d bef e d a f AIDS
● M e a a a c, e de e e e e e a ed
ade a
● C bdc d de e a a e a e a e c c HIV
○ Ca d a c a d ea e
○ K d e d ea e
○ Ma a c
● S a d c e b a e fec ae ec / e ee c d :
○ O a ea a a ca d d a
○ Bac e a f c
○ O a a e a a
○ Se e e HSV, VZV, CMV, a d HPV fec

D a a a d Od a a HIV Pa e :
● Be a e :E ca e ea e f c a e

I fec D ea e 194
Get The Step 2 CK Drills Book: usmledrills.com
The USMLE Guys Step 2 CK Crash Course usmleguys.com

● If e e f c a e: U e e d c b

B Re :
● If HSV de ec ed e ea e f c ce ac c
● If CMV de ec ed e ea e f c ce a cc
● If a ( d a c) ce de ec ed, ea a ca ( e d a e e
ed ca de e d ca e)

AIDS D a :
● W e a HIV- fec ed a e a e e:
○ CD4 ce c <200 ce / c L
○ A AIDS-def c d :
■ M
■ P e a
■ E a ea ca d d a
■ D e a ed/e a a a
■ T a f eba
■ E a a c c cc
■ HIV a d e

T ea e :A e a T e a (ART)
● ART b dec ea e e a RNA e e a d c ea e CD4 T ce e e
● Pa e d de d e a ce e e e e ART a a e, b
e f e ee f e e d be a ed ASAP:
○ D e a e f e e e c ab e a d e
○ R a -b ed da a e f a de e e c ab e a d e
○ Bce a , e f a afe a de, e c ab e e e

A e a T e a C a f ca :
● N ce de a d c e de e e e a c a e b : (Ad e e eac :
M c d a c , a fe a e ea e a , a c ea , ac c ac d
d e)
○ Te f a afe a de
○ E c ab e a d a d e
○ Abaca
○ Zd d e
● N - c e de e e e a c a e b : (N c a ec f c ad e e eac )
○ Efa e (ad e e eac : e ca d c a c de effec , QT
a )
○ Efa e
○ D a e
● Protease b : (Ad e e eac : Me ab c d c a e de a,
d e a ce, e ce a, d abe e )
○ Da a
○ Aa a a
● Integrase b : (Ad e e eac : We a )
○ Ra e a
○ D e a
○ Bce a

I fec D ea e 195
Get The Step 2 CK Drills Book: usmledrills.com
The USMLE Guys Step 2 CK Crash Course usmleguys.com

Content Re ie Questions:

A ec d e c e ec c fe e , ade a , a ,a d a f
ce . Y ec e a e a be ffe f a ac e a c HIV
fec .U ec f e ce , a c a ac e c ae e ee?

A. S a de a ca ed b de
B. P d e ee / e d c a e
C. W e ba e e c c ed b a f e a ea f e e a
D. Pa a a f ce

T e e e ce f c ef d be c de ed a AIDS-def e a
a e e da ed HIV?

A. E a a c c cc
B. HIV a d e
C. N ca d
D. P e a

W a e be e e e ea e fa a e HIV fec a d CD4 c


f 204 e e d a aa d d a a a d a a c ea e e ca ed
a e e?

A. E c ea e f c a e
B. U e e d c b
C. E c ea e ac c
D. E c ea e a cc

I fec D ea e 196
Get The Step 2 CK Drills Book: usmledrills.com
The USMLE Guys Step 2 CK Crash Course usmleguys.com

Ba ,H a ,a P P a (PCP)
Blastom cosis

P a Ba c :
● F a fec e de c O a dM R e a e a d G ea La e e

S fP a I fec :
● C ( d c e - d c e, cca a e ), fe e , c e a ,
d ea, ea , e

E a a Ba c :
● S f e e :
○ Ve c e a e c e a b de
○ B eed ce a e e ea ed b de
● S fb e e e :
○ O e e
● S f e e a ac :
○ E dd c
○ P a

Da :
● Def eda ade f ec e
● Mc c a d a ca a be ed a eda

T ea e :
● M d de a e d ea e: I ac a e
● Se e e d ea e: I d c a e c B e ea e ac a e

Histoplasmosis

P a H a :
● E de c Ce a USA (O a dM R e Va e )
● S ead a c a a ed b d ba d (ca e , fa b d b d ,
c c e c , e )

S P a H a :
● Fe e /c ,c ,c e a , a a , eadac e

C e X- a :
● M de e: E a e e f a de a d a c b a/ d a f ae
● S b a a e e: D ff e e c d a a f ae
● Pa e d ea e a f c c fec :Fb c f ae
ca a ca ed a a ce

D e a ed H a R Fac :
● T e a e ed e e a : HIV, e e
d , def c e c , fa , e a c a e

D e a ed H a F d :
● Pa c e a

I fec D ea e 197
Get The Step 2 CK Drills Book: usmledrills.com
The USMLE Guys Step 2 CK Crash Course usmleguys.com

● He a e e a
● E e a ed e a c a a fe a e
● E e a ed LDH
● E e a ed fe
● S e
● GI e f a (c ), ce a a eef ( a f , ea ed b de )
a
● Ad e a a d b a d fa c ead ad e a ff c e c
● CNS fec ( e , e ce a , f ca b a e , ce eb a a c +/- e)

Da :
● H a ( c e ba e e ,b e a )
● P ec e ( b a ed f b dc e f a c e cf b e a ,
e)
● A e de ec ( e, e CSF)
● H : S ec f c a b d e ( ec ed c c fec )

T ea e :
● M d a ca e c ee a e a <4 ee , ea e
a a ed
● M d a ca e c ee a e a 4 ee , ac a e
● M de a e e e e ca e , d e a ed, c ed d d a : T ea
a e c B( a) e ac a e

Pneumoc stis Pneumonia (PCP)

I c ed a e :D c f d be ee HIV e a e a da e
c ed a e .

P a e - fa e a e (TMP-SMX) f ce a e ed
a e a d HIV e a e :
● CD4 <200 AIDS-def c d

S N -HIV P eI c ed Pa e :
● Re a fa e, fe e , d c

S HIV P e Pa e :
● G ad a (da ee ) fe e , c , a d SOB

Lab :
● Ele ated LDH
● E e a ed be a-D- ca

CXR:
● D ff e e a e a a da e a f ae

Da :
● PCR f BAL a e
● I f e ce a f e
● S d c BAL a a d d ec a ed f f

I fec D ea e 198
Get The Step 2 CK Drills Book: usmledrills.com
The USMLE Guys Step 2 CK Crash Course usmleguys.com

T ea e :
● TMP-SMX
● If c, ea c c e d

Content Re ie Questions:

A a e eO R e Va e Re 6 da f d c ec a d d e
f b ea . T e a e bee feb e f e a 48 . V a a e ab e a a f a
e e a e f 38.1 Ce .Ba c a a fe d ffe e a be c de ed. W a
e a ae a ea e f a e ?

A. De a e a e
B. A e c B
C. I ac a e
D. A c

A HIV e a e CD4 c f 91 eM R e Va e e e e
a fe e /c ,c ,c e a , fa e, a d a a . He e a fa a d
ece a a f a ea c de ed a a e c c e ba ee a .P ca
e a ab e f e a e e a a d c e ad a de f e e e e ce f d ff e
e c d a a f ae .W c f ef ab a e e ec
ee e a e b d ?

A. E e a ed AST a d ALT
B. H ae a
C. E e a ed LDH
D. E e a ed fe

A a e AIDS a d CD4 c f 122 e e 7 da fc a dd ea


e e a d a 2-da f fe e . T e a e ec ed f a e c
e a. W c f e f ab c e da ?

A. H ae a
B. H ca ce a
C. E e a ed L- ac a e
D. E e a ed LDH

I fec D ea e 199
Get The Step 2 CK Drills Book: usmledrills.com
The USMLE Guys Step 2 CK Crash Course usmleguys.com

C a C
Coccidioidom cosis

C a ac e c :
● E de c d ,a d e f e USA:
○ Sa J a Va e Ca f a
○ A d a fA a
○ O b ea a cc f d
● M a e a a c
● W e e e , a e e 7 21 da af e e ea e a
● AIDS-def :W e d e a ed e a a a e HIV

S :
● Fe e , c e a ,c , ea , e , fa e

E a a :
● De e e a - e ca a a ( ee , a e , )
● E e a d
● E e a f e

Lab :
● N a e e a ed WBC c , bee a, e e a ed ESR

CXR:
● U aea f ae( ca upper lobes) a e a hilar adenopath a d be
mediastinal adenopath . S e e d e / - a ed ca e a be e e

Da :
● Se c e e e- ed a a (EIA) f I M a d I G. If e
EIA, c f I d ff e

T ea e :
● I c ee d de a e d ea e a c
● Se e e d ea e e ed ea f c a e
● T e e a c e IV a e c B

Disseminated Coccidioidom cosis

De e ee 6 af e e e.

Ma fe a :
● Me
● Ve eb a fec
● M a c a fec f e ee
● C a e a a e
● S bc a e f e ab ce e

Da :
● H a d ec a f c e f fec ed e

I fec D ea e 200
Get The Step 2 CK Drills Book: usmledrills.com
The USMLE Guys Step 2 CK Crash Course usmleguys.com

T ea e :
● I ac a e f c a e f -e e e ca e
● A e c Bf e ee a d e e ca e
● S ca deb de e f a a ed
● D ea e ed a c e e f a e f 1 ea

Coccidioidal Meningitis

De e ee af e e e.

S :
● Pe e , e e e, a d/ e e e eadac e; a e ed e a a ;
b ed ; a ea/

Lab :
● Occa a e a e a

CSF A a :
● E e a ed WBC (fe - e e a d ed ce ) ed c c e c , OR
● E e a ed e ( 250 /dL b c 3 a /dL CSF
b c )
● L c e

Da :
● C CSF a b de ,f a c e, CSF a e , ea ec a eac
e

T ea e :
● F c a ef fe
● S f dec e f d ce a de e
● I a eca a e c B f e e c de e ea e

Pulmonar Cr ptococcosis

C. f d fe e ed b b d ( e ).

P a I fec S :
● Va e e f a a c e a fa e
● Fe e , c , SOB, eadac e

CXR:
● O e fe ca c f ed d e , ad ace e e a. H a a d ed a a
ade a , +/- e a eff

Da :
● P e e c c cca a e , ,f a c e

I c ed a e (AIDS) a e a c a e ab e e e e d ea e, f
de e e e ce a .

I fec D ea e 201
Get The Step 2 CK Drills Book: usmledrills.com
The USMLE Guys Step 2 CK Crash Course usmleguys.com

Cr ptococcus Meningoencephalitis

S :
● Fe e , eadac e, ec ff e , b a, N/V, c a a e a e , AMS
● T ca ee AIDS a e CD4 <100 b a a AIDS-def c d , ca
be ee e CD4 c

Lab :
● CSF a a a
○ L WBC c (<50 ce / c L)
○ P e a e e a ed
○ G c e e e dec ea ed a
○ H e e e

Da :
● Se c c cca a e (C A ), CSF c c cca c e, I d a e f CSF

T ea e :
● IV a e c B a f c e( d c ea ), f ed b a
f c a e (c da a d a e a ce) f a ea e ea

Content Re ie Questions:

P a c cc d d c ca e e a ade a a d/ ed a a
ade a a d de e f ae c f ef be c e ad a ?

A. U e be
B. M dd e be
C. L e be
D. W e e e e a a f ae c e ad a

W c f ef a fe a c e e a a c cc d d c ?

A. E e a d
B. S e ca a a a
C. E e a f e
D. Pa f c c a e ce

A a e ad a ced AIDS (CD4 c 44) e e fe e , eadac e, ec ff e ,


a d b a. A ba c e ef ed. CSF a a WBC c (23
ce / c L), a e (36 /d ), a d dec ea ed c e e e (33 /dL).
W c f ef da e c e e e CSF f d ?

A. Bac e a e
B. C cc d da e
C. C c cc e e ce a
D. S a e CSF ea

I fec D ea e 202
Get The Step 2 CK Drills Book: usmledrills.com
The USMLE Guys Step 2 CK Crash Course usmleguys.com

M
Bacterial Meningitis

C Pa e :
● A e 2-50: S ,N
● Ad e 50, e a e ,a d c ed: S
,N ,L
● Pa e e ca / a e :Sa c cc a d ae b c a - e a e bac

S :
● Headac e, fe e , c a d , AMS, a ea

Lab :
● CBC d ff:
○ U a e c , a ef f ( a ef , .e. ba d a d b a )
○ Le e a e e e ca e
○ T b c e a
● C a a de :
○ Of e c e DIC
● T b dc e :
○ P e ca e
○ Ob a a fa b c
● Se e ec e , c e, BUN, a d c ea e c ce a :
○ A a e ab c ac d
○ H a e a
○ C a e CSF- -b d c e a
● CSF a a :
○ T a WBC c (ce / c L): G ea e a 1000 (ca be e ea
d ea e)
○ P e ( /dL): 100-500
○ G c e( /dL): Le a 40 ( )
○ G a a a d bac e a c e

C de a Re CT Sca P L ba P c e:
Mnemonic: TAP AS IF
Ta a
A e e 60
Pa ede a ( a e d ca e e a ed ICP)
A e ed e a a
Se e( e e e ee f e e a )
I c ed (HIV/AIDS/ a a a / e ed )
F ca e c def c

Def eDa Made W e E e :


● Bac e a a e a ed f CSF b c e
● Bac e a a e a ed f b dc e CSF e c

T ea e :
● A e 2-50: S ,N :
○ Va c c cef a e cef a e

I fec D ea e 203
Get The Step 2 CK Drills Book: usmledrills.com
The USMLE Guys Step 2 CK Crash Course usmleguys.com

● Ad e 50: S ,N ,L :
○ Va c c a c cef a e cef a e
● Ne ca / a e : a c cc a d ae b c a - e a e bac :
○ Va c c cefe e cef a d e e e e

De a e a e T ea e Bac e a Me :
● Ad e de a e a e 15 20 e bef e a e e fa b c
ad a a e a a e ca e
● C e de a e a e ad a f CSF a a a d/ e CSF b d
c e ef S .D c e ed a e a e
a
● Ea ad a f de a e a e a bee e a a d e e
e cc ca c a ea

P a f Me c cca E e:
● T ea c e c ac ( e a 8 e e, e a 3 fee a a a 7 da
de e e f )
● I d d a a e bee d ec e ed a ec e
● P a c de fa ,c f ac , cef a e

Viral Meningitis

V e c a ca e e : entero iruses, HSV, VZV, HIV, We N e ,


c cc e , aa ( )

S :
● Fe e , eadac e, b a, a ea/

CSF A a :
● T a WBC c : 5-1000 ce / c L ( a e a 250 ce / c L)
● P e : 50-300 /dL ( a e a 150 /dL)
● G c e: 40-80 /dL ( a a e)
● Def eda PCR f e e e

V a . Bac e a Me :
● Of e e a bac e a e f d , dec be ade a
e ca b c
● T e a c ca e e e ca e, e de , c ed, a ece
ece ed a b c (a e CSF f d , a ) ca ece e 48
e c a b c ea e af e b a b d a d CSF c e
● A e ec ed f a e ca be b e ed a b c

T ea e f V a Me :
● U a , e f- ed e e e ca e
● If HSV I ec ed a e ca e, c de ea e IV ac c

Tuberculous meningitis

Me ca ed b M .

I fec D ea e 204
Get The Step 2 CK Drills Book: usmledrills.com
The USMLE Guys Step 2 CK Crash Course usmleguys.com

S :
● Headac e, fe e , c a d , AMS, a ea, c a a e e a e (II a d VI)
● I ad a ced e : C a, e e , e e aa be

C ca :
● H d ce a , a e a,

CSF A a :
● T a WBC C 5-1000 ce / c L
● P e 50-300 /dL
● G c e e a 10 /dL ( )
● Ac d-fa bac ea a d c e
● N c e c ac d a f ca e (NAAT )

I a :
● Ba MRI ba CT (b a MRI efe ed)

Def eDa :
● Made a e f e f :
○ CSF e ea f ac d-fa bac
○ CSF c e e f M.
○ CSF e c e c ac d a f ca e (NAAT)

T ea e :
● A f d a d, fa , a a de, a d e a b f f ed
b a d a d fa f 10

CSF F d f Me Re e ed:
Infection Total WBC Count Protein (mg/dL) Glucose (mg/dL)
(cells/microL)

V a 5-1000 ( a e 50-300 ( a e 40-80 ( a a e)


a 250) a 150)
Bac e a G ea e a 1000 100-500 Le a 40 ( )
(ca be e ea
d ea e)
TB 5-1000 50-300 Le a 10 ( )

I fec D ea e 205
Get The Step 2 CK Drills Book: usmledrills.com
The USMLE Guys Step 2 CK Crash Course usmleguys.com

Content Re ie Questions:

W c f ef CSF f d c e a e ?

A. T a WBC c : 200 ce / c L, P e : 92 /dL, G c e: 55 /dL


B. T a WBC c : 2000 ce / c L, P e : 500 /dL, G c e: 29 /dL
C. T a WBC c : 1320 ce / c L, P e : 350 /dL, G c e: 35 /dL
D. T a WBC c : 500 ce / c L, P e : 270 /dL, G c e: 10 /dL

W c f ef CSF f d c e bac e a e ?

A. T a WBC c : 200 ce / c L, P e : 92 /dL, G c e: 55 /dL


B. T a WBC c : 2000 ce /
c L, P e : 400 /dL, G c e: 22 /dL
C. T a WBC c : 800 ce / c L, P e : 350 /dL, G c e: 70 /dL
D. T a WBC c : 5 ce / c L, P e : 270 /dL, G c e: 50 /dL

W c f ef a d ca ef a CT ead ba c e a a e
ec ed f a e ?

A. N c a d
B. A a e ab c ac d
C. T b c e a
D. Pa ede a

I fec D ea e 206
Get The Step 2 CK Drills Book: usmledrills.com
The USMLE Guys Step 2 CK Crash Course usmleguys.com

M ba T b a M ba A C
(MAC)
M c bac e T be c D ea e C e:
● P a d ea e
○ I ed a e a c d ea e
● La e fec
○ D ea e c a ed b e a e ' e e
● Reac a d ea e
○ Nd f c a ed c bac e a fec ae fec bec e ac e

Primar S mptomatic Pulmonar Tuberculosis

S :
● Fe e , e e a a , a d/ e ca a a

CXR:
● H a ade a
● P e a eff
● P a f ae

PPD T be c S Te a d IGRA :
Induration (in mm) to Criteria for Positi e PPD (48-72 hours post intradermal
be Considered Positi e administration)

5 -HIV fec
-Rece c e-c ac e e d d a ac e TB
-I e ed a e
10 -I a f c a e f TB a 5 ea
-L / e ee f TB a c ea ed
( , ee e e, fac , ea ca e fac ,
c bac e a ab, e c.)
-H f ec d e
-A e <4 ea
15 T e a f e ab e fac

I e fe - a a e ea e a a (IGRA ) a e a b d e a ca be ed ace f PPD

Reactivation Pulmonar Tuberculosis

S :
● C , e , fe e , ea , d ea, c e a , e , fa e

CXR:
● Ca itar lesion e posterior apical segment f e upper lobes e e
e e f e e be

I fec D ea e 207
Get The Step 2 CK Drills Book: usmledrills.com
The USMLE Guys Step 2 CK Crash Course usmleguys.com

Miliar Tuberculosis

He a e d e a f TB f e e a a TB f eac a
f ae TB.

S :
● Ac e: M a fa e, ARDS, e c c
● C c: Fe e , a d f c (de e d c a TB a ead , ca
affec e e a e b d ), fa e e

Lab :
● N c c, c ca e a, a c e a fb e a e e

CXR:
● Re c d a f aef d f e

Da :
● Def eda ade a eM c ef ,
BAL, e a f d, e a/ eb

O e Da c Te I c de:
● S ac d-fa bac (AFB) ea
● N c e c ac d a f ca e
● PPD e fe - a a e ea e a a

T ea e f Ac e TB:
● 2 f a d, fa ,e a b ,a d a a de
● F ed b 4 f a d a d fa

T ea e f La e TB:
● I a df 6 9 , fa f 4 , a d a d fa f 3 ,
a d a d fa e e ce a ee f 3

M cobacterium Avium Comple (MAC)

S :
● C , SOB, c e d c f , fa e, cca a e

T D c C ca P e e a :
● W e dd e a e/e de ae f COPD f d a e f ae a d
ca e e e be CXR, a d a d ea e e e e ca e ca bec e
e a e
● N dd e a e/e de e f d a ec d ca b c ec a a d
a a a d e CXR

Da (a e ed):
● S c e MAC fec
● P a ca e OR a d e CXR OR b c ec a e
a d e e CT
● E c f e a da

I fec D ea e 208
Get The Step 2 CK Drills Book: usmledrills.com
The USMLE Guys Step 2 CK Crash Course usmleguys.com

● P ec ef e aae ec e OR ec ef e BAL
OR b c bac e a a c fea e ( a a , e e ce f
AFB) a d ec e

T ea e :
● A c , fa ,a de a b a c de ( e a c ) f a e a
ca a e
● S c e e e 1-2 c e a ef 12
● CXR e e 6 e de ea e
● If c e e a e af e f ea e , add a ac e
aa e
● L e ec c de ed f e ea e fa e

Content Re ie Questions:

A a e ed e U ed S a e 3 ea a f ac ee ae f
be c a e e de c. W e be e a a ed a PPD be c e , c f e
f d a e e a e e a d d ca e a e e f ae
be c ?

A. 5
B. 9
C. 10
D. 12

W c f ef ed ca ec e ded a a a ea e f ac e
a be c ?

A. I a d
B. E a b
C. P a a de
D. P a a e

W c f ef ed ca ec e ded a a a ea e fc f ed
c bac e a c e ?

A. S fa e a e
B. A c
C. R fa
D. E a b

I fec D ea e 209
Get The Step 2 CK Drills Book: usmledrills.com
The USMLE Guys Step 2 CK Crash Course usmleguys.com

C C ,I a, P a ,a C a
Common Cold (Upper Respirator Viral Infection)

S :
● R ea/c e / ee ,c , a , b d ac e , d eadac e,
- ade fe e

Fea e :
● Be , a e f- ed, a d ( cca a ead ,
a a e ace ba , e c.)

T ea e :S eT ea f S a c Re ef
● NSAID /ace a e
● I a a a c d
● I a a a a b de

Influen a

S :
● Fe e , a a, eadac e, d c ec , a , ea

C ca :
● P ea a
Se e e d ea e c a ac e ed b
○ fe e , d ea, c a
CXR
○ b aea e c a ac e / e ed c da
● Sec da e a (bac e a )
○ S. a d S. c a e
○ I a e e f ed b e f
○ S. (CA-MRSA) e a e bef a aee e
, ea a e

I f e a T ea e :
● I f e aa a d : e a , ba a , a a , ea
● A a ed ca d ca ed :
○ Pa e e a a f f e a
○ O a e aea f f e ac ca , e a de f
d a
○ O a e e ee e , e a de fd a
○ O a e e d a 48

Acute Group A Strep Phar ngitis

S :
● P a , a e da e , a ea ede a, fe e , e de a e ce ca
ade a , ac fc

G AS e C ca :
● Sca e fe e
● Ac e e a c fe e
● P e c cca e e

I fec D ea e 210
Get The Step 2 CK Drills Book: usmledrills.com
The USMLE Guys Step 2 CK Crash Course usmleguys.com

● Reac ea
● T c c d e
● T a ea ab ce /ce
● O ed a
● S

Da :
● P e a da e e c ef A e

T ea e :
● T ea a a e a e a da e e c ef A e a d
c ca
● T ea a c e c ( ac de , ce a , c da c f e
e c a e )

C ca - Sca e Fe e :
● Sa d a e - e a d e a a ee a , d ff ee e a, b a c e
e e, be a da e e e e e e a e
a d a . Ra e e a de a ae
● C c a a
● S a be e
● P ed e ac e e a c fe e
● T ea e a ea A e a

C ca - Ac e R e a c Fe e :
● Da e ac e e a c fe e e GAS fec f ed b 2 a OR 1 a
a d2 c e a

Major Criteria Minor Criteria

J (J )A A a a

O (<3) Ca d a d a Fe e

N (N d e ) S bc a e E e a ed ESR, CRP
d e
EE e a a a P ed PR e a ECG

S S de a c ea

La e e e ae f ac e e a c fe e R e a c ea d ea e (RHD)

C ca -T cS c S d e:
● A c a ed a / f e fec
○ Re f e c cca ca a ec e e a ead
ca a ea a d e da a e

I fec D ea e 211
Get The Step 2 CK Drills Book: usmledrills.com
The USMLE Guys Step 2 CK Crash Course usmleguys.com

Da c C e a:
● H e ( cb d e e 90)
● M a e e 2f d :
○ Re a a e
○ L e
○ ARDS
○ C a a
○ S f e ec
○ E e a ac a a b e de a a

C ca - Re a ea Ab ce :
● S ff ec , a ec e e ,d a a, fe e
● U ab e a e (a , ea f a d, a ed a e a e ac ):
I ed a e ec e e a a a d ceed e e e c ca d a a e
● S ab e a e : CT f e ec c a d ffe e a e ab ce f ce .Da
f a e ab ce e e
● E c IV a b c e a f da a e ed a e f d a a e a a ed

Content Re ie Questions:

A a e f d a e AS a c cb fa c
e c bed a b c . O e ee a e e e e e a e e a BP f
72/56. W c f e f add a f d d be c e a eda
f c c d e?

A. DIC
B. Re a ea ab ce
C. A a e a a fe a e a d a a aea a fe a e 3 e e
f a
D. C ea e 2.6 /dL

A 10- ea - d a e e e a - c a c a d e ee e f
e .S e a f d a e ef a e d e de e a e a
a e ee , a e , e b ,a d . He e e ec a ed f a e
a a a e 2 ee a b e e a bee ea e a ed ca
f e. W c f e f d a e ec a be e e a
a e e e c d ?

A. K
B. Te a ec a a
C. S de a c ea
D. Ra a d d e

W c f ef a c a ac e cf d ee ca e fe e ?

A. C c a a
B. S a be e
C. Sa d a e - e a
D. Pa f e a a a d

I fec D ea e 212
Get The Step 2 CK Drills Book: usmledrills.com
The USMLE Guys Step 2 CK Crash Course usmleguys.com

R
Acute Rhinosinusitis

S :
● P e a a d c a e/c e , fac a f e , eadac e, fe e , a /de a
a , ea a

Ac e R (V a a d Bac e a ) C a f ca :
Classification S mptom Duration

Ac e R <4 ee
S bac e R 4-12 ee
C cR >12 ee
Rec e Ac e R 4 d c e e cc e ce f ac e
e ea , e f
be ee cc e ce

Classification T pe of Pathogen and Anatomical In ol ement

U c ca ed Ac e Bac e a , e e de a a ca a d aa a a
Bac e a R e
C ca ed Ac e Bac e a Bac e a , e e de e a a ca a d aa a a
R e
Ac e V a R V a

Ac e Bac e a R . Ac e V a R :
Ac e bac e a
S a e e f >10 da e e
B a c e e f e
3-4 c ec e da f fe e >39 C, e a a d c a e, fac a a
<2% f ca e a e bac e a
S ,H ,a dM
c a e
Ac e a
S a e e 7-10 da , f e a 10 da ceab e
e e cc
Fe e a 1-2 da

C ca ed Ac e Bac e a R :
● Ce ab ce f a e e b a/ b a e
● O e e f e b e
● Me
● I ac a a ab ce

I fec D ea e 213
Get The Step 2 CK Drills Book: usmledrills.com
The USMLE Guys Step 2 CK Crash Course usmleguys.com

I a :
● I a ece a a eada f c ca ed ac e bac e a
ac e a
● CT MRI f e ead b a ed f c ca ed ac e bac e a

T ea e :
● Ac e a :S e ca e
● U c ca ed ac e bac e a :A c a dca a ae
● C ca ed ac e bac e a : Ta ea e c ca

Fungal Rhinosinusitis

R Fac :I c ed a e

C Pa e :A ,F , de a ace (b ac ) d , c ae
( c c c )

F d :
● Nec c e ae /
● S f e a a e
● Pe f a f e a a e
● Pa a a a e c a

CT a d/ MRI f e Head:
● F ba

Def eDa :
● Na a e d c b e a dc e

T ea e :
● Ba ed f a a e ( c a e, a e c B) a d ca deb de e

Content Re ie Questions:

W c f ef c ca fea e c e ada f ac e bac e a


e ac e a ?

A. W e f da 6 f e af e a e e
B. S e e f 14 da e e
C. Fe e f 39.1 C f f c ec e da a e be f e
D. C c e ed ec e e b c

U c ca ed ac e bac e a be da ed a c f ef
da e ?

A. C ca d a
B. Na a c e
C. S a ae
D. CT f e ead

I fec D ea e 214
Get The Step 2 CK Drills Book: usmledrills.com
The USMLE Guys Step 2 CK Crash Course usmleguys.com

H eda ff a ade?

A. E e a ed e e a
B. CT a f e e
C. Na a e d c b a dc e
D. C ca d a

I fec D ea e 215
Get The Step 2 CK Drills Book: usmledrills.com
The USMLE Guys Step 2 CK Crash Course usmleguys.com

U a Ta I
Acute Simple C stitis

R Fac :
● Fe a e e
● HI f UTI
● Rece e a ec e (e ec a e f e c de)

S :
● U a fe e c
● U a e c
● D a
● He a a
● S a bc a
● U a NO a a d c a e/

Da :
● If ab e c ca e e fe a e a e , eda ca be ade
add a e
● F e a ca :U a - e c e (>10/ c L), a e
f e c ee ea ea d e. U ec e a a ed f ea e a
a e fac f e a ce

T ea e :
● N f a e - fa e a e

Acute Complicated Urinar Tract Infection (UTI)

Ac e C ca ed UTI:
● S f UTI ( a fe e c a d e c ,d a, e a a, a d/ a bc
a ) a f ef :
○ S e c (fe e , c , a e ed e a a )
○ CVA e de e
○ Fa a
○ Pe c/ e ea a ( ae )

I a :
● O a a ed f e e f ea e a b c , f a ac
b c ec ed, c ca a e ( e c c , ICU ad )
● CT a d c a efe ed.

Da :
● U a ( e WBC , e c e e e a e, a d e) a d ec e

T ea e :
● C ca e / ec ed a ac b c / f d e a a e :
Me e e e e d e e a c c e d da c
● If e f e ab e fac : Cef a e e ac -a bac a c f ac
e f ac

I fec D ea e 216
Get The Step 2 CK Drills Book: usmledrills.com
The USMLE Guys Step 2 CK Crash Course usmleguys.com

A a c Bac e a Pe a c :
● A c a ed ad e e e a c c e ( b e , ee b , c ea ed
e aa a )a d e f de e e e d e a c
● Sc ee ec e ef ed a 12-16 ee e a
● U ec ec de ed e e e ce f 105 cf / L f a e c bac e a
● A b c c ce ba ed c e ce b e

Content Re ie Questions:

W c f ef a f d c e ada f ac e e
c ?

A. P e e ce f 50 e c e / c L
B. P e e c ee ea e
C. P e e
D. P e e ce f RBC ca

W c f ef c c a ce d a a e ca b c ea e a e
a e e e e f e e e a c c f ea e f ac e c ca ed a
ac fec ?

A. P e a a e ee 12-16 f e a
B. U a ac b c
C. Rece a e a ad
D. Rece e - fa e a e e

W e d e a a e dea be a c ee ed f a a c bac e a?

A. 6-10 ee e a
B. 12-16 ee e a
C. 16-20 ee e a
D. 20-24 ee e a

I fec D ea e 217
Get The Step 2 CK Drills Book: usmledrills.com
The USMLE Guys Step 2 CK Crash Course usmleguys.com

T -B D a
L me Disease

Ta :I c e bef a a e c c eeB

D ea e C e f L e D ea e D ded T ee P a e :
● Ea ca ed d ea e
● Ea d e a ed d ea e
● La e d ea e

Ea L ca ed L e D ea e:
● T ec e: Da ee af e fec
● S :
○ E e a a
■ C a ca a a c c a a e - e a ea a ce ( ce a c ea f
e e a af e e e a da )
■ E a d e da ee
■ A ea 3-30 da f a c b e
○ Fa e, a a ,a a a, eadac e, ec ff e

Ea D e a ed L e D ea e:
● T ec e: Wee af e fec
● S :
○ Ne ca a fe a :
■ C a c ad f c c e , ad c e ,a dca a
e
○ Ca d ac a fe a (L e ca d ):
■ AV c d c b c , e ca d
○ Oc a a fe a :
■ C c

La e L e D ea e:
● T ec e: M ea af e fec
● L ea ( a e ee)
● L e e ce a a ( d e c )

L e D ea e D a :
● Pa e a eb :
○ R fe e c L e e de c a ea
○ S / f L e d ea e
● T e da ade ba ed a e:
○ Ea ca ed L e d ea e: P e e ce f e e a a ( L e e
eeded)
○ Ea d e a ed a e L e d ea e: C ca a fe a e ca
e ELISA a d We e B ( e e, ece a
a e def eda )

L e D ea e T ea e :
● Ea ca ed L e d ea e: T ea a d c c e, b f e f
e f 24 (Ja c -He e e eac )

I fec D ea e 218
Get The Step 2 CK Drills Book: usmledrills.com
The USMLE Guys Step 2 CK Crash Course usmleguys.com

● Ea d e a ed L e d ea e: O a d c c e, e ce e ca d
a e e e e 1 de ee (PR e a 300 ) 2 3 de ee AV b c eed ca d ac
ee e , e a ace a e , a d IV a b c (cef a e, cef a e, e c G)
● La e L e d ea e: L e a -d c c e a c
● La e L e d ea e: L e e ce a a ( e e ca a fe a ) - IV
a b c (cef a e, cef a e, e c G)

Human Ehrlichiosis and Anaplasmosis

Ta :
● H a c ce c (HME) ca ed b E a ed b
e e a c .H a a c ca a a (HGA) ca ed b A
a ed b I a dI

S :
● Fe e , c , a a, eadac e, a d b a ( ec HME)

Lab F d :
● T b c e a, e e a, a d e e a ed AST/ALT, a d e e f HME
HGA e d ec f e ce a b d (IFA) e PCR

T ea e :
● O a d c c e, f c a d ca ed e a fa

Rock Mountain Spotted Fever (RMSF)

Ta :
● R c M a ed fe e (RMSF) ca ed b fec R
a ed b R c M a W d c ,b d c , A e ca d c

S
● Da 1-4: Fe e , a , eadac e, GI e , e e e abd a a , a a, ede a
a d ee e a d e bac f a d
● Da 5 a d be d: ARDS, a ede a, ec e a a , ce eb a
ede a, a fa e

Da :
● Made c ca a d d ae ea e
● C f ed e ac e d ec f e ce ce a b d (IFA) PCR ec f c
R

T ea e :
● D c c e

Babesiosis

Ta :
● Babe ca ed b B a a ed b a ed b I c
a a f fc a a ed b d d c

I fec D ea e 219
Get The Step 2 CK Drills Book: usmledrills.com
The USMLE Guys Step 2 CK Crash Course usmleguys.com

S :
● I ee c
d d a : Fe e , c , ea , d c , a a, eadac e,
a ea, a
e a, e e a a a e, a d fa e
● I e ed d d a : Sa e a c ee a e ,b
fe e e e e, a e a de e e f fe ea e c , ARDS, CHF, DIC,
e a fa e, e c e

Lab F d :
● L e b a d e a c , a b , e c c ec ,
b c e a, e e a ed LDH, e e e , a d c ea e

Da :
● Made c ca e , ab f d ,a de e e de c a ea b d
a f c e d fec
● S e e a da c c de: b d ea de a e e ce f
Ma e e C , PCR f B DNA, d ec f e ce a b d e (IFAT)

T ea e :
● Oa a c (IV f c ed f e e e d ea e) a a a e

Content Re ie Questions:

W c f ef e ed eda fL ea ?

A. E e- ed be a a (ELISA) a d e e b
B. U a a e e
C. B d ea
D. DNA c a a

A a e e a ece c b e e A a 3 da a f ed b fe e , a ,
eadac e, a d a ea. W a e e be e a a e e f a e ?

A. C f da fR c M a ed fe e d ec f e ce ce
a b d (IFA)
B. C f da fR c M a ed fe e PCR a f ca ec f c
R
C. P ceed ea e d c c e ed a e
D. P de e ca e f a c e ef

T ef d f a Ma e e C ab d ea c e c f ef
da e ?

A. R c M a ed fe e
B. L e d ea e
C. H a a c ca a a
D. Babe

I fec D ea e 220
Get The Step 2 CK Drills Book: usmledrills.com
The USMLE Guys Step 2 CK Crash Course usmleguys.com

M -B D a
Ma a a:
● A e bef a a e c P. , P. , P.
, P. ca Ma a a

Species Incubation Period

P. 12-14 da
P. 14 da - e ea , ea e 2-3 ea
P. 14 da - e ea , ea e 2-3 ea
P. 18 da

Ma a a S a dC ca :
● U c ca ed a a a: M de a ae ec f c (feb e a ,
c /d a e , fa e/ a a e, a ea/ /d a ea, abd a a ,
a a a / a a , eadac e) c f ed a a e a a d a e ab e
a ed ca
● Se e e a a a: Se e , ARDS, c c a c a e, e ab c ac d , e a fa e,
e b a, e a c fa e, c a a DIC, e e e a e a a e
a a c a e , ce a

Lab F d :
● A e a; b c e a; c a a ; e e a ed AST/ALT, BUN, a d c ea e

L Mc c :
● G e a- a ed b d ea a e aa a a a e( c a d b d ea )

Ra d D a c Te (RDT ):
● De ec aa a a a ea e

T Ma e D a a Pa e M Ha e:
● S c e aa a a bee a a a e de c e
● A e aa ada c e (parasitemia on blood smear a dda c e
ef aa ea e )

Med ca f T ea e :
● C e- e e P. a a a: C e
● C e- e a P. :A e c b a e a (ACT)
● C e- e e -fa c a a a a: C e a e c b a
ea (ACT) AND e ca e f P. a d P. e e ea ef
e e a ea ea e e a e afe e

P ec f Ma a a:
● Ge e c ec f e b a a :
○ He b S, C, SC, E, F
○ A a a d be a a a e a
● I e ec f e e:
○ O de c d e a d ad e de c a ea e a e a a

I fec D ea e 221
Get The Step 2 CK Drills Book: usmledrills.com
The USMLE Guys Step 2 CK Crash Course usmleguys.com

○ Y c de , e a c a e , e a a e ,a d a ee e de c
a ea aa ae e a f e e e d ea e

A aa a P a :
Drug Species for Which Drug is Considerations
Effecti e

A a e- A (d e e e e -Ad e ed e da a e,
a f a b P. P. ) d a e de c a ea, a d 7 da
f e a a a-f ee e
-GI e
C e A b c e- e a P. -Ad e ed ce ee a e
d c e a d e P. ee a a, ce ee e
(d e e e e e a a a e de c e , a d ce
f a b P. P. ) ee f f ee f e
a a a-f ee e
-Safe f e e a c
D c c e A (d e e e e -Ad e ed e da e e,
f a b P. P. ) da d e e, a d da f ee
f e e
-UV e
Mef e A b ef e- e a P. -Ad e ed ee ee
(d e e e e e, d e e, a d f ee
ef a b P. f e e
P. ) -Neurops chiatric ad erse effects
-S b ad ca d a a d QT a
-Safe e a c
P a e P a f a ea P. -Ad e ed da e da
a d ed a e e e e, ce da d e e, a d
a - ea e e a f da f e e da f e e.
ef a b P. -Ca ca e e c a e a G6PD
P. def c e c a e
Tafe e P a f a ec e , -Ad e ed da ee da
a - ea e e a f e e, ee d e e, a d
ef a b P. ce e ee f e e
( e ab P. a e b -Ca ca e e ca e a G6PD
e ed) def c e c a e

Chikungun a Virus

Ta :
● A a ead a A a dA e
● A a a e a -fe a a a db d d c a f

I c ba T e:
● 3-7 da

I fec D ea e 222
Get The Step 2 CK Drills Book: usmledrills.com
The USMLE Guys Step 2 CK Crash Course usmleguys.com

S :
● Fe e , a a a (b a e a a d e c, d a > a , ff e ),
ac a ac a a a

C ca :
● Me e ce a , ac e e a fa e, ca d a dec e a
● Pa e ca de e c ca affec ed d ac e e

Lab F d :
● E e a ed LFT , b c e a, c ea ed b e

Da :
● C f ed RT-PCR c a e

T ea e :
● S e ca e f ac e (e 7-10 da )a d e de e
c ca a ea e ed e
● If e ed e ce a e c de e e e e ae TNF
b

Zika Virus

Ta :
● F ead a Aede a e a ec e

S :
● Fe e , c a ,a a a( a a d a d fee ), c c

C ca :
● G a -Ba d e

I Pe a Pa e :
● Fe a de e fe a c ce a ca de e

Da :
● RT-PCR Z a e

T ea e :
● S e ca e

West Nile Virus (WNV)

Ta :
● F ead b ( e a e be f ec e )

S :
● M aea a c, b d ca e c de fe e , eadac e, bac a , a a ,
ac a a b f a (c e , bac , a d a )
● Se e ca a fe a : Me , e ce a , a e c f acc d
aa

I fec D ea e 223
Get The Step 2 CK Drills Book: usmledrills.com
The USMLE Guys Step 2 CK Crash Course usmleguys.com

Lab :
● Pe ea b d e e a ed WBC a d - ec f c. CSF e e a ed e
a d de a e e c c e ed a ce

Da :
● P e WNV I M a b de e CSF a ee a MAC-ELISA

T ea e :
● S e ca e

Yellow Fever

Ta :
● F ead b Aede ae S A e ca a d b-Sa a a
Af ca

S f a ea af e 3-6 da .

T ee S a e f Ye Fe e :
● Pe d f fec
● Pe d f e
● Pe d f ca

Ye Fe e - Pe d f I fec :
● D a :
○ 3-4 da
● S :
○ Fe e /c , eadac e/ b a, e bac a , a a, a ea/ ,
c c a ec
○ Fa e ( e ea e fe e )
● Lab f d :
○ Le e a ea e e e a, e e a ed AST a d ALT e e

Ye Fe e - Pe d f Re :
● Af e e d f fec , cea e f 48 .T e be
c e :
○ M a e (85%) ec e a d e a a a cf e d
f e
○ 15% f a e ad a ce e e d f ca

Ye Fe e - Pe d f I ca :
● He a c a fe a :
○ E e a ed AST a d ALT, AST > ALT
○ E e a ed d ec b b
● Re a a fe a :
○ E e a ed e c ea e
○ P e a
○ O a/a a
○ Re a fa e
● GI a fe a :
○ Pa c ea

I fec D ea e 224
Get The Step 2 CK Drills Book: usmledrills.com
The USMLE Guys Step 2 CK Crash Course usmleguys.com

● Ca d a c a a fe a :
○ A a
○ B ad ca d a
○ H e
● C a a a d e a e:
○ T b c e a
○ P ed b e (PT)
○ GI b eed
○ Pe ec ae/ecc e
○ He a a
● Ne ca a fe a :
○ De
○ Se e
○ C a

Da :
● ELISA for IgM PCR f a e e

T ea e :
● S e ca e

Dengue Virus Infection

Ta :
● De e e (DENV) a e a e fF ead b A
A e

S :
● Fe e , a e a/ a ea/ , eadac e, ac a ac a a a ,
a a/b e a / c a a

De e He a c Fe e :
● De e fe e a a ea a e, e a e, b c e a

Pa a Lea a e Ide f ed a A f eF :
● 20% c ea e e a c
● P e a eff
● A c e

Da :
● I M a a ELISA de f de e a c a e 1a e

T ea e :
● S e ca e

I fec D ea e 225
Get The Step 2 CK Drills Book: usmledrills.com
The USMLE Guys Step 2 CK Crash Course usmleguys.com

Content Re ie Questions:

W c f ef a aa a a ed ca a c a ed e c a c
ad e e effec c a ae a d aa d dea ?

A. A a e- a
B. Mef e
C. P a e
D. C e

A a e a b a d a ed e U ed S a e fa a a a a e de c
e a e a e ce f P. .W c f ef ed ca a a ae
e ea - ea e e a effec e ea ef a e e?

A. A a e- a
B. Mef e
C. P a e
D. C e

A a e a e e eec e- e a P. e de c a d fa a e
a a a a a ed ca .U e e e US, e de e a fe e a e a
c ,da e , a ea, a d .W c f ef e d be c d c ed
e eda f a a a?

A. S e c b
B. L e b
C. B d ea
D. L e a d

W c f ef c a c a ed c a fe e ?

A. P a a a
B. O c
C. E
D. H e a

W c f ef ab a e be c a ac e e a a e e fe e a
e ed e e d f ca ?

A. E e a ed AST a d ALT, AST>ALT


B. E e a ed AST a d ALT, ALT>AST
C. E e a ed AST a d a ALT
D. E e a ed ALT a d a AST

W c f ef a f a a ea a e cc de e e a c fe e ?

A. A c e
B. B eed f e
C. 20% c ea e e a c a ef ba e e
D. P e a eff

I fec D ea e 226
Get The Step 2 CK Drills Book: usmledrills.com
The USMLE Guys Step 2 CK Crash Course usmleguys.com

Pa a I
C sticercosis

Ta :
● Ca ed b fec a e T , e e e fece fa a
a e ca e a e e ed

S :
● Sei ures, c a e , f ca e c def c
● If f ee f a c e e ce b c f ac , b f increased ICP
a ede a, eadac e, a ea, a e ed e a a

Ne a :
● S a (<2 c ) c c e ,e a c e , a d/ ca c f ed e eba
a d/ c ce c e e ce b ca b c e d ce a
● CT a d MRI b a a ed

Lab :
● Se e e- ed e ec a fe b (EITB) f a a e c e
d be e af e a c e c ce c f d

Da :
● P b fe e T , c ca a fe a c e d ea e
( e e a d/ e e a ed ICP) a d e a c e da

T ea e :
● E ee a d be e f ed a a aa c ea e c de c a
c ce c ( ea e ca ca e f a a a dd a a e e e a d ead
a e )
● T ea a cc d f (VPN f b c e d ce a , a e e
ed ca f e e )
● Ne ea a a a c ed ca a be da e f ec a be da e
a a e
● A a a e ec c e d a aa c ea e e
fa a a d f e e

Echinococcus Granulosus/Multilocularis

Ta :
● Ta e e fe e ca e d a d ee , a a a
dead-e d . D ea e cc f e fe f d d fece

Ma ea / fe ea a ca da d c f a d e
e a

S :
● L e e e :R e ad a a , e , a ea/
○ If c ade b a ee: b a c c, b c e a d ce, c a
● L e e :C e ,c e a ,d ea

I fec D ea e 227
Get The Step 2 CK Drills Book: usmledrills.com
The USMLE Guys Step 2 CK Crash Course usmleguys.com

I a :
● U a dfe e ed e ec ed c c e e f e e.F d
c de f d fa e c a , e e ce f da e c ,a d f da d
a d
● CT a d MRI a ed ( e ef e e e f e a )

Lab :
● Se a a ab e de a e f da e , b ELISA a d d ec e a a
(IHA) a e be a e

Da :
● C ca , a d a c e c / da d c , a d e
e ( e a a eed be e f c e )
● Ve a e ec a e a a b f a d/ e ae c c e
( f ec da ead f e fec a d/ a a a )

T ea e :
● De e d a e fac , e, a d ca fc :S e , ec a e
a a e e ,d ea (a be da e), a d/ b e a

Trichinellosis

Ta :
● R d fec f e f a d a e (bea a d a )

C ca Ma fe a :
● I e a a e 2-7 da af e e abd a a , d a ea, a ea/
● M c e a e 1-5 ee af e e a a , ce e , ea e , fe e ,
e b a ede a, b a e e a e

Lab :
● Ab a e a ed ce a e e c ,e a, e e a ed CK
a d LDH.
● P e ELISA, d ec f e ce ce, ae a a e c f
da

Da :
● Pa e fc f de c ed ea a d ,e a,
e b a ede a c f ed e e

T ea e :
● M d ca e : S a c ea e
● M de a e e e e ca e : a be da e ebe da e c c e c c e d
ea e e e e ca e

Cutaneous Larva Migrans

Ta :
● U a e e fd ca f d ca a d b ca
e fec a e e e a e c ac c a a ed

I fec D ea e 228
Get The Step 2 CK Drills Book: usmledrills.com
The USMLE Guys Step 2 CK Crash Course usmleguys.com

S :
● I a ce e a a ef f ed b c, serpiginous, ed/b
e

Da :
● E e e c a a ed a d e e ce f e e

T ea e :
● I e ec a be da e

Pinworm Infection

Ta :
● E ( ) e c e c fec e USA

S :
● Pe a a (a ) ead cac a d e ead ff e f a d
f da d e ed b e f e

Da :
● Ce a e a e e ed e a a a d c c c f e e ce f
e

T ea e :
● A be da e a e a ae

Content Re ie Questions:

Pa e e Aa aa dc bea . T ee ae e a e
c a f fe e , a a , e b a ede a, a d b a e e a e e a .
W c f ef e e da ?

A. C ce c
B. T c e
C. Ec c cc a
D. G a d a

A fa e f 3 c de da ca e e a e a bee e e e c ad e
d bed ee a d e a a c a f e a ee . W c f e f e e
be e a ea aeda ?

A. CBC d ffe e a
B. Ce a e a e e
C. P a
D. T ea a be da e

I fec D ea e 229
Get The Step 2 CK Drills Book: usmledrills.com
The USMLE Guys Step 2 CK Crash Course usmleguys.com

A 36- ea - d fe a e e ed f a ece beac aca Ja a ca a d ee da af e


e e ced a ce e a a ef ed e ef f c e ed
a c, ed e e .W c f ef ed ca d be a ae
ea e fec ?

A. D c c e
B. P a e
C. I e ec
D. R fa

I fec D ea e 230
Get The Step 2 CK Drills Book: usmledrills.com
The USMLE Guys Step 2 CK Crash Course usmleguys.com

MSK, Dermatolog , Rheumatolog


C S ,Eb ,a W C a
Shoulder Impingement S ndrome or Rotator Cuff Tendinopath

R Fac :
● S e e e ac a ab e e de

S :
● Pa b b e ead ac , ca ed e de d a ea aea a , e
a e affec ed de

P ca E a F d /Ma e e :
● P e Nee a d Ha -Ke ed e e e
● Te de e a bac a ace/ e de
● Pa e e d a c abd c a de e a a ( e a)

I a :
● D a c MSK US e f e e a d e d ed (d a c)

T ea e :
● Ice f affec ed a ea, a d ac e a ca e , ea NSAIDS,
ca e a

Rotator Cuff Tear

S :
● Pa a e a de d, ade eb e ead ac e a d affec ed de
a

P ca E a :
● Wea e e ed abd c a de e a a
● P e a f ac e
● P ed a e

I a :
● S de MSK a d e d ea (d a c)
● MRI ca e e ce a d de ee f ea , e d e ac ,a d cea
(d a c)

T ea e
● C e a
e a a e e f a a a c ff ea a d ac e a ca e
, NSAID , a d ca e a
● S ca e a f ac e, f - c e a c ff ea a d e a a a c ff
ea fa c e a e a a e e

MSK, De a ,R e a 231
Get The Step 2 CK Drills Book: usmledrills.com
The USMLE Guys Step 2 CK Crash Course usmleguys.com

Adhesive Capsulitis

S :
● F e de , de a e f a e f (ROM) f
abd c a de e a a , a bec e e e a ROM e , e
a cc eac f a e a e f

P ca E a F d /Ma e e :
● L ed ac e a d a e ROM f e e a a a d abd c

I a :
● MRI ( ca ef ed) c e f e ca e

Da :
● C ca d a ba ed a d ca e a a f d

T ea e :
● P ca ea , a e d a-a c a c c c d ec , a-a c a
d a , e

Lateral Epicond litis (Tennis Elbow)

S :
● Pa ae a eb , e ed f ea ac

P ca E a :
● Pa / e de e a e a e a e c d e a d a e e ce
● Pa e ed e e e eb e e ded
● Pa a e e a fe eeb e e ded
● B e , e eb e e

I a :
● MSK US ( f e ece a ) ca e d c e , a a ea

Da :
● C ca ba ed a de a

T ea e :
● A d ac e a ca e a d c af e ac f ca be a ded,
e b ace, NSAID ace a e , ca e a

Medial Epicond litis (Golf Elbow)

S :
● Pa ed a e b , e e be d ad a

P ca E a :
● Pa / e de e a e ed a e c d e a d a fe ce
● Pa e ed fe eb e e ded
● Pa a e e a e e eeb e e ded
● G fe ' e b e e

MSK, De a ,R e a 232
Get The Step 2 CK Drills Book: usmledrills.com
The USMLE Guys Step 2 CK Crash Course usmleguys.com

I a :
● MSK US ( f e ece a ) ca e d c e , a a ea

Da :
● C ca ba ed a de a

T ea e :
● A d ac e a ca e a d c af e ac f ca be a ded,
e b ace, NSAID ace a e , ca e a

Carpal Tunnel S ndrome

S :
● N b e / / a ed a e e e (f ee d a d ad a a f f
f d )a e a fe e ; ea a d e e e
fe e e d e , ed a ed a d a

P ca E a :
● P be e a d def c ed a e e- e a ed e f e
a d( e a e e ce a )
● P eP ae a e e
● P eT e e

E ec da c Te :
● Ne e c d c d e (c f eda )

T ea e :
● M d ca e , c c c d ec e ca a e, a c c c d
● M de a e/ e e e ca e ca dec e

Content Re ie Questions:

W c f ef e / a e e ca be ed def e da ea a e
ca a e d e?

A. Se def c ed a e e- e a ed e f e a d ca e a
B. Ne e c d c de
C. P eP ae a e e
D. P eT e e

A 46- ea - d a e e e ec c ac a f ff de . T e a e a
a a e de a e a f b e a a bee e e ed, b e ca
ba e e de . He de e a f a a. O ca e a e a e
a ed ac e a d a e a e f fe e a a a d abd c .W a e
e da ba ed e ef d ?

A. R a c ff e d a
B. R a c ff ea
C. Ad e e ca
D. G e ea e a

MSK, De a ,R e a 233
Get The Step 2 CK Drills Book: usmledrills.com
The USMLE Guys Step 2 CK Crash Course usmleguys.com

W c f ef ca e a f d c e a c ff e d a ?

A. N a a e a e f
B. P e Nee e e e
C. Ha -Ke ed e e e
D. Wea e e ed abd c a de e a a

MSK, De a ,R e a 234
Get The Step 2 CK Drills Book: usmledrills.com
The USMLE Guys Step 2 CK Crash Course usmleguys.com

M a I K
O a a K ee R e:
● I d ca e a ad a a e ac e ee a da f e
f :
○ A e 55 ea
○ U ab e bea e ( ed a e af e a d e ED) f 4 e
○ I a ed e de e f e ae a
○ Te de e a e ead f e f b a
○ U ab e fe e ee 90 de ee

Anterior Cruciate Ligament (ACL) Injur

Ca e:
● La d e a e ( a e a be d ) a e d ace e f e ba
e fe , a d a d c a e f d ec (c ), a a ee ca
ee e , a e f e ee

S :
● Of e fee a e ee, ac e f d e , a , ab f e ee,
ab bea e , dec ea ed ROM

P ca E a :
● P e Lac a e
● P ea e da e e
● P e f e

I a :
● MRI ed de f ea a d e f e ACL

Da :
● Def eda ade a MRI ee a c , e eda ade
ba ed a de a f d ( e Lac a , a e da e, f e )

T ea e :
● I ed a e e a RICE ( e , ce, c e ,ee a )a da d
e -bea
● S ca e a ca ea de e d e e f a d a e efe e ce

Posterior Cruciate Ligament (PCL) Injur

Ca e:
● H -e e a a (ca acc de ), fa /c ac a ad a e c

S :
● I e e a a: He a , ee ab , dec ea ed ROM, ab bea
e
● L e e fa /c ac a a: K ee ab , d ee eff

P ca E a :
● P e e da e e

MSK, De a ,R e a 235
Get The Step 2 CK Drills Book: usmledrills.com
The USMLE Guys Step 2 CK Crash Course usmleguys.com

● P e e a
● P e ad ce ac e e

I a :
● MRI ed de f ea a d e f e PCL

Da :
● Def eda ade a MRI ee a c , e eda ade
ba ed a de a f d ( e e da e e , e a
ad ce ac e e )

T ea e :
● I ed a e e a RICE ( e , ce, c e ,ee a )a da d
e -bea
● H ed ee b ace a a ee f e e
● P ca e a a d/ ca e a de e d de ee f

Medial Collateral Ligament Injur

Ca e :
● C ac ( a a e aea a ec f e ee), a d e f e e
e e (f e a a a a d , a e a e ee)

S :
● K ee a ( ed a ), e , ff e , c /ca c

P ca E a :
● P e a e e

I a :
● Ge e a ef ed

Da :
● H a d ca e a

T ea e :
● I ed a e e a RICE ( e , ce, c e ,ee a ) a d ace a e f
a e ef
● P ca e a

Lateral Collateral Ligament (LCL) Injur

Ca e :
● Ta a e ed a a ec f e ee, a d e f e e e e (f
e a a a a d , a e a e ee)

S :
● La e a e aea ee a , e , ab , c ca c

P ca E a :
● P e a e e

MSK, De a ,R e a 236
Get The Step 2 CK Drills Book: usmledrills.com
The USMLE Guys Step 2 CK Crash Course usmleguys.com

I a :
● N a ef ed f LCL ec ed, b MRI f e a e e f
add a a e

Da :
● H a d ca e a

T ea e :
● I ed a e e a RICE ( e , ce, c e ,ee a ) a d ace a e f
a e ef
● P ca e a e de e d e e f

Meniscal Injur of the Knee

Ca e:
● R a ee e e ee f e ed a d f e a a a ( a e
a a,c e e, a d ea e ), de e e a e ea e de

S :
● Of e a a (ab e c e ), e e 24 a e f
a a d e , a , dec ea ed ROM
● P / c /ca c / ab / ff e f ee de e e ee

P ca E a :
● P e McM a e
● P eT e a e
● P eA e e

I a :
● P a ad a (fO a ac e a e ): De e e a a d ca c f ca f e e c
● MRI ( f c de e ): Ide f e ca ea

Da :
● U a e eda ade ba ed a d ca e a . D a
ade def e b MRI a c e a de ca e a

T ea e :
● Re , ce, a d e bea ( e e e ca e , ec c e ); a e a e a
b ace f ab
● P ca e a a d/ ca e a de e d e e

Patellofemoral Pain

Ca e :
● O e e( ea e , c ea ed ca ac ), aa e ,a d a a

S :
● G ad a ac e a e ee a ; e , ed , a ,
a (a ce d de ce d ); de c bed a be a ac , e e a
a de a d e ae a

MSK, De a ,R e a 237
Get The Step 2 CK Drills Book: usmledrills.com
The USMLE Guys Step 2 CK Crash Course usmleguys.com

P ca E a :
● Pa a , a e a face / e ac e de e

Lab a d I a :
● U ece a ca e , c ca d a

Da :
● H a d ca e a

T ea e :
● Te a cea e ed ce e ace ba ac
● P ca e a

Patellar Tendonitis

Ca e
● O e e ( , d a ce , ) ff c e ec e e
f e d ea

S :
● L ed ROM e a e ee e e , a ee f e a d/

P ca E a :
● Pa e ed ee e e , e de e a fe e ee e ae a e d
ae

I a :
● T ca ef ed b MSK US ca be ed de f e d ea , e a a ,
e a c a a

Da :
● H a d ca e a

T ea e :
● Ic , a e a e d a , a d/ e a
● Te a c e f NSAID

Osgood-Schlatter Disease

P a /R Fac :
● 9 14 ea de ece ,b > , a c a a e c

S :
● A e ee a e e e f d ac a a ca a .
Pa e , , a ,c b a , ee , d ec a a.
Pa e e ed e

P ca E a :
● T b a be c e e de a a ,a d e e e ce f a b a be c e f e
e ce b e ce

MSK, De a ,R e a 238
Get The Step 2 CK Drills Book: usmledrills.com
The USMLE Guys Step 2 CK Crash Course usmleguys.com

● Pa ee e e a a e a ce a d a

I a :
● T ca d ca ed b a ad a ca fa e a ee a f e
b a be c e a a f e af

Da :
● H a d ca e a

T ea e :
● C e a e a a e e ( ce, c e f NSAIDS)
● C a f ac ( a e ab e a f a )

Bursitis of the Knee - Pes Anserinus Pain S ndrome (PAPS)

R Fac :
● D abe e , be , ee OA, fe a e e , ee aa e

S /P ca E a :
● Med a ee a , e de e a a a ed a e, a a
aa e f e ee

Da :
● H a d ca e a

T ea e :
● We , ca ea , f ee b ace, -e NSAID e

Bursitis of the Knee - Prepatellar and Superficial Infrapatellar Bursitis

Ca e :
● I fec , a a (ac e e e a a a, c c ed ee ), ,
e a da

S :
● Ac e: E e a, e , a d e de e a e e ae a ae a e d ,
ee eff , cca a fe e
● C c: N - e de , f d-f ed a e e ae a ae a e d

Lab :
● P b e e e a ed WBC c , e e a ed e c ac d
● B a f da a Ga a , bac e a c e, eb d ce c
d ffe e a ( e ed a ce a d >2000 ce / c L fa a a d e c
b ), a d a c c de f c a

I a :
● Pa ad a e f ac e e ca e f a a, e e , a d de f
f d c e f ec a e .e. b a f d acc a

MSK, De a ,R e a 239
Get The Step 2 CK Drills Book: usmledrills.com
The USMLE Guys Step 2 CK Crash Course usmleguys.com

Da :
● H a d ca e a , eGa a a d bac e a c ef e cb
a dc a de f ca f ca ed b

T ea e :
● A b c ea a db ada a e( a e e-a a e )f
e cb
● If e b c a de f ca , ea e f
● If fec c a , e ae a a d e fca f a ae a b ea ed
c e a e a a e e

Bursitis of the Knee - Baker s (Popliteal) C st

R Fac :
● O e a , e a da , e ca ea , f ee a a

S /P ca E a :
● P e ee a , a be d e ee, ee ff e , a / e dec ea e
d a ea e ee f e ed 45 de ee (F c e' )

I a :
● US: P e e ce f a a ec c a
● P a ad a :P b e de f ca f f e a ;b e ab a e
a c a ed OA, RA, a a

Da :
● P e e ce f ea a eF c e' a dc e

T ea e :
● T ea de d de , a ce e f e ee a d a-a c a
c c c d ec

Plica S ndrome

P a /R Fac :
● R ,a e c , a a (d ec b )

S :
● A e a d ed a ee a ; e ed ac ( , a ), a e,
ee , ( ed); c c ee f e

P ca E a :
● Pa ab e f b c c d e ed a fe a c d e
● S f e e ed a e a e a b de
● P e ed a a e a ca e
● Te de e f e ed a fe a c d e
● Te de e f e ed a a e a face
● T ad ce a d/ a ce
● Wea abd c
● C c / e a e ee ROM

MSK, De a ,R e a 240
Get The Step 2 CK Drills Book: usmledrills.com
The USMLE Guys Step 2 CK Crash Course usmleguys.com

Da :
● H a d ca e a

T ea e :
● Te a cea e ed ce e ace ba ac
● S - e NSAID e, c ee
● P ca e a

Content Re ie Questions:

W c f ef a e e e be e a a ed e ca f
e ee?

A. McM a e
B. P e da e e
C. Lac a e
D. Q ad ce ac e e

A 20- ea - d a e e e e ED ac a f ee a . T e a e a
a cce a d e a d c a d ec e e fe a f ed b ac e
f d ab a d ed ed a e e ED. O e a e a e a a e
Lac a e a d ea e da e e .W a e e da ?

A. A e c c a e a e (ACL)
B. P e c c a e a e (PCL)
C. La e a c a e a a e (LCL)
D. Med a c a e a a e (MCL)

Acc d eO a a ee e, c f ef a e d ece e a a
ad a f e ee a a f e e a a af e a ac e ee ?

A. A e 55 ea
B. U ab e bea e ( ed a e af e a d e ED) f 10 e
C. Te de e a e ead f e f b a
D. U ab e f e e ee 90 de ee

A 13- ea - d a e e e ec c c a f ee a . T e a e
a e f e ee a d a e e e e c a e a a a
e . He de e a a a e ee a d a e e a e e ed e .
O e a e b a be c e e de a a ,a d e e e ce f a b a be c e b
e ce a d a ee e e a a e a ce a e a a .W a
e e da ?

A. P ca d e
B. O d-Sc a e d ea e
C. B f e ee
D. O e a c a

MSK, De a ,R e a 241
Get The Step 2 CK Drills Book: usmledrills.com
The USMLE Guys Step 2 CK Crash Course usmleguys.com

A 72- ea - d fe ae a e f e a e e ec c ef ee
a .T e a e e e a a ee e a ,b a e , e ee a a
de e ed e e a 4 da . O e a a a ed be d e ee c d a ea
e e ee f e ed 45 de ee . W c f e f e e da ?

A. Ga c
B. P ea a e a e
C. Ba e (P ea ) c
D. S f e ac a

A 24- ea - d ae e ba a e e e ec c a e ee a . T e a e
e a e a bee e ce e f ced e a a d a e
e e e, a c a a d ee f e .O e a a e f
ed e a e ee e e a d a e ed ee e e .T e e a
e de e a e fe e f e a e a. W c f e f e e da
ba ed a e a d ca e a ?

A. O d-Sc a e d ea e
B. Pa e fe a a (Pa e fe a d e)
C. Pa e a e d
D. P ca d e

MSK, De a ,R e a 242
Get The Step 2 CK Drills Book: usmledrills.com
The USMLE Guys Step 2 CK Crash Course usmleguys.com

L Ba Pa
Lower Back Pain

S :
● N ec f c bac a (MSK a )

P ca E a :
● P be a a a f aa a ce , e ca e a f
ad c a ( a a e a e, a e e )

I a :
● N a d ca ed e ef ed f a a e e e :
○ P e e a d/ e ee e c def c (b a e a ea e , f
b e /b adde c , add e a e e a)
○ S f be a a c ( f ca ce , e a e , fe e ,
ea , a ea )
○ R f fec ( ece bac e a fec , IV d e, ece e d a a
ced e )
○ A e >50 ea
○ Pa e e f >4 ee

Ma a e e :
● 2-4 ee NSAID (f e)
● Ace a e
● M c e e a a (c c be a e)
● B ef d e(a e , a d)

Lumbosacral Radiculopath

S /P ca E a :
● Pa ( a , , e ec ca ), a e e a, e , f ef e , f
e , e a e a e e
● L5 ad c a : L e bac a ad a e aea e a d ef .
P bed ed e f d fe , e a de e , e
aea f e e a dd ff
● S1 ad c a :B c a ad a e f e e a df .L fa e
ef e , bed ed e f a a fe , e e e , ee f e ,
e f e a ec f e e a d e a e a ed e f e f
● S2/S3/S4 ad c a e :B c a a ad a e e e e e
e e , be a a d feca c e ce, e a d f c
● L2/L3/L4 ad c a e : L e bac a c ad a e d ea e e
e ee d e ed a e e a d e e a c f e f . T e ee ef e ca be
ed ced a d bed ed e fe a d add c a d ee
e e

U e I a S d be Pe f ed If:
● R f e a
● R f e d a ab ce
● P e e e c def c
● P e e ce f b a e a e c , a ee , add e a e e a

MSK, De a ,R e a 243
Get The Step 2 CK Drills Book: usmledrills.com
The USMLE Guys Step 2 CK Crash Course usmleguys.com

I a :
● MRI CT e e c e
● If a d e de f ca e f ad c a , e ec a (EMG) a d e e
c d c d e (NCS) ca be ed

Da :
● C ca d a ba ed a d ca e a , ca e f a a d
e ec d a c e f a e fa c e a e a a e e

T ea e :
● C e a e a a e e : NSAID ace a e , ac d f ca ,a d
ca e a
● If fa e fc e a e a a e e (4 ee ), de da c e
e a a dc de e d a c c c d ec , ca e e

Lumbar Spinal Stenosis

S :
● Ne e c c a d ca , a , b e / , e , e ea e

I a :
● MRI c a a f e a a ca a

Da :
● C ca d a ba ed a d ca e a , a c a
a f e a a ca a

T ea e :
● P ca e a , NSAID
● S ca e e (dec e e a ec )

Cauda Equina S ndrome

S /P ca E a :
● L e bac a ( , a a ad a e ( )), add e a e e a, b adde
a db e d f c , e e e e a d ef e

Ca e :
● E d a ab ce , e a a , e e eb a d c e a , ba e
d

I a :
● MRI e a ae e de ee/ca e fc dc e

Lab :
● Va de e d ca e (e e a ed WBC f fec ,a e a f a a c , e e a ed
ESR f f a a ca e; a f ab f d c e a , ba e d )

Da :
● C ca d a ba ed a d ca e a a c e
a

MSK, De a ,R e a 244
Get The Step 2 CK Drills Book: usmledrills.com
The USMLE Guys Step 2 CK Crash Course usmleguys.com

T ea e :
● Ta ed ec f c ca e (a b c f fec , e d f fa a ca e,
c e e a a d ad a ea f e a )

Osteoporotic Vertebral Compression Fracture

Ca e:
● L - a ae e , f a a

S /P ca E a :
● Pa ( a , d ) ca ed d e e, e e e d e e, ,
Va a a a e e , e e
● He
● Pa a a e e eb a ce , ea b a a e eb a c e

Da /I a :
● P a ad a bc ca e def , ed e f ac e , c e f ac e

T ea e :
● Pa c NSAID, ace a e , d de e d e e
● B aef -e a a e e ed ce a e f e f ac e

Content Re ie Questions:

W c f ef ca ee ca da e a d e?

A. E e c ac e f e a d ee
B. Sadd e a e e a
C. B adde d f c
D. L e bac a

A 32- ea - d ae a ed ca e e ec c ac a f e
bac a . T e a e ae e a be a 4 da a , d a , 5/10 e ,a d
e ca de f a e e e .O e a ee e de e a a f aa a
ce ,a e ca e a f ad c a , a a e a e, a d
a e e .W a e e be e a a e e f e a e ?

A. P a ad a f e bac
B. CT f e bac
C. L ba c e
D. S c e f NSAID f - f e ac f
e e af e 4 ee

MSK, De a ,R e a 245
Get The Step 2 CK Drills Book: usmledrills.com
The USMLE Guys Step 2 CK Crash Course usmleguys.com

A 46- ea - d fe a e e e ec c ac a f e bac a . T e a e
ae e a a a d fe d e de f e e .S ea e d e
cca a a d eed e e a e e .O ca e a e a e a e
e a d e d ce a d e a e a a ec f e .T e a e a a
d ed e f d fe ,f e ,a df e e a e a e
e aea f e e e .W a e e da ?

A. L5 ad c a
B. S1 ad c a
C. S2 ad c a
D. S3 ad c a

MSK, De a ,R e a 246
Get The Step 2 CK Drills Book: usmledrills.com
The USMLE Guys Step 2 CK Crash Course usmleguys.com

O ,O ,a B T
Osteoporosis

R Fac f Fa F ac e:
● O d a e, , e d e, f ac e, 1 de ee e a e f ac e,
b d e ,ac ab e, e ace
● D ea e c a :R e a da , ad ( e a e e a e),
a ab d de , c c e d ea e, f a a b e d ea e

Sc ee B e M e a De Te (DXA):
● A e >65 ea
● P e a a e fac ( ee ab e) f f ac e
● Me f b e a

S :
● A a c

C ca Ma fe a :
● F ac e ( e eb a , ,d a ad )

Da :
● O e f
e a e a afa f ac e OR T- c e -2.5 a da d de a
be e ad ea b e e a de a de e ed b d a -e e X- a
ab e (DXA)
● O e e a f T- c e -1 -2.5 a da d de a be e ad ea

T ea e :
● N a ac ca : Ade a e d e a ca c a d a D, e a e e c e,
ce a ,a d e ce eac e, fa e e ae e
● S e e a a D a d ca c fdea a e ade a e
● O a b a e (a e d a e ed a e) f - e a a e
T- c e be ee -1.0 a d -2.5 e a a e T- c e -2.5

Osteonecrosis (Avascular Necrosis)

Ca e :
● Ta ad ef (fe a ec f ac e, ca da d a e f ac e,
a e f ac e)
● S e c e e a
● S c e ce
● Med ca ( -d e e d , b ae )
● Ac ab e
● C ae e
● Rad a e e

S /P ca E a :
● Pa a c a (a e , c a ), a de e de , a d ae a e
f d ea e ed ROM f

MSK, De a ,R e a 247
Get The Step 2 CK Drills Book: usmledrills.com
The USMLE Guys Step 2 CK Crash Course usmleguys.com

○ Fe a ead a a c a ec e e a db c a ,
e e bea ; a e- a e d ea e e a a a d
abd c ed, a a e ed

I a :
● MRI ( d a da d) e e( - e a) ea d ea e
( e aa f a a d c e c b e)
● Add a ec d e ( - e ) a e d ea e ( e a c a a a e),
c e d b e- e f e ec

Da :
● C ca d a ba ed a d ca e a a c e
a

T ea e :
● Ra e e ca e b ae , a ea ,a c a a ,
a d a f ca ea , e e, ca ca e e ece a a d
fe f - e ea

Bone Tumor Features

O e dO e a -Be b e
-Pa e ca 20
-L ca :P a fe ( c )
-Pa a a a d e e ed b NSAIDS
-P e de e a a
-X- a a , d ce c ce c a
O e ac a -Pa e e e ad e ce (13-16 ea ) e 65
-L ca : Me a f b e (d a fe )
-L ca ed a a d e de f e a e a
-X- a ad a " b " a e f f e a
-Lab c de e e a ed a a e a a e, LDH, a d ESR
E Sa c a -L ca :L b e a d e
-L ca ed a a d e ; e e fe e , e
-F e e e /b e
-X- a de c e -ea e e c ea a ,
e e ee " a ea a ce ( e a e f e
b e ea b e f a )
-Lab c de a e a
G a Ce T f -Be ca a e e
B e -M fe e ee ad
-L ca : T ca ee ( e )
-Pa , e , ed ROM
-X- a c e e a-e ea e f b e ,
a c f ac e

MSK, De a ,R e a 248
Get The Step 2 CK Drills Book: usmledrills.com
The USMLE Guys Step 2 CK Crash Course usmleguys.com

Content Re ie Questions:

W c f ef a da e be a eada f ec ed
e ec ?

A. P a ad a
B. CT
C. MRI
D. U a d

W c f ef f d MRI c e ada f e ec ?

A. S b a ea a ce f eb e
B. D b e- e
C. B c a
D. B e a ede a

W c f ef fea e d e ee ec e af ada f e ?

A. F a f ac e f e e eb a
B. F a f ac e f e
C. T- c e -2.0 DEXA ca
D. T- c e -3.0 DEXA ca

MSK, De a ,R e a 249
Get The Step 2 CK Drills Book: usmledrills.com
The USMLE Guys Step 2 CK Crash Course usmleguys.com

O a ,R a A ,a A S
Osteoarthritis (OA)

R Fac :
● A e >50, f , fe a e e , be

S :
● J a a e e a d e e ed e , a e
af e /e e
● Pa e f a a ac ac e , ea c a a
ac ac e , c a a cca a e de f e ee a
ac e

J T ca I ed:
● F e d a e a a ea (DIP) ,f CMC ,f MTP , ee , ,
e ce ca a d e ba e face

S /P ca E a :
Ha d b e a e e e e aea de f e e a a ea
(Hebe de de , B c a d de ), e e a def , - e e de e , ed
ROM (ac e a d a e), ee ab

S a F d:
3
● <2000 eb d ce / , c ea ce ed a ce

I a :
● Pa ad a :J ace a , e e , bc da ce

Da :
● C ca d a c e a: A e 45 ea , a a e f e,
ff e a 30 e ,a d fea e a ca f e a

T ea e :
● We f e e e , ae b c a d e e ec e , a a d , ee
b ace , f e f e
● A eeded ca NSAID (f a fe ed) a NSAID , d e e,
ca ca a c ; ae a-a c a c c c d ec ( a acce e a e OA
d ea e b da a ca a e)
● T a e ace e

Rheumatoid Arthritis (RA)

R Fac :
● Fa , fe a e e , c a e e e

S /S :
● J a a d ff e ca e bef e e, e
● M a a, e , - ade fe e , bc a e d e , de e ,e ce

MSK, De a ,R e a 250
Get The Step 2 CK Drills Book: usmledrills.com
The USMLE Guys Step 2 CK Crash Course usmleguys.com

J T ca I ed:
● F e e aca aa ea (MCP) a d a e aa ea (PIP) , b
e a a ea , ,eb , de , e e a a a a ea (MTP) ,
a e , ee

P ca E a :
● Te de e a a f , a a d e c ed e e d ROM e ,
e , fe e ca a ,f e " a d f ", a ec a d
B e e def e

Lab :
3
● S a f d a e c ec f 1500-25000 eb d ce / ,
c ea ed a ce
● T b c ,a e a fc cd ea e, def c e c a e a, d e c ,
e e a ed ESR a d CRP (ESR, CRP f c ae de ee f d ea e ac )
● Of e eb a e c d e a d fac (RF) a d a -c a ed e de
a b d e (ACPA) (ACPA a e ed f a -CCP a a )

I a :
● P a ad a e a c a e e a, e f ca a ea db e,
ace a , b a

Da :
● A 3 , e RF a d/ ACPA, d ea e a 6 ee , e e a ed
CRP ESR, a d e de ce f e d ea e

T ea e :
● I a d f c ce e e ae NSAID a d/ c c c d f a c
c e a f effec f e e ae
● Fa e eac ade a e e e e e a e af e 6 , ea
e e a e PLUS d c ea d fa a a e e e ae a TNF
b

Felt S ndrome

T ad f e a da , e e a, a d e e a .

Lab /H :
● Ne e a: Ab e e c <2000/ c L, ec e bac e a fec (
a d e a ac )
● Pe ea ea a d b e a e a a ed e e ca e f
e e a (LGL e e a)
● ESR a d CRP ca e e a ed
● P e RF a d a -CCP a b d e , a d a e b a e e c a
a c ea a b d e (ANA), a -d b e- a ded DNA a b de ,a e
c a c a b d e (ANCA) a - ea b de

Ank losing Spond litis

A c a ed d d a a e c ea e (HLA)-B27.

MSK, De a ,R e a 251
Get The Step 2 CK Drills Book: usmledrills.com
The USMLE Guys Step 2 CK Crash Course usmleguys.com

S /S :
● Bac a c a be bef e a e 40, ed e , ea
,a d a a a a c d e
● E e , e , a ed f a b , /b c a , dac

A c a ed C d :
● I fa a b e d ea e (UC C ), a ,a e e

Lab :
● E e a ed e I A, ESR, CRP, a a e a a e (f b e), c c
c ca e a

I a :
● P a ad a ac ac - ace a , ce ,e e
c a e ( e a b a ), b a .S a f d c de
de e a da f e face , ba b e( a a f )

Da :
● I d d a (HLA)-B27, c c bac a , a d ac a , e
c ca fea e fa d a de c f e da e

T ea e :
● NSAID
● TNF-a a b (ada ab, f ab)
● IL-17 b ( ec ab, e ab)

Content Re ie Questions:

W c f ef c d a c a ed a d ?

A. U ce a e c
B. O e e
C. C ' d ea e
D. P a

A 39- ea - d fe a e f e a da e e ec e e a ac
fec . CBC d ffe e a ab e e c f 1000/ c L a d ca
e a e e a ed. Pe ea b d ea a d b e a e a a ae
a .W a e e da ba ed e e c ca f d ?

A. La e a a c c e e a
B. Ac e e d e e a
C. S e c fa c
D. Fe d e

MSK, De a ,R e a 252
Get The Step 2 CK Drills Book: usmledrills.com
The USMLE Guys Step 2 CK Crash Course usmleguys.com

A 54- ea - d fe ae e e ec c c a ff e a .T e a e e a
e a a e f e a df e a ae e ea d e e ed e .
S e de c be e a a ac a ,a d ca a e e d f e da a a
, af e e a dfe e a a e ec c a . O e a e e a a ea (DIP)
f eac f e a e - e e de e , b e a ROM. W a e
e da ba ed c ca e e a ?

A. V a a
B. O e a
C. R e a da
D. A d

MSK, De a ,R e a 253
Get The Step 2 CK Drills Book: usmledrills.com
The USMLE Guys Step 2 CK Crash Course usmleguys.com

S A a O
Septic Arthritis

I fec f e .

R Fac :
● IV d e/ d e ca e e , fec (e ec a e ),
e , d ea e ece a a ( e , ec ), d
a e

S :
● M a c a a ( a c a f f d ea e), e , a , dec ea ed
a e f (ac ea d a e), fe e

Lab :
● S a f da a e b d ce c >20,000 ce / c L, 75%
e c ea e c e ( e ), a eca a d e c ,
e a f dc e
● P eb dc e
● E e a ed WBC c , ESR, CRP

Rad a :
● Ide f de b e/ d ea e

Da :
● Def eda e a f d a a a d/ c e; e e
da ade f a e c ec 50,000-150,000 ce / c L, e
ed a

T ea e :
● E c a b c ba ed a a ( a e c cc e a c c )
● Ad a b c ba ed c e ce b
● Da

Osteom elitis

I fec f eb e.

S :
● Ac e e e :D a , a ,e e a, ede a a e f fec de e
e da , fe e /c
● C c e e : Occa a e e ce f d a ac , e e a ,
e e a, a d e

C ca :
● S ac , e f e fec ab ce f a , bac e e a,
b e def

Lab :
● Ac e: E e a ed WBC, ESR, CRP

MSK, De a ,R e a 254
Get The Step 2 CK Drills Book: usmledrills.com
The USMLE Guys Step 2 CK Crash Course usmleguys.com

● C c: P b e e e a ed ESR a d CRP ( fe a)
● P eb eb , a a a dc e, a d/ b d c e

I a :
● U e ad a f e 2 ee e
○ P e e ce f e e a, c , b de c
● U e MRI f d abe c a e ec ed e e f ef a df a a e
e a 2 ee d
○ Ab a a ede a

Da :
● Def eda f eb eb c e
● Pe eda f c ca a d ad a c f e e PLUS
eb dc e OR e e e e a ed ESR a d CRP
● Pe eda f b e a c e e e

T ea e :
● S ca deb de e f ec cb e
● E c a b c ea e a c c d- e e a ce a

Content Re ie Questions:

W c f ef a e c e eda f e e ?

A. Ab a a ede a MRI
B. P e e ce f c ad a
C. P e e ce f e e a ad a
D. P e e ce f b e e ea eac ad a

A 45- ea - d a e f IV d e e e e ED fe e , ee a ,
e , a , a d dec ea ed a e f ac e a d a e . Lab e c
a d e e a ed ESR, CRP. S a f da a e c ec 80,000 ce / c L,
e ed a .W a e e da ?

A. G
B. T ACL
C. Se c a
D. B

W c f ef da e ca be ed a e a def eda f e e ?

A. C ca a d ad a c f e e e e a ed ESR a d CRP
B. C ca a d ad a c f e e eb dc e
C. B e a c e e e
D. P ec e a e f b eb

MSK, De a ,R e a 255
Get The Step 2 CK Drills Book: usmledrills.com
The USMLE Guys Step 2 CK Crash Course usmleguys.com

G a P (Ca P a D a C a
D )
Gout

R Fac :
D e ( ed ea , f c e) a d a c e
Ma e e
T a a, e , e ee e
H e a
D ea e ca ae e d c ( e fe a e d de /
a a ce / e cd de )
E e defec ca e e d c (G6P def c e c )
Med ca ( a de d e c , ACE- , b-b c e , c cd )
Dec ea ed c ac d c ea a ce (CKD)

S :
G f a e: Pa , e , ed e , a , ca e a e ( a f
eaa a a ea (b e) ee), a be e e ea
, eac a e e 12-24 a d a da ee
T :C a f d a e ca - a f c c fa a a d
de c ec a e , c c a e ( e e), f e ca ed e ea
f e

Lab :
E e a ed WBC, ESR, CRP, e ae e e
S a f da a :
P e e ce f d aec a ( eed e- a ed, e a e b ef e )
Ne ed a , WBC e e a be ee 10,000-100,000

I a :
Pa ad a ( e da c ea a d ab e a ae a f d)
e e a ea a ce ( c ed- e ) f

Da :
Def eda ade b de f d aec a a
c c f a f d f a affec ed
Pe eda ca be ade ba ed ,e a a , ab , a d a
de

T ea e :
F f a e, be e a ASAP a c c c d NSAID c c c e
A ( ac e e e a e <6 /dL) ed f a e a ce e e
fa e , e e e e ce f
L fe e d f ca / ed ca e a a c d dea c a e (e ed ea ),
ac e ed c /ce a ,a d b ed a ae fac f
e be

MSK, De a ,R e a 256
Get The Step 2 CK Drills Book: usmledrills.com
The USMLE Guys Step 2 CK Crash Course usmleguys.com

Pseudogout (Calcium P rophosphate Dih drate Cr stal Deposition (CPPD))

R Fac ( e d c e e ):
S e , e e e ed ca e , a a, b ae e

S :
Pa , e , ed e , a , ca e a e ( ee f e affec ed,
b e e affec f e e e e a ) a a da ee

Lab :
S a f da a :
P e e ce f ca c aec a ( ea e b ef e ,
b d- a ed)
Ne ed a , WBC e e a be ee 15,000-30,000 ( cca a
e)

I a :
P a ad a ca a e ca c f ca ( ad de e ca a e) a d
de e e a ec a e ( bc da c , e e )

Da :
Def eda e b ef e ca c aec a de f ed
a f da a a d ca a e ca c f ca ca e ca c f ca a
ad a

T ea e :
If 2 fe e ed, a a a d a-a c a c c c d ec
T e ea ed c c c d ec ea NSAID c c c e a
c c c d
Ma e a ce c c c ef e e f ac e CPP c a a a e
e a 2 a ac a ea

Content Re ie Questions:

W c f ef ed ca effec ef e a ac e f a e?

A. P ed e
B. Na e
C. C c c e
D. A

W c f ef e e a a f da a c e eda
f a ac e a ac f ca c aed daec a de d ea e?

A. R b d- a ed c a a f da a
B. Ne ed a ce f WBC a f da a
3
C. Le c e c ce a f 20,000 e a f da a
D. Ne a e b ef e ce f c a a f da a

MSK, De a ,R e a 257
Get The Step 2 CK Drills Book: usmledrills.com
The USMLE Guys Step 2 CK Crash Course usmleguys.com

W c f ef a fac f e de e e f ?

A. T a de d e c e
B. C f ed ea
C. C f da d c
D. G6P def c e c

MSK, De a ,R e a 258
Get The Step 2 CK Drills Book: usmledrills.com
The USMLE Guys Step 2 CK Crash Course usmleguys.com

Ta a a ' A a Ga C A
Taka asu's Arteritis

R Fac :
● A a , fe a e, a e 10-40 ea

S :
● A a, a a a , a a , e , fa e, fe e , abd a a ( - a d a ),
GI b eed ( e e e c a e c e a), eaded e , c e, e ( f ca d a d
e eb a a e e affec ed)

P ca E a :
● H e e ,d ed e ea e ,b d e e d ffe e ce be ee a ,
b c a d ca ,a e a b ,e e a d , de a a e

Lab :
● P b e e e a ed ESR a d CRP, e c , b c , c c
c c a e a ( ab a e a e e WNL)

I a :
● MRA CTA d be e f ed, a e ed a a
b ead cc
● U a d e e a c e a d a a

Da :
● S / a d ca e a c e Ta a a ' a e a
(MRA CTA) a a f ea a a d/ e a ba c e f e
a a

T ea e :
● G c c c d PLUS e e ae a a e

Giant Cell Arteritis

R Fac :
● O de a e (a e e 50, b ca e a e e 70) a d e (Sca d a a
de ce )

S /S :
● Headac e ( f e e a ), a c a d ca , a e f (ca be e a e
ea e ), e , fe e , fa e, a a a , a a ,a ca e

A c a ed D ea e:
● P a a e a ca

P ca E a :
● Dec ea ed e a a e e, c e ed e a a e a d e de e a a
f e e a a e / ca , d ed e ea e ,b d e e d ffe e ce
be ee a ,b ,a c e a (a ce d a ca e )

MSK, De a ,R e a 259
Get The Step 2 CK Drills Book: usmledrills.com
The USMLE Guys Step 2 CK Crash Course usmleguys.com

● F d c ca e ae cd ca db ed a , d ca a e c
a e c e c c e a (AION)

Lab :
● P b e e e a ed ESR a d CRP, e c , b c , c c
c ca e a

Te a A e B :
● N - ec a a e ( CD4+ c e , ac a e , a ce )

Da :
● Def eda ba ed e e a a e b ,da ca a be
ade c ca a b a a ca e f e a a e ( ea e d
NEVER be de a ed f ec ed GCA b e a eb )

T ea e :
● H d e c c c d f ed b a e e d e

Content Re ie Questions:

W c f ef f d ca be ed a e a def eda f a ce a e ?

A. Te a a e b - ec a a e
B. F d c fa e ca e c e c c e a
C. B d e e d ffe e ce be ee a
D. Dec ea ed e a a e e

A 81- ea - d fe a e a f a a e a ca c e ec c
c a f .S e e a a f e a fed e e e cca a
bec e b ed. S e a e d e e a eadac e a d a e a a c a ed
c e . O e a a c e ed e a a e fe a d e a e e e de e
a a f e e a a e .F d c a e ae cd c b ed
a .W a e e be e a a e e f a e ?

A. D a ed e e e a
B. CT a a
C. A a e a a e b e
D. T ea e d e e d

W c f ef a da e a ae a eda f
Ta a a ' a e ?

A. P a ad a f ec e
B. MRI f e c e
C. Ma e c e a ce a a (MRA)
D. C e c a (HIDA ca )

MSK, De a ,R e a 260
Get The Step 2 CK Drills Book: usmledrills.com
The USMLE Guys Step 2 CK Crash Course usmleguys.com

S S a Ra a P
S stemic Sclerosis

S /S :
● CREST d e:
○ Ca c c , Ra a d e e ,E a ea d , Sc e dac ,
Te a ec a a

Add a S :
● De : Ha de a d c e f e , f a , , ce f e
f e /f e , - ede a, e e f de c e , a
ab a e
● MSK: F b f e d / ead c ac e , a a a, a a, a
● P :I e a d ea e ( afb ), a e e
● GI: A e a c a /d a ea, a b e e , feca c e ce
● Ca d : A a, c c e e ca d , e ca d a eff , ea fa e, e c e
ca d a
● Re a : Sc e de a e a c
○ O c e a fa e d e a
○ Mc a a c e ca e a
○ T b c e a
○ H e e
● GU: E ec e d f c

Lab :
● P bea e a, d e a a , e e a ed e c ea e e e , e e a ed
c ea e a e
● B a e :A c ea a b d (ANA), a e a e I (a -Sc -70),
a ce e e a b d (ACA), a -RNA e a e III

Da :
● S c e f d ef e fb aea a d e e d a e
e aca aa ea ca e a a d ab f d c
eda

T ea e :
● S :P ea , e e a e, a d/ e c e d
● Re a : ACE b
● GI: T ea d d a a fe a (a b c f a b e e ,a da ea
a e f feca c e ce, PPI e a f GERD)
● MSK: Pa ea ed NSAID
● Ca d ac: T ea d d a a fe a (a -a c , ea fa ed )

Ra naud Phenomenon (RP)

P E e :
● C d e ea e, e a e

MSK, De a ,R e a 261
Get The Step 2 CK Drills Book: usmledrills.com
The USMLE Guys Step 2 CK Crash Course usmleguys.com

S :
● E de f c d f e c c a e f e b e ed ( a a 15
), a d eed e , f e ac , e a ed e c a

Sec da Ra a d P e e See I :
● S e c ce , e c e e a , de a / ,
S e ' d e, d
● S de effec ed ca (c a ,be c )
● Ta a (f b e, b a )

Sec da Ra a d P e e Fea e :
● A e >40, ab a a f d ca a e , c ca ab fea e f c c e a c a ed
d ea e, f ed ca e/ a a ca e RP, a e e , a e c
e de

Da :
● C ca d a : Pa e f e e c d e b e e c
c a e( e b e)
○ Pe f de a ed a de a de e e fa e d ea e ae
e e ca e d e ed ca a c a a a
○ C de a f d ca a c c

T ea e :
● L fe e d f ca :A d c da d a c c d , ce a
● T ea c c e d de ec da Ra a d e e
● Ca c c a e b c e (a d e) PLUS PDE e5 b ca ae f
eeded

Content Re ie Questions:

A 42- ea - d a e e e ec c c a fd a a. O e a e a e
ed a e e e a ec a a , c e dac a d e e d e
f Ra a d e e . CREST d e ec ed. W c e c ca
a fe a a be ee a a e CREST d e?

A. C e a ea
B. Ca c c
C. C c
D. C e a ea defec (ASD)

W c f ef da c e d be ed f e a e fa a e a a
ec da ca e f Ra a d e e ?

A. CT a a f e a d
B. Rad a a e b
C. U a a e b
D. Na f d ca a c c

MSK, De a ,R e a 262
Get The Step 2 CK Drills Book: usmledrills.com
The USMLE Guys Step 2 CK Crash Course usmleguys.com

W c f ef a ca c ca c a ac e c f c e de a e a c ?

A. O c e a fa e
B. Se e e e a a
C. M c a a c e ca e a
D. H e e

MSK, De a ,R e a 263
Get The Step 2 CK Drills Book: usmledrills.com
The USMLE Guys Step 2 CK Crash Course usmleguys.com

S L E a ,A S ,
S S
S stemic Lupus Er thematosus

S /S :
● S e c: Fe e , fa e
● De : Ma a b e f a (a c ee a d e a a ab a f d ),
e ,d c d e , c e ba e e , a ec a
● MSK: A ,a a a, a a, e -e e ede a
● Re a : L e
● Ca d : Pe ca d
● He e/L : Le e a, a e a f c c d ea e e c a e a,
b c e a, ce ca a d/ a a a d/ a ade a
● P :Pe , e , e b e b
● Ne :S e, e ea e a e , e e , c
● GI: D a a

Lab :
● Le e a, d a e a, a d/ b c e a
● Ee a ed e c ea e, e a a, a, e a, a d/ RBC ca
● L C3 a d C4 CH50 c e e e e
● Ee a ed ESR a d CRP
● H e a a b e a
● P be eb a e : ANA, a -d DNA, a da b de ,a -S
(a -S ), R /SSA, La/SSB, a d U1 b ce e

Da :
● C ea f 4 f 11 c e a e:
○ P e ANA, eb a e (A -DNA, A -S , a d
a b d ), a , a ce , d c d a , a a a , e ,
e , ab a CBC, e a d de , e cd de

T ea e :
● M d: H d c e, NSAID, a d/ ed e( -e )
● M de a e: H d c e, NSAID, a d/ ed e( -e ) PLUS
a a e e e a e f eeded
● Se e e: H d e ed e PLUS c e ae c c a de ab

Antiphospholipid S ndrome

A ed de e b a d/ fe a .

S /S :
● DVT, PE, ed e c a , e, TIA, fe a , ace a ff c e c

Lab :
● T b c e a, c e e e a
● P eb a e :A ca d a b de ,a -be a2- c e -I a b de ,
a c a a e

MSK, De a ,R e a 264
Get The Step 2 CK Drills Book: usmledrills.com
The USMLE Guys Step 2 CK Crash Course usmleguys.com

I a :
● A a ae e ae be b /e b

Da :
● P e e ce f e e b /e b (DVT, PE, e) OR e a c c ca
( e a eb f ace a ff c e c / e e e eec a a, ec ec e
a e ab )
● AND ef a f ef b a e : A ca d a b de ,
a -be a2- c e -I a b d e , a c a a e

T ea e :
● He a e a ed a fa f e a a e
● L - ec a - e e a f e a a e a e bec e e a

Sjogren s S ndrome

S /S :
● Xe a (d ), c ea ed ca e , fec f e a a a d , a
ca d d a , e a ed a a a d, d e e , e , Ra a d e e ,a a a

A c a ed D ea e :
● S e c e e a
● R e a da
● S e c ce
● L a

Lab :
● P b e e e a ed ESR, CRP
● A e a, e e a, b c e a
● H e- a a b e a
● P b e e e a ed ALT, AST
● P eb a e c d : Anti-Ro/SSA, anti-La/SSB antibodies, a c ea
a b d e , e a d fac , a -CCP a b d e , ce e ea b de ,
a c d a a b de

I a :
● Ab a a a a d c a
● Ab a a a a d a (US, MRI)
○ Ga d a a e c a ec c f c , c , ca c f ca (US)
○ I e e f e a d a d ae c a (MRI)

Te /D a :
● Pa e a e c e 4
○ Sa a a db f ca c c a ade (3 )
○ A b de R /SSA a d/ La/SSB (3 )
○ Sc e e : Ref e ea d c (<5 e f e ) (1 )
○ Ab a c a face a (1 )
○ Ab a e a e (1 )

T ea e :
● A f c a ea a d/ a a, a d

MSK, De a ,R e a 265
Get The Step 2 CK Drills Book: usmledrills.com
The USMLE Guys Step 2 CK Crash Course usmleguys.com

● De e d a d e e , e d d ea e- df a e a cd
( d c e e e a e)

Content Re ie Questions:

W c f ef f d de e ea a f f ada fS e
d e?

A. P e P-ANCA
B. P e a -R /SSA a d a -La/SSB a b d e
C. Ab a Sc e e
D. Sa a a db f ca c c a ade

W c f ef c ca ce a c a ac e c fa d d e?

A. P e a e b d e e ee eec a a
B. He a e a
C. DVT f e dee fe a e
D. P e a ca d a b de

A 36- ea - d fe a e e e ec c c a f a .T e a e a bee
e e e c a f e a a e fb a d .S ea e d e
e cc e a f e a 3 da . He ab d a e a, e a a ,
a d e ANA a d a -S a b d e . O ca e a e ea a e c ed-
e a e ee e b cca c a f e .W a e e da ba ed
e ef d ?

A. S e c e e a
B. A d d e
C. R e a d a
D. S e c c e

MSK, De a ,R e a 266
Get The Step 2 CK Drills Book: usmledrills.com
The USMLE Guys Step 2 CK Crash Course usmleguys.com

P a D a
Pol m ositis

S /S :
● S e c a c e ea e ( ad a d ff c c b a , a d
f ea ed ), a a ,a a a

A c a ed D ea e:
● E a ea d , e a d ea e, ca d ac d ea e ( ca d ,
a a, ca d a fa c ), a a c ( a a , a c ea c, a c)

Lab :
● E e a ed c ea e a e, ac a e de d e a e (LDH), a d a e, AST/ALT
● M ec f c a a b d e (a c ea a b d e , a -J -1)
● M a c a ed a a b d e (ANA, a -R /SSA, a -La/SSB, a -S )
● ESR a d/ CRP a e e a ed

E ec a (EMG):
● M a c( - e a c) ca e f ea e

MRI:
● N - ec f c f d c e ede a, fa a ,a d befb
a d/ ca c f ca

M ceB :
● CD8+ ce a e e ed a fa a f ae
● D ec a f c e f be b fa a ce
● Nec c c e f be f d e fa c c e

Da :
● C ca ( e c a c e ea e ) a d ab f d (e e a ed ce
e e ) a a e c a ac e c f ed de . EMG MRI f e e ed c f
e e e ce f a a d/ de a a e a ea a ef ceb

T ea e :
● Se d a e ed e effec ed e a a e e e a e.

Dermatom ositis

S /S :
● S e c a c e ea e ( ad a d ff c c b a , a d
f ea ed ), a a ,a a a , e ee , a f d ab a e ,
G a e ,G , fac a e e a, ca c c , de a

A c a ed D ea e:
● E a ea d , e a d ea e, ca d ac d ea e ( ca d ,
a a, ca d a fa c ), a a c ( a a , a c ea c, a c)

Lab :
● E e a ed c ea e a e, ac a e de d e a e (LDH), a d a e, AST/ALT

MSK, De a ,R e a 267
Get The Step 2 CK Drills Book: usmledrills.com
The USMLE Guys Step 2 CK Crash Course usmleguys.com

● M ec f c a a b d e (a c ea a b d e , a -J -1)
● M a c a ed a a b d e (ANA, a -R /SSA, a -La/SSB, a -S )
● ESR a d/ CRP a e e a ed

EMG:
● M a c( - e a c) ca e f ea e

MRI:
● N - ec f c f d c e ede a, fa a ,a d befb
a d/ ca c f ca

M ceB :
● CD4+ ce a e e ed a fa a f ae
● I ca a e a d e fa c c a f be
● S e e ec c c e f be f d ed e f e fa c c e (b d
e e c ff e f )

Da :
● C ca ( e c a c e ea e , c a e a fe a ( )) a d ab
f d (e e a ed cee e ) a a e c a ac e c f ed de
● MRI/EMG/ ceb eeded f d a c ea

T ea e :
● Se d a e ed e effec ed e PLUS a a e e e a e.

Content Re ie Questions:

W c f ef de a ca c e a c a ed ada f
de a ?

A. Ca c c
B. He ee
C. Fac a e e a a ab a f d a
D. G a e

A 45- ea - d fe a e e e ec c c a f c e ea e . T e a e
ae a e a ced d ff c b e c b a a d a d f
ea ed .O e a e a e a d ed e e e a dfe f
b aea . Lab a ee a c ea e a e a d ac a e de d e a e. A ce
b ef ed ec c c e f be f d e fa c c e. W a e
e da ba ed e ef d ?

A. M a e a a
B. La be -Ea d e
C. P
D. G a -Ba e d e

MSK, De a ,R e a 268
Get The Step 2 CK Drills Book: usmledrills.com
The USMLE Guys Step 2 CK Crash Course usmleguys.com

W c f ef ab f d ea c e ada f ?

A. E e a ed c ea e a e
B. A -J -1 a b d
C. E e a ed ac a e de d e a e
D. Se e e e e a ed ESR

MSK, De a ,R e a 269
Get The Step 2 CK Drills Book: usmledrills.com
The USMLE Guys Step 2 CK Crash Course usmleguys.com

P a aR a aa Fb a a
Pol m algia Rheumatica

Pa e De a c :
● A e 50, fe a e > ae , e (Sca d a a de ce )

S /S :
● S e ca ec , de , ,a d stiffness a d ac a a ab
e a d a a e ; a ed ac e de abd c

A c a ed D ea e:
● G a ce a e

Lab :
● E e a ed ESR a d CRP, c ca e a

I a
● N a a ed, b fe b e ee US MRI

Da :
● A e 50
● B aea de a d/ de a
● M ff e 45
● D a >2 ee
● E e a ed ESR a d/ CRP
● E c f e d de ( a ce a e )

T ea e :
● P ed e a ed d e a ac e e e f 2-4 ee , e
a e ed e d e ac e e e
● T ea e ded b c ca a d ESR/CRP e e

Fibrom algia

S /S :
● C c (>3 ) e f e a fa a ede a,
e de e a a
● Fa e/ ee d , eadac e, a e e a, ff e , c e
d ba ce (f d), de e ,a e , GI d ba ce (IBS, GERD)

Lab :
● N ab ab a e de ec ed

I a :
● N ab a e de ec ed

W de ead Pa I de (WPI) f F b a a:
● Nec , c e , abd e , e bac , e bac 1 f eac e a ea
● R de: Ja , de , e a , e a ,b c , e e , e e 1
f eac e a ea

MSK, De a ,R e a 270
Get The Step 2 CK Drills Book: usmledrills.com
The USMLE Guys Step 2 CK Crash Course usmleguys.com

● Lef S de: Ja , de , e a , e a ,b c , e e , e e 1 f
eac e a ea

S Se e Sca e (SSS) f F b a a:
● P e e ce a ed be 0-3 de e d e e
○ 0= e 1= 2= de a e 3= e e e
■ Fa e (0 3)
■ Wa ef e ed (0 3)
■ C e (0 3)
● P e e ce e a 6 ,c e1
○ L e abd a a /c a (1)
○ Headac e (1)
○ De e (1)

Da :
● WPI >7 SSS >5 OR WPI 3-6 a d SSS >9
● S a e bee e e 3
● E c f e ca a e d ea e

T ea e :
● Ae b c e e c e
● A e d e e e aba ac a

Content Re ie Questions:

W c f ef e be ee ab a e a fa a e
fb a a?

A. E e a ed ESR
B. E e a ed CRP
C. N c ca e a
D. N ab ab a e

A 78- ea - d fe a e a ed ca e e ec c 3- ee f
de a . T e a e de e a a a e a ea, b ae a 3- ee a e
a e b aea de a d a ff e a 1 .P ca e a
e a ab e e ce f a ed ac e de abd c . Lab e e a ed ESR a d
CRP. S e de e a e .W a e e da ?

A. P a a e a ca
B. F b a a
C. O e a
D. S e c e e a

MSK, De a ,R e a 271
Get The Step 2 CK Drills Book: usmledrills.com
The USMLE Guys Step 2 CK Crash Course usmleguys.com

W c f ef ac e f e e e ca e ed
e a aea a e f a e a da ffb a a?

A. De e
B. Fa e
C. W de ead a
D. C e

MSK, De a ,R e a 272
Get The Step 2 CK Drills Book: usmledrills.com
The USMLE Guys Step 2 CK Crash Course usmleguys.com

Neph olog
E D -P a
H e ae a Ca e: Re ea e f eI ace a S ace
● T e b ea d :
○ Ta a (c e )
○ R abd
○ T d e
● Me ab c ac d
● I def c e c / e ce a/ e a
● N e ec e be a b c e
● S cc c e
● D a ed e
● P e d e ae a

H e a e a Ca e: Red ced U a P a E ce
● Dec ea ed a d e e d c
○ H e e c ad e ( e a d ea e, NSAID, ca c e b
e)
○ A e b
○ C c e a ea
○ P a ad e a ff c e c
○ Ge e c d de (c e a ad e ,G d ' d e)

H e a e a Ca e: Red ced Re e Ad e e
● P a - a d e c (a de, a e e e)
● Ad e ea a ( ac e)
● V a e-de e de e a b a ac d ( a ac b c , e )
● P e d ad e e1

H e a e a Ca e: Red ced Effec e A e a B dV e


● V
● D a ea
● Hea fa e
● C
● C d c ca e dec ea ed e f f e d e

H e ae a Ca e: Ac e a d C c K d e D ea e

S f H e a e a:
● Se e e c e ea e / a a
● ECG f d c d ef e f ea e e e:
○ Pea ed T a e e e f e PR e a a d QRS d a fP
a e QRS de e a e fa e
● C d c ab a e :R b deb a c b c , ef b d e b a c b c ,
ad a ced AV b c
● Ca d ac a a :S b ad ca d a, a e , d e c a ,
e c a ac ca d a, e c a f b a ,a da e

Ne 273
Get The Step 2 CK Drills Book: usmledrills.com
The USMLE Guys Step 2 CK Crash Course usmleguys.com

H e ae cE e e c :
● C ca f e ae a
● Se a >6.5 E /L
● Se a >5.5 E /L AND e a a e AND e e e b ea d
a ab

T ea e f H e a e a:
● S ab e e ca d ac e b a e CALCIUM
● M eK ace a insulin and glucose, sodium bicarbonate,
beta-2-adrenergic agonists
● Re e a f eb d a de d e c , ca e c a e e ,
a d/ e da

H a e a Ca e :
● Dec ea ed a c
● GI e
○ V , d a ea
● I c ea ed ace a e e f a :
○ I
○ Be a-ad e e c a (e d e ca ec a e , be a-ad e e c
a , a e c )
○ A a (e a e ab c)
○ H e a
● U a a e ce :
○ D e c (ca b ca da e b , d e c , a de d e c )
○ I c ea ed ea c c d e e
○ N - e bab e a ( .e. b ca b a e )
○ H a e e a

S fH a e a:
● M c e ea e ( a e e a e ae a)
● M cec a
● R abd
● M b a

ECG C a e : ST e e de e , dec ea ed T a ea de, ed QT e a,


c ea ed U a e a de

A a : PAC a d PVC , b ad ca d a, a a a a c a ac ca d a,
a e c a b c , e c a ac ca d a, e c a f b a

H a e a T ea e :
● P a e ace e
○ C ec a d c ec a a e e a a e a e a ef ac
a e ace e
● P a - a d e c
○ If a e a e e a a a a d a e e a
effec e e a e

Ne 274
Get The Step 2 CK Drills Book: usmledrills.com
The USMLE Guys Step 2 CK Crash Course usmleguys.com

Content Re ie Questions:

W c f ef a a a e ea e f a a e e e a e
a f 6.8 E /L?

A. I a e ca c
B. I a e a d c e
C. A de
D. F e de

W c f ef a f e ae a ECG?

A. Pea ed T a e
B. Le e f e QRS d a
C. L fP a e
D. U a e

W c f ef ed ca d e a e e e a de effec f ae a?

A. C c e
B. Ace a a de
C. T e de
D. H d c a de

Ne 275
Get The Step 2 CK Drills Book: usmledrills.com
The USMLE Guys Step 2 CK Crash Course usmleguys.com

E D -S
H e a e a Ca e :
● De d a ( ae de e ade a e e ace e )
○ GI e ( , a cf d e a, c d a ea)
○ U a e (d abe e d , cd e )
○ S e ( ea e be )
○ H aa c d ea e ( d a)
● E ce a a e
● H e c IVF

H e a e aS :
● Le a , ab , ead e e a dc a
● O c de e a
● F ca ace eb a e a e
● S ba ac d e a e
● Le e d ce e c c c ca e c a ed ac e
● Ra d c ec a ca e ce eb a ede a

H e a e aDa :
● T da e e ca e f e a e a, ea e e urine osmolalit
○ U e a (<300 / , e a a a): W - d abe e
d
○ U e a e ed a e (300-600 / ): W - cd e .
d abe e d
○ U e a (>600 / ): W - e a e a a e e .
a a d abe e d

H e a e a T ea e :
● Ac e e a e a:
○ Re ace e e a e def c 24
○ U e IV D5W (de e 5% ae)
○ F e e Na a d c e( 1 ) e d 145 eac ed,
e aea d e fe e ( 2-4 ) e Na e
a a e
● C c e a e a (>48 ) c c e e a:
○ L e e Na b 10 E /L 24 (a d ea e a 12 E /L 24
c ec )
○ U e IV D5W (de e 5% ae) f a e ab e a d ab e d c de
a e da (f ee a e e e a )
○ C ec e Na af e 4-6 a d f c ec ae a aec e
e Na 12 e Na e a a e
● C c e a e a (>48 ) c c e e a:
○ C ec e a
○ L e e Na b 10 E /L 24 (a d ea e a 12 E /L
24- c ec )
○ U e IV , e c ec f ee a e def c (0.9% isotonic
solution) a d e e ec d c ec d (D5W)

S fH a e a:
● Na ea a d a a e: Se Na 125-130 E /L

Ne 276
Get The Step 2 CK Drills Book: usmledrills.com
The USMLE Guys Step 2 CK Crash Course usmleguys.com

● Headac e, e a , b da , e e ,c a, e a a e : Se Na 115-120
E /L ( a be e e b e e a e )

H a e a Ca e : I ca dH e cH a e a
● F e : De e e f e ce a e e a d f ca c a e c ec ed e
d . If c ec ed d a e a e a h pergl cemia induced
h ponatremia
● If e e d af e d e d c ec a
e ce c a e OR e a e a a c e e e , c ec f e a e
bee d ced e a e c c a a d f , ea e serum osmolalit
○ IVIG ea e
○ Ma
○ P ae e
○ Ue e e

P e d a e a:
● H e de a e e e a e e e d c ce a e
ea ed ce a a a e e ab ( a ab a fac , ec f c e e ab
a a e )
● P f ca e e a e Na e e
● T e a acc a e ead e ab e d , a ea e e ca ed d ec
e e ca be e f ed

H cH a e a Ca e :
● ADH e e a e a ae
○ P a d a
○ Bee d e a a, ea a d a d e ( a e de )
● A ae e f ADH b a ed ed
○ Se e e e a a e (GFR <15 L/ )
○ T a de- d ced a e a
● U e ed ADH ec e ead a ed ed
○ Red ced effec e a e a b d e ( ea fa e, c )
○ H e a

S d e fI a aeA d e cH e Sec e (SIADH):


● Fa e e ec e f ADH
● ADH e dec ea ed e a d c ea ed c ce a f e
● ADH e c ea ed a b d a e (TBW)
● I c ea ed TBW e e d c ce a ad
● I c ea ed TBW ca e c ea ed a d e ce , c a e e
e ace a f d e ad a b e f e a e a

SIADH Ca e :
● Ca ce
○ S a ce ca ce
● Med ca
○ A de e a ,a c c ,a c a ,a ca ce d
● Ne ca
○ S e, a a, ac a a e a e, fec
● S e
● Ad e a ff c e c

Ne 277
Get The Step 2 CK Drills Book: usmledrills.com
The USMLE Guys Step 2 CK Crash Course usmleguys.com

● H d
● Id a c

SIADH Lab F d :
● H a e a (<135 E /L)
● H a (<275 / )
● U e a >100 /
● U e d c ce a a >40 E /L
● N a a a d ac d ba e a (f c c e d ea e)
● L e c ac d e e d e c ea ed e c e ea d e d

H a e a T ea e :
● If ac e a e a (<48 ) OR e e e a e a (<120 E /L)
○ Re e a a a dfe e d a d e c ec
○ M e
○ Ra e e d b 4-6 E /L 24- (f a c AND ac e
a d ca a e e Na b 4-6 E /L e a 6 )
○ IVF a b f 3% saline (h pertonic saline) a d ad acc d e
d e e e ea d
● If c c a e a (>48 ) a e Na f <130 E /L AND e e e
ac a a a
○ Re e a a a dfe e d a d e c ec
○ M e
○ I c ea e e e Na 4-6 E /L e a fe
○ M e 3% saline b e
● If c c a e a (>48 ) as mptomatic a e mild to moderate
s mptoms a e Na f 120-129 E /L
○ U a d e e e a a
○ DO NOT USE 3% SALINE
○ Ide f a d e e e ca e f a e a ( ea c d ca
a e a, ed ca ca a e a)
○ L ae a e
● If c c a e a (>48 )a a c a e d de a e
a e Na <120 E /L
○ Re e a a ,fe e e d c ec
○ 3% Sa e f
○ +/- dDAVP e e a d f c ec
○ Ra e e d b 4-6 E /L 24-

SIADH T ea e :
● L d e c e e- - e ca a >1
● O a a ab e f d a a ca d e Na >120 E /L
● U ea ( ace f f d e c a d a a ab e )
● Va e ece a a

O c De e a S d e (ODS) Ce a P eM e (CPM):
● R fac c de: Se d 105 E /L e ,c c > ac e, e a d ae
f c ec f e Na ( ea e a 4-6 E /L 24- )
● Ne ca c de: D a a, aa ae ad a e , e e ,
e a , b da ,c a

Ne 278
Get The Step 2 CK Drills Book: usmledrills.com
The USMLE Guys Step 2 CK Crash Course usmleguys.com

Content Re ie Questions:

If e ca e f e a e a a a e c ea , c f ef e ca e be
de e e ee ?

A. Se Na
B. U e a
C. ACTH a e
D. H e ea e e

W c f ef ae bec ca f ea ed ac e e a e a?

A. O c de e a
B. F ca ace eb a e a e
C. S ba ac d e a e
D. Ce eb a ede a

W c f ef IV f d d be a ae ea a c ca e a e c
a e e e c?

A. Lac a ed e
B. 0.9% a e
C. 0.45% a e
D. D5W (de e 5% ae)

W c f ef c d e a e a a e a ?

A. SIADH
B. H e ce a- d ced a e a
C. H a e af IV a
D. H a e af IVIG

W c f ef f d a ae a a e ac e a e aa da
e Na c ce a f 122 E /L?

A. 3% a e
B. 0.9% a e
C. 0.45% a e
D. D5W (de e 5% ae)

W a ea ae ae c ec e Na a a e c c a e a
a a ca d a a e Na f 118 E /L?

A. I c ea eb 4-6 E /L 24
B. I c ea eb 4-6 E /L e a 6
C. I c ea eb 12-16 E /L e a 6
D. I c ea eb 12-16 E /L 24

Ne 279
Get The Step 2 CK Drills Book: usmledrills.com
The USMLE Guys Step 2 CK Crash Course usmleguys.com

N a
Ne a S :
● M c : Re a c c a d e a a
● O e : Na ea, abd a a ,fa a , a e c ,fe e c
d ff c , d a, e e e c a a
● Pa e a a be a a c

S e T e: Ca c
● Ca c a a e:
○ Ma e a d 75% f a e
○ Rad a e
● Ca c a e:
○ U 15% f e
○ Rad a e
○ U a f a a e e f H >6.5

R Fac f Ca c S e :
● L f d a e( ade ae e ac f d e ) ead e
e
● H e ca c a d e aae e e , ec ae e e
● De ca c , a ,a d a e; a a e, -da a a e ,
d , c e, f c e
● Pa e , e aa d , d abe e , a d ed a e d e

S e T e: C e
● Ge e c ca e: Pa e a a e fa fc a
● Ab 1% f ad e , 5% f e c de
● Rad ce
● He agonal c ec a UA
● P e c a de- de e

S e T e: S e
● Made f a e a ae
● Ca ed b e UTI ea e- d c a (P )
● M ec fe a e
● Ca f a a c e fa ed e
● Rad a e
● Re a e c a e e a d ea e a b c

S e T e: U c Ac d
● Ma e 5-10% f e
● M a fac e e (ac d c) e H
● Med ca c d e ,c c d a ea, d abe e e ,a d c ac d
e d c c ea e f c ac d e
● Rad ce
● T ea e a a a a , c ea ed f d a e, a e da e
b (a )

Ne 280
Get The Step 2 CK Drills Book: usmledrills.com
The USMLE Guys Step 2 CK Crash Course usmleguys.com

Ne a Da :
● Be e a eda : CT abd e a d e c a
( - ad a -d e)
○ U a d f e a f e eed ec e CT ca f e ca e f
e a e fe- e
● Ma a e e de e d e e e a d ca
○ S e 5 : Pa ed (NSAID d), f d , a d c a e a
e c ec ef a a
○ S e >5 a d 10 d a e e : T ea f d , a ed , a e
c ec e, a d e e a a b c e ( efe ed) ca c c a e b c e
○ S e a fa a ed ca a a e e a d e 1.0-1.5 c e:
T ea c a e ee c
○ >1.5 c , a d c (c e ca c aae d a e), e
c e ca :U ee d c c efa e a

P beC ca f Ne a : Ob c eU a
● I c ea e e d f UTI
● H d e
● E d- a e e a d ea e (ESRD)

Med ca T e a f P e e fS e Rec e ce:


● T a de d e c
● P a c ae
● A

Content Re ie Questions:

A a e e ef f a a a d e a aa d f d a e e a a c a ee
a .A d e e a ed. W e e f a a , a e f e e
be e e ?

A. S e
B. U c ac d
C. Ca c ae
D. C e

A a e fa a a d e a a f d a ea6 e ed a
ee CT abd e a d e c a .W c f ef ed ca
d ca ed a a e e f e a ?

A. Ke ac
B. Ta
C. N fed e
D. C c e

Ne 281
Get The Step 2 CK Drills Book: usmledrills.com
The USMLE Guys Step 2 CK Crash Course usmleguys.com

W c f ef ea e d be a ae e a a e e fa a e
a d ec e c ac d d e e ?

A. U a ac d f ca
B. I c ea ed f d a e
C. Pa a a e e
D. A

Ne 282
Get The Step 2 CK Drills Book: usmledrills.com
The USMLE Guys Step 2 CK Crash Course usmleguys.com

U a I
R Fac :
● O de a e
● H f e a c
● Va a b
● Obe

T e fU a I c e ce:
● U e c c e ce
● S e c e ce
● M ed c e ce
● O ef c e ce

U e c I c e ce:
● Ca ed b de e ac
○ Id a c
○ S a c d
○ A a ca b adde ab a e
○ I ba a ce e b adde c b e

S e I c e ce:
● I c c e c def c e c
○ Dec ea ed c e a c a e
● Ue a e b
○ Dec ea ed f e e ab e e cf c a e
○ M ec be a d a a c db

O ef I c e ce:
● U a O e Ob c
○ Pe c a a e
○ Fb d
● D ed/ a ed de ac
○ S a de effe e e e da a e
○ Pe ea e a
○ O ed e f e b adde

I c e ce W -U :
● Ide f e e f c e ce a a e
● Med ca a ca ead c e ce (a aa ,d e c ,
a a e c / eda e )
● U a
● B adde e e f a e d ca e c e ce

I c e ce T ea e (I a ):
● L fe e d f ca :
○ We be e a e
○ F d e c (1.8 L/da )
○ E a caffe e
● B adde a
● T ca a a e e f e e a d e f e a e

Ne 283
Get The Step 2 CK Drills Book: usmledrills.com
The USMLE Guys Step 2 CK Crash Course usmleguys.com

● Ke e e e c e ( e e e cf ce )

U e c I c e ce T ea e :
● A ca c ed ca
● Be a 3-ad e ce a
● B ec f e de ce
● Pe c a e ba e e a ac a e d a
● S e (a e a c a , a d e , a b c ca e e )

S e I c e ce T ea e :
● E e a de ce ( e a )
● S e ( d e a )

M ed I c e ce T ea e :
● T ea ba ed ed a

O ef I c e ce T ea e :
● S e c ec b adde e b c
● C ea e e ca e e a

Content Re ie Questions:

A fe a e a e de c be e e a f eed a e a cc ed a e
a ea a e f e. Y ad e a a f Ke e e e c e , fe e d f ca ,a d
b adde a b e a e e e e .W a ca f
a ac ca ed ca d be a aef a a e c d ?

A. Ca c c a e b c e
B. Be da e e
C. A ca c
D. Be a-b c e

A a e c e ec c ac a f a c e ce e e e e a
c , c a bee cc f e a 2 . S e de e a a , fe e , c ,
d a. S e a a ff a a c db . Ba ed e , a e
e da ?

A. U a ac fec
B. S e c e ce
C. U e c e ce
D. O e f c e ce

W c f ef c d c a ac e ed b a ed de ac ?

A. U a ac fec
B. S e c e ce
C. U e c e ce
D. O e f c e ce

Ne 284
Get The Step 2 CK Drills Book: usmledrills.com
The USMLE Guys Step 2 CK Crash Course usmleguys.com

R a,Ba ,a P a Ca a BPH
S e Re a C :
● Ma d , e a,a d a
● Ma a de a ca ed
● Lac f e a a d c a
● Ma e a ec c US
● Ma a a e a ec
● N c a e a ce e CT

C e Re a C :
● I c ea ed f a a c
● I e a c e ed a
● N c ea de a ca ed
● Se a , c a , ca c f ca
● Ma a b da dc cc e
● C a e a ce e CT

Re a Ce Ca c a:
● He a a, e a a , f a a ,a d e
● L e d f c (e e a ed e a a e a a e)
● Fe e
● A e a( fc c d ea e) e c (e e a ed e e )
● H e ca ce a ( c b e e , PTH P)
● Da ed : Pa a e ec e ec

B adde Ca ce :
● R fac :
○ S
○ W ace e e( d ea, a , e e, bbe , ea e )
○ Pe c ad a
○ C cc
○ C c a de
○ O d, e, a e
● F d :
○ T ca a e e a a( a , e e c c c) a e
e 35
○ A a e e e c ,fe e c ,d a
○ U a e RBC a d e e ca e f ed e (f d
a dd )
○ C stoscop e d a da d f d a

Be P a c H e a a (BPH):
● Be a f ae a a e 45
● S : N c a, f e e c , e c , c e ce, / ea ea , e a c ,
a d
● D a ec a e a : N - e de , e a ed, e ca , , a d cc a
e ce a a ( a a e) f e ae
● P a e- ec f c a e (PSA) ca e e a ed BPH
● Da f BPH d e NOT e ec f a b

Ne 285
Get The Step 2 CK Drills Book: usmledrills.com
The USMLE Guys Step 2 CK Crash Course usmleguys.com

P a e Ca ce :
● O e f e c ca ce e
● U a a a cb e e e e a , e a a,
e a e a
● L e d f a e ca ce c ea e f:
○ PSA e e a ed ab e e a e f e a e a ec
○ PSA c ea e b >0.75 / L e ea
○ DRE e ea ae a a d e , d a , a e ca ,
a e a be a e a ce a
● Re e b c f da , a ec e d f fe e ec a c a
ea 10 ea

Content Re ie Questions:

W c f ef c a ac e c a c a ed a e e a c ?

A. P e e ce f d , e a,a d c a a d
B. Ma e a ec c a d
C. C a e a ce e fc CT
D. Lac f e a c a d

W c f ef e d a da d f eda f b adde ca ce ?

A. U a
B. C c
C. C ec
D. Pa a e ec

W c f ef f d d a ec a e a c e eda f
be a c e a a a e a a e ca ce ?

A. P a c d e
B. P a c d a
C. G e ce a a ( a a e) f e ae
D. A e f e ae

Ne 286
Get The Step 2 CK Drills Book: usmledrills.com
The USMLE Guys Step 2 CK Crash Course usmleguys.com

A K I (AKI)
AKI Def :
● Pa e de e a f ef c e a:
○ I c ea e e c ea e b 0.3 /d 48
○ I c ea e e c ea e f 1.5 e ba e e, e 7 da
○ U e e <0.5 / / e 6

Ca e f AKI:
● Pee a
○ H e
○ H e e c ae
○ A e c e e a a c a a e a
● I ae a
○ G e a d ea e
○ Re a b a d ea e
○ Re a e a d ea e
○ Re a a c a d ea e
● P e a
○ Ob c e e a

P e e a AKI I a Lab Va e :
● FENa % = ((UNa SC )/(SNa UC )) 100
● U e Na c ce a a <20 /L e e a AKI
● B d ea e (BUN)/c ea e a >20 a e e a AKI
● FENa <1% a e e a AKI
● FEU ea <35% a ee a ( e ead f FENa f a e d e c )

P e a AKI:
● U a ac b c a eea e a ac e
● S c de a/a a, e e a ed e c ea e, abd a /f a a ,
e a a
● I a e ed a d best e f a ac b c ultrasound
● T ea e e b c d e e a ac ( e )

I a e a AKI:
● U a
○ Red b d ce ca : P fe a e e e
○ Ga a a de e a ce ca /f ee e e a ce : Ac e b a ec
○ P e a e a a: N fe a e e a
○ WBC , WBC ca , a d RBC : Ac e e a e
● +/- Se c e a d e a b

Ne 287
Get The Step 2 CK Drills Book: usmledrills.com
The USMLE Guys Step 2 CK Crash Course usmleguys.com

Content Re ie Questions:

W a e be e de e e f a ac b c e e e a ac
e ?

A. U a
B. U a d
C. U e c e
D. K d e b

A a e a a d c e ed e de f e d e a a a d b e e
ee a e c ea e b 0.53 /d . W c f ef FENa % c e
a e e a ca e f AKI?

A. 0.80%
B. 1.5%
C. 2.1%
D. 3.0%

W c f ef ada cc e f ac e d e ?

A. I c ea e e c ea e b 0.3 /d 48
B. I c ea e e c ea e f 1.5 e ba e e, 7 da
C. U e e <0.5 / / e 6
D. I c ea e e e b 0.25 / 6 af e 1L NS f db

Ne 288
Get The Step 2 CK Drills Book: usmledrills.com
The USMLE Guys Step 2 CK Crash Course usmleguys.com

C K D a (CKD)
Def :
● K d e da a e dec ea ed d e f c (a ea ed b eGFR) f 3
e d a .

CKD S a ed U :
● Ca e f CKD
● GFR (G a e )
● Ab a (A a e )

CKD S a e P e:
● Ide f f e
● De e e d f de e CKD c ca
● I a a ae a a e e

C c K d e D ea e (CKD) S a e :
GFR Stage GFR (mL/min/1.73 m2)

G1 90
G2 60-89

G3a 45-59
G3b 30-44
G4 15-29
G5 <15

Urine Albumin Stages Albumin E cretion Rate (AER)


in mg/da

A1 <30
A2 30-299
A3 300

CKD a d Ca d a c a D ea e R Fac :
● R fac f ca d a c a d ea e (HTN, a e, d abe e , d de a, )
e e e a aea c a ed e d e f c
● R fac e f CKD ( e a, a e a, ca c ba a ce, a e ed b e
e a e ab , e e a ed c e ) e a e ac ca d a c a ea

T ea e Red ce CKD a d Ca d a c a D ea e R :
● Sa a e GFR e a 60 ( f da )
● U e ACE b a e e c d abe c CKD f BP c a d e a
ec

Ne 289
Get The Step 2 CK Drills Book: usmledrills.com
The USMLE Guys Step 2 CK Crash Course usmleguys.com

● Re a be ef f e e BP c (<130/80) f a e e e ce f
e a 1 /da

Ma a e e f CKD:
● T ea e e b e ca e f d e
● T ea e e ae f e f CKD
○ A a e BP c
○ A ae c ec
○ L e a ( e ACE ARB)
○ P e e c
○ S ce a
● T ea CKD c ca
○ T ea e e ad ( d e c , HD)
○ C ec e ec e ba a ce ( ed , HD)
○ C ec e ab c ac d ( d b ca b a e, HD)
○ T ea a e a (ESA , IV )
○ T ea d de a ( a , e a e )
○ H e ae a( a e b de , ca c e c )

Da Pa e CKD:
● I d ca :
○ U e c e ca d , e , e ce a a
○ Pe e ef ac e e ad
○ M dc e a e
○ Ref ac ac d , e a e a, a d e ae a
○ W e a a (a e a, a ea, a d/ e )
○ W e fa ea d aa e

A e a CKD:
● N c c c ca e a
● Dec ea ed d c fe e e d e
● Da fe c ( e e ca e f a e a)
● T ea e e - a a e (ESA ) a d/ IV
● Ta e a a Hb e e 10-11.5 /dL e d a e f ESA be

H e a e a CKD:
● E ce fa e f a e CKD
● H e a e a ead e ec e f aa d e (PTH) c ec
e ae aa d ca ce a
● Sec da e aa d a ead e a e d ( e f b a,
e a ac a, a d ad a c b e d ea e)
● T ea dea ae e c a d a e b de , ca c , a d/
ca c e c

Ne 290
Get The Step 2 CK Drills Book: usmledrills.com
The USMLE Guys Step 2 CK Crash Course usmleguys.com

Content Re ie Questions:

W c f ef e a a e ea e f a a e a f
CKD e e c e a a d f d a e e c e ca d e de ce f
e d a ade?

A. I d e ac
B. P ed e
C. C c c e
D. D a

A a e CKD f d a e a e a a d af e a - e ca e f
a e a a e bee ed .W c f ef H b a e a ae a a e
be ea ed f a e a e e - a a e d e CKD?

A. 8.0 9.0 /dL


B. 8.5-9.5 /dL
C. 10.0-11.5 /dL
D. 11.5-13.0 /dL

W c f ef ac ca fc c d e d ea e?

A. Me ab c ac d
B. A e a
C. H ae a
D. V e e ad

Ne 291
Get The Step 2 CK Drills Book: usmledrills.com
The USMLE Guys Step 2 CK Crash Course usmleguys.com

A I a N (AIN), P a D a ,a A T b a
N (ATN)
Acute Interstitial Nephritis

S :
● Ra
● Fe e
● E a
● A a a
● O a
● Na ea/
● S e e a a c

Ca e :
● D :
○ A b c (be a- ac a , f a de , f e , fa )
○ D e c ( a d a de- e)
○ NSAID
○ A
● A e d ea e:
○ Sa c d
○ SLE
○ S e '
● I fec :
○ Le e a
○ Le a
○ S e c cc
○ CMV

Lab :
● E e a ed e c ea ec ce a
● E a
● +/- P e a
● U e ed e a c de WBC , WBC ca , a d RBC ( e a a)
● FENa >1 ca e

Da :
● Def eda ade d e b , e eda acce ab e
e d ca e AIN a ed c e e a dc d
e d ce a
● Kd e b f d :
○ I e a ede a a d f a e c d T c e , c e ,e

T ea e :
● Ce a f ca a e d (fa cab e)
● G c c c d ea d e b c e AIN

Ne 292
Get The Step 2 CK Drills Book: usmledrills.com
The USMLE Guys Step 2 CK Crash Course usmleguys.com

Prerenal Disease

Ca e :
● V e
GI (
○ , d a ea)
He
○ a e
Med ca
○ (d e c )
T a a (b
○ ,c e e a f ac e ca d ace
e e a )
○ I ade a e f d a e
● Hea fa e
● C
● H e
○ H e c c
○ Med ca (a e e e )
○ Ca d e c c
○ Se c c
● D affec e a affe e a d effe e a e e
○ NSAIDS
○ Ca c e b
○ ACE-I
○ ARB

Acute Tubular Necrosis (ATN)

Ca e :
● A e a e ca e a e e a d ea e ead e a c e a
● Se :
○ H e
○ Ac a f e b e d f bac e a
○ I c ea ed eac e e ec e
○ A e a f a ac e e
○ I c ea ed c e e ea e
● Ne
○ A c de
○ Rad c a a e
○ C a
○ Va c c
○ He e
○ Ma e d

S f P e e a D ea e a d Ac e T b a Nec (ATN):
● S fc d c a e e de ca e ( ea fa e, c , a a,
e c.)
● P e e a d ea e a e e a e e ca e d e c
● O c a a be ee ATN

P e e a D ea e Ac e T b a Nec (ATN):
● Response to fluid replacement

Ne 293
Get The Step 2 CK Drills Book: usmledrills.com
The USMLE Guys Step 2 CK Crash Course usmleguys.com

○ G d a da d f d be ee e e a d ea e a d ATN af e
a aef d e c a e e e f e , e c ea e
e e e ba e e 24-72
○ Se c ea e e ba e e = e e a d ea e
○ Se c ea e e a e e a ed = ATN
● FENa
○ FENa a <1% e e a d ea e
○ FENa a 2% ATN
● U a
○ N a f d e e a d ea e
○ ATN dd b ca , e e a ce ca , a d f ee e a b a
e e a ce

P e e a D ea e a d Ac e T b a Nec (ATN) T ea e :
● A aef d e c a a a e c e d a c a a d
de ade a e e f f e d e
● Ce a f e ca e e e f e e a

Content Re ie Questions:

W c f ef c d c e ef d f dd b ca a d
f ee e a b a e e a ce a ?

A. P e c cca e e
B. Ac e e a e
C. P e e a d ea e
D. Ac e b a ec

W c f ef ed ca ac a a d e ef e ed c e effec e a
affe e a e ed a ?

A. Na e
B. L
C. L a a
D. F e de

W c f ef c d c e a d e b e a
ede a a d f ae c d T c e , c e ,a de ?

A. M a c a e d ea e
B. Ac e e a e
C. A d e ( e ed a e )
D. Ra d e e e e

Ne 294
Get The Step 2 CK Drills Book: usmledrills.com
The USMLE Guys Step 2 CK Crash Course usmleguys.com

N S
C a ac e c :
● Ede a
○ Pe b a ( e ), eda ede a
● H ab e a (<3.5 /d )
● P e a (>3.5 /24 )
● H e de a
● L d a

Ne cS d e Se e a:
● P e def c ( a )
● H a a b e a (I fec )
● H e a (f e ee ab e a)
● T b e b
○ DVT, PE, a d renal ein thrombosis

Me b a Ne a (MN):
● S b d f be e a de ead d c e da a e ead e c
d e ( a MN)
● Me a a be d e a de ( a MN, ec ec da MN)
● Ca e f ec da MN: SLE, d (NSAIDS), fec (He B, C, ), ca ce
● T c e f e e a ba e e e ba e c c
● I Ga dc e e de f e ce ce
● Da d e b OR e a -PLA2R e f a d e f c
a d e de ce f ec da ca e f MN
● T ea e ca e, e e ea f f e

M a C a e D ea e (MCD):
● CHILDREN, ad
● T ce d ce e e fa e a e eab fac , ca f ce
f
● P a ec da (d (NSAID ), ca ce ( e a c), a e )
● Normal e a c c (LM)
● NO I G c e e de f e ce ce
● Diffuse effacement f e d c e f ce e e ec c c (EM)
● F da ad e e d e b ;c de e e a e >50
/ /da a d ab e a
● T ea c c e d

F ca Se e a G e ce (FSGS):
● Ad , e c de ce Af ca A e ca a d e
● P a FSGS ( e c d e), ec da FSGS ( be , HIV, e ), e e c
FSGS ( a e e a c d)
● Ide f d e b
● Se e a ce f e LM
● U a I G c e e de f e ce ce
● P d c ef ce efface e EM

Re a A d :
● D ff e e a de fa a e ae a LM

Ne 295
Get The Step 2 CK Drills Book: usmledrills.com
The USMLE Guys Step 2 CK Crash Course usmleguys.com

● A d C ed a a e- ee b ef e ce de a ed
● A dfb EM
● AL a d = de f b c a ( c a a a f
e a f ca ce)
● AA a d = de fa dA( e a da )

Content Re ie Questions:

W c f ef f d c e e a a d ?

A. D ff e e a de fa a e ae a c c
B. Ma ed e a de c a fT c e
C. C ed a a e- ee b ef e ce de a ed
D. A dfb e ec c c

W c f ef d be a a ae e / ced e a e a def eda f


e ba e a ?

A. Se ab e e
B. 24- e e e e
C. A c ea a b d e e
D. K d e b

W c f ef c d c a ac e ed b a f d c c a d
f e ce ce a d d ff e efface e f d c ef ce e e ec
c c ?

A. Me b a e a
B. M a c a e d ea e
C. P fec e e
D. A d

Ne 296
Get The Step 2 CK Drills Book: usmledrills.com
The USMLE Guys Step 2 CK Crash Course usmleguys.com

G
C a ac e c fG e e :
● He a a
● P e a
● Le c a
● H e e
● Re a ff c e c
● O a

A S d e (He ed a Ne ):
● M a e e e c d e IV c a e f e d e ( e ), c c ea
( e e a ea ), a d e e (a e e c )
● U a fa f e a a, e a fa e, a d deaf e
● Da ca be ade ec a e e c e
● Da ca be ade d e b
○ L d a f e a a de a f e GBM e ec c c
○ I a a e f e IV c a e ab e ce a ab a
e IV c a e c a f e GBM
● N d ea e ec f c ea e , ACE- b f a e a e e e a

I A Ne a :
● H e c de ce 20 a d 30 , Ea A a a d Ca ca a > Af ca
● S a c e a a( e a a c c e a URI,
e eaf e )
● Sec da I A e a c , ce ac d ea e, HIV
● Ge e a be d ea e c ea d e eda ade ba ed c ca
a d ab
● Def eda ade d e b ( ef ed f e e f d f
e e a ed e c ea e e e e e e ce )
● D ff e mesangial fe a a d a e a LM
● IgA deposits e mesangium f e ce ce
● De e de e mesangium EM

P e c cca G e e :
● M c c d e a d ad e 60, a e e a fe a e
● Occ 1-3 ee af e A e a a d 3-6 ee af e A e
fec
● Lab f d : Complement C3 and CH50 are decreased, positi e strepto me test
(a - e ,a - a da e, a - e a e, a - c a de-ade e,
d c e da e, a -DNa e B a b d e )
● C ca d a ba ed ac e e a d ece A e fec ( a
c e e e e )
● Kd e b a ef ed
● E d ca a fe a a d e e LM
● Starr sk " a a a e f C3 de e e a ca a a a dI G
de f e ce ce
● Humps d e- a ed be e a de (I G a d C3 de ) ee EM

Me b a fe a e G e e (MPGN):
● MPGN af d , a ec f c c ca d a

Ne 297
Get The Step 2 CK Drills Book: usmledrills.com
The USMLE Guys Step 2 CK Crash Course usmleguys.com

● MPGN de f ed a d e b
● I ec e - ed a ed MPGN: He a Ba dC ,c c bac e a,
a e d ea e, c a a a
● C e e - ed a ed MPGN: Ge e c e f a fc e e fac
● MPGN ec e c e e ed a ed: TTP, HUS, TMA
● H e ce a a d c e f e e a ba e e e ba e LM
● I b a d/ c e e de f e ce ce (de e d
e f MPGN)
● S be d e a a d e a a de EM
● L e c e e e e a e

Ra d P e eG e e (RPGN):
● M ec de ad ,b be a d d a
● Ca ed b e f ee c d :
○ A - e a ba e e e b a e (GBM) a b d d ea e
○ I ec e - ed a ed
○ Pa c - e( a ANCA- e a e e a c , ec GN
ec e de )
● H e ce a c c fe e a crescent c e e e a f LM
● L ea de f I G a -GBM (G d a e) d ea e ee
f e ce ce
● B ea e GBM ee EM
● C e ce a d e d e e e a , ca e e be
d e

Lab Te Pa e S ec ed f G e e :
● Se C3 a d C4 c e e e e
● ANA e e
● A -d DNA e e
● ANCA e e
● A -GBM a a b d e e
● Se f e a Ba dC , HIV e
● Se f ee c a a d e f a

Content Re ie Questions:

A 7- ea - d b ec c c ef c a f e a a. T e c d a a fa
f e a a, e a fa e, a d deaf e . W c f e f f d d
e ec ee e ec c c fa d e b ?

A. L d a f e a a de a f e GBM
B. D ff e efface e f e d c ef ce e
C. P e e ce f a dfb
D. N a ca f d e ec c c

Ne 298
Get The Step 2 CK Drills Book: usmledrills.com
The USMLE Guys Step 2 CK Crash Course usmleguys.com

W c f ef c d c a ac e ed b a ea de f I G ee
f e ce ce?

A. A - e a ba e e e b a e (G d a e) d ea e
B. P e c cca e e
C. A d e ( e ed a e )
D. M a c a e d ea e

W a e e da a 25- ea - d e ae a f e a
be 48 a a d e a a a ea e a 24 ?

A. A d e ( e ed a e )
B. I A e a
C. P e c cca e e
D. He c a

Ne 299
Get The Step 2 CK Drills Book: usmledrills.com
The USMLE Guys Step 2 CK Crash Course usmleguys.com

A Ba D

N a P :
● N a a e a b d H: 7.35 7.45 (7.40)
● N a CO2: 35 - 45 (40) H
● N a HCO3-: 22 - 28 (24) E /L

Ide f P a Ac d-Ba e d de :
1) L a H
2) If H e a 7.35, ac de a c ec f e e HCO3- (<22) d ca e ab c
ac d PaCO2 (>45) d ca e a ac d
3) If H ea e a 7.45, a a e a c ec f e e HCO3- (>28) d ca
e ab c a a PaCO2 (<35) d ca e a a a

Metabolic Acidosis

Ca e :
● Sae c c ea e ac d d c
● Sae c dec ea e ac d ec e f e d e
● L f HCO3-
● D ac d

E a a Me ab c Ac d :
W e F a:
PaCO2 = 1.5 [HCO3-] + 8 2

A Ga :
A Ga = Na - (C + HCO3-)

N a A Ga Me ab c Ac d :
● HH ea e a
● A Add d ea e
● R Re a b a ac d
● D D a ea
● A Ace a a de
● SS ac e
S Sa e f

A Ga Me ab c Ac d :
● M Me a
● U U e a ( d e fa e)
● D D abe c e ac d ( e e e .e. Be a- d b a e, ace ace c ac d, a
e ce a)
● PP e e c ( e ca e f a e a ,da e a )
● I I a d ab e
● L Lac c ac d ( c e a)
● EE e e c (e e e eed e- a ed ca c aaec a )
● S Sa c a e

Ne 300
Get The Step 2 CK Drills Book: usmledrills.com
The USMLE Guys Step 2 CK Crash Course usmleguys.com

De a-De a Ra A Ga (AG) Me ab c Ac d :
De a-de a a :
(AG)-12/24-(HCO3-)

● A de a AG/de a HCO3 <1:


○ C e normal AG e ab c ac d
○ Ea e 4 e a b a ac d bef e d e f c de e a e ( a
e ce ;a e e RTA a -a a e ab c ac d )
○ C d ( e ac d , e e e ) ee d e f c a a d e
ac d a ca be e c e ed
● A de a AG/de a HCO3 e a 1 d ca e e e ce f AG e ab c ac d (
e ed ac d-ba e d ba ce)
● A de a AG/de a HCO3 a f 1-2 d ca e e e :
○ H AG e ab c ac d ( ac c ac d ) ee d e f c a ed
ac d a ae e c e ed a d e a c c a OR
○ Me ab c a a c e a AG e ab c ac d OR
○ H AG e ab c ac d a d e a e a a e e a ed ba e e HCO3 a a
e fc c e a ac d
● A de a AG/de a HCO3 a >2:
○ Me ab c a a c e a AG e ab c ac d OR
○ H AG e ab c ac d a d e a e a a e e a ed ba e e HCO3 a a
e fc c e a ac d

Metabolic Alkalosis

Ca e :
● M e e f d e ce ( a e a)
● C ac a a
● E e a a ad a
○ E ce e b d/b d d c (c a e e ab ed d b ca b)
○ S d b ca b e ea ac c ac d e ac d
● L f d e f d e
○ M e a c c d e ce ( a ad e )
○ L a de d e c ( ec da e ad e )
● L f d e f GI ac
○ V a a a a c c ( f HC )
○ D a ea f a a e ab e (d a ea a a c a ed ac d f
f b ca b )

A a e Re a C e a Pa e Me ab c A a :
● I c ea e a e a PaCO2 b 0.7 H f e e 1 E /L e e HCO3-

Respirator Acidosis

Ca e :
● Dec ea ed e e a (e a ae da e)
● I c ea ed dead ace
● I c ea ed e ab e e e a ed CO2 d c

Ne 301
Get The Step 2 CK Drills Book: usmledrills.com
The USMLE Guys Step 2 CK Crash Course usmleguys.com

A a e Me ab c C e a Pa e Re a Ac d :
● HCO3- c ea e b 1 E /L f eac 10 H e PaCO2 ac e e a
ac d
● HCO3- c ea e b 4 E /L f eac 10 H e PaCO2 c c e a
ac d

Respirator Alkalosis

Ca e :
● H e e a
○ Ne ca ( a , a c a ac /a e , e)
○ A ed e
○ H e a( a de, d ea e)
○ Pe a c

A a e Me ab c C e a Pa e Re a A a :
● HCO3- dec ea e b 2 /L f eac 10 H dec ea e PaCO2 ac e
e a a a
● HCO3 dec ea e b 5 /L f eac 10 H dec ea e PaCO2 ac e
e a a a

Ne 302
Get The Step 2 CK Drills Book: usmledrills.com
The USMLE Guys Step 2 CK Crash Course usmleguys.com

R a T b a A (RTA)

T pe 1 (Distal) Renal Tubular Acidosis

E : Fa e f H+ ec e e f e

Ca e :
● A e d ea e (S e , RA)
● H e ca c a ( e aa d , ac d )
● Ge e c d de ( a e e c d d)
● Med ca (a e c B, f fa de, )

Re I :
● U e H 5.5 ea e
● Dec ea ed ea e ce
● H ae a

T pe 2 (Pro imal) Renal Tubular Acidosis

E : Fa e f a b ca b a e eab

Ca e :
● Fa c d e
● M c a a a e
● Ge e c d de

Re I :
● U e H e a 5.5
● H ae a

T e 3 (M ed) Re a T b a Ac d :
● Ra e e e c d de ca ed G ba d-Va e d e ca ed b ca b c a da e
2 def c e c a a fea e f e 1 a d 2 RTA
● A c a ac e ed b e e , ce eb a ca c f ca , a d e ec a d ab
● Ra e ee e US, a e ae ca f A ab c de ce

T pe 4 Renal Tubular Acidosis (RTA)

E :H ad e ,ad e e e a ce

Ca e :
● D abe c e a
● C c e a e
● Ac e e e
● Med ca (NSAIDS, ca c e b , ACE , ARB , a - a d e c )
● C e a a ed ad e

Re I :
● U e H e a 5.5
● H perkalemia

Ne 303
Get The Step 2 CK Drills Book: usmledrills.com
The USMLE Guys Step 2 CK Crash Course usmleguys.com

Content Re ie Questions:

W c f ef c d e e a e e a ed a a e ab c
ac d ?

A. D a ea
B. Ace a a de e
C. S ac e e
D. I a d e

W a da a a e CO2 e a c e a be f a a e a e
b ca b a e e e f 18 E /L?

A. 33.0-37.0 H
B. 34.0-38.0 H
C. 32.0-36.0 H
D. 32.5-36.5 H

Ca c a e ea a ef a e : Na: 140 / C : 100 / HCO3: 18.

A. 20 E /L
B. 22 E /L
C. 24 E /L
D. 26 E /L

W c f ef c d ca e e a e a a a ea
f ed b a e e a ed a - a e ab c ac d ae e d ea e c e?

A. I ab e ed e
B. A ed e
C. A e c B ed e
D. E e e c c

W c f ef - a e ab c ac d c d c a ac e ed b a e H 5.5
ea e , dec ea ed ea e ce ,a d a e a?

A. T e 1 (d a ) e a b a ac d
B. T e 2 ( a ) e a b a ac d
C. T e 4 e a b a ac d
D. D a ea

W c f ef e a b a ac d a c a ed e e e ce f e ae a?

A. T e 1 e a b a ac d
B. T e 2 e a b a ac d
C. T e 3 e a b a ac d
D. T e 4 e a b a ac d

Ne 304
Get The Step 2 CK Drills Book: usmledrills.com
The USMLE Guys Step 2 CK Crash Course usmleguys.com

Neurolog

La b -Ea S a M a aG a
Lambert-Eaton S ndrome

Pa :
● A b de a a e a c a e- a ed ca c c a e

S /P ca E a :
● P a c e ea e ( e e, d ff c a d f ,
c b a ) ed b ef e e c e ac a , a e ed a , ce
ac e /c a , dec ea ed dee e d ef e e a ef e a ed b ef e e
c e ac a ,d , ,a dd a

Lab :
● P ea b de P/Q- e a e- a ed ca c c a e (a -P/Q- e VGCC)

E ec da c Te :
● Re e e e e a a e e c eee a c d c e ac
e a a de ( e a 100% c ea e)

Ma a c :
● A a e be e a a ed f a a c (e ec a a ce ca ce )
○ CT f e c e , abd e ,a d e d be e f ed ea c f
a a c
○ SOX a b d e e f a ce ca ce

Da :
● C ca e e e e e a f d c e
La be -Ea S d e

T ea e :
● T ea de a a c
● F de a e/ e e e ea e ea a fa d e
● I a e e b IVIG f fa e e d a fa d e

M asthenia Gravis

Pa :
● A b de a a e a c ace c e ece / ece -a c a ed e
f da e e c a c

Ne 305
Get The Step 2 CK Drills Book: usmledrills.com
The USMLE Guys Step 2 CK Crash Course usmleguys.com

S /P ca E a :
● P ,d a, d a a, d a a, ea e c e , a ce
ea e ( b , ec )

M asthenic Crisis

P ec a Fac :
● I fec , e , a a, c db , ed ca ( e c a b c e ,a b c ,
be a-b c e )

S :
● W e e a /b ba ea e

T ea e :
● I ba a d/ ec a ca e a e a
● IVIG a a ee d e e d

Lab :
● A b de e ace c e ece (AChR-Ab), c e- ec f c e a e
(MuSK-Ab), -de e ece - e a ed e 4 (LRP4) c ae
ace c e ece -a c a ed e

E ec da c Te :
● Re e e e e a e e dec ea e c d c e ac
e a a de eac b e e

A c a ed D ea e:
● T c ,a e e a cd de
○ C e CT MRI d be e f ed e c af e a e a
a da

Da :
● C ca ( a )e e ea a b d e (AChR-Ab, MuSK-Ab,
LRP4), a d/ e ec da c e

T ea e :
● S a c ea :P d e
● I e e e a : P ed e a a e
● M a e cc ea e : P a a e e IVIG
● T ec (f a e e a e <60 ea d)

Ne 306
Get The Step 2 CK Drills Book: usmledrills.com
The USMLE Guys Step 2 CK Crash Course usmleguys.com

Content Re ie Questions:

F a e da f La be -Ea d e, c f ef e d be
ef ed?

A. EKG
B. CT f e c e , abd e ,a d e
C. C e ad a
D. DEXA ca

A 74- ea - d a e a 40- ac - ea f e e ec c ce
ea e . T e a e ae e a d ff c a d f a a dc b
a .S e ae e e a fe e a ced e a ed e a dad
c .O e a e e a ef e a f e a e a ef e . O e e e e e a e
e a e a a e e c eee a c d c e ac e a a de f
150% c a ed ba e e. Ba ed e ef d , a e e f f
ca ce affec a e ?

A. L ca c d
B. S a ce ca ce
C. S a ce ca ce
D. L ade ca c a

A 56- ea - d fe a e f a e a a e e e a e e.
T e a e a a e HR 106 BP 102/67 S O2 71 T 101.6. Y ec be a e c
c .W c f ef e a ae e e a a e e ?

A. I ba a d ec a ca e a
B. IVIG
C. P a a e e
D. H d e e d

Ne 307
Get The Step 2 CK Drills Book: usmledrills.com
The USMLE Guys Step 2 CK Crash Course usmleguys.com

G a -Ba S ,B ,a C Ja bD a
Guillain-Barr S ndrome (GBS)

P eced I fec :
● Ca bac e e , CMV, EBV, HIV, a

S :
● S e c c e ea e ( f e be e a d ), ab e dec ea ed
ef e e , bac a , d a a, fac a e e a e , e e e e a a d b ba
c e ea e , a e e a f e a d a d fee , a e a

Lab :
● CSF e e a ed a e , a WBC
● A -GQ1b a b d e M e F e d e( a a f GBS)

E ec da cS de :
● Ne e c d c d e (NCS) a d e ec a
○ De e a f dec ea ed e ec d c e c ,
c d c b c
○ A a f dec ea ed d a a d/ e a de

I a :
● MRI c e /e a ce e f e a eca a e e a d f e
ca da e a

Da :
● Rece (12 28 da ) URI d a ea e
● B a e a , f acc d ea e f e b c e d d ed ab e dee
e d ef e e
● CSF e e a ed e e e a d CSF WBC c <50 ce / c
● E ec cf d c e GBS
● N e de f ab e ca e f ea e

T ea e :
● Mec a ca e a f e e e ca e d e e a /b ba
ea e
● Pa a ee a e e b (IVIG)

Botulism

Pa e :
● C d b

Ne 308
Get The Step 2 CK Drills Book: usmledrills.com
The USMLE Guys Step 2 CK Crash Course usmleguys.com

R e f I fec :
● F db e- e ca ,c e ca c a a ed f d, e ( fa <1 ea ),
d

S :
● B aea ca a e a e (b ed , ,d a, a , fac a
ea e , d a a, d a a), e c de ce d ea e , d a a a c
aa , a ee ,c a , ac e a a d e a a , fe e

Lab a d I a :
● T ca ab a e

E ec da cS de :
● E ec a a d e ec d c d f d c de dec ea ed c d
c e ac e a,f e e c - a e e e e e e e a

Da :
● B aea ca a e a , e c de ce d ea e , e ,a d
fe e

T ea e :
● I ba a d ec a ca e a f e e d e a fa e
● B a
● Deb de e f db e ca e, a a e ff db e e ca e( e
aa c e e e )

Creut feldt Jakob Disease

Pa e :
● P

S :
● Ea : Dec ea ed c ce a , ac f d e a d e ,a a a d
de e , a a c a , e a a
● Ra d e e de e a
● M c af e be a ed, a , b ad e a, d a, d , a a a,
e ef e a, e Bab , a c ,a e c

Lab :
● P e CSF ea - e a - d ced c e (RT-Q IC), e 14-3-3 e
CSF

Ne 309
Get The Step 2 CK Drills Book: usmledrills.com
The USMLE Guys Step 2 CK Crash Course usmleguys.com

I a :
● B a MRI (d ff e ed a ) e e e a f e ce eb a
c e ,c a , ca da e ead, a d a e
● EEG c b- a c e dc a a ec e e

Da :
● Def eda ade ba b ( a ef ed, c f ed
a )
● P bab e d a ade f C e fe d Ja b D ea e a e
ea - e a - d ced c e (RT-Q IC) OR e e de e a PLUS
○ 2 f ef :M c , a da e a a da d f c ,
ce ebe a a d f c ,a e c

T ea e :
● N ea e , d ea e ca fa a 1 ea

Content Re ie Questions:

A 47- ea - d fe a e e e e a ac a f d ff c b ea .V a
c de BP 107/64 HR 87 T 98.6 SPO2 88%. T e a e 6L e a a a ca a
a d be a e .A c e e a de a e a e
bec e d e ed a d a e e b ea a bec e e d ff c . S O2 80% 6
L a a ca a. W a e e be e ?

A. S a IVIG e a
B. S a a a ee
C. E e e ac e
D. E e e ba a d ec a ca e a

A 52- ea - d fe a e e e e a ac a fb aea e ea e . T e
a e e e c c e ea e e e a a e e e ed e
e a 8 .S e e e c a a de f 3 da f d a ea a ee a . O
e a dee e d ef e e f e a e a a e ab e b a e a . W a e e f d
CSF a a e c ca e e a ?

A. CSF e e a ed e
e e a d a WBC c
B. CSF a e e e a d a WBC c
C. CSF a e e e a d e e a ed WBC c
D. CSF e e a ed e e e a d e e a ed WBC c

Ne 310
Get The Step 2 CK Drills Book: usmledrills.com
The USMLE Guys Step 2 CK Crash Course usmleguys.com

A 68- ea - d fe a e e e f e .T e a e ba d e a
e e a 1 e a e a ac ed e ab c ce a e a d e f ac e f
da e . He e e a a e c a d f e d , fa , a d ac e a
e ed e .O e a e a e de ae c af e be a ed a d
a de f ed e e e a . H e ef e a, e Bab ,a d a c e
e e e e ed. A ba c e ef ed a d CSF a a ae e a
e ea - e a - d ced c e .W a e e da ?

A. A e e d ea e
B. C e fe d Ja b d ea e
C. F e a de e a
D. Va c a de e a

Ne 311
Get The Step 2 CK Drills Book: usmledrills.com
The USMLE Guys Step 2 CK Crash Course usmleguys.com

M S (MS) a A La a S (ALS)
Multiple Sclerosis (MS)

S /S /P ca E a :
● O c e , e c ea a e a, e d a a ,L e e ,
U ff e e , eadac e, e , b e /b adde d f c , de e ,
a a a e , a a e a, a c , a d f c

M e Sc e (MS) D ea e S a e :
● C ca a ed d e: F e de f MS c ca
● Re a -e MS: A ac f b e e f c ee
ec e
● Sec da e e MS: Re a -e MS e e a e e e ce ead
e f ( ea e )
● P a e e MS: P e e d ab f e fda

Lab :
● Ce eb a f d c a I G ba d

I a :
● H e e e e ( a e ) e e e ( de e )f d e
a e a ea f e b a a d e a c d MRI

McD adDa cC e a f MS:


● 2 e a c a ac 2 ec e d MRI e
● 2 e a c a ac 1c e d MRI e a d e e
fa e e
● 2 e a c a ac 1c e d MRI e , e e f
a e e b add a e MRI f e a a e a ea f CNS
● 1 a ca ac 2 ec e d MRI e d e a
e MRI (b e a c a d e a c e e c a ed d MRI)
CSF c a ba d
● 1 a ca ac 1c e d MRI e a d add a e
(a a c) MRI f e a a e a ea f CNS d e a e MRI (b
e a c a d e a c e e c a ed d MRI) CSF
c a ba d

T ea e :
● D ea e- d f e a e : Na a ab, c e ab, a e ab, fa ab,
c ad b e
● MS e ace ba :S c e -d e e d ea ,a d f fa , a a
e c a e
● S -ba ed a a e e

Ne 312
Get The Step 2 CK Drills Book: usmledrills.com
The USMLE Guys Step 2 CK Crash Course usmleguys.com

Am otrophic Lateral Sclerosis (ALS)

L e M Ne S /S :
● Wea e , ed ced ef e e , cea , fa c c a , cec a ,f d ,
a d f c ( add )

U e M Ne S /S :
● Wea e , e ef e a a c ef e e , a c ,c

B ba S /S :
● D a a, d a a, e d b ba affec ( a ae a ,c ),
a a

C eS :
● E ec ef c , be a a c a e ,f e a de e a

A cS :
● C a , a e c

Ne c a Re a Fa e:
● Tac ea, acce e a c e e, f e e a e b ea be ee
d , a ad ca abd a b ea
● Re dea ac e a d e a e e a

E ec da c Te :
● N c d c b c e e a d c d c e c e ca a,
e a de a d e c e ca a
● M c e de e a e e (f b a e a , e a a e )
● C c de e a a d e e a e e

I a :
● MRI ca a ALS

Lab :
● T e c de e d ea e ( cca a c ea e a e e e a ed ALS)

Da :
● S / f e e e a d e e d ea e e b
b d e e , OR / f e e e e d ea e 2
e b b d e e , AND
● Ab e ce f e de ce f e d ea e ce e a a ca e e e a d/ e
e d ea e

Ne 313
Get The Step 2 CK Drills Book: usmledrills.com
The USMLE Guys Step 2 CK Crash Course usmleguys.com

T ea e :
● R e a d eda a e( e f d ea e)
● S -ba ed a a e e

Content Re ie Questions:

A 30- ea - d fe a e e e e a c a f e e e. T e
a e ae a be a a fe a a d a e bee e e e
e. T e a e a e c e c a ,b f e e 6 a
e ed e a ed ea e e e c a ed f 10 da a d e ed
. O MRI e e e ea d e e e aef d e e a e a ea
f eba a d e a c dc e d e a a d ea e . W a
e e da ?

A. M e ce
B. A c aea ce
C. P
D. L e d ea e

P e e ce f c f ef CSF f d c e eda f e
ce ?

A. G c e e e f 2 /d
B. O c a b G ba d
C. P e e e 50 /d
D. T a e b d ce c 2000 ce / c L

W c f ef a a a c a ed eda fa c
aea ce ?

A. D a a
B. Fe e f
C. Fa c c a
D. H e ef e a

Ne 314
Get The Step 2 CK Drills Book: usmledrills.com
The USMLE Guys Step 2 CK Crash Course usmleguys.com

S a H a L a V
Meniere s Disease

S :
● Ve , ea ,

Da :
● 2 e e de a 20 12
● P e e ce f a d ( ,d ed ea ,f e )
● A d e e e e a - d-f e e c ea
● E c f e ca e

T ea e :
● Va d a e a (be a e) d e c ( d c a de/ a e e e) e e
e
● Ac e ea e f e be da e e e b a e a (c a e a )
a d a e e c ( da e )f a c a ed a ea

Benign Paro smal Positional Vertigo (BPPV)

S /S :
● Ve a c a ed ead e e , a

P ca E a :
● P e D -Ha e a e e

Da :
● Re ea ed e de f e (<1 ) c a e be f ead e e
● C f ed b a d D -Ha e a e e

T ea e :
● Pa c e e a e e (E e a e e , Se a e e)
● P e e a e ea be a e

Vestibular Neuritis and Lab rinthitis

S :
● Se e e e (ca a ea/ ), a ab
● C d c de ed ab e ab e e e aea
ea

P ca E a :
● S a e aea a ( a) a ca be e ed a
f a

Ne 315
Get The Step 2 CK Drills Book: usmledrills.com
The USMLE Guys Step 2 CK Crash Course usmleguys.com

● P e ead e e

T ea e :
● P ed e a e

Lateral Medullar Infarction (Wallenberg S ndrome)

S /S :
● D ff c , a d , a ( ea / ed ad a e a de), b ed
/d a, a , a a d/ a a a f e aea a
● L f a a d e ea e e a ec aae a a d b , aea
fac a a
● I aea H e' d e
● D a a, d a, d a a, cc
● Fa e f a a c e a d ee

P ca E a Te (HINTS - Head-I e, N a , Te f S e ):
● Head e ef e e e ed
● N a a c a e d ec (de e d a e)
● Te fS e

Da :
● CT MRI aea ed a fa c

T ea e :
● PA e a ae
● S a c ea e (feed be, eec ea , a ed ca , e c.)

Presb cusis (Age-Related Hearing Loss)

S :
● S e c ea ( fe e c ) e e e e ea , ,
e

P ca E a :
● N a e ea ee c
● W e ed ce e ea def c

Da :
● C f ed aa d a

T ea e :
● F e ea e ea ad

Ne 316
Get The Step 2 CK Drills Book: usmledrills.com
The USMLE Guys Step 2 CK Crash Course usmleguys.com

Acoustic Neuroma (Vestibular Schwannoma)

S /S :
● Hea , , ead a , e a f ee , fac a ae ,
a d/ a ( e a e a a)

P ca E a :
● Webe e - d aea e e a ea
● R e e -a c d c >b ec d c b ea

A d e :
● A e c e e a ea

I a /D a :
● MRI CT e e e a a d ca a bee e e
ce ebe ea e

T ea e :
● S ca ad a ea e e

Aminogl coside Ototo icit

Pa :
● Da a e e c c ea a d/ - e e a ce

S :
● Hea ,d e b / e ,

Pe e :
● O ce-da d
● M e c ce a

Da :
● Pa e a da d e c e da

T ea e :
● W e b e, c a e d ffe e a b c e e f e c

Ne 317
Get The Step 2 CK Drills Book: usmledrills.com
The USMLE Guys Step 2 CK Crash Course usmleguys.com

Content Re ie Questions:

A 38- ea - d fe a e a e e ec c ac a fd e .T e a e
ae a e e e e ead a d , e e e e ce e f 10-15
ec d bef e e e. O ca e a , a ed d
D -Ha e a e e.W a e e da ?

A. Me e e d ea e
B. Be a a a e
C. Ve b a e
D. Lab

W c f ef a c a ac e cf d f Me e e d ea e?

A. Ve
B. Hea
C. H e d e
D. T

A 62- ea - d fe a e a ed f e ee e .T e ce de f ed a a a ac
fec c ca ed b a d e a a . Af e 3 da f ea e e a e
e ea .W c f ef ed ca e c b e e
c e a e e e e c ?

A. Cefa
B. P e ac - a bac a
C. Me e e
D. Ge a c

Ne 318
Get The Step 2 CK Drills Book: usmledrills.com
The USMLE Guys Step 2 CK Crash Course usmleguys.com

D a
Mental Status Scales

M -Me a S a E a a dM ea C eA e e :
● U ed c ca c ee a e f dc e a e de e a
● B e 30- ca e
● B a e e a , e ,c ce a , a a e, a a f c

Al heimer Disease

S :
● Me def c , f , a ed e ec ef c , be a a d ba ce,
e e , fac d f c , f a e ed ee , d a a

A e e T :
● M ea C eA e e (M CA) M -Me a S a e E a a (MMSE)

I a :
● MRI ed ced ca a e (a e ad ed), ed a e a be a
● A d PET ca de ec a d a e

Lab :
● L ce eb a f d (CSF) A 42
● I c ea ed a a a d -a

Da :
● P e e ce f e e e de e a d e , dec e f a
e ba e e f c , e fe e ce da ac e , de / a c a c
d de e c ded, c e a e c f ed a a e a d ed eab e
f a
● AND c e a e e a ed a ea 2 f e f :
○ D f c e a ed e e be e f a
○ D f c e a ed a c e
○ D f c e a ed a a ab e
○ D f c e a ed a a ef c
○ C a e e a a a e be a

T ea e :
● C e ea e b :D e e , aa a e, a e
● N- e -D-a a a e (NMDA) ece a a : Me a e
● V a E e e a

Ne 319
Get The Step 2 CK Drills Book: usmledrills.com
The USMLE Guys Step 2 CK Crash Course usmleguys.com

Vascular Dementia

R Fac :
● D abe e , HTN, e de a, , CAD, A-f b, da e

S :
● Step ise c e dec e, a ed e ec ef c ,c e a e ,
de e , c de / a c a , e d b ba affec ( a ae
a )
● Acc a f e ce eb a a e e d ea e (a a a, f
e a a d ea e , a a e , b ad e a, d , e ef e a)

A e e T :
● M ea C eA e e (M CA) M -Me a S a e E a a (MMSE)

I a :
● MRI fa c , c b eed , e a e e e e , a d/ ba a

Da :
● S f a c a de e a f e OR e a c e
e/ce eb a a e e d ea e a d de e a a ce eb a c a f d ae
ff c e ca e c e a e

T ea e :
● A a e ea e f e acc d a e e b e
● T ea fac f ec e e (HTN, d abe e , e c.)
● If e ec e dec e, c e ea e b e a (d e e , a a a e,
a e)

Frontotemporal Dementia

A e fO e:
● Pea e 60-69 ea (ea - e de e a)

Be a a Va a S :
● D b ( a a e be a ), e a ,c e be a ,a a , f
e a , f

I a :
● MRI f ca f a e a a

Ne 320
Get The Step 2 CK Drills Book: usmledrills.com
The USMLE Guys Step 2 CK Crash Course usmleguys.com

Da :
● Be a a a a e e e c d ed a d a e a 3 e f e
f :a a , fe a ,d b ,c e be a ,
e a ,d e ec e e c c f e

P a P e e A a a Va a S :
● A a a f d e

I a :
● MRI f ca a ef a e a be

Da :
● P e e ce f d ff c aa e, c e a ca ef a ed f c ,
a a a be e e e a c f ac f f ca
a ( e) a e ca e ( b e f ca a ef a e a
be ) a d e c f e c a a a d c a cd de

T ea e :
● M df ee e , c ea ed e
● Se e a e b (c a a ) a d e

Dementia with Lew Bodies

S :
● C ed f c (e ec ef c , a a f c ,a da e /a e e
d ed), fea e f a ( ead- e d , e e , b ad e a),
de a ed a d ec e a a c a , REM ee be a d de , ec e fa
dde a a, c e( a c e )

I a :
● MRI e e a f ed a e a be a d ca a c e ( e
A e e )
● SPECT PET ed ced d a e a e a e ba a a a
123
● M ca d a c a - a e d e-MIBG

Da :
● Pa e de e a a d 2 f e f :
○ C a e c ( a e /a e e )
○ Rec e a a c a
○ Pa
○ REM ee be a d de
● OR e f e ab e e f ef b a e :
○ SPECT PET ed ced d a e a e a e ba a a a
○ M ca d a c a - a e 123 d e-MIBG

Ne 321
Get The Step 2 CK Drills Book: usmledrills.com
The USMLE Guys Step 2 CK Crash Course usmleguys.com

○ P a REM ee a a

T ea e :
● C e a d be a a :C e ea e b (d e e , aa a e,
a e)
● Se e e c c : L -d e a ca a c c ( e a e)
● Pa : Le d a
● REM ee be a d de : Me a

Content Re ie Questions:

A 61- ea - dae b ec cb fe d e be a a c a e .T e a e
fe e e a e be a a ae b c, f e e a a e .
He a a ee e ac fa e be c d a dc d e e
e a e affec a e a d ca a d . He a c ee ed a c a
ac e ef a d c e e e ed. Ba ed e e , a
f f de e a e be affec a e ?

A. De e a e b de
B. F e a de e a
C. Va c a de e a
D. A e e d ea e

W c f ef f d c e eda fA e e d ea e?

A. L ce eb a f d A 42
B. I c ea ed ce eb a f d a a
C. M ca d a c a - a e 123 d e-MIBG
D. MRI ed a e a be a

A 74- ea - d a e f CVA b ec cb a
ca e a e . T e e fa e ffe ed a e 3 ea a c a ef e a e
e d a ea e ef e . T e a ced d a c c a e e e fe
fa e ' c ,a d ece fa e c f ed ab eea d e .
W c f ef e f de e a c e ?

A. De e a e b de
B. F e a de e a
C. Va c a de e a
D. A e e d ea e

Ne 322
Get The Step 2 CK Drills Book: usmledrills.com
The USMLE Guys Step 2 CK Crash Course usmleguys.com

Ca a H a a a C ba H a
Subdural Hematoma

Ca e :
● Ta a, ce eb a a ,a b c ed

S :
● Ac e: Headac e, , a a a, c a a e e a e , c a d ,c a
( e e c d e d af e )
● C c: L - eaded e , eadac e , c e a e , e ce, e e

I a /D a :
● N c a ead CT -de (ac e) de e/ de e (c c)
c e ce c c ec , a c a a ,b c d a a ac e

T ea e :
● S ca e a a e ac a
● N ea e a a e e ac a a e e a d e a ead CT

Epidural Hematoma

Ca e:
● Ta a ba e ead ea f e dd e e ea a e ( fe )
a e e ea a e

S :
● L fc c e ,d e , eadac e, ,c f , e ae ,
e e ,c a( e e c d e d af e )

I a /D a :
● N c a ead CT e - a ed b c e a e c d e c
a a ,b a c d a a ac e

T ea e :
● C a a d e a a e ac a , a f e b eed e e
● N ea e a a e e e a ead CT a d c e e c b e a f
e a e d a e a a d

Cerebral Midline Shift and Herniation

Ca e :
● S bd a e d a e a a, , ab ce , ace eb a e a e, a e
c e c e, b c e d ce a

Ne 323
Get The Step 2 CK Drills Book: usmledrills.com
The USMLE Guys Step 2 CK Crash Course usmleguys.com

S :
● Md e f : A e ed fc c e , a ede a, C ad
( e e , b ad ca d a, e a e a ), c a
● U ca e a : A e ed e a a , a e a CN III a ( a d a
e e , e e de a aea a dd a d), c aae a e ae ,
dec ca e dece eb a e

I a /D a :
● F d e f : CT MRI f e ead d a ce a d e c e( e
e cd ) a f ed
● F e a ( ca ) e a : CT MRI f e ead ed a d ace e f
e c (e a be) f e e a (ae a e c e), a e ca
fe db a d ace e

T ea e :
● E e e c e ca dec e (de e d ca e f e e a ed ac a a
e e ca d e f e a )
● P e , ca e a a a ed ead e e a , e a ea ,
a d e e a dec ea e ICP

Content Re ie Questions:

A 88- ea - d a e e e e a af e a fa . T e a e e ea ed e
ac b e a a d ead e de a . T e e a fc c e b
e e a e e eadac e. N c a ead CT a -de c e ce c
c ec ac e e e cc e c c e a a .W a e
e da ?

A. S bd a e a a
B. E d a e a a
C. G b a a f e
D. Ob c e d ce a

Da a e c f ef b d e e e c ca e fe d a
e a a ?

A. A e e ea a e
B. M dd e e ea a e
C. Ve eb a a e
D. I e a ca d a e

Ne 324
Get The Step 2 CK Drills Book: usmledrills.com
The USMLE Guys Step 2 CK Crash Course usmleguys.com

W c f ef ae e a ae e a a e e fa a e de f ed
c ea ed ac a a e e ead ca e a ?

A. Ne ca dec e
B. E e a f e a e ead
C. Ma e a ea
D. H e a

Ne 325
Get The Step 2 CK Drills Book: usmledrills.com
The USMLE Guys Step 2 CK Crash Course usmleguys.com

Ta I A a (TIA) a S
Transient Ischemic Attack (TIA)

Ta e e d f e c a ca ed b f ca ce a e e e (b a ,
a c d, e a) c e a, ac e fa c e e .

Ca e :
● E b
● Lac a / a e e a e e e
● La e a e , -f

S :
● D e , e a a, a a a, d a, d a a, ea e b e (face, a ,
a d/ e ), /c a

I a :
● Brain MRI (d ff - e ed a ) de f TIA
● Neuro ascular imaging c d CT a a (CTA) a e c e a ce
a a (MRA), a d/ a d (ca d d e a a , a ca a
D e a a )
● Cardiac e aluation a 12- ead ECG a d b e 24- ca d ac ,
b e ec ca d a

Lab :
● CBC, b d c e, PT, PTT, INR, b d d

T ea e :
● If f TIA, a a a e f a a c d e
● A a fb a a e b c ce f TIA: T ea a fa
● I e a ca d a e e (50-99%) ca TIA ea ed ca d
e da e ec
● S a e e d ea e ca TIA ea ed a aee,a e e e, a d a

Ischemic Stroke

Ca e :
● T b
○ A e ce ,a e (Ta a a G a ce ), d ec
● E b
○ Hea a a, a a d ea e, ece MI, fec e e d ca d ,
ca d a
● S e c ef
○ Ca d ac fa e (MI, a e , a a), e a
● C a a d de

Ne 326
Get The Step 2 CK Drills Book: usmledrills.com
The USMLE Guys Step 2 CK Crash Course usmleguys.com

○ P e C S def c e c , c e ce a e a, c e a e a, a d
d e

He a cS e:
● I ace eb a e a e
○ H e e , a a, ce eb a a da a ,a c a a ea ,d
(c ca e), a c a a f a ,a e e
● S ba ac d e a e
○ R ed a e (be a e ), a c a af a

S /S fS e Ba ed I ed Ve e :
● Pe e a e e : Wea e f e face, a , a d e ( e e ae )
● Ve eb ba a : C a a e e a e , d e ,d a, a a a, a ea/ ,
d a a, d a a, cc ,c a
● A e ce eb a a e : M a d/ e def c e ec a e e ,b a a
a d face
● M dd e ce eb a a e : M a d e def c f e face a d a e a e ,
e a a
○ If d a e ee bea a a
○ N -d a e ee be aea a a e ec
● P e ce eb a a e : H e a a, a a c a ,ca a e e
III a

Da cI a :
● N c a ead CT a d de f ace eb a b eed a d f e ea fa c
( f a - e a e d ffe e a e ba a a a, c ca
a e a a d ca efface e , b c a fS a f e) a e ca
a a
● Ma e c e a ce a (MRI) d ff - e ed a (DWI)
e e da de ec ac e c e c e( ed e f e e beca e
f a d a d f ead CT)
● ECG a d 24- ca d ac a e f a a a a ca e e b

Treatment with Intravenous Alteplase (tPA) for Ischemic Stroke

C e a:
● C ca f e c def c d e c e c e
● O e f e a 4.5 a ea e
● N c a d ca ( ece c e e a e, c e a c a a ed, ce a
a a ce )

Ne 327
Get The Step 2 CK Drills Book: usmledrills.com
The USMLE Guys Step 2 CK Crash Course usmleguys.com

Mec a ca T bec T ea e :
● Mec a ca bec ca be ed f e a e c c a a a a ea e
cc ca e ac e c e c e, f de f ed 24 f
e

A c a a T ea :
● If ca d e b c ce de ec ed (af b, ef e c a b , b a c a ed
ec a ca a e) -e a c a a a fa d ec a
a c a a

A aee T e a :
● A aee e a a 48 f f ea e f ac e
c e c e ca d e b c ( -e d a a aee e a
a a dc d e f - TIA c e c e)

Intracerebral Hemorrhage

S :
● Headac e, , b da ,c a
● I c ea e c def c (ac e e, e e) de e de e e a d a ea f
ba ed

T ea e :
● S a c a a a da aee d a d e e ea c a a ed ca
● A e eb d e e a a e e
● Ee a ead ff e bed, cd e a , e c a da a e f
ce eb a f d, e e a , a ac cc a

Subarachnoid Hemorrhage

S :
● W eadac e f fe a dde , a d e , ec ff e , ,
fc c e , e e

I a /D a :
● N c a ead CT e de eb d e ba ac d ace
da c
● O ce d a ade d a b ac a a (DSA) CT a a
ed de ec ac a a a e

Lab :
● L ba c e e e a ed e e e, e e a ed RBC c , a c a

Ne 328
Get The Step 2 CK Drills Book: usmledrills.com
The USMLE Guys Step 2 CK Crash Course usmleguys.com

T ea e :
● Ma a cb d e e <160 H ea a e a e e <110 H
d e (ca c c a e b c e)
● S ca c e d a c a c e a ea e

Content Re ie Questions:

A 80- ea - d fe a e b e a ef ded ea e . T e a e e
e a e e e ced ea e a d f e ef de f e face, a , a d e f e a
36 b fe a d a ab e eac e e ca 911 e da e a ed a d
f d e . Head a de f e a ac e c e c e, a d f e - ECG a d
ec ca d a d e de f a e a ca d e b c ce. W a e
a a e ea e f a e ?

A. Wa fa
B. IV a e a e
C. A
D. Mec a ca bec

W a e a a f e af e e a f fac f ed ac e c e c
e a IV a e a e ca be ed f ea e ?

A. 3
B. 3.5
C. 4
D. 4.5

W c f ef f d ba c e c e eda f
S ba ac d He a e?

A. E e a ed e e e
B. E e a ed a WBC c
C. E e a ed RBC c
D. Xa c a

Ne 329
Get The Step 2 CK Drills Book: usmledrills.com
The USMLE Guys Step 2 CK Crash Course usmleguys.com

S
Ca e :
● Ac e c e c e a c e, ba ac d e a e, bd a e a a,
a c a af a ,ba , ec a a, a a c b a , b a ab ce ,
e , e de e e a e de e a, d a a (a c , be d a e e ),
e ab c ba a ce ( e ce a, ca ce a, a e a,
a e e a, e a)

T e :
● S e ,fa , d c

A a:
● R e a c e a , fac a d a c a ,d

Focal Sei ures

F ca Se e I a ed A a e e :
● A a ( e ea d a e , fac a ac ,c e , ac )

F ca Se e Re a ed A a e e :
● S a ba ed e a f ec e a d ed b e e( eec
d ff c e , ee fa ,d ed a a e ce )

Generali ed Sei ure

Ge e a ed T c-C c Se eS :
● L fc c e ,e e f bac a d b , a e a (c a ),
e fe c ac , eb ,f a a a , fb e b adde
c

A c Se e :
● Ra d fc f e ce (e )

Ab e ce Se eS :
● See c d d, b ef a e de e ac cc 10-100 f e a
da

M c c Se e S :
● Ra d, b ef cec ac ( a a ) ca a ed c c e

Ge e a ed C eSa E e c :
● 5 e fc e e
● 2d c ee e e c e e ec e fc c e be ee e e

Ne 330
Get The Step 2 CK Drills Book: usmledrills.com
The USMLE Guys Step 2 CK Crash Course usmleguys.com

P c a P a e:
● Pe d f e e a e - c ca be a a c
c a ac e ed b c f , f ca e c def c (T dd a a ,a a a,
b e )

Lab :
● G c e, e ec e , ca c , a e , CBC, e a a d e f c e ,
a ,a d c c ee a e f e ab c d ba ce fec ead
e e

E ec ca d a :
● A e f a a ead c e(a e a e e)

E ec e ce a a (EEG):
● E e f ab a e ( ee 25% f a e e e )

I a :
● Ba MRI a e f c a ab a e c a ca e e e

Epileps

C ca C e a:
● 2 ed e e e a a ed b e a 24
● 1 ed e ea da bab ff e e e ba ed e a a d
e ec e ce a a (EEG)

T ea e :
● F c e a e e c , IV be da e e( a e a )a d -ac
a e ed ( a a e, f e , e e ace a )
● A e ed ea a e a f ec e e e /af e
e e e c 2 e e e (ca ba a e e, e , a a e, aba e ,
a e, a a e)
● Ab e ce e e ea ed e de

Content Re ie Questions:

W c f ef e e ea a ae e a a a a e a
e e c ?

A. Ob a IV acce
B. T a fe EEG ab c f ac e e e
C. Ad e IV a e a a d -ac a e ed
D. Ad e IV a ae

Ne 331
Get The Step 2 CK Drills Book: usmledrills.com
The USMLE Guys Step 2 CK Crash Course usmleguys.com

A 5- ea - d c d b ec c c a f a e . H fa e ae a
ec d e ac a d a e ff ace f a d 15 ec d a d e
fa e d e ca e a c d a e d a e. A EEG ef ed e
c e a ab e ce e e. W c f e f e a ae a
ea e ?

A. La e
B. Ace a a de
C. T a ae
D. E de

A 41- ea - d a e e e e a f fc c e .H b e ed
ee e a d ae e a e fe e d, beca e ff, a d a ed e b
e a ca . T e a e a a ed d e de. O e a a e ace a b e.
T e e ae c e c f ef d f e e?

A. A c e e
B. Ab e ce e e
C. F ca e e a ed a a e e
D. Ge e a ed c-c c e e

Ne 332
Get The Step 2 CK Drills Book: usmledrills.com
The USMLE Guys Step 2 CK Crash Course usmleguys.com

T a C a
Essential Tremor

S :
● B a e a ac e f a d a da ( e e ead a d ce), cc
e e a d e a d ac e a
● Made eb a e ,e c e e
● Re e ed b a c

I a :
● O e ed f a e a e ca e e a e a d Pa
d ea e e a ( a ece a )
● S aa d a e a e a a e e a e

Da :
● Ac e e e f 3 ea e affec b aea e b ( e e
ead, ce) e e ca ab a e

T ea e :
● T ea a d e

Ph siologic Tremor

S :
● N b e de a c c a ce b ca a e f e e a ed a e c
ac (caffe e, d , ed , e )
● Ac e ade e e e
● Re f be e e a f e ec a fac d e
c e a de e a e

Cerebellar Tremor

Ca e :
● M e ce , e affec e ce ebe

Pe e a :
● Te e a e a da ac e a e , ce ebe a (a a a,
d e a), ba f e ead a d ec

Parkinson Disease

S :
● M :Te (a e , , a a e a ), d (c ee ead- e),
b ad e a, a ab

Ne 333
Get The Step 2 CK Drills Book: usmledrills.com
The USMLE Guys Step 2 CK Crash Course usmleguys.com

● N :C ed f c /de e a, c ( a c a ), de e ,
c a

I a :
● O e ed f a e a e ca
● S aa d a e a e a aa d a e def c e c

Da :
● Pa e b ad e a PLUS e e d e a d a c ea , be ef c a e e
d a e c ea
● T ee d be fa e aeda e (ce ebe a ab a e , a e a e )

T ea e :
● Le d a (d a e ec )
● D a ea (b c e)
● M a e da e e B (MAO B) b (a a e, e e e)
● A a ad e
● A c e cd ( e e d , be e)

Wilson Disease

S /S :
● Ne :Te , a ,d a a, d a, a ab a e , c ea
● L e : RUQ a , a d ce, e a e a (c ), a c e , e e a
● E e: Ka e -F e c e

Lab :
● C b - e a e e ca e a
● T b c e a
● L e e f e ce a , e e a ed a c e e ce
● L a a a e aa e
● E e a ed e a a fe a e AST ALT >2
● Dec ea ed c ac d e e

I a :
● Abd a US, CT, MRI a e a c ea ; a , d a e (c );
a d/ e e a de e d d d a a e

Da :
● Da ade e ce a e e , Ka e -F e c e ,a d
e e a ed a c e e ce ( e b e e ca a a be e ed f
ab e e aee ca )

Ne 334
Get The Step 2 CK Drills Book: usmledrills.com
The USMLE Guys Step 2 CK Crash Course usmleguys.com

T ea e :
● T ea e ac e c ea a e (D- e c a e e e)
● Ac e e fa e ea ed e a a

Huntington Disease

Ca e:
● A a d a HTT e e a

S /S :
● M : C ea ( a , e a, e ce e e ), e e f
a c ,d a a, d a a, a ca a , a
e ef e a,
● M d/c :I ab , de e , a c a , e ec e dec e
(e e a e 24- e a d ca e)

I a :
● Ba MRI ca da e a

Da :
● C ca fH ' D ea e e e e c e ( a e ed
a a a f HTT e e e a )

T ea e :
● S e ca e
● T ea c ea e c a a e a e e2 b (de e abe a e,
e abe a e) f c ea de e ec d- e e a a c c ae ed
(a a e, e d e, a a e)
● De e a da e ea ed SSRI ( e a e, f e e)

Content Re ie Questions:

A 31- ea - d a e e e ec c a c ef c a f e .T e a e ae a
f e a 3 ea e a ced a e affec b a d a da c e e
e e a d d a ea a e . He e a ac ee a e e e be e . O
e a e e ed e a e a e a ca ,b e
e ca ab a ed. W c f e f e e da ?

A. E e a e
B. P c e
C. Pa d ea e
D. W d ea e

Ne 335
Get The Step 2 CK Drills Book: usmledrills.com
The USMLE Guys Step 2 CK Crash Course usmleguys.com

W c f ef f d c e eda fW d ea e?

A. H e ce a e e
B. Ka e -F e c e
C. E e a ed a c e e ce
D. C b - e a e e ca e a

A a fa e a a f e , a 72- ea - d a e de e aa d a e
a e a c aa d a e def c e c . W a e
ca a c a ed e e da ?

A. P c
B. R d
C. B ad e a
D. C ea

Ne 336
Get The Step 2 CK Drills Book: usmledrills.com
The USMLE Guys Step 2 CK Crash Course usmleguys.com

H a a
Migraine

P d a S :
● Ya , de e , ab ,e a

M a e A a:
● P be a (b , , a e ), a d ( , ea ),
e ( ), (e e e )

M a eT e :
● S e , e a , ea , ea e c a e

M a e C a ac e c :
● U a e a , e e e, a eadac e a 4 72 , a ea/ ,
b a, b a, a d a

Da :
● C ca d a , a ea 5 eadac e c a 4-72 ,a d a e2 f e
f c a ac e c :U ae a, de a e- e e e a , a , ade e
da ac e . A ac da c f a ea e f ef : b a,
b a, a ea,

T ea e :
● Pe e a e ea e be a b c e ( e ), a de e a (a e),
a c a ( a a e)
● Ac e a e a ac ea e NSAID ( a e ), a ( a a ),
a e e c( e c a de), ca c e e- e a ed e de (CGRP), a a
( e e a )

Tension-T pe Headache

C a ac e c :
● B ae a, d- de a e e d c eadac e f e / e e/f e a
a e da , be ba b a, a a a
e c a a ( ead, ec , de ) ce

Da :
● A ea 10 e de , d a f 30 1 ee , f a ea
a d a ea f ef :
○ Pe e a f a , b a e a ca , d- de a e e e ,
ade e da ac e . Pa e ca a e 0-1 f e f e
e de: ba ba

Ne 337
Get The Step 2 CK Drills Book: usmledrills.com
The USMLE Guys Step 2 CK Crash Course usmleguys.com

T ea e :
● P e e a e ea e c c c a de e a (a e)
● Ac e ea e NSAID ( b fe )

Cluster Headache

C a ac e c /D a :
● 5 e e ee ae a b a, a b a , a d/ e a eadac e f 15
3 d a . Pa e ca a e eadac e be ee 1 e e e da a d 8 da
a de e e b f ef a ac :
○ Se e f e e e a a
○ A c aea a ( a a c e , ea
c c a ec , ac a , , , e e d ede a, fac a ea )

T ea e :
● P e e e ea e ea a
● Ac e ea e e ( efe ed) a

Idiopathic Intracranial H pertension (Pseudotumor Cerebri)

Pa e De a c :
● T ca e e fe a e f c dbea a e

S :
● Headac e, a b c a , a e

P ca E a :
● Pa ede a, ca a e e a , e ea a fed

I a :
● MRI a e c e a ce e a (MRV) a empt sella, flattening f
e e a ec f e globe

Lab :
● E e a ed e e e ba c e ab a e ee CSF
a a

Da :
● S / f c ea ed ac a a e e e e def c a e ed
e e fc c e e e a ed e e e ba c ea d a
CSF a e a d e ca e f ac a a e e MRI MRV

T ea e :
● S e f e ac c e ,c e e

Ne 338
Get The Step 2 CK Drills Book: usmledrills.com
The USMLE Guys Step 2 CK Crash Course usmleguys.com

● T ea e da
ace a a de a d a e f eadac e a eeded
● If a e , ca e e ace e f CSF c
e e ea fe e a

Brain Tumor

S /S :
● Ne e eadac e a e e 50, e a ea , e
Va a a a e e, e e e , e , e c (fe e ,
fa e), e e ca ( e e ,c ed f c , f ca ea e )

I a :
● Ba MRI a d c a de f

Da :
● Made def e e e a e

T ea e :
● Se d dec ea e e a ede a f a e a e e a ed ICP
● I d d a a a e e ad a ,c e ea ,a d e ba ed e,
ade, a d ca

Content Re ie Questions:

A 31- ea - d a e e e c c c a f ec e eadac e. T e a e ae
f e a 4 da e a e ad eadac e 4-5 e da a eac ef a d 30
e .T e aee e e c e a be d e e e c c e e e
c c a ec a d ac a a e a ea. W c f ef e be
a ea e da f a e d a ac e a ac ?

A. Ib fe
B. A e
C. O e
D. Me

Ne 339
Get The Step 2 CK Drills Book: usmledrills.com
The USMLE Guys Step 2 CK Crash Course usmleguys.com

A 24- ea - d fe a e e e ec c ac a f eadac e. T e a e ae a
e e a e ea e a bee e e e c e e e eadac e a 24 .
S e ae e a a a ca ed ab e e ef e ea d a / bb a .
S e fee a ea e eadac e a d e e eadac e e d e
a db .W c f ef e e da e e e ?

A. B a
B. M a e
C. C e eadac e
D. Te - e eadac e

W c f ef f d a c a ed eda f d a c ac a a
e e ?

A. Pa ede a
B. E e a ed e e e ba c e
C. S ca a e e a
D. Xa c a f CSF

Ne 340
Get The Step 2 CK Drills Book: usmledrills.com
The USMLE Guys Step 2 CK Crash Course usmleguys.com

Obstetrics and G necolog

N a P C a P a a E
R a
P a C a e :
● Re a a a e a c e a
● Dec ea ed FRC, ERV, a d e d a e
● U c a ed FEV1 a d FEV1/FVC a
● I c ea ed e e a (d e a e da e)
● O a dfa f e b
● Ra fda a a e e a e ,b da a e c ea
c ea e

Re a /Ge a C a e :
● A a e e ab c c e a e a a a b dec ea e
b ca b a e e e
● I c ea ed d e e
● I c ea ed e a a af a d c ea ed GFR (dec ea ed CR a d BUN)
● D a ed e a e a d ca ce
● H a e a, e a, c a, ce a, ab e a
● Ue d ace a d a d f a e b adde , dec ea ca ac ( a fe e c ,
e c , c a)

Ca d a c a C a e :
● I c ea ed ca d ac
● I c ea ed ea a e
● Dec ea ed e c a c a e a ce
● S e c a d a c ea ed c de, e d e a ac c d c
● Red ced a c ff e
● Dec ea ed b d e e

He a ca C a e :
● P ca e a f e a c
○ B a a e a d ed b d ce a e a d, b a a e
e a d a ea e de ee ca d
○ Dec ea ed b d c
○ I c ea ed a a a c a e
● P b c ae
○ Ve a ( e e ,c e f abd a e b e )
○ E d e a da a e (de e )
○ I c ea ed c a a fac

Ob e c a d G ec 341
Get The Step 2 CK Drills Book: usmledrills.com
The USMLE Guys Step 2 CK Crash Course usmleguys.com

M c eea C a e :
● E a ed e ca e d
● L a e a
● I c ea ed f ce e
● Sac ac a d bc de
● Ede a, c ca e e e ( ed a )

Ga e a C a e :
● GERD/a a
○ Dec ea ed e e a ea ce ed e e ad a d e e e
○ I c ea ed a-abd a e e
● Abd a b a /c a d e e e e

E ercise in Pregnanc

Be ef :
● Ma a e d a ce, f e b , e , ba a ce
● Dec ea ed a e f e ce e e a a e dec ea ed f de e
e a a d abe e a d fe a ac a a de e
● Red ced a c e ce, MSK a

E e c e Rec e da :
● A ea 30 e f e e c e 5 da a ee
● Pa e e e f ca f e ca c e e e e c e,
e e e d ad a 30 e 5 da a ee

C a d ca Ae b c E e c e D Pe a c :
● Ce ca ff c e c , e e ec d/ d- e e b eed , ace a e a af e
ee 26, ed e b a e , e a e ab a f e a e ab ( e
e a ), eec a a, e a c - d ced e e , ea d ea e, e c e
d ea e, e e e a e a

E ec e A d:
● C ac f e abd e , ac e fa , a, c ba
d ,e ec e ef ed a e (a ca a c e ),
ea - e a ce e f e ec e a a a a e e,
e e a d c c a e c ca ca e

Ob e c a d G ec 342
Get The Step 2 CK Drills Book: usmledrills.com
The USMLE Guys Step 2 CK Crash Course usmleguys.com

Content Re ie Questions:

W c f ef ca d a c a c a e a a ca c a e f
e a c ?

A. Dec ea ed e c a c a e a ce
B. I c ea ed b d e e
C. I c ea ed ea a e
D. I c ea ed ca d ac

W c f ef e a c a e a a ca c a e f e a c ?

A. I c ea ed e a a a f
B. I c ea ed GFR
C. I c ea e HCO3 eab
D. I c ea ed d e e

W c f ef ac a d ca ae b c e e c e d e a c ?

A. Ce ca ff c e c
B. R ed e b a e
C. Re c e d ea e
D. A a

Ob e c a d G ec 343
Get The Step 2 CK Drills Book: usmledrills.com
The USMLE Guys Step 2 CK Crash Course usmleguys.com

P aa V
T e e:
● <28 ee f e a :Pe aa e e 4 ee
● 28-36 ee f e a :Pe aa e e 2 ee
● >36 ee f e a :Pe aa e e 1 ee de e

I a Pe aa V :
● Idea d 1 e e a e a a a e f 10 ee e
● C e f e c de:
○ Pa ed ca a d b e c
○ U a e a e d ca c a e e a ed da e f de e a d
e a a a e
○ U a d de e e e a a a e a d de f fe a af a
○ E a e e c c ee
○ Lab

S a da d Lab :
● ABO a d R D e a d a b d c ee
● CBC
● Fe e e
● R be a e c c ee c f a dd c e ed f a ce a
a ce a acc a
● NAAT e f e e d ce a a a e f c a da
● Se c e f HIV, , e B
● U e e , ec e
● C f e a c e e be a CG

Ge a a A e 15-24 Wee :
● D d e c ee : Sec d- e e ad e e (AFP, CG, e , b A
e e )
● Ne a be defec c ee : Ma e a e a a-fe e a d a d
e a a
● Fe a c a a a e c ee :U a da e a a a e be ee 18-22
ee

Ge a a A e 24 28 Wee :
● Ge a a d abe e c ee : O e- 50- a a c e e a ce e . If e
ada c fa 100- a a a c e e a ce e ef ed
● Ad e a -D e b R D- e a e a e a 28 ee f e a
● A e a c ee

Ge a a A e 28 36 Wee :
● STI c ee f a e a c ea ed
● Sc ee f fe a e c a df - a e

Ob e c a d G ec 344
Get The Step 2 CK Drills Book: usmledrills.com
The USMLE Guys Step 2 CK Crash Course usmleguys.com

● B eec , a e e, b e e e a ef e e a ce a c e a 36
ee f e a

Ge a a A e 36 41 Wee :
● G B be a- e c e c cc c ee a 36 37 ee 6 da
● Pa e ed ca e ad ab a d de e a d a c de a

Content Re ie Questions:

H fe e d e aa cc e a a e a e a a a e be ee
28-36 ee ?

A. E e 1 ee
B. E e 2 ee
C. E e 3 ee
D. E e 4 ee

A a e a a a e da -D e b be ad e ed R D- e a e
a e ?

A. 24 ee
B. 28 ee
C. 32 ee
D. 34 ee

D c f ef e a a a e a ae c ee e a a e f
G B be a- e c e c cc ?

A. 32 ee
B. 34 ee
C. 35 ee
D. 36 ee

Ob e c a d G ec 345
Get The Step 2 CK Drills Book: usmledrills.com
The USMLE Guys Step 2 CK Crash Course usmleguys.com

N a Lab a B P a
Fa e Lab "B a -H c C ac ":
● Pa f / - a f e ec ac c ca be e a e a
● B a -H c c ac d ca e ce ca d a
● B a -H c c ac dec ea e e e e
● B a -H c c ac a ecede e a e a e f ab

Stages of Labor

F S a e:
● Be f e e ab be c e e ce ca d a (10c )
● C f ae a e a d ac e a e
○ La e a e c de e d f ce ca d a f 0 6c
○ Ac e a e c de e d f ce ca d a f 6 10 c

Sec d S a e:
● T e e df f ce ca d a de e f e bab
● C f e a e a e a d ac e a e
○ Pa e a e a f f ce ca d a ac e eff b e e a
a e de e e bab
○ Ac e a e a f a f ac e eff b e e a a e e e
e bab bab de e ed

T d S a e:
● T e e d be ee de e f e bab a d de e f e ace a

D a Ce ca E a a :
● Occ a ad , e e 2-4 d e1 a e, e e 1-2 d e2
a e
● U ed de f ce ca efface e , d a , a d fe a a
● Pe f ed ad e a a e a/a e e a
● If e a e e e fee e e ( e ef d a f ce bef e
)
● T a e a ab a e fe a ea a e

D a 1c O e T H Ac e P a e:
● Ad e c

Ac eP a eA e :
● Pa e ed e b a e a d 6 c ce ca d a ce ca
c a ef 4 ade a e c ac
● Pa e ed e b a e a d 6 c ce ca d a ce ca
c a ef 6 ade a e c ac
● Ac e a ea e ea ed ce a ea de e e e e e e

Ob e c a d G ec 346
Get The Step 2 CK Drills Book: usmledrills.com
The USMLE Guys Step 2 CK Crash Course usmleguys.com

O tocin

I d ca :
● I d c f ab , a f ab ca e f e e e a, a a e e f
c ee e ab e ab ,c a e a e

Ad e e Effec :
● Tac e
● Ue e a , e c , e
● H e
● H a e a

Fe a S a :
● Fe a a a -5 0 c : P e e a f e fe ab e e c a e , f e
e e a a ab e efe ed a f a a
● Fe a a a 0c :T e e e a ( ca e ead) be ee e c a
e , a e a ed a
● Fe a a a 0 5 c : N be f ce ee be d ee a ed a

B eec P e e a :
● F a b eec : B f e ed a d b ee e e ded fee e e ead
● C e e b eec : B a db ee f e ed
● I c e e b eec : O e b c e e f e ed

S :
● S bc a d c f , c e e abd e

Da :
● Abd a e a a de a de f eb c a d/ fee a e
e e a

T ea e :
● E e a ce a c e (ECV) a 36 ee f e a (3 a e e b e)
b e e a a b f cce f
● If 3 a e a ECV fa , a a e a a 1 e da
● P a ed ce a ea f b eec e e a e de ea e ed e e a ce a c
e
● Ra e , a f a a b b eec e e a e e e ced c ca a d
e df Ce a ea b

Ob e c a d G ec 347
Get The Step 2 CK Drills Book: usmledrills.com
The USMLE Guys Step 2 CK Crash Course usmleguys.com

Content Re ie Questions:

W e e ce d a ed 8 c a e a ea d a e f ab ?

A. F a e, a e a e
B. F a e, ac e a e
C. Sec d a e, a e a e
D. Sec d a e, ac e a e

W c f ef a ad e e effec f c ad a ?

A. Tac e
B. H e
C. H a e a
D. U e ea

W c f ef e f b eec e e a c a ac e ed b b be f e ed
a db ee be e e ded fee e e ead?

A. Pa a b eec
B. F a b eec
C. C e e b eec
D. I c e e b eec

Ob e c a d G ec 348
Get The Step 2 CK Drills Book: usmledrills.com
The USMLE Guys Step 2 CK Crash Course usmleguys.com

B a :C I a C a
Breastfeeding

Be ef I fa :
● N ,a c ba a d d a ac , ed GI f c a d
de e e , c ca be ef , e f ec c e e

Be ef M e:
● R f a b d ed ced, de a ed e f a , ed ced f
ce a ca ce (b ea , a a , e d e a ca ce ), ed e , fa e
e e e a , c ca be ef , ec ca be ef

C a d ca B ea feed :
● M e a HIV, HTLV I II, eb a fec ; fa a ca c a ac e a; e
ec ea a d

Te a C a d ca B ea feed :
● M e fec ed b ce ac e HSV e e e b ea , f a
ce a ed ca

C a d ca D ec B ea feed ( a e e feed fa ):
● M e fec ed ea ed ac e be c a ac e a ce a e e a a
5 da de e a d 2 da f de e

C D ff c e Re a ed B ea feed :
● N e a
● E e e
● A e a de a
● N e a c c
● P ed d c
● Ga ac ce e
● Lac a a a
● B ea ab ce

Lactational Mastitis

S /S :
● Red, a f , e b ea a e fe e /c , a a, aa e a de e af e
12 b e eac ea a ade a

R Fac :
● E e e , e ab a /c ac , e e bab

Ob e c a d G ec 349
Get The Step 2 CK Drills Book: usmledrills.com
The USMLE Guys Step 2 CK Crash Course usmleguys.com

Da :
● C ca d a , a d ca d a a d b ea ab ce

T ea e :
● I a c dc e e , NSAID , c ed b ea feed
● Af e 12 ,c e ab e ea e a d add d c ac ce ae
If a f MRSA, e TMP-SMX c da c

Breast Abscess

S /S :
● Pa ab e a a e de a d f c a , fe e , aa e
● Of e e e ce f c e a f ece ea e f a

I a /D a :
● C f ed f dc ec ee a d

T ea e :
● D a a e ( eed e a a ca ) AND a b c ea e dc ac
ce a e
If f MRSA e TMP-SMX c da c

Content Re ie Questions:

W c f ef da e c e a b ea feed a e de e a
a e a e a ab e a e b ea e c ca f d ?

A. Lac a a a
B. B ea ab ce
C. Ga ac ce e
D. B ea e e e

W c a da c ca d ca ed a a e ec ed f a a b ea
ab ce ?

A. Ma a
B. U a d
C. CT
D. MRI

Ob e c a d G ec 350
Get The Step 2 CK Drills Book: usmledrills.com
The USMLE Guys Step 2 CK Crash Course usmleguys.com

W c f ef ea e d ca ed a a e ac a a a ?

A. S b ea feed
B. Ib fe e
C. D c ac
D. C d c e e

Ob e c a d G ec 351
Get The Step 2 CK Drills Book: usmledrills.com
The USMLE Guys Step 2 CK Crash Course usmleguys.com

H a M ,G a a T ba N a a, a
H Ga a
H datidiform Mole

C e e M e:
● S e fe a fa e e
● Lac f e b /fe a c e
● D ff e ba c e a a
● D ff e da d f e fc c
● Ka e 46XX 46XY (a f e )
● H e f e a a ba c e a a

Pa a M e:
● P ca a e fe ed b 2 e
● P e e ce f e b /fe a c e
● F ca da d f e f ec c WITH ca
● F ca ba c e a a AND ba c a c
● I c e ef 57
● Ka e ca be 69XXY 69XYY 69XXX ( a e a a d a e a c e )
● L e f e a a ba c e a a

S /S :
● Pe c a / e e, e a ed e , a a b eed , a ea/
● La e f d c de e d , eec a aa e a 20 ee
e a a a e, b a e a eca e a a c

Lab :
● Se CG e e f e a a a e

I a :
● Ta a a a d fea e :
○ C ee a e a c : Lac f e b /fe a da cf d a
a ea a ce, a d b e e e ce f a a eca e c
○ Pa a a e a c :P b e e e ce f e b /fe , e e ce f
a c f d, ace a S c ee e a e , a d/ ec e c f
c c

Da :
● H cc f a

T ea e :
● Ce ca d a , e ec c c a a ,a dc e a e
● Se a CG e e af e c e a e a e f e a a ba c
e a a

Ob e c a d G ec 352
Get The Step 2 CK Drills Book: usmledrills.com
The USMLE Guys Step 2 CK Crash Course usmleguys.com

Gestational Trophoblastic Neoplasia

H ca T e :
● H da d f e
● C ca c a
● P ace a e ba c
● E e d ba c

S /S f L ca ed D ea e:
● Of e a f a e a c c a e c a / e e, a a
b eed , a ea/ , e d , a a eca e c ,a d ae
eec a a

S e f Me a a :
● M c e e ,f ed b e a a, CNS, a d e

L Me a a S /S :
C e a ,d ea, c , e , a d/ e a fa e

Va a Me a a S /S :
● Va a b eed a d/ e d c a e

CNS Me a a S /S :
● D e , a ea, eadac e, e a , a d ba ce , ed eec ,
e a e , a d/ e e a ed ac a a e e

L e Me a a S /S :
● Ja d ce, abd a /bac a a cc a e e ea a e

Lab :
● E e a ed CG e e ( a a dc ca c a)
● T df c e ( e e e d e e d e e e a ed CG e e )
● ALT a d AST e e (e e a ed ca e e ea a )

I a :
● Pe c a da e f e a a ba c e a a
○ I a e e a ea a a -def ed e e a a ec c a ea
be a e e
○ C ca c a a ea a a e e e e (d e a ea f ec a d
e a e) e e a be a e aa e
○ P ace a e ba c a ea a a e ec c- a e e a
b c ca d dc e a d a ade e e a
a

Ob e c a d G ec 353
Get The Step 2 CK Drills Book: usmledrills.com
The USMLE Guys Step 2 CK Crash Course usmleguys.com

○ E e d ba c fe ae a d e a
a
● C e X- a a e f e a ea a

Da :
● U e d , e e ed a eada a d ea e
a ed ba ed , ab , a d a
○ F a a e a c f ee CG e e a ea e a 3- ee e d
CG e e c ea e e 10% 3 e aae a e e a 2- ee e d
eda f e a a ba c e a a ade

T ea e :
● L - c ca c a a e e: T ea e e aea d
e a e CG e
● S a e I ace a e ba c e e d ba c ea ed
a e ec
● H - c ca c a a e e ea ed e de, e e ae
ac c D, a d a e a c c a de a d c e
○ H - ace a e ba c e e d ba c e
c e e e e ec

H peremesis Gravidarum

R Fac :
● M a e a ce , e e a ,f e a c

S :
● Na ea, , e , eaded e

Lab :
● P be ae ca d c e c e ab c a a , a e e a,
ca ce a, e , c ea ed e a c , e e a ed BUN a d e e a ed e ec f c
a , e e a ed ALT a d AST (ALT e e a ed e)

I a :
● If e ef ed a a f e a c - , e c a d d be
ef ed e a aef e a a ba c d ea e e e a

Da :
● C ca d a c a ac e ed b e e ,e >5% e- e a c
e e a ef e e e de f ab e ca e

T ea e :
● P d e-d a e cc a e

Ob e c a d G ec 354
Get The Step 2 CK Drills Book: usmledrills.com
The USMLE Guys Step 2 CK Crash Course usmleguys.com

● IVF e ace e a e, de e d e e fc d

Content Re ie Questions:

W c f ef c a ac e c c e eda fac ee a
e a c ?

A. P e e ce f fe a c e
B. I c e ef 57
C. S e fe a fa e e
D. Ka e f 69XXY

W c f ef c a ac e c c e eda fa a a a
e a c ?

A. P e e ce f fe a c e
B. 46XY a e
C. D ff e ba c e a a
D. D ff e da d f e fc c

A a e ee e a da a bee ffe f 2 da f a ea a d
ade a e a f d a e e e e c f e
f ab ab a e ?

A. H ae ca d c e c e ab c a a
B. H ae ca d c e c e ab c ac d
C. H e ae ca d ec e c e ab c a a
D. H e ae ca d ec e c e ab c ac d

Ob e c a d G ec 355
Get The Step 2 CK Drills Book: usmledrills.com
The USMLE Guys Step 2 CK Crash Course usmleguys.com

H D P a
Chronic H pertension in Pregnanc

C a ac e c :
● Pa e da f e e e a c
● H e e e e 20 ee e a a a ea d e f 12 ee af e
de e

Fe a R :
● S a f e a a a e, ee de e , b

Ma e a R :
● S e ed eec a a, a e a e, ace a ab , e, e a
ff c e c , MI

Ma a e e :
● E ab ba e e ab
● O ea e e e ed ca
● L -d e a af e 12 ee e a e e eec a a
● Sc ee f fe a e c a 28 32 ee a d ce ee e e
b ca f e a 32 ee
● Med ca ed ea e e e a c c de e d a, abe a ,
d a a e, a d/ fed e

Gestational H pertension

C a ac e c :
● Ne e f e e a e a a a e f 20 ee e a a
e f e d- a da a e

Lab :
● E e e a a a d e a e a 24- ec ec
● E e fe d a da a e e ec a ab a e aee ,
c ea e, a d/ e a c a a a e

Ma a e e :
● Se e e e e ea ed a e e e ( e e e-e
e d- a da a e) a f SBP 130 150 H a d DBP 80 100
H
● Wee b ca f e e e a ce a f ea c
f d a a 32 ee e a
● A e fe a e e 3-4 ee

Ob e c a d G ec 356
Get The Step 2 CK Drills Book: usmledrills.com
The USMLE Guys Step 2 CK Crash Course usmleguys.com

Preeclampsia

C a ac e c :
● Ne e e e a e a a a e f 20 ee a a e a f e
e e AND e a OR a f ef : e a ff c e c ,
b c e a, e a c d f c , ce eb a / a , a ede a

P eec a a Se e e Fea e :
● A f ef :
○ SBP 160 a d/ DBP 110 2e de a ea 4 a a
○ Ce a e e d f c
○ Re a ff c e c
○ He a c d f c
○ T b c e a
○ P a ede a

H P eec a a Ma Ma fe :
● P e a: 300 a 24- e c ec e :c ea e a 0.3
● Re a ff c e c : D b f ba e e c ea e e c ea e f >1.1 /dL
● T b c e a: P a e e c <100,000/ c L
● He a c d f c :L e a a a e ab e 2 e e f a e ee
e e e ad a /e a c a e e ed ca
● Ce eb a / a : Headac e, f a , b ed ,d a, b d e
● P a ede a: Ne ea/d ea

Ma a e e :
● De e f 37 ee e a a a e f e e e fea e 34 ee e a a a e
● E ec a a a e e be a e a e f fe a a ab e e a a a eb
e a 34 ee
● A e e e f e e e ee e e 160/110 ( e d a, abe a ,
d a a e, a d/ fed e)
● F eec a a e e e fea e e a a a d a e e
a a e fa e

HELLP S ndrome

S /S :
● R e ad a /e a c a , a d ce, a ea/ , e a,
e e , eadac e, a c a e

Lab /D a :
● He de a ed b 2 f e f :
○ Se e e a e a, d e b d
○ LDH 2 e e f a a b

Ob e c a d G ec 357
Get The Step 2 CK Drills Book: usmledrills.com
The USMLE Guys Step 2 CK Crash Course usmleguys.com

Pe○ ea ea de f c c e a db ce
○ E e a ed e b b 1.2 /dL
● AST ALT 2 e e f a
● P a e e <100,000 ce / c L

T ea e :
● G e a e fa e f e e a
● If e a a a e 34 ee ab e, de e af e e ab ed
● If e e e c ca (ec a a, ab ace a, e, a ede a, e a c
b eed , AKI, DIC) e e fe de 34 ee b a a ab e a e, de e af e
e ab ed
● If fe a de e fe a a ab e a e f e a e e a , de e af e e
ab ed
● If e eec ca ( ee ab e) a d e fe de 34 ee b a a ab e
a e, e be a e a e a d de e af e 48 f e a d fe e a ab e

Eclampsia

C a ac e c :
● Pa e e a a e e , eec a a, HELLP d e de e
e e a ed c-c c e e f e - e c a
● Ec a a a cc a e a , a a , a

I a :
● MRI CT f e ead f d f a e c ede a f e e ce eb a
e e e (c e e e be e e e ce a a d e)

Da :
● C ca d a ba ed e e ce f e e ed de f e a c
c-c c e e c a a a e e ca e f e e /c a

T ea e :
● P ace a e aea a d e e a eb ea e face a
● T ea f e e SBP 160 DBP 110
● Ma e fa e e e ec e e e
● P de e

Magnesium Sulfate To icit

M e cc e e a ff c e c a d c ea a ce.

S :
● P e f de e d e a e c ce a
○ Dee e d a ef e a a 7 10 E /L

Ob e c a d G ec 358
Get The Step 2 CK Drills Book: usmledrills.com
The USMLE Guys Step 2 CK Crash Course usmleguys.com

○ Re a de e / aa a 10 13 E /L
○ A a a >15 E /L
○ Ca d ac a e a >25 E /L

T ea e :
● Ca c c ae

Content Re ie Questions:

A 33- ea - d fe a e a e be ee b e OB GYN d c f a f - .T e
a e a da ed e e a a e a a a e f 14 ee a d da e
ff ce e b d e e 150/92. S e de e ed a ea c d 13 ee a . W a e
a aeda ba ed e e f d ea e?

A. Ge a a e e
B. T a e e e f e a c
C. C c e e
D. P eec a a

A a e a 35 ee e a a a e e e e a eadac e. T e a e
e e e a 168/112 a d ab a b ; e ea ea c c e ;
e e e e e a ed AST, ALT, a d b b ;a d b c e a. W c f e f a
ea e f e e a ?

A. Labe a
B. Ma e fa e
C. P de e
D. Be a e a e

W c f ef ef a ea a a f a e fa e c
e a a e be ea ed f ec a a?

A. L f dee e d ef e e
B. Re a aa
C. C d c ab a e
D. Se e

Ob e c a d G ec 359
Get The Step 2 CK Drills Book: usmledrills.com
The USMLE Guys Step 2 CK Crash Course usmleguys.com

P aa I
TORCH I fec :
T-T a
O-O e ( , a ce a, a B19, a)
R - R be a
C-C e a
H - He e e

To oplasmosis in Pregnanc

Ta :
● C a a ed de c ed ea a d c a ca e

Ma e a S fP a I fec :
● U a a a c, b beb ae a - e de ce ca ade a , fe e /c ,
da e , a a, eadac e , d ff e ac a a a

Congenital To oplasmosis

Fe a I fec :
● Ta ace a a fa a ae a fec

Ne aa S /S :
● C e , ac a a ca c f ca , d ce a , e e e a e e a ,
a d ce, fe e , ade a , e

Lab :
● Q a a e e T I G, I M, a d I A
● P b e a e a, b c e a, e a
● P beee a ALT/AST, a a d d ec b b

CSF A a :
● E e a ed CSF e (>1 /dL) a d e a a- ec f c PCR

I a :
● CT f e ead ac a a ca c f ca , d ce a , a d/ c ca a

Da :
● C f ed T - ec f c b e a a- ec f c
PCR e CSF

T ea e :
● P e a e fad a e f c ac d

Ob e c a d G ec 360
Get The Step 2 CK Drills Book: usmledrills.com
The USMLE Guys Step 2 CK Crash Course usmleguys.com

S philis in Pregnanc

Ta :
● Se a a ed

Fe a I fec :
● Ta ace a a

Sc ee :
● A a e c ee ed a f e aa
● If f fec , c ee a a a be ee 28 a d 32 ee a da
de e

Ea C e a S /S (U de 2 Yea ):
● Ja d ce, e a e a , e e a ed ade a , e e a ab a e , a ,
a a d c a e

La e C ea S /S (O e 2 Yea ):
● F a b , be a a d b e, add e e, a a, H c ee ,
be a , c ea ca , e a ea , e e a ea ,
e ec a d ab , a e ed d ce a , abe

Lab :
● Le e a e c ,a e a, b c e a

CSF A a :
● Reac e CSF VDRL; e e a ed CSF, WBC , a d e

I a :
● L -b e X- a ea ea ce ba d , e a e ea c e ,
We e ,W be e

Da :
● D ec a a f T. b da f e d c c
● P ef e ce a b d a
● P e a a ( ace a a d b ca c d)
● P e e c eac (VDRL RPR e )

T ea e f P e a Pa e S :
● Pe c G be a e( ef de e a e ad e e c ea f
e c a e e e )

T ea e f Ne a e/C d C e a S :
● Pe c G be a e

Ob e c a d G ec 361
Get The Step 2 CK Drills Book: usmledrills.com
The USMLE Guys Step 2 CK Crash Course usmleguys.com

● Ja c -He e e eac a cc

Congenital Varicella

Ta :
● Pe - - e c ac

C e a I fec :
● Ta ace a a aa a e a d e d ec c ac

C e a S /S :
● L b e ,a e c e c b e , de a a a e f ca , e ec a
d ab , d ce a , c ce a , e e , c e ea , ca a ac , b
ab a e

Lab :
● VZV PCR e f fe a b d a cf d

I a :
● 5 ee af e ae a fec ef a a c a d f e fe

Da :
● P e VZV PCR a d a d ab a e

T ea e :
● T ea c ca ed a ce a e a a e a ac c a d a ce a
e a e a a e a e ac c
● G e a ce a- e e b (VZIG) e e a a e VZV
e e
● G e e ae f fec ed e VZIG a a b e af e b , de
d c a ca e f e ae e ab ed c e a a b

Parvovirus B19 Infection in Pregnanc

S /S :
● Fe a , d fe a , fe a e a eff , e ca d a eff ,a c e ,
ede a

I a :
● U a d a fe a d

Lab /D a :
● P e ae a e B19 I G a d I M a b d e
● P e ae a e a B19 DNA PCR

Ob e c a d G ec 362
Get The Step 2 CK Drills Book: usmledrills.com
The USMLE Guys Step 2 CK Crash Course usmleguys.com

● P e a B19 DNA PCR f a cf d


● P e fe a e B19 I M a b d e

T ea e :
● I a e e fe a a f fb d d c

Zika Virus Infection During Pregnanc

Ta :
● M -b e e
● A a ed e a b fec ed a e

Ma e a S /S :
● Fe e , ac a a a ,a a a, c c

C e a S /S :
● M c ce a , c a ab a e , e e ,ca a d , e e a
ea , e ab a e ,c ac e , fe a e c , fe a

I a :
● CT MRI f e ead e defec c d ac a a ca c f ca ,
c ca ab a e ,a d e c e a

Lab/D a :
● P e a e a RT-PCR f a a d a I M
● P e e a e RT-PCR f a a d a I M ( b a ed 2 da f
b )

T ea e :
● S e ca e a d a a e e fc ca

Congenital Rubella

Ta :
● Ta ace a a

Ma e a S /S :
● Fe e , c a, c c , e a,c , a a a, ade a ,
ac a a a

C e a S /S :
● Hea , ca a ac , c e a a c a, ea defec , "b ebe ff "
e , c ce a , b e , e ec a d ab , ad ce b e d ea e

Ob e c a d G ec 363
Get The Step 2 CK Drills Book: usmledrills.com
The USMLE Guys Step 2 CK Crash Course usmleguys.com

Lab /D a e Ne a e/I fa :
● P e be a- ec f c I M a b d e
● Pe e be a- ec f c I G a b d e
● P e a c e
● P e be a RNA PCR

T ea e :
● S e ca e a d a a e e fc ca

C tomegalovirus Infection in Pregnanc

Ta :
● Pe - - e c ac , e a ec e, a f / a a a

Ma e a S /S :
● P a , , a , a a, a a a, eadac e

Fe a I fec :
● Ta ace a a

C e a S /S :
● Se e a ea , e a e e a , a d ce, e ec ae, c ce a ,
a ef e a a a e, a e , e e , e ec a d ab

Lab :
● T b c e a, a e a ( e c)
● E e a ed AST, ALT, d ec a d d ec e b b

I a :
● CT, MRI, a d e e c a ac a a ca c f ca , e c ae
a c a , e c e a , a d/ e a e d ea e

Da :
● V a c e PCR f c e a f e aa e( 3 ee fb )
● A cf d a c e CMV DNA e CMV I M a b d fe a b d

T ea e :
● T ea a c fa c f ed CMV fec a d e de ce f e d- a
a cc ( fe- ea e ca e ) a a cc ( - fe- ea e )

Neonatal Herpes Simple Virus

Ta :
● I a e e ( a e), e aa ( c ), aa

Ob e c a d G ec 364
Get The Step 2 CK Drills Book: usmledrills.com
The USMLE Guys Step 2 CK Crash Course usmleguys.com

Ne aa S /S :
● S e ce ,c c a e e a, ca a ac , c e , a ce a e e
● A a , e a , feed , fe e , e ,f a e f a e, e e
● D e a ed d ea e e e a e : M ca d , e a,
e a c e , d e d ea e, e a , a c e , e fa e, d e a ed
a a c a c a a , ec e e c

Lab :
● Ab a e de e d a ed (e e a ed e a a a e , d ec
eb b e a, DIC, b c e a, e e a)

CSF A a :
● L - a c e e e , c ea ce e c , d e e a ed e ,a d
e HSV DNA PCR

I a :
● Af e da ee f d ea e, CT MRI f e ead a de c e e
e a ea f e b a , a e c a b a ede a, e a e

Da :
● C ca e a c e, HSV DNA PCR, e a a e
a d d ec f e ce ce a a

T ea e :
● IV ac c

Content Re ie Questions:

W a e a a e ea e f a e a a e f d a ea fec
a e c e c ?

A. D c c e
B. C f ac
C. Ge a c
D. De e a f ed b e c ea

W c f ef c e a fec a c a ed c e , ac a a
ca c f ca ,a d d ce a ?

A. T a
B. S
C. Va ce a
D. HIV

Ob e c a d G ec 365
Get The Step 2 CK Drills Book: usmledrills.com
The USMLE Guys Step 2 CK Crash Course usmleguys.com

W c f ef a fe a a c a ed Pa B19 fec d
e a c ?

A. Fe a
B. H d fe a
C. T a e e ca d a eff
D. Sadd e e

W c f ef c e a fec e e e ea , ca a ac ,
"b ebe ff " e , c ce a , a d ad ce b e d ea e?

A. He e e
B. C e a
C. R be a
D. Z a

W c f ef CSF f d c e ada f e aa e e
e ee e fec a eac ed e CNS?

A. CSF c ea e c
B. E e a ed c e e e
C. L e e e
D. D a

W c f ef ed ca ed ea c e a c e a e de ce f
fe- ea e e d- a da a e?

A. Ac c
B. Ga c c
C. O e a
D. Za a

Ob e c a d G ec 366
Get The Step 2 CK Drills Book: usmledrills.com
The USMLE Guys Step 2 CK Crash Course usmleguys.com

T a D a T A F a E
Drug Teratogenic Effect(s)

ACE- b a d O da , a a a, fe a e c ,
ARB ca a b e a a, fe a e ,a d d e ef
Wa fa I a e e e c , a a a a, f
ec da e
L Eb e a a

Ca ba a e e Ne a be, ca d a c a,a d a ac defec ; fac a


d
Va c Ac d L b ab a e , a b f da, dac , c ef a a e, a a e a
defec , ad a , fac a d

P e C ef a a e, e c a e a defec , fac a d ,
e ec a d ab

Eb e A a :
● T c d e a ca ed b def ed a d d aced c d a e eaf e
● A c a ed ca d ac defec a a c de a a a d/ e a e a defec ,
acce c d c a a , a a e a e, b c d a c a e, a e
d c a e
● Da ed a ac c ec ca d a

Drug Teratogenic Effect(s)

Re d A c a c a d ea defec , c a, a a, ca a a

Me e ae L b defec , c a a, c ce a , e c ,
dec ea ed f ca f e ca a

De be C ea ce ade ca c a f a a, e a af a

T a d de L b defec , a e a defec e a e a

M M b d e, b- ed c defec

M c e ae fe CNS, ea , d e ,d a b a d ea defec ; c ef / a a e;
ca a e

Fe a A c S d e:
● Fac a fea e c a e b de , , a eb a f e
● De e e a , be a a , c -e a , a d/ c e be
● M c ce a , b a c a defec , a d/ ec e feb e e e

Ob e c a d G ec 367
Get The Step 2 CK Drills Book: usmledrills.com
The USMLE Guys Step 2 CK Crash Course usmleguys.com

Content Re ie Questions:

W c f ef ed ca ca ed a a e a e ca e Eb e
a a ?

A. Te ac c e
B. P ac
C. L
D. T a d de

W c f ef a fac a def a c a ed fe a a c d e?

A. T e b de
B. Dec ea ed f ca f e ca a
C. S
D. S a eb a f e

Te ac c e ed e a c a c a ed a fe a ab a ?

A. L b defec
B. Tee a
C. Ne a be defec
D. O da

Ob e c a d G ec 368
Get The Step 2 CK Drills Book: usmledrills.com
The USMLE Guys Step 2 CK Crash Course usmleguys.com

Pa aP a, P a aA aS , Va a P a, a
Pa a Ab
Placenta Previa

P ace a e c e e d e a a f e e a ce ca .

R Fac :
● P ace a e a, e e a , C- ec

S :
● U a a e a a b eed af e 20 ee e a
● Occa a e ec ac , a ,a d a a b eed

I a /D a :
● P ace a e ada ed e a a a a d ace a e
e e d e e e a ce ca
● L - ace a d a ed e ace a ed e 20 f e a ce ca

T ea e :
● W b eed :
○ M a e a e d a c a d fe a ea a e
○ Lab c d CBC, c a a de , e, a d c ee /c (de e d
e d f eed a f )
○ IVF a db d d c a f
○ E e e c C- ec d ca ed e :
■ L fe- ea e e a e e e f d a db d
d c
■ Ge a a a e f 34 ee ea e f ca b eed
■ T e a e ac e ab
■ Fe a ea a e d ca e e dec e
W b eed :
○ Re ea a a a a d a 32 ee a d 36 ee
○ Sc ed e C- ec f 36 ee 37 ee a d 6 da
○ If - ace a e e a d e ace a ed e 10 c e e
ed e f e e a , c ed e C- ec
○ If - ace a e e a d e ace a ed e 11 20 f e ed e
f e e a , a f ab

P ace a Acc e a S ec :
● P ace a acc e a: Ab a a ac e f ec c e e
● P ace a c e a: Ab a e e a a d a ac e f ec c e
e

Ob e c a d G ec 369
Get The Step 2 CK Drills Book: usmledrills.com
The USMLE Guys Step 2 CK Crash Course usmleguys.com

● P ace a e c e a: Ab a e e a a d a ac e f ec c e
e e e e e a ad ace a

R Fac :
● La e fac ace a e a af e a e C- ec
● P e e e e c ad a , a e a a e >35 ea , a , a a
e a f e ace a

S /S :
● B ,c , fe- ea e b eed a e ed a a ace a e aa

I a /D a :
● T a abd a a d/ a a a a d b
○ M -ea e ace a ( ac f e e ) e ace a ac ae c
a ea a e a a ace a ce ace e dd e f a b e
c ed b de e ed e
○ L f e ec c a ea be d e ace a
○ S f ace a e c e a c d f e e e be ee e b adde
a a d e e e a e face, b d e e ee e e d f ace a
e b adde , ba f e e e b adde e e e ace a
a ac ed, a ee a e e e e e aa de e d e b adde

T ea e :
● Sc ed e Ce a ea e ec be ee 34 ee a d 35 ee 6 da f
e a
● Ra e e ec e a ca be a e ed f a e de e f ec dbea
● If f de f ed d Ce a ea ec ,c e e a ac a d de a
e a ae e e a de e f a e a f e e
● If f de f ed a a a de e , e e e a a e e f e a ea d
e ec ef ed

Vasa Previa

P e e ce f fe a b d e e e e e a ce ca .

R Fac :
● Ve a e b ca c d e ,b bed ace a, ace a e a/ -
ace a, e e a

S /S :
● D a a a e a a a a e e e e ba e e e ce ca

● A e f e b a e , a e (fe a ) b eed e fe a ea ae
ab a e a d b e fe a dea e

Ob e c a d G ec 370
Get The Step 2 CK Drills Book: usmledrills.com
The USMLE Guys Step 2 CK Crash Course usmleguys.com

I a /D a :
● Ta a a a d a e ba fe a e e( ) a e
20 f e e a ce ca

T ea e :
● G e be a e a ea e a a a e f 28-32 ee
● H a ad a e a a a e f 30-34 ee
● De e f fe a Ce a ea ec a e a a a e f 34-35 ee , bef e e
f e ba e e f ab
● E e e c Ce a ea ec e ca e f e f e b a e , ab , fe a ea ae
ab a e , a a b eed c f ed be e fe a b d a A e
K e a e -Be e e

Placental Abruption

Pa a c ee e aa f e ace a f e e de e .

R Fac :
● P e ace a ab , abd a a a, c ca e e, , e e ,
e e ab a e

S /S :
● Va a b eed , abd a /bac a ,f a d e de e a a ,
c ac , fe a ea a e ab a e

Lab :
● P b e DIC
● A e a a a be e e de e d de ee a d eed f e a e

I a :
● U a d e ace a e a a

Da :
● C ca d a ed b ab a d a

T ea e :
● M a e a e d a c a d ab ab a e IV f d a d b d
d c e ace e a eeded
● C fe a ea a e
● Ce a ea de e e e a a e e d a ca ab e, f ca
c a a e e , fe a ea a e a e ea ( f a a de e
e , ef a e e -a ed a a b )
● If e a a a e e a 34 ee a b d c a a , ea
c e a e a a e e 37 ee

Ob e c a d G ec 371
Get The Step 2 CK Drills Book: usmledrills.com
The USMLE Guys Step 2 CK Crash Course usmleguys.com

Content Re ie Questions:

A e a a e ece a 20- ee c ee a dd a e aa .O
a abd a a d eeae e e a a ace a ce ace e
dd e f a b e b de e e e ace a a -ea e a ea a ce
d e ac f e e .W a e e da e e e a df d ?

A. P ace a ab
B. Va a e a
C. P ace a acc e a
D. M a e a c

W c f ef c d c a ac e ed b ace a e c e e d e a
a f e e a ce ca ?

A. P ace a e a
B. P ace a acc e a
C. P ace a cea
D. P ace a ecea

A e a a e ada f a a e a ad ed e a a e a a a e
f 30 ee a c a f a c ed ed Ce a ea de e a 34 ee . T e a e e
be a e a e e fe a a a 30 ee e a a a e. A 32 ee e
f e b a e cc a a b eed c f ed be e fe a b d a A e a d
ea fe a ea a e. W a e a ae e e ?

A. E e e c Ce a ea de e
B. E ec a a a e e
C. T a f a a de e
D. T a a a a d

Ob e c a d G ec 372
Get The Step 2 CK Drills Book: usmledrills.com
The USMLE Guys Step 2 CK Crash Course usmleguys.com

S a Ab ,T a Ab ,S Ab ,a
E P a
Spontaneous Abortion

R Fac :
● A e d , ab a ae a a e
● U e e ab a e (a a ca a , ade , e a, a e e
ad e , a e e ad e )
● A d d ea da e fac
● E d c e d ea e (PCOS, e ac e a, c ed DM, d d ea e)
● T b a (fac V Le de , e S def c e c , b G20210A a )

C e e Ab /C eePe a c L :
● S a , e -c ac ed e
● O e c ed ce ca
● M a a a b eed
● M dca a
● E e e a e e fe a e a d

T ea e ed Ab /T ea e ed Ea P e a c L :
● C ed ce ca
● Va a b eed
● U a d a e e e a c fe a ca d ac ac

I c e e Ab /I c e e Ea P e a c L :
● O e ce ca
● Va a b eed ( c ea )
● P e e ce f c a /c ac
● P be a a f e a a e a e e a ce ca
● Ue e e e c ac ed, b a be a e a e ec ed f e a a a e
● Pa a d c f c ce be a d

M ed Ab /De a ed Ea P e a c L :
● Fe a de e bef e 20 ee e a
● +/- a a b eed
● Ce ca ca c ed
● U a d a e e e a a ac +/- a fe a e NO fe a ca d ac
ac

I e ab e Ab :
● O e ce ca
● Va a b eed
● Pa f c a

Ob e c a d G ec 373
Get The Step 2 CK Drills Book: usmledrills.com
The USMLE Guys Step 2 CK Crash Course usmleguys.com

● P b e fe a ca d ac ac a d
● P be e f e ba e

Pe a c L T ea e :
● E ec a a a e e a ae 13 ee e a a a e c f a
e e a a e , e e CG e e , a d/ a d
● Med ca a a e e f 1 2 e e e a c fe ef ed b

● S ca a a e e e ea a f 1 e e e a c a d2
e e e a c ea ed d a a d e ac a
● S ca a a e e f e d a ca ab e ab

Septic Abortion

S /S :
● Fe e , e a a d c a e, a a b eed , e c/abd a a

Lab :
● B dc e
● Te f STI
● CBC d ffe e a , CMP, c a , ac a e e e , a a d ec e, b d e
a d c ee

I a :
● O ef ed f e a e e d a ca ab e
● U a d a e a ed d c f c ce

Da :
● C ca d a ba ed / e a c /e a
de 20 ee e a a a e

T ea e :
● He d a ca ab e a e e e e c a (IVF , a e ), b ad
ec IV a b c , a d ed a e e ea a ,c e f e ac a ed e e
e
● If e d a ca ab e: Pe f a d e e e ac a a d f e
ca e f de f ed, ea e c ab b ad ec IV a b c ,
e ea a ,c e f e ac a ed e e e

Ectopic Pregnanc

Fe ed e a a a ca a ee d e e e e ca .

Ob e c a d G ec 374
Get The Step 2 CK Drills Book: usmledrills.com
The USMLE Guys Step 2 CK Crash Course usmleguys.com

M C L ca :
● A a f e fa a be

R Fac :
● Pe c f a a d ea e, ec c e a c , ba e , e DES
e e

S /S :
● Va a b eed , abd a/ e c a
● If ed b e ac ca d a, e , a a b eed , abd a / e c/
de a

Lab :
● CG a a ae (<35% e 2 da )

I a :
● Lac f a e e e a c a a a a d
● P e e ce f e a e e e a a ac a ac e b ee a a a
a d

Da :
● Def eda a f ef :
○ W e d c f c ce e ed e ea a a
a ea CG e e f a a
○ P e e ce f e a e e e a a ac a ac e b ee
a a a a d
○ H cc f a f ca e ec

T ea e :
● Me e aea aef ef ca e :
Pa e
○ e d a ca ab e, a a a a d fe a
ca d ac ac , CG e e 5000, acce c ef - ca e f e
● Me e ae a aef ef ca e :
○ Pa e e d a ca ab e, f e d /c e e,
e a / e a c/ e a c ab ab a e , e e ce f c e ab e
a e e e a c , PMH c a d ca e e a e (TB, e c
ce d ea e)
● S ca T ea e :
○ E e a ec a

Ob e c a d G ec 375
Get The Step 2 CK Drills Book: usmledrills.com
The USMLE Guys Step 2 CK Crash Course usmleguys.com

Content Re ie Questions:

W c f ef c a ac e c c e ada f ea e ed ab ?

A. Va a b eed
B. C ed ce ca
C. U a d a e e e a c
D. U a d fe a ca d ac ac

Me e ae a aef ed ca ea e f ec c e a c a e ce c f e
f c c a ce ?

A. Pa e a acce c ef - ca e f e
B. T a a a a d fe a ca d ac ac
C. CG e e 4,500
D. Pa e e d a ca ab e

W c f ef c a ac e c c e ada f e ab e ab ?

A. C ed ce ca
B. Va a b eed
C. Pa f c a
D. R e f e ba e

Ob e c a d G ec 376
Get The Step 2 CK Drills Book: usmledrills.com
The USMLE Guys Step 2 CK Crash Course usmleguys.com

T A a D S
F -T e e C b ed Te :
● Pe f ed be ee 10 ee a d 13 ee + 6 da e a
● C f ae a e be a- CG, a e a e e a c -a c a ed a a
e A (PAPP-A), a d c a a ce c a d
○ D S d e: I c ea ed c a a ce c , c ea ed be a- CG, a d
dec ea ed PAPP-A
○ T 18: I c ea ed c a a ce c , dec ea ed be a- CG, a d dec ea ed
PAPP-A
○ T 13: I c ea ed c a a ce c , dec ea ed be a- CG, a d dec ea ed
PAPP-A

Sec d-T e e Q ad e Te (Q ad Sc ee ):
● Pe f ed a 15 18 ee + 6 da e a dea (a a e a 22 ee + 6 da )
● C f ae a e a a fe e (AFP), CG, e ,a d b A
○ D d e: Dec ea ed AFP, c ea ed CG, dec ea ed e , c ea ed
b A
○ T 18: Dec ea ed AFP, dec ea ed CG, dec ea ed e , a b A
○ T 13: N a a e f AFP, CG, e ,a d b A
○ Ne a be defec : I c ea ed AFP

C cV Sa :
● Pe f ed be ee 10 a d 13 ee f e a
● I d ca : A e e 35, ab a f - e e a d c ee e , e
c d c a e e cd de , a e a ca e f c a/ e e c
d de
● F e ce ce b d a (FISH) f ed b a e c a a de
e e cda
● C ca : Me b a e e, e a c , a a b eed , fe ae a
e a e

A ce e :
● Pe f ed dea be ee 15 ee 17 ee + 6 da f e a (ea e a 11
ee )
● I d ca : Ge e c a a , a e fe a e b a e c a e a,
e a a e e a be defec fe a fec
● F e ce ce b d a (FISH) f ed b a e c a a de
e e cda
● C ca : Fe a , d ec d ec fe a , ea a e f a c f d,
c a c e aa , d c fb e f a

Ce -F ee DNA c ee :
● Pa e dec e a e ced e
● Pe f ed a 10 ee e a a a e ae

Ob e c a d G ec 377
Get The Step 2 CK Drills Book: usmledrills.com
The USMLE Guys Step 2 CK Crash Course usmleguys.com

● NOT d a c fa e d
● Pe f ed b d a ae a e a da a fe a ce -f ee DNA f d e
ae a a a
● M e e a d ec f c c ee f d d e, 18, a d
13
● I d ca ed : Ab a f - e e c b ed e ec d- e e ad e e ,
a e a a e e 35, e a e d e a c , ae a c a
a ca

D S d eS /S a B :
● Head a d ec ab a e c d e ca cf d , a a eb a f e ,
f a a a b d e, e a a a e f e ec , d e
● Ha d ab a e c d a e e a a c ea e, b ad a d , d a a f
e dd e a a f eff f e
● Ca d a c a defec c d c c eea e c a e a defec ,
e c a e a defec , a a e a defec
● D de a a e a e

Content Re ie Questions:

O af e e c b ed e , c f ef e c e d
d e?

A. N a c a a ce c , c ea ed be a- CG, a d dec ea ed PAPP-A


B. I c ea ed c a a ce c , c ea ed be a- CG, a d dec ea ed PAPP-A
C. I c ea ed c a a ce c , c ea ed be a- CG, a d c ea ed PAPP-A
D. I c ea ed c a a ce c , c ea ed be a- CG, a d dec ea ed PAPP-A

O a ec d- e e ad e e ( ad c ee ), c f ef e
c e d d e?

A. Dec ea ed AFP, c ea ed CG, dec ea ed e , c ea ed b A


B. I c ea ed AFP, c ea ed CG, dec ea ed e , c ea ed b A
C. Dec ea ed AFP, dec ea ed CG, dec ea ed e , c ea ed b A
D. Dec ea ed AFP, c ea ed CG, c ea ed e , c ea ed b A

W c f ef ca ee c de d d ea b ?

A. T a e e a a c ea e
B. U a a eb a f e
C. C eea e c a e a defec
D. C e - ed a e ac a

Ob e c a d G ec 378
Get The Step 2 CK Drills Book: usmledrills.com
The USMLE Guys Step 2 CK Crash Course usmleguys.com

C a ,P Lab , a P P ab R
M ba
Chorioamnionitis

R Fac :
● I c ea ed e f ab , c ea ed d a f ed e b a e , d a a a
e a ( e), ec - a ed a c f d, e a ac fec

S /S :
● Fe e , a e a a d fe a ac ca d a, e e e de e , e / a d a c
f d

Lab :
● E e a ed WBC
● A cf da a e a a a dc e, c e 14 /dL, WBC
3
c ce a >30 ce /
● P eb dc e ( b a ed f e ec ed)

C ca :
● C ca f ab c d eed f Ce a ea de e , e ea , a
b eed +/- eed f b d a f
● R f e a d fec a e e c d e c e c b eb ,
C- ec d fec ,e d e , e c ab ce

Da :
● Nea a a ac ca d a b c e fa cf d de def e
da

T ea e :
● B ad ec a b c (a c e a c )
● Va a Ce a ea de e
● C e ec c fe a
● Ace a e

Preterm Labor

Lab be bef e e a a a e f 37 ee .

R Fac :
● P ee b ; fec ; e e ab a e ; ed a a d e ac a ;
ce ca e ; abd a e e e a ; e e a c e a;a e
de 17 e 35; e ; ad ca ab ; b a ce

Ob e c a d G ec 379
Get The Step 2 CK Drills Book: usmledrills.com
The USMLE Guys Step 2 CK Crash Course usmleguys.com

e e bacc , c ca e, e ; e e a ; a a b eed ; fe a
af a ; da

S /S :
● P e ab a e a e e e ce c a e a c ac , e e
e e , a a d c a e f c a d/ b eed , e bac a

Lab :
● Fe a f b ec ca be e ed a e/ e a e a a a a e ea e e .
P e/ e e a e ed ed c e d f a e ee b

I a :
● Bef e 34 ee e a a a ea a a a a d a ce (<30
) c ea ed f ee ab

Da :
● 4 e e ec ac e e 20 e ( 8 60 e ) PLUS e f e
f :
○ S ec e a d a ce ca e a ce ca d a 3c
○ Ta a a a d ce ca e <20
○ Ta a a a d ce ca e e a 30 b e a
20 e fe a f b ec

T ea e :
● A 34 ee ae a e ad ed f de e
● Le a 34 ee ce ca d a f 3c ce ca e <20 20
30 e fe a f b ec :
○ Ad e be a e a e, e c c ( d e ac ) be a e a e a
e , a b c f GBS c e a
● Ma e fa e f e a a a e 24-32 ee

C de B P e a e a e a E e a ed R f eF C d :
● Re a ab a e
● Nec e e c
● Re a f e a
● Pa e d c a e
● I a e c a e a e

Preterm Prelabor Rupture of Membranes

R e f e ba e 37 ee e a .

R Fac :
● Sa e a ee ab

Ob e c a d G ec 380
Get The Step 2 CK Drills Book: usmledrills.com
The USMLE Guys Step 2 CK Crash Course usmleguys.com

S /S :
● F d ea f e a a c fe a dde a e ea ,b a
a be a e ff d ea c e e

S ec E a :
● P ff d e e a a a

Lab :
● N a e a e H f f d a 7.0 7.3
● Fe a e e a c f d d ed c c e de

I a :
● If ee e a a d da (a cf d de f
5c a e ca c e <2 c ) c e da

Da :
● C ca d a ba ed a de a f d f ff d e e
a a a
● If , a d da c e da
● If a cf d e a a e, c e ca e a e a e a
fe a e ca be ed a e eda

T ea e :
● Pa e e a a a e <34 ee ae ab e ea be a e a e,
a b c a ed e f a a c , IV a c f 48 , e
a a c f 5 add a da a d f e ec a a a e e
● Pa e e a a a e 34 ee ae ab e d be de e ed
● E ed de e e a d e f e a a a e f a e ab e ( ea
fe a e , ace a ab ,c a , c d a e)

Content Re ie Questions:

W c f ef c e ee e ab e f e ba e ?

A. P ff d e e a a a ec e a
B. N a e a e H f f d a 7.0 7.3
C. Fe a e a ed e a c f d d ed c c e de
D. A c f d de >7 c

Ob e c a d G ec 381
Get The Step 2 CK Drills Book: usmledrills.com
The USMLE Guys Step 2 CK Crash Course usmleguys.com

A a e 37 ee a d 4 da e a a bee ab f 40 .I e a
e a e a bec e feb e a d ac ca d c, fe a ac ca d a a e e e
.O ca e a e e e de a a .C a ec ed.
W c f ef ea e d ca ed?

A. Va a de e
B. Ce a ea de e
C. A c e a c
D. U e e a a e

A a e a e a a a e 35 ee a d 2 da e e e a c a f
e ec ac .Ue ec ac a e cc 4 e e e 20 e .O d a
ce ca e a e a e ed be d a ed 4 c . N e c ca ae ed, a d
e fe a d e a e ab e. W a e e be c e f ac ?

A. Ad a e f de e
B. Ad e be a e a e a d a a e e ec a
C. Ad e d e ac a d a a e e ec a
D. Pe f e e e Ce a ea de e

Ob e c a d G ec 382
Get The Step 2 CK Drills Book: usmledrills.com
The USMLE Guys Step 2 CK Crash Course usmleguys.com

F a M
A e a Fe a M :
● Fe a e e c
● N e e
● B ca f e
● C ac e e
● U b ca a e a d e a e e

Fe a M e e :
● P e a a e c e ed a ae e fe a e e c d a a ea d
a a ea ac e :
● Q a a e:
○ C ac ea ca e de f ee dec ea ed fe a e e f
ba e e
● Q a a e:
○ C ac ea ca e de f e a e a a e e de e
fee e a 10 c e 2c ec e d a e e e fe
a ac e

N e Te :
● N a e e c e a de ce e abd e c ea e fe a
ea a e a d a e a c ac

Ba e e Hea Ra e:
● Be 110 b b ad ca d a
● 110-160 b a ae
● Ab e 160 b ac ca d a

Va ab :
● F c a f e ba e e ea a e c ae e a a de a d f e e c
ae a a ab a d a e ea ed e a 10- e d
● A de ea ed f ea

T e f Va ab :
● A de de ec ab e ab e a ab
● A de 0-5 b a a ab
● A de 6-25 b de a e a ab
● A de >25 b a ed a ab

Acce e a :
● Ab c ea e fe a ea a e a a e a a a e 32 ee , a 15 ec d ,
a d ea 15 b ab e ba e e

Ob e c a d G ec 383
Get The Step 2 CK Drills Book: usmledrills.com
The USMLE Guys Step 2 CK Crash Course usmleguys.com

Ea Dece e a :
● G ad a dec ea e fe a ea a e a f e e ec ac , e ad f
e fe a ea a e a d e ea f e c ac cc a e

La e Dece e a :
● G ad a dec ea e fe a ea a e de a ed , e ad f e
dece e a cc af e e ea f ec ac

Va ab e Dece e a :
● A ab dec ea e fe a ea a e f 15 b f ba e e a 15 ec d 2
e f e e ba e e. T e e , ad , a d e d f e dece e a
fe a cce e e ec ac

Reac e Te :
● Te c de ed eac e e a a e a a a e f 32 ee e eeae2
e fe a ea a e acce e a (a ea f a ea 15 b ab e ba e e a a
ea 15 ec d ) bef e e ba e e, a cc a 20- e e d

N eac e Te :
● C e a e f eac e e e af e f a ea 40 e
● N eac e e ca d ca e fe a e a e ab c ac de a, fe a a ,
fe a ee , fe a e c ab a e , fe a ca d ac a a e , e , ae a d
e ca d ac effec , a e a
● If e eac e, e e e ea e e 30 e , e b ac c a ,
ef ac ac e e b ca f e e

Bioph sical Profile

C e f e Te :
● Fe a b ea e e (2 )
○ 1e de f c b ea e e f 30 ec d a 30- e
e d
● Fe a e e (2 )
○ 3d ceeb d / b e e 30 e .A e de f ac e
c e e c ed a e e e
● Fe a e (2 )
○ 1e de( ) f e e f e fe a e fe a b e fe
● A cf d e (2 )
○ T e e dee e e ca c e 2c
● N e e (2 )
○ Reac e e

N a Re :
● 10/10 8/8 f e e ef ed

Ob e c a d G ec 384
Get The Step 2 CK Drills Book: usmledrills.com
The USMLE Guys Step 2 CK Crash Course usmleguys.com

● 8/10 ( f a cf d c ed 2 )

E ca Re :
● 6/10 ( f a cf d c ed 2 )

Ab a Te :
● 6/10 8/10 f ae e f a cf d
● 0-4/10 a a ab a

Contraction Stress Test

C a d ca :
● P ee ab , e e e ab e f e ba e , ace a e a, a a e a,
e C- ec , e e e

Te :
● I f f c ee c ac cc 10 e

Re :
● P e e (ab a ) cc e a e dece e a ae e e f 50% f
c ac
● Ne a e ( a) e e a e dece e a f ca a ab e dece e a
e e

U b ca A e a D e A e e :
● Wee b ca a e a d e a e e d be e f ed e a ce
ec ed fe a e c
● H e c da cf e b ca a e a a f d
● Ab e e d-d a c e c (AEDV) a d e e ed e d-d a c e c (REDV) a e
ab a a d d de e a e AEDV a 34 ee a d REDV a
32 e e f e fe a a

Content Re ie Questions:

D a e e , a ad a dec ea e fe a ea a e a f e e e
c ac , e ad f e fe a ea a e a d e ea f e c ac cc
a e c e c f ef ?

A. Acce e a
B. Ea dece e a
C. La e dece e a
D. Va ab e dece e a

Ob e c a d G ec 385
Get The Step 2 CK Drills Book: usmledrills.com
The USMLE Guys Step 2 CK Crash Course usmleguys.com

Ab ca f e e ef ed f a e a fe a a e e .O a de a e
e dee e e ca c e fa cf d 3c a d 30 e 1e de f
c b ea e e f 30 ec d d a de f ed, 3 d c e e b e e
a e de f ed, a d e de f e e f e fe a e/ b e fe ae
de f ed. T e e e eac e. Ba ed e ef d a eb ca f e
c ea d a d e e c e d ca e.

A. 10/10 a e
B. 8/10 a e
C. 8/10 ab a e
D. 6/10 e ca e

W a d a be e a a f d ac ac e e de c ea e e
e d fc ac ?

A. O c
B. Te b a e
C. N fed e
D. Ma e fa e

Ob e c a d G ec 386
Get The Step 2 CK Drills Book: usmledrills.com
The USMLE Guys Step 2 CK Crash Course usmleguys.com

G a a D ab M
R Fac :
● H f e a a d abe e e e a c , fa f d abe e , BMI
>30, e ce e e a e a a a e 18-24 ee , f PCOS a ed
c e e a ce, de a e a a e, - e ace

Sc ee :
● Pe f ed a e a a a e 24-28 ee
● F e e e- 50- a a c e e a ce e
○ Ad e ed e ad e f da a ea
○ Mea e e c e c ce a af e 1
○ If c e e e 140 /dL: Sc ee e. M e Se 2(f e a e
f e e eeded)

Da :
● Sec d e e ee- 100- a a c e e a ce e
● Te ef ed a a e a fa ed f a ea e
● Te da c e 2 ab a a e b a ed

T ee- 100- a Oa G c e T e a ce Te :
Time Since Carpenter and National Diabetes
Oral Glucose Coustan (plasma or Data Group
Load serum mg/dL) (plasma mg/dL)

Fa 95 105
1 180 190
2 155 165
3 140 145

M :
● Pa e e a a d abe e e d e f- b d c e e e b
fa (bef e b ea fa ) a d a e e 1 2 a d a f eac ea
● Fa c e d be <95 /dL
● 1- a da c e c ce a d be <140 /dL
● 2- a da c e c ce a d be <120 /dL
● Sc ee a d a 36-39 ee a e f ac a
○ Ce a ea de e c ed ed a 39 ee e e a ed fe a e 4500
a a e e a a d abe e

T ea e :
● F - e ed ca a c e
● M de a e e e c e f a e c a d ca

Ob e c a d G ec 387
Get The Step 2 CK Drills Book: usmledrills.com
The USMLE Guys Step 2 CK Crash Course usmleguys.com

● If ed ca a c e a e fa , f - e a ac ea
● F a e dec e ab e a e , ef ca be ed

Content Re ie Questions:

W c f ef e ed f c ee f e a a d abe e e a d
-d a cf ed de ?

A. He b A1C
B. O e- 50- a a c e e a ce e
C. T ee- 100- a a c e e a ce e
D. O e- 5 IU e

A a e a 38 ee e a e a a d abe e e a a c ee
a d ef ed c e fe a e a ed fe a e f 4600 a .W a
e efe ed e f de e e e ef d ?

A. I ed a e d c f ab
B. I d c f ab a 39 ee
C. I ed a e Ce a ea de e
D. Ce a ea de e c ed ed a 39 ee

A a e a a a e d e a a d abe e e be c ee ed f e a
a e ?

A. 20-24 ee
B. 24-28 ee
C. 28-32 ee
D. 32-36 ee

Ob e c a d G ec 388
Get The Step 2 CK Drills Book: usmledrills.com
The USMLE Guys Step 2 CK Crash Course usmleguys.com

La T P a C a
Stillbirth

Fe a dea af e 20 ee e a a a e.

T ea e :
● A a e dec de f de e
○ S a e ab ca cc 1-2 ee af e fe a dea , b a f
a e ab c ea e e f de e b dc
● Be ee 20 a d 23 ee + 6 da : Pe f d a a d e ac a
● 24 ee f e a : Ad e f ce fa ab e ( eed f
fa ab e ce )a d e d ce ab c

Meconium Aspiration

Pe e :
● Pe e e (>41 ee ) de e
● I a a fe a ea (FHR) a d e d f fe a e a

S /S :
● Mec - a (a c f d, fe a a a d e ), fe a e c ,
e a d e /de e /fa e, e ca de e

I a :
● C e X- a a ea , ea de e
● F - c e X- a a fa e f eda a , e f a ed ,a d
d ff e a c de e

Da :
● Ne b e a d e a da f ef :
○ P e e ce f ec - a ed a cf d
○ C e X- a f d c e da
○ I ba ed e b e e ce f ec e ac ea

T ea e :
● D ef a a a a ea c e d ac ea c
● S e ca e f e e a e e d ac ea ba ba ed ca
e aa aa ee ( e d e a fa e, e a ,a a b c ,
ea a e <100 b ) e aa e e ca e
● C ec e ab c ab a e , a a ade a e BP, ac c a b c
bac e a e aa d e ed
● A a c fa ad ed a da d e

Ob e c a d G ec 389
Get The Step 2 CK Drills Book: usmledrills.com
The USMLE Guys Step 2 CK Crash Course usmleguys.com

Macrosomia

Fe a e ab e 4500 (9 b 15 ).

R Fac :
● P e e a c , a e a d abe e , e ac c fa , ae a be
e ce e e a a e a , a e fe

Pe e :
● E c a e e- e a c e be e a e ; c ec a e
d abe e e , e ec a e e a ;a da a e da d e ce e
e a a e a d e a c

C ca :
● Fe a c ca c de: B , e a d e d e, ec
a a , e aa a
● Ma e a c ca c de: M e e e e a a ace a , Ce a ea de e ,
e ee a e a e
● Pe a a c ca e b c d ce a, c e a, a a

Shoulder D stocia

S de ab e a e e af e de e f e fe a ead.

R Fac :
● Mac aa d e fac c c b e ac a

C ca :
● Te a / e a e b ac a e a , c a c a f ac e, e f ac e,
a a, fe a dea

Ne a a B ac a P e Pa :
● C5 a d C6 ca e f ea e e a d e a add c a d e a
a
● C5, C6, C7 c de ab e C5 a d C6 f ea a a d
fe f e a df e
● C5-T1 de e d e e ca ca e ea aa a ff e
fe fa a a dH e d e
● C8 a d T1 ca e e e ded , a e f ea , f e ed f e ,a dH e
d e

T ea e :
● S a e
● C f e a e

Ob e c a d G ec 390
Get The Step 2 CK Drills Book: usmledrills.com
The USMLE Guys Step 2 CK Crash Course usmleguys.com

● Ca a e e a ea ca e de e
● D a b adde , f d e ded
● Pe f e McR be a e e,a d f fa e ef McR be a e e
a bc e e
○ Ha e a a b ae a ee c e ( e fe e )a d f
a e fa a ea a a e e a b ca a /f d ec
e ed ada d aea

Content Re ie Questions:

A e a e be e a a ed af e a de e c ca ed b de d c a. O e a e
ef f ea a ed a d e e ded, a d e e a add c ed a d e a a ed.
Add a , ee a fe f e a df e e ef a d. W a e e f e
b ac a e ae e ed ?

A. C2, C3, C4
B. C4, C5, C6
C. C5, C6, C7
D. C7, C8, T1

W c f ef e e e a aef e a e b a e
e a d e a d S O2 f 62% de f ed ec - a ed a cf da b ?

A. I a a a a ea ec c
B. I ed a e e d ac ea ba
C. Ad e -bab e
D. P ac c a b c

W c f ef a e aa c ca d ec a c a ed de d c a?

A. B ac a e a
B. A a
C. C a c a f ac e
D. H ce a

Ob e c a d G ec 391
Get The Step 2 CK Drills Book: usmledrills.com
The USMLE Guys Step 2 CK Crash Course usmleguys.com

C a
Intrauterine Device

Be ef :
● Effec e, -ac , e e b e, d e e e da ad a b a e be
effec e, e e e e

Me a B eed IUD:
● C e IUD ca ca e e , ea e , a d/ e a f e e
● Le e e IUD ca ca e e e e a e ea

S de Effec a d C ca :
● I c ea ed f e c fa a d ea e, e f a f e e , IUD
a /e , fc ace fa e ae f ec c e a c

Etonogestrel Contraceptive Implant

Be ef :
● Effec e, -ac , e e b e, d e e e da ad a b a e be
effec e, e e e e

Me a B eed :
● Ca ca e e e e e a e e

S de Effec a d C ca :
● He a a, a ,e f e a , fec , e e ( ed a
a eb ac a c a e e e a e e)

Depot Medro progesterone Acetate Injection

Be ef :
● Re e bec ace (3 )

Me a B eed :
● Ca ca e e e e e a e e

S de Effec :
● Re fe a a e 1 ea , dec ea ed b e e a de , be e a

C a d ca E e -P e C ace :
● T bacc e e e a e f 35, e d ea e/ , e e ce f e e
e - e e ca ce (b ea ), e a C a e ce a a a d ,
e e , a e fac f ca d a c a d ea e f

Ob e c a d G ec 392
Get The Step 2 CK Drills Book: usmledrills.com
The USMLE Guys Step 2 CK Crash Course usmleguys.com

c e c ea d ea e e, b a, a a ea d ea e, a e
a a, a a e (21 da )

Combined Estrogen-Progestin Contraception ( a a , a de a ac , a )

Be ef :
● Effec e, de e a c cec , a d e e b , ed ced a f
e d e , e e a a c f a , e ac e a d , ea e
f fa e

Me a B eed :
● Ma ed ce b eed d e e

S de Effec :
● I c ea ed f e b e b , d c ea e b d e e, c ea ed
ce de , a ea, b a , b ea e de e , a ( a a ), e ea
( a a )

Emergenc Contraception

C e IUD:
● M effec e, ca be ed 5 da af e ec e, effec e e a d e f BMI,
ca be ed a a e f e e a c ce

Le e e IUD:
● Ne effec e, ca be ed 5 da af e ec e, effec e e a de f
BMI, ca be ed a a e f e e a c ce

U a Ace a e ( a ):
● Ca be ed 5 da af e ec e, ca be ed LH ea f e a
c c e, e c eeded, ed ced effec e e BMI >30

Le e e ( a ):
● Ca be ed 3 da af e ec e, ca be ed LH ea f e a
c c e, e c eeded, ed ced effec e e BMI >30

Ob e c a d G ec 393
Get The Step 2 CK Drills Book: usmledrills.com
The USMLE Guys Step 2 CK Crash Course usmleguys.com

Content Re ie Questions:

W c f ef ac a d ca c b ed e e - e c ace
e?

A. M a e a a
B. S e
C. C
D. A a

W c f ef f fc ace ca be ed f e e e c c ace
f ec ed ec e a a e e a a f fc ace ?

A. C e IUD
B. U a ace a e a
C. C b ed e e - e a
D. Le e e IUD

W c f ef de effec c ee de ed e e e ace a e
ec ?

A. Ve b e b
B. H e e
C. Dec ea ed b e e a de
D. Va

Ob e c a d G ec 394
Get The Step 2 CK Drills Book: usmledrills.com
The USMLE Guys Step 2 CK Crash Course usmleguys.com

M a C a M a
Me a C c e:
● Me a c c e be e e (da 1)
● T ef c a a e be e e a de d e da e LH e (14-21
da e f c a a e)
● T e ea a e be e da f e LH ea de d ce e e be (14 da
e ea a e)

Ea F c a P a e:
● I c ea ed f e e c f ad - e ea e (G RH) e
● I c ea ed f e e c f G RH e e c ea ed f c e- a e
(FSH) c ce a

M d-F c a P a e:
● I c ea ed FSH c ce a ca e de e e ff c e , ead c ea ed
e ad d c
● I c ea ed e ad c ce a ca e e ee d e fe a a d
c ea ed be f e d e a a d

La e-F c a P a e:
● C ed c ea e e ad c ce a ca e dec ea e FSH a d LH d e
e a e feedbac
● I c ea ed e ad c ce a ca e e ee d e c e
● S ed a f cea e

O a :
● E ad c ce a ea e da a
● M dc c e e cc c e a e feedbac ac LH a d FSH e
feedbac , da a c e LH c ce a (a d a e e FSH)
● LH e ead e f e a a f c e a d e e ea e f e c e

L ea P a e:
● C e d ce e e e (a d a a e a f e ad )
● P e e e ca c , a d de e e e ee d e
c ea ed ec e
● P e e e ac a e a e feedbac FSH a d LH e e eeded ee c
e f c
● LH e e bec e a c e a e , ca b e e d e
e e e a d e ad e e
● D f e e ea de e ca e e d e (e d f e ea
a e)

Ob e c a d G ec 395
Get The Step 2 CK Drills Book: usmledrills.com
The USMLE Guys Step 2 CK Crash Course usmleguys.com

Menopause

A e ea f 12 a a e e 45 ea d a b ab e a e
ca ca e.

S /S :
● H fa e , a a d e , de e , ee c a e

Ne a e Effec fL E e :
● L b e e a de , ca d a c a d ea e

T ea e :
● Pa e de a e- e e e ac a f fe ( a fa e ,
a ee d ba ce, a d/ dc a e ) d be ea ed c b ed
e e - e ea ( ed e e ea f a e
e ec )
● D a f e a a e ea d be ed 3-5 ea ca e
● If a e a c a d ca e e e a , SSRI, SNRI, aba e ca be
ed ea
● If a a a , -d e a a e e ea ae
ea e

Content Re ie Questions:

W c a f e e a c c e be e da f e LH ea de d ce e e
be ?

A. Ea f c a a e
B. La e-f c a a e
C. L ea a e
D. O a

W c f ef ed ca a aef ea e f e ee fa e
ca dec ea ed a f fe a a e a f e b e b ?

A. E e ea
B. E e - e ea
C. Me ca ba
D. C a a

Ob e c a d G ec 396
Get The Step 2 CK Drills Book: usmledrills.com
The USMLE Guys Step 2 CK Crash Course usmleguys.com

A 49- ea - d a e a ad e a e e f e a ea , e a e a e d
10 a .S e e deb a fa e c e fe e e a dfe e
a a e e a ead ee . S e e e a f fe e e e ce
be a . W a e a a e ea e da f a e ?

A. L -d e a a e e
B. E e ea
C. E e - e ea
D. N ea e d ca ed

Ob e c a d G ec 397
Get The Step 2 CK Drills Book: usmledrills.com
The USMLE Guys Step 2 CK Crash Course usmleguys.com

P a C a
Uterine Aton

R Fac :
● P ed ab , e a e a e, ed e f e e , e a ed
ace a, e d e

S /S :
● D e ded, f , b e , a a ea f b eed f de e
● P be a a a f e a ed ace a e d e e e

Da :
● C ca d a f e f af e d a e f ab

T ea e :
● E ab IV acce a d de f da db d d c e c a a eeded
● Ue e a a ea dc e
● Ue cd ( c , ca b e a e, e e e, )
● If b eed c e : e ec e e , a e e ba ca e e ,
a e e ac , e e a e e - a a a e a
● H e ec a e

Uterine Inversion

R Fac :
● Ra d ab , ed ab , b ca c d, e f e e ea a , e e
ab a e , e a ed ace a, ace a acc e a ec , ac a

S /S :
● S a d f e ce a a, ac f a ab e a ed
f d abd a e a , a a b eed , e abd a a , a ee

I a :
● U a d a e e b a a ca ed de e e a da
ab a c f e e ef d

Da :
● C ca f d c e da

T ea e :
● Ma a c ec e e ed e bac a a
● S e cd
● Ob a IV acce a d de IVF a d b d d c e c a a eeded
● If ace a a ac ed, d e e e bac a a

Ob e c a d G ec 398
Get The Step 2 CK Drills Book: usmledrills.com
The USMLE Guys Step 2 CK Crash Course usmleguys.com

● Re a e cd ce ace a e ed a d e ed a
● S e d e f cefa f e d e a
● If a a c ec fa , ca c ec ef ed

Postpartum Urinar Retention

Ca e
● P de da e e d ab

R Fac :
● E d a a e e a, e -a ed de e , e , a

S /S :
● U a e c , c e ce, f e e c , e e ea , a d e ,
a , fee f c eee

Da :
● Lac f a e a f a a de e
● Lac f a e a f ca e e e a f Ce a ea
de e

T ea e :
● I e e ca e e a

Postpartum Endometritis

R Fac :
● Cesarean deli er , fec (c a , a a GBS E. c c a ,
a a S. a e c a , HIV), ce a c ead c ea ed f fec
( ed ab , e ce ca e a , e a fe a , ed e f
e ba e , a a e a f e ace a)

S /S :
● Fe e , c , ac ca d a, e abd a a , e de e a a f e e ,
e c a, eadac e, a e a

Lab :
● E e a ed WBC ef f

Da :
● P e e ce f a ea f ef :
○ U e e e de e abd a a a b ab e a e ca e
○ P e c a
○ Fe e

Ob e c a d G ec 399
Get The Step 2 CK Drills Book: usmledrills.com
The USMLE Guys Step 2 CK Crash Course usmleguys.com

T ea e :
● E ca b c (c da c a d e a c ,a d f e e 24 add
a c )

Content Re ie Questions:

A 34- ea - d fe a e a c e ed e d a e f ab . O e a e a e
e d a ca ab e, b a d e ded, f,b e fe . W a e be a
ea e f a e c d ?

A. U e e a a e
B. U e e c e e
C. I a e e ba ca e e
D. H e ec

W c f ef d d be a aef e ea e f e ea ?

A. O c
B. Me e e
C. Te b a e
D. Ca b e a e

W c f ef a d a f eda cc e a f a
e d e ?

A. U e e e de e
B. S a d f e ce
C. P e c a
D. Fe e

Ob e c a d G ec 400
Get The Step 2 CK Drills Book: usmledrills.com
The USMLE Guys Step 2 CK Crash Course usmleguys.com

V a a Va a D a
Vulvar Lichen Planus

S /S :
● V a , a (b e e ), d ae a
● G a ed e e a e f e a e ae/ e b de f e OR
c ace a e OR e ea e e e a d e a a a ea

Da :
● C ca d a ba ed ca c e e a
● Def eda ba ed b a d

T ea e :
● F - e ca c c e d
● Sec d- e ac

Vulvar Lichen Sclerosus

S /S :
● V a ,d ae a, d a, a a d c f
● Pa e c aea ca d e a d c a e ce a e ,a d a e e d e
e c a f d b c ;f ; ca d e e e a a

Da :
● C ca d a ba ed ca c e e a
● Def eda ba ed b a d

T ea e :
● F - e ca c c e d
● Sec d- e ac

Vulvar Cancer

R Fac :
● V a c e ce , , f ce ca ca ce , def c e c

S /S :
● V a e ( ce , a e a ), , a , b eed

Da :
● V a b c e a ca ce

Ob e c a d G ec 401
Get The Step 2 CK Drills Book: usmledrills.com
The USMLE Guys Step 2 CK Crash Course usmleguys.com

T ea e :
● De e d a :S e , ad a e a , a d/ c e ea (c a )aea
ea e

Vaginal Cancer

R Fac :
● HPV fec , , DES e e e

S /S :
● A a c a a a , a a b eed , a a d c a e (b d
a d )
● If ca ce a e e ded ca a a d GI a be e e

Da :
● Va a b c e a a ca ce
● T ca :
○ O de a e a e a ce ca c a
○ Y e a e DES e e e a e c ea ce ade ca c a

T ea e :
● De e d a e , ad a e a , a d/ c e ea (f ac
a d/ c a )aea ea e

Atrophic Vaginitis (Vulvovaginal Atroph )

R Fac :
● C d e e e ( e a e, a a a ff c e c , b a e a
ec ,a e e cd , e e a ed ac e e )

S /S :
● V a a d e ; ; a ; b eed ; d a e a; f e ; efa a d
; f ec e / b ca ,ea c ,a d ae
● U a f e e c , e a a, d a, ec e UTI

Da :
● C ca d a ba ed e e ce f e e c d a dca c e e a

T ea e :
● F - e ea e a a b ca
● Sec d- e -d e a a e e

Ob e c a d G ec 402
Get The Step 2 CK Drills Book: usmledrills.com
The USMLE Guys Step 2 CK Crash Course usmleguys.com

Vaginismus (Genito-Pelvic Pain/Penetration Disorder)

S /S :
● Pa a d/ a f e a a e e a e de ce f fec
de a , e e e cf ce d e e a
● P e e f a ea 6 ca a e d e

T ea e :
● De e a ec e (Ke e e e c e , e f a a d a )

Bartholin C st and Abscess

Ba C a d Ab ce e :
● L ca f e Ba a d :4 c c a d8 c c a d e a a
f ce
● Fea e f Ba c : Pa e d a f f a , fda a e e e c
c ea e
● Fea e f Ba ab ce : Se e e a f a ( a be f c a , a , a d/
e de ), b e fe e , f d a a e e e e ee e c

T ea e :
● Ba c e a 3c d be ea ed a c e e a d ba
● Ba c 3c a e ea ed c a d d a a e a d ace e fW d
ca e e
● A ed Ba ab ce e d a e c a dda a e ef ed
● F Ba ab ce e 3 c a e,c e f f d a d a b c ea e
( e - fa e a e) W d ca e e ace e

Content Re ie Questions:

A fe a e a e e e e ED c a fa a f a .O e a e a
ca ed a e 8 c c a d e a a f ce. O a a e a a a d
f c a a d ee e e e e de e e e . Ba ed e ef d , a e
e da ?

A. V a c e c e
B. A c a
C. Ba c
D. Ba ab ce

Ob e c a d G ec 403
Get The Step 2 CK Drills Book: usmledrills.com
The USMLE Guys Step 2 CK Crash Course usmleguys.com

A 37- ea - d fe a e e e ec c ac a fa a e a. O e a e
e a ea a a a ed e e a e f e a a e e b de
a e e .T e a e e d e ab a a e e f e e .W a e
e da e e ef d ?

A. V a c e a
B. V a c e c e
C. V a a a
D. T a a c e a

A 51- ea - d a e e e ec c ac a f a ec e. S ce
e a e, e a e a ed a a d e , e ,a da f ec e a d
b ca .S e a a e ed e e ec e a a b ca a
e e .W a e e be e a a e e f a e c d ?

A. L -d e a a c c e d
B. U e f a a d a
C. L -d e a a e e
D. T ca ac

Ob e c a d G ec 404
Get The Step 2 CK Drills Book: usmledrills.com
The USMLE Guys Step 2 CK Crash Course usmleguys.com

B a Ma a B a L
B ea Ca ce R Fac :
● BRCA1 BRCA 2 a , a e fa f b ea a a ca ce ,
b a ca e a a, fc e ad a e a , c ea a e, ea
e a c e, a e e a e

B ea Ca ce Sc ee :
● A e 40-49:
○ S a ed dec - a be ee d c a d a e , b e c ee
a a e e 1-2 ea
● A e 50-74:
○ Sc ee a a e e 1-2 ea
● A e 75 a d de :
○ Sc ee a a e e 2 ea f a e fe-e ec a c 10
ea e
● C ca b ea e a a d b ea e f-e a ec e ded f c ee
a a c, a e a e a e
● If ab a ee c ee a a ,da c a a
ef ed:
○ If d a c a a a a /be f d , e ea c ee
a a
○ If d a c a a a bab e be f d , e ea d a c
a a 6
○ If c e e f ca ce , b e e a c e eed e
b

Be Le f e B ea :
● Ha a a: E ca a ed a e a ade f a d a,fb , a d fa
c e
● L a: T ade f fa ce
● Fa ec :F b ea a a ca e e a a a ca c f ca
a a c a a c

Breast C sts

S /S :
● A a c a ab e b ea a

I a :
● B ea a d ed d ffe e a e ea dc e c
● S ec fea e c de be dc e , e c c c bed, a d
a e a ec c
● C ca ed c a e -e e e a ec e d e ec e c deb , a d e e
a e fea e f ec

Ob e c a d G ec 405
Get The Step 2 CK Drills Book: usmledrills.com
The USMLE Guys Step 2 CK Crash Course usmleguys.com

● C e c fea e c de c a , c e a, b c ca d dc e ,
a d/ a ec e cc e

Ma a e e :
● S ec d e ef e a a e e ( e a ) a d f e eed e
a a ef ed f f fec e e
● M c ca ed c d e ef e a a e e , b f ca e ed a
b be , e ea a 6 a a ed b e e c e eed e
b f c e be a a c de e
● C e c e e a e- ded c e eed e b de e e f be
a a

P fe a e Le W A a:
● Sc e ade :Fb ec a e a d a ce ( b a)
● Rad a ca : F b e a c c e ad a d c a d b e , e e ca e c
● D ca e a a: I c ea ed be ce d ca ace
● I ad c a a a: Pa a ce c f a a fac e
● S ca e c d ca ed f e a c ad a ca e e c e
eed e b

F b ade a:
● P e e a a e -def ed, b e a b ea e a
● S ec a e de e de e
● M c a e a ed 15-35
● B ea e a a d a c e f b ade a a a e e c eb
3-6 a a d b ea e a f -

P fe a e Le W A a:
● A ca b a e a a: U a f d c de a e e a a e b ea
e , ce d a ed c e eed e b e c a b ea b d ca ed f
a e e e e e c a b ea b
● L b a ca c a :U a f d c de a e e a a e b ea
e , ce d a ed c e eed e b c a c b a ca c a fea e
a a a f - ,a e e f b a ca c a fea e a a
e c a b ea b
● A ca d c a e a a: T ca a ea a e c ca c f ca
a a , ce d a ed c e eed e b , e be e e c a b ea
b

Breast Cancer

S /S :
● B ea a c - b e, a d, e a b de , f d c a
d , c e , e e a, a d/ a a ade a

Ob e c a d G ec 406
Get The Step 2 CK Drills Book: usmledrills.com
The USMLE Guys Step 2 CK Crash Course usmleguys.com

C S e f Me a a :
● B e: Bac a
● L e : Ja d ce, abd a a
● L :D ea, c
● B a : Headac e, a ea, ea e , e e , e a a c a e

I a :
● Da c a a f d c e b ea ca ce c de:
○ H -de a c a ee a e ca ce
○ G ed c ca c f ca , f e de / a

B ea U a dF d C e B ea Ca ce I c de:
● A a a d d c a , c a , ca c f ca , ec c e be a e
a de

MRI F d C e B ea Ca ce I c de:
● S c a ed a e a a , e a e a e a ce e , a d e a
e a ce e f e a

Be I a M da W e Pa ab e B ea Ma De ec ed:
● Pa ab e b ea a a e de 30:
○ Pe f a d
● Pa ab e b ea a a e 30-39:
○ E e e f a d da c a a
● Pa ab e b ea a a e e 40:
○ Pe f da c a a

H ca T e f B ea Ca ce :
● I f a d c a ca c a, f a b a ca c a, ed d c a / b a
ca c a

B ea Ca ce Ge e E e :
● H a e de a fac ece 2 (HER2), e e ece (ER),
e e e ece (PR)

Breast Cancer Staging

T S a e (P a T ):
● T1: P a 20 e ea ed f ed e ed e f e c e
● T2: P a >20 a d 50 ea ed f ed e ed e f e
c e
● T3: P a >50 ea ed f ed e ed e f e c e ,
d ec e e ec e a a d/

Ob e c a d G ec 407
Get The Step 2 CK Drills Book: usmledrills.com
The USMLE Guys Step 2 CK Crash Course usmleguys.com

● T4: P a fa e d ec e e ec e a a d/

Re a L N de :
● N0: N de e a a e
● N1: M ab e e aea a a de/ de
● N2: F ed e aea a a de/ de
● N3: Me a e a f ac a c a / e a a a / ac a c a
de/ de

D a Me a a :
● M0: N d a ea a
● M1: D a ea a

Management of Earl -Stage Breast Cancer

Ea -S a e B ea Ca ce (T1N0-T2N1):
● B ea e , ca e a a fa a de a d d ec f ece a
● P b e ad a e a de e d e f b ea e
● P e a a e e ece - e b ea ca ce : T ea a
a aa e b
● P e e a a a e de 35 e ece - e b ea ca ce e
35 - fea e : T ea a a e a da aa e b
● P e e a a a e e 35 - fea e e
ece - e b ea ca ce : T ea a fe
● If e- e a e b ea ca ce : T ea c e e a (d bc ,
c c a de, a d ac a e )

L ca Ad a ced B ea Ca ce (T3N0-T4N3):
● C e ea ( a ab f HER2 e b ea ca ce )
● B ea e a d ca e a a f de f c e ea
● P e a a e e ece - e b ea ca ce : T ea a
a aa e b
● Pe e a a a e de 35 e ece - e b ea ca ce e
35 - fea e : T ea a a e a da aa e b
● Pe e a a a e e 35 - fea e e
ece - e b ea ca ce : T ea a fe
● If e- e a e b ea ca ce , eed ea a a c e ea ,b f
HER2- e a a cc ee e e f e b e e ca
e ec cc , c e a ab f 1 ea

Ma a e e f Me a a c B ea Ca ce :
● F e e e a a c b ea ca ce f - e ea e e d c e ea
a a aa e b

Ob e c a d G ec 408
Get The Step 2 CK Drills Book: usmledrills.com
The USMLE Guys Step 2 CK Crash Course usmleguys.com

● F HER2 e e a a c b ea ca ce f - e ea e a ab,
e ab, a d ac a e
● C e ea e a e ea e a a e

Content Re ie Questions:

W c f ef a fac f e de e e f b ea ca ce ?

A. BRCA1 a
B. La e e a c e
C. La e e a e
D. M e e e a a b ea ca ce

W c f ef f d ec e a e b ea c c a ed a
c e b ea c e ee a d?

A. T c c a
B. T c c e a
C. P e e ce f b c ca d dc e
D. A ec c

A 50- ea - d fe a e ee a e c c f a a a ca . S e a a ed ca
a d f ca fa . He ca e a a.T e a e ae e
fee e ab a b ea ca ce a e be f e d a ece da ed. G e e
a e a ea d ee a f d , c f ef a da e
a a e be a e f e e e ce f b ea ca ce ?

A. Sc ee a a
B. D a c a a
C. B ea a d
D. B ea MRI

A 28- ea - d a e e ec ac c a f a e b ea a .O e a a
b ea a a a ed c - b e, a d, e a b de d
ee ab e e a . N ade a ed. W a e e be e
a a e e ?

A. D a c a a
B. B ea a d
C. C e eed e b
D. Rea a ce a d f e e e eeded

Ob e c a d G ec 409
Get The Step 2 CK Drills Book: usmledrills.com
The USMLE Guys Step 2 CK Crash Course usmleguys.com

A 56- ea - d a e da ed f a d ca b ea ca c a ba ed c e
eed e b .T e a 18 e ea ed f ed e ed e f e e
c e a d eeae ab e e a a e aea a a de .N d a
ea a a e de ec ed. W a ea a e b ea ca ce a ba ed e e
f d ?

A. T1N1M0
B. T2N1M0
C. T1N2M0
D. T2N1M1

A 67- ea - d a e da ed f a b a b ea ca c a ba ed c e
eed e b .T e a 25 e ea ed f ed e ed e f e e
c e a d ee d ec e e e . T e e a e f ed e e aea
a a de a d a ea a e e. W a ea a e b ea
ca ce a ba ed e ef d ?

A. T2N2M1
B. T2N3M1
C. T4N3M0
D. T4N2M1

Ob e c a d G ec 410
Get The Step 2 CK Drills Book: usmledrills.com
The USMLE Guys Step 2 CK Crash Course usmleguys.com

D aa U Ab a
Primar D smenorrhea

S /S :
● L e abd a/ a bcc a a a be 1-2 da bef e a e a
b eed a a d a f 3 da ad a
● Na ea, d a ea, a d eadac e a e e e e

Da :
● C ca d a ba ed e de ce f e d de a ca ef
a

T ea e :
● E e c e a d ea ad a ca
● NSAID a d a c ace

Se a e U e :
● M c e ea a
● Ide f ed a d e ee e e ce f e d e a ca e e a a ed b
a e a d de a ef d
● I c ea ed f a e ab , ec e ca a e, ace a ab ,a d
b eec e e a
● T ea e c de e ec f e e e e a c c e , f
a a ed

Bc aeUe :
● Occ e e a d c a a a e a c ee f e
● U a d f d c de ed 1 c ea dd e e e d e a
ca e
● I c ea ed f a e ab , fe a dea , ee b , fe a
e c , a d b eec e e a
● Pa e a c a ca e a c c e ca de e e
e f ca e

I ef aeH e :
● I e ae e e a b
● If c ec ed e e a e, a e a c e a e a e e a e ea,
e a c , c c c abd a/ e c a , a defeca , a d ff c
● O e ce a ,ab b c f e a a a ed
● U a d ca c f e e e ce f a e ba ec e a ef ae
e
● T ea e ca e a e ca c e e b a e a d e ac a f e
b c ed c e

Ob e c a d G ec 411
Get The Step 2 CK Drills Book: usmledrills.com
The USMLE Guys Step 2 CK Crash Course usmleguys.com

M e a A e e :
● Ab e ce f e a a ab e ce af a f e e
● NT3, HNF1B, a d LH 1 e e a
● Of e a e e aea e a a e e , e c e e d e ,a d
eea a a e
● Pa e e e a a e ea
● Va a d e ee e a a d e e a e e a e a a a d ec da
e c a ac e c
● T ea e a a d a ca c ea fa e a a

Endometriosis

P e e ce f e d e a e de f ee d e .

S /S :
● D a e a, d e ea, a a a , fe
● De e d ca ab a e e b eed , a e ca ,b e a d
b adde d f c

I a :
● U a d MRI a de f a a c d e a a a ca ca

Da :
● Def e da ed eb a a c

T ea e :
● M d- de a e a ea NSAID a d c b ed a c ace e
● If NSAIDS a d a c ace e fa , G RH a (e de) ed a d add-bac
e (e e e d e c b ed a c ace e )
● If ab e ed ca e a fa , ad e a aa e b
● If a a ac e a fa , ea a a c f def eda a d ca
e ec f e

Adenom osis

E d e a a d a d af d e e .

S /S :
● D e ea; e a a; e c e a a f, b e, d ff e e a ed
e

C ca :
● M ca a e, ee b

Ob e c a d G ec 412
Get The Step 2 CK Drills Book: usmledrills.com
The USMLE Guys Step 2 CK Crash Course usmleguys.com

I a :
● Ta a a a d a e c c e f e e , f b de
be ee e d e a d e , e a c , e a ee e e ,
ea a ad a e f ee d e

Da :
● Made af e c de f ca f e ec

T ea e :
● H a ea f a e de c dbea , e ec def e
ea e

Leiom oma

R Fac :
● N a , ea e a c e, b ac ace, ed ea c ,ac e

S /S :
● D e ea, e a a, d a e a, e c a , c a , d ff c e
e b adde , c ea ed f b e b (f e e a ca a), fe
● E a ed e e a c c e a b e f e ee e a

I a :
● Pe c a d e -c c c bed ded a e c ae ec c
● If e f b d de e e a - e ca c f ca e a a be ee
ad be d e ca c f ca

Da :
● C ca d a ade ba ed a

T ea e :
● F e e a aa d de e e a c , e c c ec
ef ed f b c a fb d
● Pa e ac e de e a c ca be ea ed c b ed
e e - e c ace e f ed ca fa a ad - e ea
ea ca be ed
● F a e d a bea c d e ef ea d a e fa ed ed ca
ea , ec , e ea e e b a , e ec aea

Ob e c a d G ec 413
Get The Step 2 CK Drills Book: usmledrills.com
The USMLE Guys Step 2 CK Crash Course usmleguys.com

Content Re ie Questions:

A 31- ea - d fe a e e e ec cf a f e a e ab .A
e ,a a d ef ed c de a ef d f e e a d
e a a e e d e a ca e c a e d ded b e. W a e e
da ?

A. Ade
B. B c ae e
C. Le a
D. Se a e e

W a e a a e ea e f a 28- ea - d fe a e a e
ca a e d e e e e ce f a b c a fb d c cc 70% f e e e
ca a d de e f e c dbea ?

A. N e e
B. C b ed e e - e c ace e
C. H e c c ec
D. H e ec

C c aec f e a ae c e a c a ed c f ef c d ?

A. P c c a d e
B. A a d a c c d e d ea e
C. Ade
D. E d e

Ob e c a d G ec 414
Get The Step 2 CK Drills Book: usmledrills.com
The USMLE Guys Step 2 CK Crash Course usmleguys.com

O a a T ,O a a C R ,M
Ovarian Torsion

R Fac :
● O a a c , a a e a , e a a

S :
● Pe c/abd a a , a ea/ , ad e a a , fe e

I a :
● Pe c a d c a a e a ed, d a a ee e e
f ce e e e ea , a ca a be a e e e

Da :
● V a a f d e

T ea e :
● De f e a be a - ec c da a e

Ovarian C st Rupture

R Fac :
● O a d c , a a e

P ec e:
● E e e c ace e

S /S :
● Se e e aea e ad a abd a a ( a , f ca ) f a a
ec e
● La e e e a e ca ca e e abd a a d/ de a
● P be - ade fe e

Lab :
● CBC be e a c , fe e a a e
● Ob a b d ea dc f e e d a c ab
● STI e a d a ef ed f fe e e e e a ae e c fa a
d ea e UTI
● Se CG d be e a e e c de ec c e a c

I a :
● U a d ad e a a f d e e aef d c e
a a c e

Ob e c a d G ec 415
Get The Step 2 CK Drills Book: usmledrills.com
The USMLE Guys Step 2 CK Crash Course usmleguys.com

Da :
● C ca d a ba ed ff , ab , a d a e c f e
c d

T ea e :
● If c ca ed ( e d a ca ab e, f ca d e a c ,a d
c b eed ), a a e a e b e a
● If c ca ed ( e d a ca ab e, f ca d e a c , a d/
c b eed ), a e a a e e f d , a f ,a d e
e a e, a a ae

Mittelschmer

F cee a e e a d e a d e ea e f e c ae e e e .

S /S :
● D a aea d-c c e e abd a a c a be acc a ed b
a a f a a d c a e b eed a 2 da

Da :
● C ca , ba ed

T ea e :
● NSAIDS ace a e
● If a ed ca ae ade a e, a e c ace

Content Re ie Questions:

A 30- ea - d fe a e e e e a abd a a .T e a e ae e a
a a a ec e e ba d e e fe a dde e e e a e
e ad a c a a d f ca . Lab a e a a a d e e a c e
e a e. U a d a ad e a a f d e e .B a e a ea f
ca a e. T e a e d a a ed a d a ea a . Ba ed
e ef d , c f ef da e ae e ?

A. O a a c e
B. Ec c e a c
C. A e d c
D. O a a

Ob e c a d G ec 416
Get The Step 2 CK Drills Book: usmledrills.com
The USMLE Guys Step 2 CK Crash Course usmleguys.com

W c f ef e e a a dae c e a a ?

A. E a ed a d ded a
B. P e e ce f f c e e ea e a
C. H e e ec e e
D. O a ca ed a e e e

W c f ef ea e da e a aef ea e c e ?

A. Na e
B. Ace a e
C. C c be a e
D. C b ed e e - e a c ace e

Ob e c a d G ec 417
Get The Step 2 CK Drills Book: usmledrills.com
The USMLE Guys Step 2 CK Crash Course usmleguys.com

E a a O a a Ca
Endometrial Cancer

Pa :
● T eI :
○ G ade 1 2
○ Ma fe d e a ca ce
○ E e - d ced
○ Of e eceded b a ca c e e d e a e a a
○ Fa ab e
● T e II :
○ G ade 3
○ 10-20% f e d e a ca ce
○ Ma be -e d e d ( e , c ea ce , c )
○ Le fa ab e

R Fac :
● U ed e e ea , a fe , a e e a e, a , c ea ed a e,
be , PCOS, L c C de d e

P ec e Fac :
● E e - e a c ace e

S /S :
● Ab a e e b eed

Da :
● T e a ef e d e a b , c e a e, e ec c e
d ea e

L -R E d e a Ca ce T ea e :
● G ade 1 2, ca ed ee d e ad e a af e e ,
a - ( c ea ce e )
● Pe f a e ec ,b aea a - ec (BSO), a d e a a e
de f e a e e d ea e

H -R E d e a Ca ce T ea e :
● G ade 3, c ea ce e ade ca c a ; a e III/IV d ea e
● Pe f a e ec a d BSO ad a c e e a (ca b a
ac a e ) a d/ ad a

Me a a c E d e a Ca ce T ea e :
● G a f ea e a a
● C e e a (ca b a a d ac a e )

Ob e c a d G ec 418
Get The Step 2 CK Drills Book: usmledrills.com
The USMLE Guys Step 2 CK Crash Course usmleguys.com

● If HER2 e ea a ab
● If ac e a -def c e c ae e ab e- e e ea
e b ab

Ovarian Cancer

R Fac :
● Ea e a c e, a e e a e, a , d a e, BRCA1 a d BRCA2 a ,
L c d e, fa

P ec e Fac :
● B aea a - ec , e ec , ba a , a c ace e , a ,
b ea feed

S /S :
● S bac e e e a c de b a , a ea, ea a e , a fe e c
e c , abd a a d e , e c a , a a b eed , ad e a a
e ce a
● Ac e e e a (ad a ced d ea e) c de a c e , e b e b ,
b e b c , e a eff

I a :
● Pe c a d a ad e a a a dc d ae (
a a ), c e a ,a da c e a c e a a ca ce
● If e c a d f d c e a a ca ce , CT f c e ,
abd e , a d e ef ed a e f e e ce f e

B a e :
● P e ca ce a e 125 (CA 125)

Da :
● Made a ca c f a f e b a ed b aea
a - ec b f e e e

Sa :
● S a e I: Ca ce c f ed e a e fa a be
● S a e II: P a e ea ca ce ca ce e a e fa a be
e e be e cb
● S a e III: P a e ea ca ce ca ce e a e a d/ fa a be ,
c f ed e ea e de e e a d/ e e e e ea
de
● S a e IV: P a e ea ca ce ca ce e a e a d/ fa a be ,
c f ed ead

Ob e c a d G ec 419
Get The Step 2 CK Drills Book: usmledrills.com
The USMLE Guys Step 2 CK Crash Course usmleguys.com

T ea e :
● F a e ca ce c f ed e a e ( - ade a d ea a e):
H e ec BSO e ec a d e c a d a aa c ade ec
( e a ) a ae
● F e e ade a d a e c e ea a d e a a ed

Content Re ie Questions:

W c f ef a fac f de e e d e a ca ce ?

A. La e e a e
B. P c c a d e
C. L c d e
D. E e - e a c ace e

W c f ef a c a ed eIe d e a ca ce ?

A. G ade 1 e d e d ca ce
B. P eceded b a ca e d e a e a a
C. E e - d ced e d e a ca ce
D. Se e

W c f ef b a e ae ed a a ca ce e a d
ec e ce?

A. Ca ce a e 125 (CA 125)


B. Ca b daea e 19-9 (CA19-9)
C. Ka e -3 (KLK3)
D. Pe c ea a - e c a ca b d e ( -ANCA)

Ob e c a d G ec 420
Get The Step 2 CK Drills Book: usmledrills.com
The USMLE Guys Step 2 CK Crash Course usmleguys.com

C a Ca
Ce ca Ca ce Sc ee :
● A e 21-29:
○ Ce ca c ( a ea ) e e 3 ea
● A e 30-65:
○ Ce ca c ( a ea ) e e 3 ea
○ HPV e e e 5 ea
○ C -e (HPV e ce ca c ) e e 5 ea
● A e e 65:
○ S a ed dec - a a e , f e c ee d c ed f a e
ade a e c ee a d a e c ed a e 74 f e a e
a a fe e ec a c f a ea 10 ea

Ab a e Sc ee :
● Pa e 25 de CIN 1 eceded b - ade a ae e a e
(LSIL), a ca a ce f de e ed f ca ce (ASC-US), e a ef
ae e a e a a c (NILM) e f HPV: Ne e 1- ea f -
HPV-ba ed e
● Pa e 25 de CIN 1 eceded b a ca a ce ca e c de
- ade a ae e a e (ASC-H): Ne e HPV-ba ed e
1- ea
● Pa e 25 de CIN 1 eceded b - ade a ae e a e
(HSIL): Ne e da ce c a ced e f a e a f e a f
c dbea , e 1- ea f - c c a d HPV-ba ed e
● Pa e 25 de e e CIN 2 3: T ea e ec e ded/ efe ed, e
e da ce c a ced e e e c d fe c e, e ec ca
e c ced e (LEEP), a e c eb
● Pa e de 25 CIN 1 eceded b LSIL ASC-US: Ne e e ea
c a 1 ea
● If e a LSIL ASC-US, e ea c af e 1 ea
● If e ASC-H HSIL, e f c c
● Pa e de 25 CIN 1 eceded b HSIL: Ne e c c a dc a
1 a d 2 ea
● Pa e de 25 CIN 1 eceded b ASC-H: Ne e c a 1 a d 2 ea
● Pa e de 25 CIN 2: Ne e c a dc c a 6 a d 12
○ If CIN 2 e a f 2 ea ( b e a c a dc c a 6
e a ) e CIN 3 e e e da ce c a
ced e
● Pa e de 25 CIN 3: Ne e da ce c a ced e
● A ca a d a ce (AGC) (a bca e e e ce a ca e d e a ): Ne e
c c a d f 35 de , f fac f e d e a ca ce e e e
de 35, e d e a a
● AGC bca e a ca e d e a ce e e e d e a a d
e d ce ca a

Ob e c a d G ec 421
Get The Step 2 CK Drills Book: usmledrills.com
The USMLE Guys Step 2 CK Crash Course usmleguys.com

Cervical Cancer

R Fac :
● HPV- e a ed ( e a ef f e a e c e, a a - e a a e
a e e a a e , e )
● S , c ec c a , a c ace e e

S /S :
● B eed d /f a a ec e, a a d c a e, e a a a
b eed , be a a f e e ce a

HPV:
● HPV a c a ed ea a ca e f ce ca ca ce , HPV 16 a d 18
c de f ed b e

Da :
● C c d ec ed b da ce c a ced e c a c d fe
c e, LEEP, a e c eb de f a c e ce ca ca ce

Sa :
● S a e I: Ca ce c f ed e ce
● S a e II: Ca ce ade be d e e e e e e d f e
a a e e c a
● S a e III: Ca ce ade e e d f e a a, e e d e e c a , ca e
d e , e e e c a a-a c de
● S a e IV: Ca ce e e ded be d e e e e b adde ec d a
a

Ea -S a e Ce ca Ca ce :
● I c de S a e I ce ca 4c ea e d e
● P a ea e ca a a e e ( ad ca e ec )

L ca Ad a ced Ce ca Ca ce :
● I c de S a e I ce ca >4 c ea e d e S a e IV
● P a ea e c e ad a c a

Ob e c a d G ec 422
Get The Step 2 CK Drills Book: usmledrills.com
The USMLE Guys Step 2 CK Crash Course usmleguys.com

Content Re ie Questions:

A 22- ea - d fe a e de e e ce ca ca ce c ee ce ca c . He
e ce ca ae e a e a a 1 (CIN 1) eceded b - ade a
ae e a e (LSIL). W a e e be e a a e e ?

A. Re ea ce ca c 1 ea
B. C c
C. C d fe c e
D. L e ec ca e c ced e (LEEP)

A 40- ea - d fe a e de e e ce ca ca ce c ee ce ca c . He
e ce ca ae e a e a a 3 (CIN 3). W a e e be e
a a e e ?

A. C a dc c a 6
B. C a dc c a 12
C. HPV-ba ed e 12
D. L e ec ca e c ced e (LEEP)

W a a e f ce ca ca ce acc a e c a ac e e ce ca ca ce c b
e ade e e d f e a a?

A. S a eI
B. S a e II
C. S a e III
D. S a e IV

Ob e c a d G ec 423
Get The Step 2 CK Drills Book: usmledrills.com
The USMLE Guys Step 2 CK Crash Course usmleguys.com

Pediatrics

N a a Ca
APGAR Sc :
● Acti it ( e e a d ce e):
○ Ac e (2 ), f e ed a a d e (1 ), e (0 )
● Pulse ( ea a e):
○ >100 b (2 ), <100 b (1 ), e e (0 )
● Grimace ( ef e e e ab e ):
○ P e e c a c ,c , a a (2 ); e , e
fe fe e e (1 ); /f acc d (0 )
● Appearance ( c ):
○ P (2 ); b d ,b ee e e (1 ); c e e b e/ a e (0
)
● Respiration (b ea a e ):
○ Re a b ea (2 ), e a b ea (1 ), e (0 )
● Sc e 7-10 a e a , c e 6 d ca e d e

Ne b P a /Vacc a :
● He a B acc a :
○ W 24 f de e , ef d e f e a B acc e
● P a e e a K def c e c b eed :
○ I a c a d e f a K
● E e a :
○ W 2 f de e , ee c a c e e e
c cca a a e a

Ne b Sc ee :
● Sc ee f e ab c a d e e c d de :
○ B d e 24-48 f de e
● Sc ee f ea :
○ O ac ce a a ed a d ba e e e ed
c ee f ea , dea ef ed bef e a d c a eb ae
a 3 af e b
● Sc ee f c ca c e a ea d ea e:
○ P e e c ee
● Sc ee f c a ed eb b e a:
○ Mea e a e b b

Ped a c 424
Get The Step 2 CK Drills Book: usmledrills.com
The USMLE Guys Step 2 CK Crash Course usmleguys.com

Neonatal H perbilirubinemia

R Fac :
● Sb e e a , a d ce de e e a 24 f de e ,
e c d ea e (G6PD def c e c ), f ca e a a( ) f b a a,
e c e b ea feed >10% e , ea A a de ce , e a a a e <37
ee , a b b >95 e ce e

Ca e :
● E e a ed b b d c ( e c d ea e, b ea d fb d a
e a a/b e), b ea feed fa e ( e a a d c ea ed e e e a c
c c a fb b ), b c ( c ea e e e e a c c c a fb b )

Def :
● Be e aa eb b e a c a d ce f e e b
● Se e e e a a eb b e a def ed a a b b >25 /dL
● E e e e aa eb b e a def ed a a b b 30 /dL

C ca Ma fe a :
● Ja d ce f e face e e b d , c c a c e , ca e f e b b e a
(b / e a a, a f a e ad e e )
● B b - d ced e cd f c
○ Ac e b b e ce a a
■ Me a a e e f a ee e a c
c a e/ e e
■ C e e f a c ed ab e c
■ M ce e e e f a a a e a e a d
a a d e c
○ C cb b e ce a a ( e ce )
■ Se e a ea
■ L a ad a e
■ C ea, e ,a dd a
■ De a e a e a a

Ma a e e :
● T e f a a e e de e d e f de e e ee e aa
eb b e a (def ed a a b b >25 /dL) de e d - ec f c
b b a e a d e e ce f fac ( c a e a a a e)
● P ea ed a a ea b e e e e e e aa
eb b e aa d ea e e e eb b e a f e e ae a a c
a de e e ae ee
● E c a e a f e e ed f a c e ae de a e/ e e e
b b - d ced e cd f c

Ped a c 425
Get The Step 2 CK Drills Book: usmledrills.com
The USMLE Guys Step 2 CK Crash Course usmleguys.com

Neonatal Respirator Distress S ndrome

Ca ed b a ac f a fac a .

S /S :
● Tac ea, e c a a d bc a e ac , a a fa , dec ea ed b ea
d ,e a

I a :
● C e ad a d ff e e c a a d a f d b
a b c a a d e

Lab :
● H e a ABG, be a e a

Da :
● Re a d e af e b ( e e e ) c a ac e cc e
ad a f d

Pe e :
● Pe a a e <34 ee e a e a e aa c c e d f de e
a c a ed e e 1-7 da

T ea e :
● F e ab e e e a e (ba e a ,f ed b ba f
e e ) f a e c HR <100
○ O ce ab ed HR >100, fac a ca be e
● If e fa e a d e b ab e, CPAP ed
○ If CPAP a e a e be eac ed, ad e fac a

Transient Tach pnea of the Newborn

E ce fe a a e a f d c a e bee c ea ed f e bed f e ca
a ede a.

S /S :
● Tac ea, a d e e e ca e e a , a a fa , ec a/ bc a
e ac

I a :
● C e ad a d ff e a e c a f ae , e a c a a
e a a f e ,f d e e ba f e

Ped a c 426
Get The Step 2 CK Drills Book: usmledrills.com
The USMLE Guys Step 2 CK Crash Course usmleguys.com

Da :
● Re a d e af e b c a ac e cc e ad a f d ,
e 12-24

T ea e :
● S e e a e a d a a ca a
● S e ca e ( a c feed , e c ed e e )

Content Re ie Questions:

1- e af e b , a e b fe a e ed be a ca e e a d
e e . He ea a e 45 e a e a a d e a a
b b ee e e .W a a e 1- e APGAR c e?

A. 3
B. 4
C. 5
D. 6

A ae a e b a 38 ee e a a a e ac ec e af e b . C e
ad a d ff e a e c a f ae , e a c a a e a a f
e ,a df d e e ba f e .T e a e aced 30% e e a e
e f a d eed f e af e 12 .W a e e
da ?

A. Ne a a e a d e d e
B. B c
C. C cfb
D. T a e ac ea f e e b

A e b b e a f a eb .T ec c aa de eb d
a ea a d ced, a d e a e e e e e a ca dd a .T e
ae ae e a e e de e ed a e d c b a c a
ed. Lab a a b b f 39 /dL. Of e f , c e a ae
e e ?

A. IV a c c a d cef a e
B. P ea
C. E c a e a f
D. L e a a

Ped a c 427
Get The Step 2 CK Drills Book: usmledrills.com
The USMLE Guys Step 2 CK Crash Course usmleguys.com

C a P a
Bronchopulmonar D splasia

C c d ea e def ed a e e e e a a 36 ee e a
a e.

R Fac :
● P ee b (e ec a GA <28 ee ), a e e e c , ec a ca
e a , e c , fec / fa a , ae a

S /S :
● Tac ea, ec a a d bc a e ac , b e a e a d/ e a
ee e

I a :
● C e X- a : C a e e a a a d d ff e a e , e fa ,a d
fb cc a e c a c c a ea

Lab :
● ABG e aa d be e ca a

Da
● Need f e e e a a 36 ee e a a e (c f ed
e ed c e )

T ea e :
● I a e e e ca e f e aa d e ca a a e ea ed ec a ca
e a a d e e a e ,a da a d a e e e
a f a CPAP/ -f a a ca a e e a e , e
e e a e a e, a d f a a
● S e ca e ( a ,f d e c , f a
e e )

Retinopath of Prematurit

I c ee e a a c a a ee e ee fa ead a defec .

R Fac :
● Pe a , b e ,c bdc d ( a e c a e a e,
b c a d a a), fec , e ce a, eed f a e e
( ec a ca e a , a f )

S /S :
● P be a a e , e c a, a, a , ab

Ped a c 428
Get The Step 2 CK Drills Book: usmledrills.com
The USMLE Guys Step 2 CK Crash Course usmleguys.com

Sc ee :
● A aef a a e b e 1500 e a a a e 30 ee
e ee a ef ed b ed a c a

T ea e :
● La e c a a a -VEGF a e (be ac ab af be ce ) f e ee
ca e , c ed e a ce f e e e e d ea e

Germinal Matri Hemorrhage and Intraventricular Hemorrhage

R Fac :
● Ge a a a e <32 ee , b e <1500 , ed e c a ec a ca
e a f e a d e d e, c a , eed a fe a
e ca e fac

P ec e Fac :
● A e aa c c c d ea

S /S :
● Ma ca e a e a a a c, b c a e ed c c e ,
ab a e e a d e e , dec ea ed e e , a, e a
b ea a e
● I e e e ca e c a, e e , e a b ea /a ea, e d a c ab
( e , b ad ca d a), dece eb a e , f ed ,b a e
f a e e

I a /D a :
● Ca a a d e e ce f b d e e a a e ce
be e c a d aa .S e e b da a ed e ce eb a
ae c a
● Pee fa de 32 ee e a a a e d ece e e a c a a a d
a e f e e a e

Lab :
● CSF a a d be e f ed af e c a a a da e e f e a ed 3
a d a 4 e ce
● P e e ce f ed b d ce , e c ce a ,a d a c ac e
a e c a e a e

T ea e :
● S e ca e c d c e e a f e d a c , e a / e a ,
f d a ,a d
● T ea e f e e (f e e ) e e f e c da a e (f - e
e ba b a )

Ped a c 429
Get The Step 2 CK Drills Book: usmledrills.com
The USMLE Guys Step 2 CK Crash Course usmleguys.com

● P e a c e c a d aa c ea ed ac a a e e ea ed
e ca e e ( e c a acce de ce) e e CSF

Content Re ie Questions:

W c f ef ff c e a eada fb c a d a a a
a e e e ac ea, ec a a d bc a e ac ,a de a
ee e ?

A. C d c
B. Need f e e e a a 36 ee e a a e
C. A ca d abd e
D. PA c e ad a e " ee e

W c f ef f d CSF a a ae a ee e a a a d
a e c a e a e?

A. E e a ed c e
B. P e e ce f ed b d ce
C. E e a ed e c ce a
D. Xa c a

W c f ef a fac f e de e e f e a a a d
a e c a e a e?

A. C a
B. Ge a a a e <32 ee
C. B e <1500
D. A e a a c c c d ea

Ped a c 430
Get The Step 2 CK Drills Book: usmledrills.com
The USMLE Guys Step 2 CK Crash Course usmleguys.com

N aa G a a C a a I
Neonatal Gonococcal Infection of the E es

S /S :
● C c
○ P f e, e e da e e ee e f ee e d
○ T ca e e 2-5 da af e de e
○ C ca c de e e ec ea ead ce a ca
a d beb d e

Lab /D a :
● C e f ee da e ef N.

T ea e :
● S ed e (IV IM) f cef a e

Pe e :
● E c a c e e 2 fb

Disseminated Gonococcal Infection

S /S :
● A , ca ab ce , e e

Lab :
● E e a ed WBC, ESR, a d CRP; eb dc e
● If a e e a f da a e ed a ce WBC
c f >50,000, e a f dc e
● If e e e e e a ed CSF WBC c a d e , dec ea ed CSF c e,
e CSF c e

Da :
● S a f d, CSF b dc e ef N.

T ea e :
● IV IM cef a e

Chlam dial Infection of the E es

S /S :
● C c
○ W e d, ae d c a ea d d e f ee e d ee

Ped a c 431
Get The Step 2 CK Drills Book: usmledrills.com
The USMLE Guys Step 2 CK Crash Course usmleguys.com

○ W e e e e, e d e b a e f a e da e b d- ( b
b d- a ed), e e e e f e e e d , c e ed e e a
c c a, f a f a a e( c a )
○ S ca a e 5-14 da af e de e

Lab /D a :
● C c a ab e c e c ac d a f ca e (NAAT )

T ea e :
● O a e c a a c ( f de e c e )

Chlam dia Trachomatis Pneumonia

S /S :
● S acca c cc a , ac ea, e a, a ea, a a
c e , d fe e ( afeb e), a e a c a
● S de e 4-12 ee af e de e

Lab :
● N a WBC c e a

I a :
● C e ad a d ff eb aea a d e ca e a f ae a d
e fa f e

Da :
● C ca a d a f d c f ed a a ab ac ea a ae (f
ba ed) e c e c ac d a f ca e (NAAT )

T ea e :
● Oa e c a a c ( f de e c e )

Content Re ie Questions:

H af e de e d e c c d e c cca fec f ee e ca
e e ?

A. 2-5 da af e de e
B. 5-14 da af e de e
C. 4-12 ee af e de e
D. 3-6 af e de e

Ped a c 432
Get The Step 2 CK Drills Book: usmledrills.com
The USMLE Guys Step 2 CK Crash Course usmleguys.com

A 12-da - d c d de e a d, a e d c a e f ee e a e a a d e f
ee e d .Ac c a ab e e f C. ac a c e c ac d a f ca
e .T e a e ea ed a e c .W c f ef c d e
a e a a e e a ed f de e ?

A. J e e d a c a
B. Re e d e
C. Ve c e e a ef
D. P c e

A 8- ee - d fe a e b e a a c ef c a fc .F e a 4
da e a e a e e e ced a acca c cc a a d a a
c e a e a d, a e d c a e f ee e a d d e f ee e d .O
e a e a e ac e c a d a e a e ea d a f e d . C e ad a
d ff e b a e a a d e ca e a f ae a d e fa f e .W c f
ef e e da ?

A. A a
B. C a d a ac a e a
C. E
D. F e b d a a

Ped a c 433
Get The Step 2 CK Drills Book: usmledrills.com
The USMLE Guys Step 2 CK Crash Course usmleguys.com

C a Ma a
T ac e e a ea F a:
● T e A: E a ea a e a f a (8%)
● T e B: P a ac e e a ea f a d a e a ea a e a (1%)
● T e C: P a e a ea a e a d a ac e e a ea f a (84%)
● T e D: P a a d d a ac e e a ea f a e e e a ea
a e a (3%)
● T e E: N e a ea a e a b e e ce f a ac e e a ea f a (H e)
(4%)

C ca Ma fe a :
● P da (fe a ea a e a e e )
● Af e de e fa e e e ce ab feed, c , e a d e , a c
d e a da a e a

Da (T e C):
● T a a ca e e e ac 10-15 c a ed a d X- a c f
c ed ca e e e e e a ea c

T ea e :
● S ca a a f ee a a d ea e ac e e a ea f a
a
● Ec ca d a a d e a a a a e f e c e a defec

Congenital Diaphragmatic Hernia

Pe aa F d :
● U a d abd a a e e ec e a d da

S /S :
● Re a d e , ac f b ea d ( e e ce f b e d ) e
affec ed de, ca d abd e

Da :
● Made a ( a d e aa c e X- a b e ec e
ca e e a e)

T ea e :
● I ed a e e d ac ea ba a d ec a ca e a (e ac ea
e ba e e a - ECMO - f a e fa ec a ca e a ),
e d a c , NG be dec e , ca e a e ab ed

Ped a c 434
Get The Step 2 CK Drills Book: usmledrills.com
The USMLE Guys Step 2 CK Crash Course usmleguys.com

Gastroschisis

Pe aa F d :
● U a d abd a a defec e a f e e e (
ac/c e e b a e) ca e f e b ca c d e e
● E e a ed a e a e a a-fe e (MSAFP)
● O da da a be e e , e a a ead ec ,
, a a , a e a, e , ef a
● U c f a e a e e c e a a a c defec

S /S :
● B e ( c e e b a e) c f a ed/f b c/ a ed d
a e b ca abd a a defec

T ea e :
● Wa b e e e a ede ,f ed b a c a
● P ace OG be a d dec e ac
● P ace IV e f d a d a b c (a c a d e a c )
● S ca ace e f eb e e abd a ca a dc e f e abd a
a defec

Omphalocele

Pe aa F d :
● U a d abd a a defec e a f abd a c e (b e
+/- e ) c e ed b a 3- a e ed ac/ e b a e a d ca ed d e a e ba e f e
b ca c d c d e a e a e f e ac
● P da
● C f a e a e e c e a a a c defec
● H e a e f fe a a e d (e ec a e e e e a )

T ea e :
● Wa b e e e a ede ,f ed b a c a
● P ace OG be a d dec e ac
● P ace IV e f d a d b ad ec a b c
● S ca a ce e ed c , e e e ec e f da f e a e, a d
a c e

Ped a c 435
Get The Step 2 CK Drills Book: usmledrills.com
The USMLE Guys Step 2 CK Crash Course usmleguys.com

Content Re ie Questions:

W c f ef e f ac e e a ea f a c a d c a ac e ed
b a e a ea a e a d a ac e e a ea f a?

A. T e A
B. T e B
C. T e C
D. T e D

W c f ef c ca fea e ca ee a c ?

A. E e a ed a e a e a a-fe e
B. Pe b ca abd a a defec
C. Lac f a e b a e c e eb e
D. P e e ce f e c e a a a c defec

W c f ef c a ac e c aea c a ed a ce e?

A. He a f abd a c e c e ed b a e b a e
B. Pe b ca abd a a defec
C. He a ed abd a c e e e c de e e
D. N a b ca c d e

Ped a c 436
Get The Step 2 CK Drills Book: usmledrills.com
The USMLE Guys Step 2 CK Crash Course usmleguys.com

G D
C stic Fibrosis

Ge e c :
● A a ece ed de ca ed b CFTR e e a

Ne b Sc ee
● Se eac e e (IRT) a d DNA a a f a CFTR e e

Effec O a S e :
● P a :C cc c , ee ,d ea, b c ec a ,
e a/c a ( e d a , S a a e ), a a ,
● GI: Pa c ea c ff c e c , a c ea , ec e ,d a e a b c e
d e, c a
● GU: C e a b a e a ab e ce f e a defe e

Da :
● Sb da ed c cfb , a ea 1 a e c e
c cfb , e e b c ee e PLUS e e c e de ce f
a ed CFTR e e , e e a ed ea c de e ( 60 /L), a a e a
d ffe e ce ab a

T ea e :
● CFTR d a e a (fa ed f e a e d d a a )
● A a c ea a ce ed ca c d c e e a , DNa e, a ed
e c a e
● C ca c f f ec ef e d a f a a
c e e
● A b c ea e f a e ace ba
● Pa c ea c e e e ace e ea f a c ea c ff c e c

Fragile X S ndrome

Ge e c :
● X- ed d a d e ca ed b a ed FMR1 e e ca ed b e a f
CGG e ea

S /S :
● I e ec a d ab , ADHD- e , e e , aeb e e , ab ,
-a c ed a a e, a e ea , e f e ead a d c , a, e c a
e a e e

Ped a c 437
Get The Step 2 CK Drills Book: usmledrills.com
The USMLE Guys Step 2 CK Crash Course usmleguys.com

Prader-Willi S ndrome

Ge e c :
● Ab e ce f e e f e e ae a c e 15 11.2-13

S /S :
● Ne a a a b ead a a a d/ d ff c feed ead
fa e e
● C d e a d ad e ce de e e a a a d be e a
a a e e , ad , e e

Friedreich Ata ia

Ge e c :
● A a ece ed de ca ed b F N e e a e ce de
e ea (GAA)

S /S :
● Aa a fa b , ea e , e ,d a a, f dee e d
ef e e , e c ca d a , d abe e e , ea , b adde
d f c

Content Re ie Questions:

W c f ef e b c ee e ae ed c ee f c cfb
e b ?

A. Se eac e e (IRT) a d DNA a a


B. S ea c de e
C. Na a e a d ffe e ce ea e e
D. Se a a ea d a e e e

A 6- ea - d a e be ee a e c c f a a a e a .T e a e a a f
e aa a a ead feed d ff c a d fa e e. S ce c d d, e
a e a de e ed e a a bec be e. He a ffe f e e c
a fe a a e .W a e e cc d c e e e ?

A. F ed c a a a
B. F a e X d e
C. P ade -W d e
D. C cfb

Ped a c 438
Get The Step 2 CK Drills Book: usmledrills.com
The USMLE Guys Step 2 CK Crash Course usmleguys.com

W c f ef c d c a ac e ed b a ce de e ea ca d f c
e F N e e?

A. F ed c a a a
B. F a e X d e
C. P ade -W d e
D. C cfb

Ped a c 439
Get The Step 2 CK Drills Book: usmledrills.com
The USMLE Guys Step 2 CK Crash Course usmleguys.com

M ab D
P e e a:
● A a ece ed de e def c e e aa e d a e
● Ne b c ee ef ed ( a ec e )
● If d e a e c e e ed, e e b e e ec a d ab , e e ,
d e b ea e, ec e a, fa ,b ee e
● Da ed e e a ed e e aa e e e a d e e c e
● Ma a e e dea e c a d e aa ec a f d

Ne a -P c D ea e:
● A a ece e d ea e ca e a e def c e c e
acc a f e
● T ee e : A, B, C
○ T eA e eea d c A e a Je , dea a d
2-3 ea d
● S / c de c e ed e ac a, f ea ,
a ef e a, a, e ea e a , e a e e a
● T ea e e ca e

Ta -Sac D ea e:
● A a ece e d ea e ca ed b a def c e c be a- e a da e A e
acc a fG 2 a de
● M ec A e a Je dea a d 2-5 ea d
● S / c de c e ed e ac a, f ea ,
e ef e a, a, e a c b d e a d e e
● T ea e e ca e

K abbe D ea e:
● A a ece e a a ed de
● Ca ed b def c e c f a ac ce eb da e ead e acc a f
c e, da a ce a a d e ea e e
● C ca a fe a c de b a c ,a a a, ca ,
e ef e a a c e d e a ef e a e e a e a
e e
● Pa c de f e a d de d a, e e ce f b d ce
● T ea e e a e c e ce a a a

Fab D ea e:
● X- ed ece e a a ed de
● Ca ed b defec e a d a e a a- a ac da e A (a a-Ga A),
e acc a f b a ce a de (Gb3)
● C ca a fe a c de e a c b a , d e d ea e ( e a),
e a ec a a , a ea a , GI d ba ce, ca a ac , ef e c a e ,
e

Ped a c 440
Get The Step 2 CK Drills Book: usmledrills.com
The USMLE Guys Step 2 CK Crash Course usmleguys.com

● T ea e e e e ace e ea aa a, c c ea e
e e ac ce a a a f Fab d ea e

Ga c e D ea e:
● A a ece e a a e d ea e
● Ca ed b def c e c f c ce eb da e e acc a f
c ce eb de
● C ca a fe a c de e a e e a , b c e a, e e a,
b e a , e c
● O a Ga c e ce e e ( ac a e f ed d c e
c ed e a e)
● T ea e e e- e ace e ea

Me ac a c Le d :
● A a ece e a a e d ea e
● Ca ed b def c e c f a fa a e A e e acc a f ce eb de
fa e
● C ca a fe a c de e e f , a a a, ab a a ,
e ea e a , a, ca
● T ea e e ca e

H e S d e:
● A a ece e a a ed de
● Ca ed b def c e c f a a-L- d da e e acc a f
c a ca ( e a a fa e a d de a a fa e)
● C ca a fe a c de c ea c d e e a ab a e (d
e ), c a e fac a fea e , e e e e ec a d ab , e a e e a
● L fe e ec a c 5-10 ea
● T ea e ec b a a a-L- d da e

H e S d e:
● X- ed a a ed de
● Ca ed b def c e c f d a e 2- fa a e e acc a f
c a ca ( e a a fa e a d de a a fa e)
● L fe e ec a c 13-20
● S a f d a H e d e c ea c d a d ea a a
e e ca a, e a ,a d
● T ea e ec b a d ae fa a e

H c a:
● A a ece ed de
● Ca ed b def c e c f c a e be a- a e e e e a ed c e e
e e

Ped a c 441
Get The Step 2 CK Drills Book: usmledrills.com
The USMLE Guys Step 2 CK Crash Course usmleguys.com

● C ca a fe a c de Ma fa d a ea a ce, de e e a de a ,
b e b , e e e e a ea e ce , e , ec a e
● T ea e c de a B6, B12, a d f a e e e a a d
a a e e /a c a a f d ca ed

Le c -N a S d e:
● X- ed ece ed de
● Ca ed b def c e c f a e- a e b a fe a e (HPRT)
e a c ea ed d c f c ac d
● C ca a fe a c de e f- a be a , de a ed de e e a
e e , e ec a d ab , d a, , a ed d e f c
● T ea e a , ca b d a/ e d a, d a e a , a e d

G c e S a e D ea e I ( G e e D ea e):
● A a ece ed de
● Ca ed b def c e c f c e-6- ae d a e e e e
c e acc a
● C ca a fe a c de fa e e, ce a, e e (f
ce a), e a e a , ac c ac d , e ce de a, e ce a
● T ea e c de a a a ae c e e e f e e feed

G c e S a e D ea e II (P e D ea e):
● A a ece ed de
● Ca ed b def c e c f a ac d a a- c da e e acc a f
c e e a dc a
● C ca a fe a c de e e e a (f bab ) a d ca d a
(b e c a e ) ead ea fa e
● T ea e e e e ace e ea

G c e S a e D ea e III (C D ea e):
● A a ece ed de
● Ca ed b def c e c f c e deb a c e
● C ca a fe a c de ce a, e a e a , e e a ed e
a a a e , e ac d , e de a, e e a ce a , e ,
ea fa e
● T ea e c de a a a ae c e e e feed

G c e S a e D ea e IV (A de e D ea e):
● A a ece ed de
● Ca ed b def c e c f c e ba c e e e ab a c ed
c e
● C ca a fe a c de e a e e a a a de e a c ,
ce a, a d e a e fa e
● T ea e c de e a a a

Ped a c 442
Get The Step 2 CK Drills Book: usmledrills.com
The USMLE Guys Step 2 CK Crash Course usmleguys.com

Content Re ie Questions:

W c f ef c d c a ac e ed b a c e ed e ac a, f
ea , a ef e a, a, e ea e a ,a d e a e e a
e f a acc a f e ?

A. Ga c e d ea e
B. Le c -N a d e
C. Ta -Sac d ea e
D. N e a -P c d ea e

Le c -N a d e c a ac e ca a c a ed c f ef e ab c
d ba ce ?

A. H e ce a
B. H e ca ce a
C. H e ae a
D. H ab e a

W c e ab c d de a e e c ea c d ,d e , e ee
e ec a d ab , a d e a e e a ?

A. H e d e
B. H e d e
C. Me ac a c e d
D. P e e a

Ped a c 443
Get The Step 2 CK Drills Book: usmledrills.com
The USMLE Guys Step 2 CK Crash Course usmleguys.com

I
C Va ab e I def c e c (CVID):
● B ce e e b ab e d ffe e a e a a ce
● Red ced I G a d ed ced I M a d/ I A e e ab e e e a
● M a e de e CVID af e be , 20 -30 f ec e ,
a , a d GI fec
● Pa e a e e a ed fH d a d -H d a
● T ea e IVIG

I A Def c e c :
● M c def c e c
● M a e a a c, e e e ec e , a , a d GI
fec
● A a ac c eac ca cc b d d c
● A c a ed ce ac d ea e, IBD, e 1 d abe e , SLE, G a e , e a d a
● Lab / de ec ab e I A, a I Ga dI M e e
● P ac c a b c f ec e fec
○ I b e ace e ea ( I A) f a b c a e fa
● D b d d c d a eI A e ed

X-L ed A a a b e a (B -T e A a a b e a):
● X- ed d de ca ed b a e BTK e e eeded f e de e e fB
ce
● Red ced B c e , ac f a a ce , a d b ca c ea ed
e ca a ed bac e a a d b db e e e fec
● Rec e fec de e af e 3 ( c ca a e a be
S . ,S . , H. e B, a d P ),
e ec a ec e ed a, e a, a d
● Pa e a e ab e ce f a d ade d
● L /ab e I M, I G, a d I A e e a d /ab e a b d e a
● Af e b e e c f ed, f c e ef ed ab e
CD19+ B ce eb da d a BTK e e c f da
● T ea e IVIG a d bea c a

Se e e C b ed I def c e c (SCID):
● N a / c ea ed I M def c e c e I G, I A, I E c d e CD40L
def c e c
● Rec e a d a fec
● N a b d e e a e
● N a a B ce , e B ce
● T ea IVIG a d a e ec e a e c ce a a a

A a a-Te a ec a a:
● ATM e e a e a ed DNA e a ec a

Ped a c 444
Get The Step 2 CK Drills Book: usmledrills.com
The USMLE Guys Step 2 CK Crash Course usmleguys.com

● Ce ebe a a a a, d ff c f de e e e , e a ec a a f e c c a
a d face, a fb
● Rec e a d a fec a d b c ec a d e fec
e ca a ed a ( e c cc )
● I c ea ed f a a d e e a
● Lab c de e e a ed a a-fe e , b , a d T ce def c e c e
● A b c a a d c e c ea a ce f ec e a fec , IVIG f
fa e f e e ea e

W -A d c S d e:
● X- ed e ed a d de
● Rec e fec d e ada ea d ae e def c e c ( , a , GI
fec ,a d e )
● T b c e a ca b eed
● Ec e a ( e e e fec ), a ed de ( a c , IBD), a d
ca ce ( aa d e e a)
● Lab T ce e e , / a I G/I M e e , a d I A/I E e e
● T ea ed ac c a b c , IVIG, ad a ed (f CMV) a e e ea
b c e a, a e ea ( ab), a d def e ea e
e a e c ce a a a

D Ge eS d e:
● Ca d ac defec (ASD VSD , c a e , e a f Fa , a c a )
● C a fac a defec ( a a eb a f e , e e , ea ab a e ,
b ad a a b d e)
● H a c/a a c ed ced CD3+ T ce a d ec e , a
a d GI fec
● C ef a a e
● Pa a d a a( ca ce a)
● 22 11.2 de e
● T ea c a a e a e c ce a a , c ec f ca ce a,
ca e a f ca d ac defec a d c ef a a e

C e e S e Def c e c e :
● C1, C2, a d C4 def c e c a c a ed a ed de ( e c
e e a ) a d ec e e c bac e a fec e ca a ed bac e a
(H. e B, N. ,S . )
● C3 def c e c a c a ed ec e S . fec
● C5-C9 def c e c a c a ed ec e Ne e a fec
● T a e cc e e (THC) a a ed c ee f c e e def c e c ,
/ de ec ab e CH50 a e e e c eec e e def c e c
● A a e d ece e e c cca a d H. e B acc a a d be
c e ed ee ca e af e ea f fec

Ped a c 445
Get The Step 2 CK Drills Book: usmledrills.com
The USMLE Guys Step 2 CK Crash Course usmleguys.com

Content Re ie Questions:

A e b a e de f ed a a e e a c a fac a defec c d e e ,
ea ab a e , a d a b ad a a b d e. U f e - e a f Fa e e
a e a c ef a a e a d ca ce a. W c f e f f d c a ac e ca
a ee e a e e e c e ff d a c a ed a e ab e
d de ?

A. T 18
B. D de a a e a
C. Ab e / a c
D. W

W c f ef def c e c e a c a ed a a ac c eac d
b d d c ?

A. I A def c e c
B. X- ed a a a b e a
C. C a ab e def c e c
D. Se e e c b ed def c e c

W c f e ec ca a fe a ca a c a ed W -A d c d e?

A. H ca ce a
B. T b c e a
C. Ec e a
D. Rec e fec

Ped a c 446
Get The Step 2 CK Drills Book: usmledrills.com
The USMLE Guys Step 2 CK Crash Course usmleguys.com

C a H a D a
Atrial Septal Defect

S /S :
● S a defec a a c
● La e defec ca e ea fa e, e a e a , fa e e, a ad ca
e b , ec e e a fec

A c a ed C d :
● H -O a d e, d d e

Ca d ac E a :
● Md c e ec ea d be e e ec d ec a ace, f ed de
S2

I a /D a :
● Ec ca d a (d a c) ca de f e, ca ,a de ae e f
ASD
● ECG incomplete right bundle branch block in V1, ed PR e a
(e ec a c ea a e), a d c c e a e e fe ead ( c a eR
a e ea )
● C e X- a c ea ed a a c a , ca d e a ( a a a d
e c a d a ) f ASD a e

T ea e :
● De e d ea d e f defec , a e c e a cc
● I d ca f ca c e c de a e ef - - , ea
e a e e , a ec c a
● S ca c e ca c de e c a e c e ca d a b a a d
ace e f e ca d a Dac ac

Ventricular Septal Defect

S /S :
● S a defec ca be a a c; de a e- - a e defec ca ca e ea fa e,
e a e a , a a e , ac ea, ac ca d a, fa e e

A c a ed C d :
● Fe a a c d e, d e (13, 18, a d 21)

Ca d ac E a :
● Pa ab e fe a 3 4 ef ec a ace a e ef e a b de
● B c ea d be a e ef e a b de
● Da c b e ea d a e a e ( de a e- a e defec )

Ped a c 447
Get The Step 2 CK Drills Book: usmledrills.com
The USMLE Guys Step 2 CK Crash Course usmleguys.com

● L d S2 a e ef e e a b de ( a e defec )

I a /D a :
● Ec ca d a (d a c) ca de f e, ca ,a de ae e f
VSD
● ECG c ea ed a e(a R a e ) ead V5 a d V6 ( ef e c a
e ) ead II, III, a d aVF
● C e X- a c ea ed a a c a a a d e a ed ef a
a d e ce

T ea e :
● S a ,a a c VSD ca be a a ed c ef -
● M de a e- - a e defec a a ed ed ca f e de, ac e, a d
a
● Pa e fa ed ca a a e e a e e (e e a ed
a a e e e) a e ef - - ca ef e c a d a
e e ca c e a ac

Patent Ductus Arteriosus

Defec :
● T ec ec be ee e a a e a d ea a (d c a e ) fa
c e

R Fac :
● P e a ;b a a de; fe a e e ; c e a be a; c d c a ; H -O a ,
D Ge e, D d e

S /S :
● La e PDA ca ca e ea fa e, e a d e , fa e e, a d f
c ec ed c a

Ca d ac E a :
● C ac e- e be ea d e ef f ac a c a a ea, de e
e e, b d e , d S2

I a /D a :
● Ec ca d a (d a c) ed c a e c ec e a
a e a d ea aa de ae ea f ef - - a d e a
a e e e
● ECG b- e c a e
● C e X- a a e PDA a c ea ed a a c a a a d
e a ed ef e c e a d a

Ped a c 448
Get The Step 2 CK Drills Book: usmledrills.com
The USMLE Guys Step 2 CK Crash Course usmleguys.com

T ea e :
● P ee fa a e a a ed e ef e ca e, e e COX
b ( d e ac ) f a ac ca c e, e e c a e a ca e e
cc ,a d fa e fa ca a
● I e fa d ca f PDA c e( ec a e ca e e cc ca
a ) c de: P e d ca d , a d b e PDA, ef a a e c a e a e e ,
ef - - , a a e a e e ( d- de a e)

Coarctation of the Aorta

Defec :
● Na f e de ce d a a, a ca ed a e e f ed c
a e

A c a ed C d :
● T e d e

S /S :
● I e e a e: Hea fa e, ca d e c c ,d ea, e a e a , ab e fe a
e a cc e e PDA c e
● I de fa /c d e : C a d ca e e c e, e cb d e e e
e c a ed ea ( e e ea , e e e ), a ab e
de a f e ad a a e e fe a a e e

C ca :
● I ac a a a ca e ,a cd ec , a ba ac d e a e

I a :
C e X- a b c f e e b a d a 3-
Ec ca d a a e e f e de ce d a aa d
ab a f ac ec aca a d e c a e a e e ca be ee c
f a d ed- a e D e

Da :
● S c e e e e e e e , / ea ab e b d e e e
e e e e , a d b ac a -fe a de a c a ac e c ec ca d a

T ea e :
● I e e d ca ed f PDA e ed f a e a e, f e cb d
e e ad e >20 H , f e e e de ce fc aea f a ,
e e ce f e e ea fa e d e c a ca f ea a
● I e e c de e e e ba a a +/- e ace e

Ped a c 449
Get The Step 2 CK Drills Book: usmledrills.com
The USMLE Guys Step 2 CK Crash Course usmleguys.com

Tetralog of Fallot

A a c Defec :
● Ve c a e a defec
● O e d a a
● R e c a f ac b c
● R e c a e

A c a ed S d e :
● D d e a d D Ge e d e

S /S :
● Pa e b e ee e c a f ac b c :I ed a e f d
c a e ed a e e
● Pa e b d de a e RVOT b c :I a a a c, e e
e cc d e a e cc f e RVOT ca c a ( e d f
e c e e e a ca e a e e ed RVOT b c )
○ Te e a fe a c a , a d a d dee e a ,a d ab
fa
○ O de c d e e e e c a e e f e ec e be ec a c
a

Ca d ac E a :
● P e a ab e RV e
● Ea c c c ea d a e ef e a b de c, a ,
c e ce d -dec e ce d e ad a

I a :
● Ec ca d a ca de f e c a e a defec , e e f RVOT b c ,
b d e e (a cac ,c a a e )a a
● ECG ca de f f a a e a e e ( e P a e V1) a d
e c a e ( a de a , a R a e V1, a d dee S a e V5
V6)
● C e X- a "b - a ed" ea ed a e

Da :
● Made c a ac e cf d ec ca d a

T ea e :
● Ne a e e e e RVOT b c eed a a d E1 e a (a ad )
ee d c a e ae e d ca e a
● M d- de a e RVOT b c ea ed ca e a a 3-6 (c e VSD,
e e e b c b e a RVOT)

Ped a c 450
Get The Step 2 CK Drills Book: usmledrills.com
The USMLE Guys Step 2 CK Crash Course usmleguys.com

Ebstein Anomal

R Fac :
● L e e e a

A a c Defec :
● T c d a e d ace e f e a d e defec e eaf e
ead e a
● A a ed f e e ce( c ac , - a ed, d a ed)
● A c a ed defec c de a a e a defec (ca ca e - - ef ,
a ad ca e b )

S /S :
● C a ,d ea, ea fa e, a e d e , e a e a

Ca d ac E a :
● S 1 a d2 ea d , ea cc c , c a c ea e
e a

I a /D a :
● Ec ca d a a ac e f e e a c d a e eaf e d aced
ad e a e (d a c), c d eaf e defec , c d e a , a a ed
f e e c e (e a ed a d d a ed)
● ECG be b deb a c b c , e P a e V1 c e
a a e a e e
● C e X- a ca d e a

T ea e :
● Ne a e e eec a eed a a d E1 ea (a ad ) ee
d c a e ae e d ca e a
● C d e ea fa e ea ed ed ca e, d ,a df e de
● S e c de b e c a e a

Transposition of the Great Arteries/Vessels

A a c Defec :
● A ac ec e e cea d e a a e c ec e ef
e ce
● S a de e d ( e c a a a e a defec ae d c
a e )

S /S :
● C a , ac ca d a

Ped a c 451
Get The Step 2 CK Drills Book: usmledrills.com
The USMLE Guys Step 2 CK Crash Course usmleguys.com

I a /D a :
● Ec ca d a (d a c) a ac ec ed e e cea d e
a a e c ec ed e ef e c e a d de f e ee
cc (VSD, ASD, a e d c a e )
● C e X- a a "e a a ea a ce f e ea

T ea e :
● P a a d E1 e a (a ad ) ee d c a e ae e d ca
e a
● If e e e e a e e , a e e ba a a e def e
ca e a
● A e a c ea a ca e e ea e e ea ae

Content Re ie Questions:

W c f ef a ac c ea e ae ed c ea ae d c
a e ?

A. P a a d E1 (a ad )
B. I d e ac
C. T e e
D. M e

W c f ef c e ad a c e c aca f ea a?

A. R b c
B. B - a ed ea ed a e
C. E - -a- a ea a ce f e ea
D. T ac ea d ace e

W c f ef a da e /d a c e ef a d c
e ed de a def eda ca e fc e a ea d ea e ee
e a e / fa ?

A. ECG
B. C e ad a
C. Ec ca d a
D. CT f e c e

Ped a c 452
Get The Step 2 CK Drills Book: usmledrills.com
The USMLE Guys Step 2 CK Crash Course usmleguys.com

W c f e ef d c a ac e ca ee e a f Fa ?

A. Ve c a e a defec
B. O e d a a
C. A a ed e ce
D. R e c a f ac b c

W c f ef d / ed ca e ed d e a c c a ac e ca
a c a ed e de e e f Eb e a a e fe ?

A. Wa fa
B. Ca ba a e e
C. C ca e
D. L

W c f ef ed ca ca be ed ee ed c a e ae e d
ca e a a a e a f e ea a e e / e e ?

A. P a a d E1 ea (a ad )
B. I d e ac
C. L
D. S ac e

Ped a c 453
Get The Step 2 CK Drills Book: usmledrills.com
The USMLE Guys Step 2 CK Crash Course usmleguys.com

C a A G a D

Age Gross Motor Skill

2 A e ead ead e ed
3 Ab e de a ed, e e e f ea e e
4 S e ed, ab e f f f bac a ed
5 Ab e f f bac f a ed, ab e a

6 S a ed
8 C a d ca
10 Sa d e a d eda d a a d ed
1 ea Ab e a e de e de e ( a )
18 Ca a ba , ea e e e ,a d
2 ea K c a ba e d, ca a d e e d e a ac b
fee eac e
3-3.5 ea Ca de a c c e, ca ba a ce 1f f 3 ec d , ca a
a e a fee e a f
4-5 ea Ca ba a ce 1f f 4 ec d , ca ca c a b ced ba
5-6 ea Ca a d a d e a a e a e fee eac e
6-7 ea Ab e
7-8 ea R de b e, ab e d a e a

Age Fine Motor Skills

2 H d a e f aced a d, d a d e e
3 W ac b ec
4 H d a d e f e e, eac e e e b a d
5 Ta fe b ec f a d a d
6 Reac e e a d
8 S e c / b ec af e a be
10 Ga e de f de f e a d b
1 ea H d aca a d ca c bb e af e be ,a e ac 2 c be

Ped a c 454
Get The Step 2 CK Drills Book: usmledrills.com
The USMLE Guys Step 2 CK Crash Course usmleguys.com

18 Ca ac 4 c be
2 ea Ca aea a e
3-3.5 ea C e a c c e a d ca c c ( fce )
4-5 ea C e a ae
5-6 ea C e a e, e f a e
6-7 ea Ca e e
7-8 ea N e e e e a a e , e e ae f ea d e e e

Age Language Skills

2 Ae d ,c ,d a ca e
3 Ma e d e e , a ea e
4 M e ead d ec fa ce, a d
5 Sa e d a e, a e ea d
6 S e a e d
8 L f fa e be e a ed, ae d
10 Wa e db e bac , e ee -a-b
1 ea F e- e c a d , a de ed b ec
18 Ide fe ( ) fa a e e e a ed, e 10 25 d
2 ea F 2 e c a d , e 2- d e e ce
3-3.5 ea U e 3- d e e ce , e be f e e
4-5 ea F 3- e c a d , e e ,a d 100% e be
5-6 ea 2000- d cab a , e be , de a d a d ef ef
6-7 ea K da f e ee , a e ea f fa a d
7-8 ea U de a d a d ef e , e e

Ped a c 455
Get The Step 2 CK Drills Book: usmledrills.com
The USMLE Guys Step 2 CK Crash Course usmleguys.com

Age Cogniti e/Emotional Skills

2 Rec e a ca e a e , ec ca
3 L a d ac e e f ,e e e d ea e, a
4 M b ec , ec e ae ' ce ( c )
5 T ead f d ed b ec c a e e, f
a ac e ca e e
6 De ae a e a e , a e a d e a a f / b ec
8 L f b ec a fa e ef , f a e f ad a
e ( a e )
10 U c e dde de c , de a e fea
1 ea Lf b d e
18 Ca ac a f b ec , a ee d e
2 ea Ab e e fa a b ec , aa e a
3-3.5 ea K a e, ae e
4-5 ea C 4, a c a e a , c e
5-6 ea Ha a f f e d , ead a d 25 d
6-7 ea Ca ef e add / b ac a d ead a d 250 d
7-8 ea Ca ead de e de , a a e a

Measuring Growth

Le /He :
● Rec be e ea ed c d e <2 ea da d a d e c de 2
a d de
● U ed de ec ab a e
○ Pa e 2 ea a d de d a ea 4 c / ea
○ D e ea be a a e 5 10 c / ea

We :
● N a e be ee 5% a d 95% a da d ed c a
● Ne b ca e 5-10% f e e af e b e b e b
da 14

Head C c fe e ce:
● Ma da ee ea ed f b 3 ea a dc a ed a a da d ed
c a

Ped a c 456
Get The Step 2 CK Drills Book: usmledrills.com
The USMLE Guys Step 2 CK Crash Course usmleguys.com

Ta e S a e - Ma e a d Fe a e P b c Ha :
Stage Pubic Hair Pattern

Sa e1 N a e a
Sa e2 A ba e f e a ab a a e bc a c a c ed a d
Sa e3 P bc a ead e b b e a a e, a da e a d ec ed
a a e2
Sa e4 Ad bc a a ea a ce ead ed a face f
Sa e5 Ad bc a a a e b de

Ta e S a e - Ma e Ge a a:
Stage Male Genitalia

Sa e1 N ca f be
Sa e2 G f e e a d c c a e c a c a d e e
Sa e3 G f e (e ), c ed f e e
Sa e4 De e e f e a b ead , c ed f e e
da e f c a
Sa e5 Ad e a a (e d f e a a a )

Ta e S a e - Fe a e B ea :
Stage Female Breast

Sa e1 N ca f fe a e b ea de e e
Sa e2 B ea b dd (e e a f e b ea a d a ae), e a e e fae a
Sa e3 C ed e a e e f a e a a d b ea e aa f e c
Sa e4 Sec da d f ed f e a e ae a d a ae de e
Sa e5 Ad fe a e b ea a ae e d ab e e b ea c d e e
ece f e a e a.

Ped a c 457
Get The Step 2 CK Drills Book: usmledrills.com
The USMLE Guys Step 2 CK Crash Course usmleguys.com

Content Re ie Questions:

W a e e a e a e de e e a e e f a ed d be
e ec ed be eac ed a c d a ae ee e de e e a e e ?

A. 2
B. 4
C. 6
D. 8

W a Ta e a e a ed b ad bc a a ea a ce ead e ed a
face f ?

A. Ta e a e2
B. Ta e a e3
C. Ta e a e4
D. Ta e a e5

I fe a e a e a Ta e a e a ed b e e e ce f b ea b d (e e a f e
b ea a d a ae) a d e a e e f e a e a?

A. Ta e a e1
B. Ta e a e2
C. Ta e a e3
D. Ta e a e4

Ped a c 458
Get The Step 2 CK Drills Book: usmledrills.com
The USMLE Guys Step 2 CK Crash Course usmleguys.com

P a P a Ca
Vaccination Schedule

He a B Vacc a :
● 3 a d e e a b ,1 2 , a d 6 18

R a Vacc a :
● E e 2-d e e e a 2 a d 4 OR 3-d e e e a 2, 4, a d 6

D e a, Te a , a d Pe (DTaP) Vacc a :
● 5 a d e a 2 ,4 ,6 , 15 18 , a d 4 6 ea

H T e B Vacc a :
● E e 3-d e e e a 2 ,4 , 12 15 OR 4-d e e e a 2
,4 ,6 , 12 15

P e c cca Vacc a :
● 4 a d e a 2 ,4 ,6 , 12 15

P Vacc a :
● 4-d e e e a a e 2 ,4 , 6 18 , 4 6 ea

I f e a Vacc a :
● 2 d e , e a a ed b 4 ee f c d e 6 8 ea a e ece ed fe e
a 2 f e a acc e d e . O 1d e eeded f e c da e6 8 ea
a e ece ed 2 f e a acc e d e f e f ea ( )
● 1d ef a a e 9 de , a a

Mea e , M , a d R be a Vacc a :
● 2 a d e a 12 15 a d 4 6 ea

Va ce a Vacc a :
● 2 a d e a 12 15 a d 4 6 ea

He a A Vacc a :
● 2 a d e a a e 12 de , a d 2 d e6 af e ef

Te a ,D e a, a d Pe (Tda ) Vacc a (b e ):
● 1 d e a e 11-12 ea

H a Pa a Vacc a :
● If a e 9 14 ea : 2-d e e e a a e 9-14 a d 6 12 ae
● If a e 15 de : 3-d e e e a d e, 2 d e 1-2 ae,a d3 d e
5-6 af e 1 d e

Ped a c 459
Get The Step 2 CK Drills Book: usmledrills.com
The USMLE Guys Step 2 CK Crash Course usmleguys.com

Me c cca Se A, C, W, Y Vacc a :
● 2 d e a a 11 12 ea , 16 ea

Me c cca Se B Vacc a :
● 2-d e e e a ea 1 a a , a e 16 23 ea

P e c cca Vacc a :
● 4 a d e a 2 ,4 ,6 , 12 15

Preventative Screening

Hea R A e e :
● A ea a e a ce a d e d ca be ee a e 4 a d 21 ea a e
e a d c ee de ce ( e ea d e , ac ce ,
a e )

V R A e e :
● Sc ee de ec a ac defec , a b a, ab c d e <5 ea
a d a e e a a ea a e a ce
● C d e 5 a d de a e e a a ea a e a ce ,a d a
ac ea e e a a e 5, 6, 8, 10, 12, a d 15 ea

De e e a -Be a a Sc ee :
● 9-, 18-, 24- a d 30-

Sc ee f A S ec D de :
● 18- a d 24-

Sc ee f I Def c e c A e a
● Pe f ed a 9 12
● T e fac ( a /d e , be , e a , b
e ) f def c e c a e e e f e e ab c ee

B d Lead Le e Te :
● 1 2- ea - d a f ef e e a fac : L e ce a ea
25% e f b bef e 1960, 5% e c d e 1-2 ea d b d
ead e e f 5 c /dL e, da a c d ce a ea b d ead e e

A a Sc ee f Ac ,D , a d T bacc U e:
● A e 11 ea a d e

A a De e Sc ee :
● A e 12 ea a d e

Ped a c 460
Get The Step 2 CK Drills Book: usmledrills.com
The USMLE Guys Step 2 CK Crash Course usmleguys.com

Content Re ie Questions:

W a e ea e a e a ea c d ec e ded ece e e f e a acc e?

A. 1
B. 6
C. 1 ea
D. 6 ea

T ed e a, e a ,a d e (DTaP) acc a dea e a a e a ?

A. 2-d e e e a 11 12 ea , 16 ea
B. 4-d e e e a a e 2 ,4 , 6 18 , 4 6 ea
C. 3-d e e e a 2, 4, a d 6
D. 5-d e e e a 2 ,4 ,6 , 15 18 , a d 4 6 ea

T e ea e , ,a d be a acc a dea e a a e a ?

A. 2-d e e e a 12 15 a d 4 6 ea
B. 2-d e e e a ea 1 a a , a e 16 23 ea
C. 1 d e a e 11-12 ea
D. 4 a d e a 2 ,4 ,6 , 12 15

Ped a c 461
Get The Step 2 CK Drills Book: usmledrills.com
The USMLE Guys Step 2 CK Crash Course usmleguys.com

P a GI D a
P loric Stenosis

R Fac :
● Ma e e , f b c d, e e a , fa , ac de e( fa
e)

S /S :
● N b ec e , a ab e e- e a e RUQ

Lab :
● H c e c ae c e ab c a a
● E e a ed BUN a d c ea e (de e d da a )

I a :
● U a d f - e "a e" a e e e a d ab a a e
c ceda ee, c e ,a d e

Da :
● C f ed e e c ca a a f e- e a c a ac e c
a df d

T ea e :
● C ec e ec e ab a a df d def c
● P

Intussusception

R Fac :
● P ca ead , GI fec

S /S :
● Se e e c a abd a a c a e e a d e e e
a d bec e e f e e , a ab e a a e- a ed a e de f e
abd e ( d- e ad a ), b d / c (c a e ) cc b d
, b e b , e a

I a :
● U a d f - e "a e" e e e
● Abd e X- a e f ed a e f ef a ( e e e )a d a
ec f c f d e d e ded f b e , c e ce a ed f e de
ec a f e a eb e, a e ade f ad ce c c e

Ped a c 462
Get The Step 2 CK Drills Book: usmledrills.com
The USMLE Guys Step 2 CK Crash Course usmleguys.com

Da :
● V a a a d

T ea e :
● If f ef a , ef - ea e ed c
● If f ef a , ac e , fa ed e a e ed c a a ed ca
a a c c ed c

Intestinal Malrotation with Volvulus

S /S :
● B , abd a a , abd a d e ,b d , d abd e ,
e d a c ab

A c a ed W :
● C e a da a a c e a, Mec e d e c , ea defec , a e a ( e e ,
b a ,e a )

I a :
● Abd a X- a f ee a e ea a f e f a
● U e GI c a d aced d de c ce a ea a ce
● W " e f e e e e c e e ee a da b e
ae a d e e e

Da /T ea e :
● F d e c a
● E ca b c c ea e fb e f a
● E e e e a e e a a d Ladd ced e ed ce e d f
f e

Meckel's Diverticulum

T ed e c f ed e e e d c fa b e a e.

S /S :
● U a a a c
● Pa e GI b eed c ca a ef d ee a a e e a c e a

I a :
● Mec e ' ca e f ed e d a ca ab e a e c ea ed c e
(99 ec e e ec e a e) de f ec c a c c a
● C a e e e ca e a ed ca e f b GI b eed e
a f b de f aba c f e e e e e c a e feed e
d e c

Ped a c 463
Get The Step 2 CK Drills Book: usmledrills.com
The USMLE Guys Step 2 CK Crash Course usmleguys.com

C ca :
● I ce , ,d e c , f ed e c

T ea e :
● A a c Mec e ' d e c de f ed a :N ea e e ed
● A a c Mec e ' d e c de f ed c de d ca abd a
e a : Re ec e d e c
● S a c Mec e ' d e c a a ed acc d a ca a e
a e ec f ed e c

Congenital Aganglionic Megacolon

S /S :
● Fa e a ec ,b , abd a d e ,d a ec a e a
ca f cef e f , a a ce

A c a ed C d :
● D d e

A c a ed C e a Defec :
● Ref ac e a (a a , a), ea , d e , a ac , a d ea defec

Da c Te :
● Da c e ec a b ab e a ce
● C a e e a d a ed c a ea a c e e c
be a a
● A ec a a e ac f e a a f e e a a a ce

Da :
● Lac f a a ec a b

T ea e :
● S ca c ec c ea a a a be ee e ea b e a dd a
ec c e ea e e e ce f c

Imperforate Anus

S /S :
● Fa e a ec , abd a d e , e ea , a
f e a a e a(ff a e e )

I a :
● Abd a ad a ed de f e f a e a a d ca f ec ac
● CT c a ca de f e e ce f f a ( a a, a , e ea )

Ped a c 464
Get The Step 2 CK Drills Book: usmledrills.com
The USMLE Guys Step 2 CK Crash Course usmleguys.com

T ea e :
● A a c e fa f a, f a fa a a e , a d c ea a
a a be ee e ec a c a d e e f ed a a e

Duodenal Atresia

S /S :
● P be da e ,b e e af e f feed , b e fa e
a ec

A c a ed C d :
● D d e, VACTERL, ea defec

I a :
● U a dd e 2 /3 e e d b e b bb e f d a ed f d-f ed
ac d ec ad ace d a ed e e
● Af e b abd a X- a ef ed c f b c , c ec f
e e e ,a de a e ca . D b e b bb e a ee e e
ab e d a a
● If f e e e abd a X- a , a d a d b e b bb e
d a a e e , e a e a c a d ef ed e
a a

Da /T ea e :
● S ca a a a dc ec

Jejunal and/or Ileal Atresia

S /S :
● V a d abd a d e 2 da fb , b e fa e a
ec , be da e

I a :
● Of e de ec ed 2 /3 e e a d
● Af e b abd a X- a ef ed c f b c , c ec f
e e e ,a de a e ca . T e b bb e ee d b bb e
ca ed b e e ce f a e e

Da /T ea e :
● S ca a a a dc ec e d- -e d a a

Ped a c 465
Get The Step 2 CK Drills Book: usmledrills.com
The USMLE Guys Step 2 CK Crash Course usmleguys.com

Necroti ing Enterocolitis

R Fac :
● Ve b e (<1500 ) a d ee b (e ec a <32 ee e a )

S /S :
● B , d a ea, fe e , abd a d e /abd a a e e a/
d a /c e , e a c e a, e a , a ea, e a fa e

Lab :
● L ab e e c
● T b c e a, ed PT a d PTT, c ea ed D-d e , dec ea ed e fac
V, dec ea ed f b e c ce a
● H a e a, e e a ed c e
● E e a ed ac a e e e , e ab c ac d
● P be eb dc e

I a :
● P efe ed a da abd a X- a c a e a f e
b e ( e a e a ), be fb e a f ed a
e e c e e e b e ec a d be e e e fb e
ef a a cc ed

Da :
● C ca a d a c e da

T ea e :
● NPO, TPN, a c dec e , IV f d , c ec e ec e , d
e d a ca d e a a eeded
● E c b ad- ec a b c
● Se a ca e a , ab a d a de e e f a e ed ca
ea
● If a e e e f e e e a ea , ca e e
eeded e e a a a a e ea d a a e

Ped a c 466
Get The Step 2 CK Drills Book: usmledrills.com
The USMLE Guys Step 2 CK Crash Course usmleguys.com

Content Re ie Questions:

A 6- - dc d e e e a .I a e a b
b f e a 1 a bec e b .T e ae a e e e e ce f c a
e - e eda e.O ca e a e a e e a ca dc e abd a
a a . A a a e- a ed a a a ed e e ad a . W a e
e da ?

A. P c e
B. I ce
C. Mec e ' d e c
D. I c e c c

W c f ef da e e e e a e GI b eed a
2- ea - d c d?

A. A e d c
B. I ce
C. Mec e ' d e c
D. I e a a a

W c f ef ab ab a e c a ac e ca ee a e c
e ?

A. H c e c a e c e ab c a a
B. H ec e c a e c e ab c a a
C. H c e c a e c e ab c ac d
D. H c e c e a e c e ab c a a

W c f ef da c e e d a da d ed da ec e a
a a c e ac ?

A. C a e e a
B. Rec a b
C. A ec a a e
D. C c

W c f ef c d c e a c a ed a e b bb e
abd a ad a ?

A. Nec e e c
B. D de a a e a
C. Je a a e a
D. Mec e d e c

Ped a c 467
Get The Step 2 CK Drills Book: usmledrills.com
The USMLE Guys Step 2 CK Crash Course usmleguys.com

W c f ef f d abd a ad a c e ada f
ec e e c ?

A. P e a e a
B. D b e b bb e
C. T e b bb e
D. D a ed c f a a a

Ped a c 468
Get The Step 2 CK Drills Book: usmledrills.com
The USMLE Guys Step 2 CK Crash Course usmleguys.com

P a U D a
Vesicoureteral Reflu

Re ade e e f ef e b adde e e a ac ( e e a d d e ).

T e f Ve c e e a Ref :
● P a e c e e a ef e fa ed c e f e e e e ca c d e
c e a a e ca e e be c e ed d c ac
● Sec da e c e e a ef e a ca d e e e
b adde ee e e e e ca c e d c ac

G ade f Ref :
● G ade I: Re ade f f e e b ee eac e d e
● G ade II: Re ade f f e e b ee a d e d e
ca d a f e e a e
● G ade III: Re ade f f e e b ee a d e d e ca
d de a e d a f e ee a d e e a e
● G ade IV: Re ade f f e e b ee a d e d e ca
d a a d e f e ee,d a f e e a e a d ca ce
● G ade V: Re ade f f e e b ee a d e d e ca
d a a d f ca f e ee a e ca ce b ed, f a a
e ,a d be a e a ef

S /S :
● Feb e UTI

C ca :
● C c d e d ea e

Lab :
● U a c e UTI ( e e ce f bac e a, e e c e e e a e, e
f e )
● If b d e ed, b e e e a ed c ea e

I a /D a :
● K d e a d b adde a d d e de e d ade
● V d c e a (d a c) ef f ef e b adde e
e e +/- e d e

T ea e :
● S e a ce f ade I a d II ef , ea e f a UTI
● Da a e - fa e a ef ade III-IV ef , f
UTI, b adde a d b e d f c

Ped a c 469
Get The Step 2 CK Drills Book: usmledrills.com
The USMLE Guys Step 2 CK Crash Course usmleguys.com

● S ca c ec f ade IV-V ef e e a e 2 ea fa e de
c de ade III-IV ef fa ac c e a

H pospadias

C e a defec e e e f e e aa eef e c , e af f
e e , e de de f e a f e e a e a a e e e .

S /S /D a :
● V a ed ca fa e a e e a e e e , ab a e ea ,
d a ded e ce, ab a c a e f e e

T ea e :
● C c c c a d ca ed
● S ca c ec ef ed a 6 -1 ea

Cr ptorchidism

Te a a de ce ded e c b a e4 .

S /S :
● A e c a e e ce f e e ce e c ,
be a a f e e e a ca a abd e

R Fac :
● S a f e a a a ea b b e , e a

A c a ed C d :
● K efe e d e, P ade -W d e, Ka a d e, a d e e
d e, N a d e, abd a a defec , e a be defec

C ca :
● Te c a a a, a e a, a a a f a , dec ea ed fe

Da :
● Made da c a a c ( c c e ea e )

T ea e :
● Oc e f de ce ded e e af e f a d bef e ea fa e
● If e c a a ce cc , c e d be e f ed f
de f ca

Ped a c 470
Get The Step 2 CK Drills Book: usmledrills.com
The USMLE Guys Step 2 CK Crash Course usmleguys.com

Content Re ie Questions:

W c ade f e c e e a ef a c a ed e ade f f e b ee
a d d e ca d a f e e a e ?

A. G ade I
B. G ade II
C. G ade III
D. G ade IV

W a c d c a ac e ed b a e a e ca ed a e af f e e ?

A. H ad a
B. Ve c e e a ef
C. C c d
D. E d d

W c f ef c ca ae a c a ed c c d ?

A. C
B. I a e a
C. Te c a ca ce
D. Dec ea ed fe

Ped a c 471
Get The Step 2 CK Drills Book: usmledrills.com
The USMLE Guys Step 2 CK Crash Course usmleguys.com

Ka a a D a a R S
Kawasaki Disease

S /S :
● Fe e , c ( a be e a d c ac ed ed ), a , c c , / f
ec a e f e a d a d fee , ac ca d a, e ca d a eff ,c a a e
d a a e ,a e ce ca ade a ,a , GI d ba ce
(d a ea, , ec f c abd a a ), c , ea

R Fac :
● A a e c ,a e de 5

Lab :
● T b c , b e a e a, e e a ed fe , e c a ef f , e e a ed
ESR a d CRP, a c c e f WBC , be d/ de a e e e a
AST/ALT, a e a

I a :
● Ec ca d a bec a a e d aa a da e

Da :
● Fe e a 5 da 4 f e 5 c e a:
○ P a
○ O a c ( a be e, c ac ed ed , f a ed a )
○ Ha d f c a e (ede a, de a a de e a ,e e a f e
a / e )
○ B ae a c c
○ Ce ca ade a

T ea e :
● IVIG e a , a , a d +/- ed ef e a e f IVIG
e a ce
● M e ea ec ca d a a a d ee f a ec ca d a
a, efe e f ab a e e
● M f Re e d e

Re e S ndrome

R Fac :
● U e fa feb e c d e (e ec a f e a a ce a)

S /S :
● Se e e , e a e a ,c f /e a e e e , ab a
a dc a

Ped a c 472
Get The Step 2 CK Drills Book: usmledrills.com
The USMLE Guys Step 2 CK Crash Course usmleguys.com

Lab :
● E e a ed a a fe a e , ea e a, ed b e,
ce a, e ab c ac d , b e e e a ed e e e, a d c e
e e CSF a a

I a :
● U a d CT ca f e e e a c ea
● CT MRI f ead a be a d ff e ce eb a ede a

E ec e ce a a :
● Wa ef c e e e

L e B :
● D ff e a b a ea ce de f a e a c e

Da :
● C ca f d , ab , a d a da

T ea e :
● I ba a d ec a ca e a f c a e a e
● E e a ed ac a a e e ea ed a , de a e a e, a d
e e a
● Me ab c d ba ce c ec ed IV f d a a e e de e ec e a d
c e
● Lac e ea ea e a
● V a K, a e e , f e f e a a c ec c ab a e
● A e e ed ca e e e e e
● C ba e c fe e

Content Re ie Questions:

A 3- ea - d feb e c d ec e f f e a a bee e a b ae
c e fe e a d de a e ef. F da af e f be a , e c d de e
e -b d a d a bec e e a c a d de e ed ab a .
O e a e a e a ed. Ba ed a d e e c ca f d a e, c
f ef c d d e ec d e a e?

A. Ka a a d ea e
B. Re e d e
C. I ce
D. Pe c ce d ea e

Ped a c 473
Get The Step 2 CK Drills Book: usmledrills.com
The USMLE Guys Step 2 CK Crash Course usmleguys.com

W c f ef ea e e e a aef ac dda ed
Ka a a d ea e?

A. IVIG e a a d a
B. A a ea dc c a de
C. A c a da c
D. N ea e ( e ca e)

A 4- ea - d fe a e e e e a a c ef c a f fe e . T e fe e a bee
e e f 7 da . S e a a ee a a a e ,b e a ce c ed
de e e , e ae b e e a.O e a e a a ac a
a e a de e e ,b ed c ac ed ,b aea e da e c c ,
a d ce ca ade a . Ba ed e e , c f ef c ca
e a f de e ?

A. I e ab e a f ec
B. Lef - ded ea fa e
C. R - ded ea fa e
D. C a a e a e

Ped a c 474
Get The Step 2 CK Drills Book: usmledrills.com
The USMLE Guys Step 2 CK Crash Course usmleguys.com

M a D a J I a A
Duchenne Muscular D stroph

X- ed ece e a ed (DMD) e e ead e e ce


de e e a a d ea e .

S /S :
● O e be ee 2-3 ea d, de a , add a / e a , a ce
ea e affec ed f , ca f e a e e , G e ' , cec a ,
/a ef e a, a

C ca :
● Ca d ac c ca c de d a ed ca d a , a e a ,c d c
ab a e /a a
● O c ca c de f ac e d e fe e fa ( b )a d c c c d
e a ( e eb a )

Lab :
● Se c ea e a e 10-20 ea e a e e f a, e e
e a a e
● E e a ed AST/ALT a d a d a e

Becker Muscular D stroph

S /S :
● Sa e a D c e e c a d , ae a e f e (5-60 ea ), e e ee
( e a a ed)

Lab :
● Se c ea e a e5 ea e a e e f a

Duchenne and Becker Muscular D stroph

Da :
● If c ca f d ea e a d e e a ed CK e e e e , e f ec a e e c
e de f a DMD e e ( ca d a c)
● If ab a de f ed e e ca a , ceb
c e a dd a a ed c f da

T ea e :
● G c c c d ea e ( ed e def a ac )
● D ea e- df ea e de e d ec f c a ( d e , a e ,
ee e , ca e e )
● S e ca e

Ped a c 475
Get The Step 2 CK Drills Book: usmledrills.com
The USMLE Guys Step 2 CK Crash Course usmleguys.com

Oligoarticular Juvenile Idiopathic Arthritis

Fe a e affec ed e a a e , ea e a a e 2-3 ea , b ca de 10
ea .

S /S :
● 4 ed ( a ee /a e , e e eb / )
c ae e ,
e de , a d ca e ; e ; e ; e -e d ce a c

Lab :
● A c ea a b d e f e e
● P be d a e a, be dee a ESR a d/ CRP

Da :
● A 4 e 6 , e a b ab e ca e

T ea e :
● M d d ea e ( d , c ac e fac ) ea NSAID a d
a-a c a c c c d
● M de a e/ e e e d ea e ea NSAID , a-a c a c c c d ,a d
e e ae

Pol articular Juvenile Idiopathic Arthritis

Fe a e affec ed e a ae , e b da be ee 2-5 a d 10-14 ea .

S /S :
● A >4 e 6 (f e , e , , ee , a e , ,eb )
c ae e , e de , f e e ca , a d ca e , e

Lab :
● P be d a e a, e e a ESR, a d/ e a a b e a
● A c ea a b d e f e e

Da :
● A >4 e 6 , e a b ab e ca e

T ea e :
● Me e a e a TNF b a d NSAID
● Add a-a c a c c c d ec fc ed d ea e ac af e 3

S stemic Juvenile Idiopathic Arthritis

Ma e a d fe a e e a affec ed, e <17 ea .

Ped a c 476
Get The Step 2 CK Drills Book: usmledrills.com
The USMLE Guys Step 2 CK Crash Course usmleguys.com

S /S :
● A ( , , ee , a e c ) fa be f , da
fe e , a
● Mac a e ac a d e (c ca ) b e a , fe e , c a a ,
ade a , e a e e a , e a eff , a d/ e ca d

Lab :
● E e a ed WBC c , fe e e e , e ed a ce, be
a e a, a d b c
● ESR d a a ca e e a ed a d e e a ed CRP
● E e a ed fe
● M d e e a ed AST/ALT a d D-d e
● E e a ed b e e , ab e a
● W e ac a e ac a d e e e : Pa c e aa de e e e
fe , AST/ALT, a d D-d e ; fb e e a; a d ed PT a d PTT

Da :
● Da d a fe e f 2 ee a da f 6 ee a de c f e
ca e

T ea e :
● M d- de a e ca e ea NSAID
● I e e e ca e e e NSAID a e: Add a b ca e ( c ab,
a a a, ca a ab) a d f e e ff c e add c c c d

Content Re ie Questions:

A 4- ea - d fe a e a e e e ec c ac a f a .T e a e
ae a e e f e ,a e a e ee a da e ae a f .E a a
a f e a e a b aea ee a d a e e a d e de e
a a .U ec f a e a e . Lab a e e a ed ESR a d
e ANA, e eb a e .T e a e a ad ece fec , fe e ,
e ab e ab a e .W a e e da ?

A. O a c a e e d a ca
B. P a c a e e d a ca
C. S e c e e d a ca
D. S e c e e a

Ped a c 477
Get The Step 2 CK Drills Book: usmledrills.com
The USMLE Guys Step 2 CK Crash Course usmleguys.com

A 3- ea - d a e b e ed a c a d e a e ed a .H ae e e a e
a e fe e ,a d e a e ced e a e ec a a e ca f ce .
W e a ed a df a ea ed , e a e e a d e e c b
e e e e a d . Ba ed e e ca f d , a ab ab a e
d e ec be e e ?

A. L CSF c e
B. E e a ed e c ea e a e
C. A -d DNA a b d
D. Mac c c a e a

A 15- ea - d a e a e e e a .T e a e e d e a e ,
ee , a d a e b a e a c a ea e a d a e e de e a .T e a e a
e d e da fe e c e e a ea d f e acc a ed b a a b
a a c e e a ed, a d a - c .T e e a e bee e e
f 6 ee . Lab a e e a ed WBC c , b c ,a e a a d a a ca
e e a ed ESR. ANA e a e. W c f e f c d e e da ?

A. O a c a e e d a ca
B. P a c a e e d a ca
C. S e c e e d a ca
D. S e c e e a

Ped a c 478
Get The Step 2 CK Drills Book: usmledrills.com
The USMLE Guys Step 2 CK Crash Course usmleguys.com

P a M a D a
Developmental D splasia of the Hip

R Fac :
● B eec , fe a e e , add , fa

S /S :
● P eO a a e e (<3 ), e Ba a e e (<3 ), e
Ga ea , e T e de e b e c e ( e a e ca a ), ed
abd c (>3 )

I a :
● U a d f e ed 6 fa e c f ca e a f d
● H ad a ed a 6 a d de c f ca e a f d

Da :
● Made ba ed c ca ab a e , ca be c f ed da c a
(US/X- a f e )

T ea e :
● Abd c (<6 )
● C ed e ed c de a e e a (>6 )
● M X- a e e e de e e

Legg-Calv -Perthes

S d e ca d a c e ec (a a c a ec ) f e .

S /S :
● H a / d e be ee 3-12 ea , e ac b fe
e e ed e ed ca

I a :
● Ea X- a fe a, ae a e f d ea e fa e a a d ea
f e fe a ead (def )
● MRI ca e ab f a fe a e ea a c a ac bef e X- a
c a e a e ee

T ea e :
● S e e e ca e e c a e fe a ead e ace ab

Ped a c 479
Get The Step 2 CK Drills Book: usmledrills.com
The USMLE Guys Step 2 CK Crash Course usmleguys.com

Slipped Capital Femoral Epiph sis

R Fac :
● Obe , ,e d c e ab a e

S /S :
● O e 8-15 ea d, a d , , , ee a (d , ac ,
- ad a )

I a :
● X- a f e a a ea be a e d ace e f e fe a
e ( ce c ea ff a c e)
● MRI de f e ded b ede a

Da :
● C ca a ( ad a MRI) c e eda

T ea e :
● A d e -bea e
● O e a e ab a

Scoliosis

S /S :
● La e a c a e f e e >10 de ee ; a a a d ffe e d a ce a
;a e f de , ca ae, a d/ a ; e Ada f a d be d e

Sc ee:
● Mea e a e f a

I a :
● X- a f e ea d e de ee a d d ec fc a e, a e a
eea a
● MRI f e e ed f a / e c ae e e f C bb a e c ea e b
10 de ee a ea

Da :
● Da c a (X- a , MRI) c a e f e e a C bb a e >10
de ee a a e 10 ea de

T ea e :
● C bb a e f 11 29 de ee : C e b e a f e
● C bb a e f 30 39 de ee : B ac
● C bb a e be ee 40 a d 49 de ee : B ac e

Ped a c 480
Get The Step 2 CK Drills Book: usmledrills.com
The USMLE Guys Step 2 CK Crash Course usmleguys.com

Content Re ie Questions:

A 8- ea - d fe ae e e ec c a . T e a e de e a a aa d
ae e ca eca a e ac e e be a , b e a bee d e a f e
a ee . T e a e ac b e e ed e ed ca .I a
def f e fe a ead d ca fa e a a d ea .W c f e
f e e da ba ed e ef d ?

A. De e e a d a a f e
B. S ed ca a fe a e
C. S e c e e d a ca
D. Le -Ca -Pe e d ea e

W c f ef a e e /e ed a e f e e e ce f de e e a
d a a f e ?

A. O a a e e
B. Ba a e e
C. Ada f a d be d e
D. Ga ea

A ad a a 12- ea - d be e fe a e a a ea be a e
d ace e f e fe a e e e e fe .W c f ef
c d e ba ed e e a e ?

A. De e e a d a a f e
B. S ed ca a fe a e
C. E aca a c a e c f ac e
D. Le -Ca -Pe e d ea e

Ped a c 481
Get The Step 2 CK Drills Book: usmledrills.com
The USMLE Guys Step 2 CK Crash Course usmleguys.com

P a T
Neuroblastoma

S /S :
● S de e d a ca a d e e ce f e a a c d ea e
● P a abd a : Abd a a (f ed a d f ), f e , a d/ a ;
e e ;c e f e e e (c a , b c ) b adde
(e e , dec ea ed b adde ca ac )
● P a ac c : T ac ea de a / b c , e e a ca a d e
● P a a a e eb a :H e d e, e d a a c dc e

Pa a e a cS d e :
● Va ac e e a e de (VIP) ec e - e e e ec e da ea a d
ae a
● O c c d e-a a a, a d e e e e ,a d c f e
/e e e

Me a a e :
● S ead b e a ca a c e a, fe e , e , a ,a d
● S ead e b a b e ca ecc , ,
● S ead ca a e bc a e d e
● Re a de ead (de e de ca f a )

Lab :
● E e a ed a ca ec a e e ab e , a a de c ac d (VMA), a d
a c ac d (HVA)

B :
● B f e a b e a a ae f ea a e e e

I a :
● CT MRI ed f a , e a a ea ee e e a d a c a
ca c f ca

Da :
● Def eda ade e e e f a e OR
b e a a ae ea a e a ca ec a e e ab c
d c

T ea e :
● De e d e e c e ea , ca e ec , e a e c e ce
a a a , ad a ea ,a d a e a ce e a c ea a
ed

Ped a c 482
Get The Step 2 CK Drills Book: usmledrills.com
The USMLE Guys Step 2 CK Crash Course usmleguys.com

Retinoblastoma

A e f e a de 3, 1-2 ea d a RB1 e e.

S /S :
● Ab a e ef e ( e c a); ab a d/ a ; be
e -c c c bed, a ce /c a / , f , f ab e ae a a e ee a

I a :
● U a d d e- a ed e a a d ff e ca c f ca
● MRI f eba a d b e a

Ge e c Te :
● A e f RB1 e e a

B :
● C a d ca ed ( f eed )

Da :
● Made ba ed ff e e e a a d a f d c e d ea e

T ea e :
● De e d e , ea e a f ed a ea e da e
c d a ca e c e e ,c e ea ,e c ea , a e
c a a , a d/ c ea

Wilms Tumor

A e f e a de 10 ea , de 5 ea .

S /S :
● Abd a a ,f e , a d/ a , fe e , e a a, e e

I a :
● Abd a a d e a
● CT c a MRI ca de f f ca e e a cc ed fa c e
e e a d e acc a e c a ac e e e
● CT f ec e de f e

C d A c a ed W T :
● WAGR d e (W , a d a, e a defec , a d
e a da / e ec a d ab ) ca ed b T1 e e de e
● Bec -W ede a d e (W , a ce e, ac a,
ac a)

Ped a c 483
Get The Step 2 CK Drills Book: usmledrills.com
The USMLE Guys Step 2 CK Crash Course usmleguys.com

● Da d e (W , e a d ea e, ae e d e a d ) ca ed b
T1 e e a

Da :
● Def eda ade ca e c ed

T ea e :
● A a e e ec ab e d de e , a d de e d a ea d
ece e c e ea a d e e e d ea e a ea ed ad a
ea

Content Re ie Questions:

A 3- ea - d a e a e e e e a a c ef c a f abd a f e .
O e a e a e ed a e b a e a ac fa a d c a e e a ed a d
dd a ed. O e a e a , e e e ce e e e c a d e a e a a
a ed e ec a d ab . Abd a e a ef af a a a ed b de
f e abd e . G e e ef d , a e e ca e f a ?

A. I ce
B. N -H d ' a
C. W
D. Ne b a a

A 1- ea - d c d be e a a ed a a e c d b ed a c a . O e e e a a
ab a e ef e ed, a a ce , ae a a ee . G e e ef d ,
c f ef e d be c a d ca ed a a f e - a d ea e f
e e c d e e e ca f d ?

A. T b
B. U a d f ee e
C. Ad a f a ea e e a ef a e ee a de eda
D. C e ea

W c f ef ab a e a c a ed e ba a?

A. O c a b ba d e CSF
B. P e e ce f 30% b a ce eb e a
C. E e a ed a a a de c ac d (VMA) a d a c ac d (HVA)
D. Mac a e d-c a b d e (f a ac a e )

Ped a c 484
Get The Step 2 CK Drills Book: usmledrills.com
The USMLE Guys Step 2 CK Crash Course usmleguys.com

P a A a I a A a
Croup

V a a ac e e fa a f e a a d b ca a .

S /S :
● Ba c , a d , a e e , fe e e e e d ea e ca
a ed e c a a d bc a e ac , dec ea ed/ab e b ea d ,
ac ca d a, c a , a d a e ed c c e

I a :
● PA c e ad a e " ee e d ca b c a

Lab :
● P be , a, e e a ed WBC c

Da :
● Made c ca ba ed , a a d ab e ed

T ea e :
● M d a a ed a e e ca e a d ed e f
de a e a e ed e
● M de a e e eec ea ed a eb ed e e e,
de a e a e, a d d f ed a e . If e e , ca be e ea d f
c a e e , d be ad ed f a d ca e

Bacterial Tracheitis

Bac e a fec f e f e f e ac ea f a fec a a f


ec e bac e a ad .

S /S :
● C , d , e a d e , fe e , a e e , d a aa d e e e ca e
a ed ec a a d bc a e ac , a e ed e e f c c e ,c a

I a :
● A e e ad a ee e d ca b c ac ea
a a d aea ec ad a ac ea a e a

Da :
● Def eda ade d ec a a a c a d
ac e b c c . Ma ca e d a ed e e ba ed a e
e a fec f ed b c ca a d ad a c f bac e a ac e

Ped a c 485
Get The Step 2 CK Drills Book: usmledrills.com
The USMLE Guys Step 2 CK Crash Course usmleguys.com

T ea e :
● I ba f e a c ed, IV f d a da b c ( a c c
cef a e), a f eb ed e e e

Epiglottitis

I fa a ( ca d e fec ) f ee a d d a c c e .

S /S :
● C d e de 5 e e " d" e, d , e a d e ,a e ,
fe e , d , e a, a ce
● I c d e 5a d e a d ad e ce : D , e a,a dd a aae
c ; e e be e a d e

I a :
● La e a ec ad a e" b e e e a e a ed e

Lab :
● Ob a ed AFTER a a ec ed

Da :
● Def eda e e d ec a a f e ed e f be c
a a c d ec a c OR " b aea ec ad a

T ea e :
● Sec e e a a a e f e e a fa e( a ea - a
a a b c )
● If a a a ea ff c e ae be da c - ,e ea a ea
a bed de f e e a de e a a be a d
● If a a a ea ff c e ae ,a e a e e e e a
c f ab e ( d ae a )a da da a f / ca e e
a a ca e c a e a d ab a a b c , a d ad e
d f ed e
● B dc ea de cc e( ba ed a e ) d be b a ed
ad a fa b c ( a c c a d cef a e)

Peritonsillar Abscess and Cellulitis

S /S :
● U ae a e a a d ec e ,f c a de a f e a e
c a a e a de, " a " ce, fe e , d ,d a a, , ce ca a d
b a db a ade a

Ped a c 486
Get The Step 2 CK Drills Book: usmledrills.com
The USMLE Guys Step 2 CK Crash Course usmleguys.com

I a :
● I a a b a db a a d de fe e a ce a a e e
f e e a d ab ce a a ec -f ee ca a e a b de

Da :
● Made c ca a ed a d aced f c a a d a de a ed e
c a a e a de c ec f e ae a c f ab ce

T ea e :
● Sec e e a a f d ca ed
● F ea e f e a ce IV a b c (a c - bac a c da c
f d d ea e, add a c c f de a e e e e d ea e) e ca e
● F ea e f e a ab ce , d a a e (e e eed e a a c a d
d a a e), IV a b c ( a e a e a ce ), a d e ca e a e
a ae ec e e ed f e ee e a a b c fa e f
ab ce e e da a e

Retrophar ngeal Abscess

S /S :
● S ff ec , a ec e e ,d a a, d a a, d , fe e
d e e f e e a ea a
● T e a a b c a e e ea f ad e ead e ff
, a e a e ac , d , ac ea, a d a a de ea

Lab :
● S d be b a ed af e a a ec ed ca e e a c e
● CBC e e a ed WBC c ,b dc e cca a ae e

I a :
● La e a ec ad a de f e e a ea ace
● CT (be da ) ca de f ab ce c ee e a ce e a d ca
a a e e a ea a

Da :
● Def e e de f ed b CT f e ec d ca d a a e, e e
ba ed c ca a d a e a ec ad a

T ea e :
● Sec e ea a a e e a c e
2
● S ca d a a e e ab ce 2.5 c a d a e (c ee e a ce e
a d ca ) f ca a a c e
● If ab ce a e a 2.5 c 2 a e, a f a b c a e
(a c - bac a c da c , de a e- e e e ca e add a c c )

Ped a c 487
Get The Step 2 CK Drills Book: usmledrills.com
The USMLE Guys Step 2 CK Crash Course usmleguys.com

Bronchiolitis

V a fec ( a RSV ) f eb c e c de de 2 ead


ee a d/ ae .

R Fac :
● P e a , b e ,c bdc c d ea e, a e de 12 ee ,
ec d- a d ee e, da ca e a e da ce

S /S :
● I a e e a ac fec (1-3 da f ea) f ed b fe e , c ,
ac ea, a ea, ee , ae , ec a a d bc a e ac , a a fa

I a :
● N ca b a ed e a e da be c de ed, b c e
ad a a e b c a c e , e fa , a d a c a e ec a

Da :
Made c ca a e de 2 ea d e e a
e f c ea ed f e a ee a d/ c ac e

T ea e :
● M d ca e a a ed a e e ca e
● M de a e- e e e ca e a a ed a e e ca e c d f d
a a e e , e e a e , a a c
● Pa e a ea, e e a ed CO2 ABG d ca dec e a f c ,
e ae e e e a e a e a a ed ec a ca e a

Pertussis

T ee S a e f Pe :
● Ca a a a e: M d c a dc a
○ La 1-2 ee
● Pa a a e: Pa f e eec , ea , a d c a ac e ed b
a ec fc e a cc be ee c e
bed ea, c a ,a d a .T ec f f ed b a f
f ced a a d fe - e
○ La 2-8 ee
● C a e ce a e c a ac e ed b ad a e e fc e ee

○ A ea e a d a fc 112 da

Lab :
● Le c c

Ped a c 488
Get The Step 2 CK Drills Book: usmledrills.com
The USMLE Guys Step 2 CK Crash Course usmleguys.com

● Bac e a c e PCR

C ca :
● We ,a ea, e a, e e , dea

Da :
● Ca be ade c ca a e, b fe ade def e bac e a
c e PCR

T ea e :
● A c f e ea e f e a e a d a fc ec ac f e
a e
● A a e feed d ff c e ,a ea, e e , e a, e a d e ,
a d a e <4 ( e a de f ) e e a a

Foreign Bod Aspiration

S /S :
● C ee b c f e ac ea e e e a d e , a e ed e a
a ,c a , ab ea c ,d
● Pa a b c e e c , d ,d ea, ee , ac ea,
dec ea ed b ea d

C ca :
● P e a( - b c e, ec e ), ab ce , e

I a :
● C e ad a a f e b d ( f ad a e), ed a a f a a f
de c a ef e b d , ce c d a e b c (e e e
e f a ed ), be f aed a e b c ( - b c e
e a)

Da /T ea e :
● V a a a d e a ff e b d db c c

Content Re ie Questions:

W c f ef c d a c a ed a" b aea ec ad a ?

A. F e b d a a
B. Re a ea ab ce
C. E
D. C

Ped a c 489
Get The Step 2 CK Drills Book: usmledrills.com
The USMLE Guys Step 2 CK Crash Course usmleguys.com

A 4- ea - d fe a e a e e e e ED e a d e .T e a e a ea
a a d b e ed ea f ad be a e a e ac , e d ,
a d ac ea. P e e ac e ead f 82%. W a e a ae
e e ?

A. Bed de c e X- a
B. Ob a a e f ca e a e
C. I ba a d e a
D. D a b d c e

W c f ef e e ec e ded a a e a ed e f
e de f b c ?

A. F d a a e e
B. G c c c d
C. S e e a e
D. Na a c

Ped a c 490
Get The Step 2 CK Drills Book: usmledrills.com
The USMLE Guys Step 2 CK Crash Course usmleguys.com

C Ab
Red F a Pa e H :
● H d e c e a e de ee d f
● Ca e a e de e f a a de e e e ce f
● C a e e ed
● H f ca d ffe e e b e e e ed e a a e
● I e b a ed e c de , e , ef c ae e ca e e
e e / e f

P ca S f Ab e: B
● A b e a fa <6
● H a b e a
● Pa e ed b e (be , a d, )
● B e ca e ea , b c , ec ,

P ca S f Ab e: B e F ac e
● M e f ac e d ffe e a e f ea
● R b f ac e
● S f ac e c de de 18 , ea ab e
● L b e f ac e a fa
● Me a ea c e f ac e e ea e aa

P ca S f Ab e: I e a B
● C a e eb c a e dee e, c c a , 8-12 da ee b
● I e b , a e -def ed e e ee eb e d a d a
be
● B a c ea de a ca ed ed e ef a b ec ed b ec d( ,
c )

P ca S f Ab e: Ab e Head T a a
● Re a e a e
● I ac a a a ( bd a e a e, f ca c ca c ,ba e ,
ba ac d e a e)
● S f ac e

Se ual Abuse in Children

S f Ab e:
● Fe ea , b f e ad f aae
● E e a, ea , a d/ ab a f e ab a, , a d/ e ; ab e e ;
a a d c a e
● Pe e e e a, ab a ,b e
● A ede a a d b e d c a , ace a f e ec
● P e e ce f STI e

Ped a c 491
Get The Step 2 CK Drills Book: usmledrills.com
The USMLE Guys Step 2 CK Crash Course usmleguys.com

C ec f F e c E de ce:
P -d c e a f ca f ab e
S ab c ec e e
F e a ca f c de c be cac ea a a

T ea e :
Pe be a c d e e STI d ece e a b c a e a a
be a c d e a e bee e a ab ed
P be a fe a e a e d be ffe ed e e e c c ace
Refe a f e a ea e a a
Re a ca e a aea e

Factitious Disorder Imposed on Another (Munchausen S ndrome b Pro )

Ca e a e Ac :
● S a e e a e a ed
● S a e fab ca ed
● Ca e a e e a ca e e

Ma a e e :
● S a ece a ed ca / ced e a d e e eeded a c a acce ,
f c d ec e ce , a d c a c d ab e ec a

Content Re ie Questions:

W c f ef b ea c f a e a b ca ed b ca ab e
f a c d?

A. A 9- ea - d e dee e, c c a , 10 da ee b ca ed e
b c
B. A 10- ea - d a e fca b ed a a d aea f ed a
de f e
C. A 2- ea - d e e e b a e -def ed e e ee eb
e d a d a be a d a f eb c
D. A 11- ea - d a dee eb a c ea de a ca ed ed e ca ed e
e bac

W c f ef a e a c a ed ca f ab e ead a a
a c d?

A. Be a e eC ce fW
B. S bd a e a e
C. Re a e a e
D. F ca c ca c

Ped a c 492
Get The Step 2 CK Drills Book: usmledrills.com
The USMLE Guys Step 2 CK Crash Course usmleguys.com

W c f ef f ac e c f c d ab e a 8- ea - d?

A. C a c a f ac e
B. S ac d a e f ac e
C. T f ac e
D. M e f ac e d ffe e a e f ea

Ped a c 493
Get The Step 2 CK Drills Book: usmledrills.com
The USMLE Guys Step 2 CK Crash Course usmleguys.com

P chia
Ma D D ,P D D ,
A D ,C a G ,P a
B /D /P
Major Depressive Disorder

DSM C e a:
● 5 ( c de de e ed d f ee ea e) c ca e
c ca f ca d e / a e a dae e e d e a e 2- ee e d,
c ca be a b ed a e ed ca / c c d effec f b a ce e,
a d f a a/ a a:
○ Depressed mood most of the da , almost e er da or e er da
○ Loss of interest in almost all acti ities nearl e er da
○ C a e a e e ( c ea e dec ea e) e c a e ( c ea e
dec ea e)
○ S ee d ff c e ( a e a) ea e e da
○ P c e a da da
○ L fe e da
○ Fee f e e
○ Dec ea ed c ce a dec e e da
○ S c da dea , ec f c a f c c de, c de a e
● M e c SIG E CAPS (S ee , I e e , G , E e , C ce a , A e e,
P c , S c da dea )

T ea e :
● C c e c e a (CBT, e e a c e a ) a d a ac e a
● F e ea e ed ca c de e ec e e e a e b (SSRI )
c a e a e, c a a ,e c a a ,f e e, a e e

S c da Idea a dA e - T ea e :
● S ab e a e de e d b d a ca ed b c de a e
● Red ce ed a e fa e ( a a , a e )
● C ea e a afe a
● T ea de c d ( a ac ea , c e a , e ec c e ea )

Persistent Depressive Disorder (D sth mia)

DSM C e a:
● De e ed df f e da , f da f 2 ea
● P e e ce f 2 f e f :
○ S ee d f c ( a e a)
○ Fee f ee e
○ L e e
○ P c ce a dec e e
○ A e ec a e
○ L e f-e ee

P c a 494
Get The Step 2 CK Drills Book: usmledrills.com
The USMLE Guys Step 2 CK Crash Course usmleguys.com

● Pa e a bee ab e a d de e ed df e a 2
a a e
● N be e e a ed b e c c d , effec fa b a ce, ed ca c d
● S ca e f ca d e / a e ff c

T ea e :
● F - e ea e c ea a d a ac ea e ec e e
e a e b

Adjustment Disorder with Depressed Mood

● S d ee c e a f a de e ed de
● S e e e e a ec f c c ca e ( )

DSM C e a:
● De e ed d, ea f e , fee f ee e e e ec f c
c ca e ( 3 f e e)
● Re e e a a e e ce e a d/ a / ca f c
● N e c d de be e f , ae a e ace ba f
ee c a cd de
● Pa e e e e c be ea e e
● U e e a f e / e c e e ce , d e e e <6

T ea e :
● P c ea

Complicated Grief

Da cC e a:
● Be ea e e f 6 (e e ade f c a/ e d ffe e ce ) a
ca e f ca d e a ed f c a b ab e c a/ e
e e
● 1 f ef be e e :
○ Fe e e ab e decea ed
○ Fe e e e fee a fe ea e e decea ed f
e ee e e
○ Rec e de e decea ed dea , a e
decea ed bea ab e
○ Pe e , e e f e decea ed
● 2 f ef be e e :
○ Fe e a e ec c a ce c e e ce f e e
dea
○ I ab acce e dea
○ Pe e fee f c , b, da ed ce e dea
○ A e e ad e dea
○ I e ee a eac e de f e dea
○ A d e e/ a a e e e d f e decea ed a d/ a
ee, c , ea , e b ec a a ea ea e
decea ed

P c a 495
Get The Step 2 CK Drills Book: usmledrills.com
The USMLE Guys Step 2 CK Crash Course usmleguys.com

T ea e :
● C e-be a a ea ada ed f c ca ed ef (c ca ed ef ea )

Postpartum Blues/Depression

P a B e :
● O e 2-3 da f de e
● Re e ee
● Da ed e a e a 3-4 de e e (d e ee c e a f
a a a de e )
● T ea e ac f a ( ea e f de e a a a
de e ) ea a ce a d

P a U a Ma De e :
● A a e d be c ee ed a ea ce a f de e (10- e
Ed b P a a De e Sca e)
● Da cc e a e a ea a f a de e ed de e 4
ee a
● F d- de a e a a a de e f e ea e
c ea
● F e e e (7-9 de e e , c da dea ) a a a
de e ea a de e a ed ca (SSRI) c ea

Postpartum Ps chosis

O e ca 2 ee f de e .

S :
● Ha c a , de , b a e be a , d a a

Da :
● Pa e be e a a ( de ) e a d de

T ea e :
● E e e e e e a e be ef a e e fa ( fa c de )a d
a a ab e
● T ea a ec d- e e a a c c ( a a e, e a e, e d e)
● Add f be d a e e, a d ab e , a d/ a a de e a ba ed
de da

P c a 496
Get The Step 2 CK Drills Book: usmledrills.com
The USMLE Guys Step 2 CK Crash Course usmleguys.com

Content Re ie Questions:

A 21- ea - d a e e e ec c a c ef c a f de e ed d. F e a
2 e a e e e ced de e ed df f e da , ea e e da . He de e
a ece fe c a e a b a ce e. He e d e f ee c
e b a d a ed d ff c fa a ee . He ef
fee f e e a d a d ff c c ce a c . He de e a
c da dea .W c f ef e a ae e e a a e e ?

A. Ad e e a e
B. I a e a a
C. E ec c e ea
D. Ad e a e a

A 29- ea - d fe ae b e c c b e fa e . T e a e ece a eb 11
da a a d f e a 2 da e fa e a e e a de a ed b a e be a a d
d a ed .T e a e ae e be e e a e FBI a acce
e e e e ce e a .S ee d e e be ef a e e
be ed e c e a a e . He fa e a e da e e
a e e e b a d e e c ce ed e ed c ed d e bab e
ef ec e f FBI a e . W a e e be e a a e e ?

A. P e e e a e be ef a e e fa a d c a c a a
B. T ea e a e d
C. C e-be a a ea
D. Rea a ce a d

A 67- ea - d a e e e ec cd e fee e . T e a e ae fe d ed
a ea a d 4 a a d ce a e e a bee ab e f c a . He
ae e a fe e e e fee a fe ea e fe a d fee e ee
e e , e ee ea d c d e a d a dc de c f e
b . He a e e a e da e ec c a ce f e dea c a
a e 4 b ea ca ce . He e d e fee e a b e e da ce e dea . W a
e e da ?

A. Ma de e ed de
B. Pe e de e ed de
C. C ca ed ef
D. Ad e d de de e ed d

P c a 497
Get The Step 2 CK Drills Book: usmledrills.com
The USMLE Guys Step 2 CK Crash Course usmleguys.com

G a A D , Pa D ,S a A
D ,P ba
Generali ed An iet Disorder

DSM C e a:
● Pa e a e e ce ea e a d cc e da a f 6
, ab a a e f e e /ac e
● T e a e a be a a e
● T ea e a c a ed 3 f ef :
○ Re e e ed e
○ Ea fa ab
○ I a ed c ce a e ee e d e ba
○ I ab
○ M ce e / e e
○ D ff c ee

T ea e :
● C e be a a ea a ac e a
● F e a ac e a : SSRI ( e a e, c a a ,e c a a ,f e e,
a e e) OR SNRI ( e afa e, d e e)
● If e e ed ca af e 4-6 ee , c a e a d ffe e SSRI/SNRI
● If a a e e a SSRI/SNRI d e, add b e

Panic Attack

E de f e e fea /a e a ab a a dd a f e 1 .

DSM C e a:
● A e e ab a f fea d c f a ea e , e
e e ce f 4 f e f :( e c = STUDENTS 2 FEAR 3 C )
S - S ea
T-Te b / a
U - U ead e /d e / - eaded
D - De e a a /de ea a
E - E ce e ea a e, a a
N - Na ea/abd a d e
T-T / b e
S-S e f b ea / e
Fea f c / ca
Fea f d
C-C e a
C-C /e ce e a e a
C-C e a

Panic Disorder

DSM C e a:
● Pa e e e e ce ec e e ec ed a c a ac
● 1 a c a ac a bee f ed b 1 f e e /b f ef :
○ Pe e ab a add a a c a ac

P c a 498
Get The Step 2 CK Drills Book: usmledrills.com
The USMLE Guys Step 2 CK Crash Course usmleguys.com

○ De e e f a ada e be a e a ed e a c a ac
● S b a ce e a e ed ca c d e c ded a d e c d de be e
f

T ea e :
● I a a a e e CBT a ac e a
● F e ed ca SSRI, f e e a e a e SSRI, f a a e e
add be da e e(f f b a ce e d de )

Social An iet Disorder

DSM C e a:
● S ca a ca a ed fea a e f c b e
● Pa e fea ac a a a ca e e e ba a e , ead e e ec
ca e ffe e e
● T e ca a ea a a ca e fea a e
● S ca a a e a ded a e ded e e e fea a e
● Pa e fea a e d ae e ca a
● T e fea , a e , a da ce a 6 a d ca e c ca f ca d e
a e
● S b a ce e a e ed ca c d a e e c ded a d be e
e a ed b a e c d de

T ea e :
● I a ea e c e-be a a ea a ac e a
● F e ed ca SSRI/SNRI, f e e a e a e SSRI/SNRI, f
a a e e add be da e e(f f b a ce e d de )

Specific Phobia

C ca f ca fea fa a c a ec f e a ead a da ce be a .

S ec f e :
● S a a ( a e , e c ed ace )
● A a ( a e , de , d )
● Med ca ced e/ ( eed e , b d)
● Na a e e ( e ,b de f ae )
● O e (c , , d e )

DSM C e a:
● C ca f ca fea a e f a ec f e
● T e ec f e ea a a e ed a e fea a e
● T e ac a ea ed b e ec f e ca e fea a e f e
ac a da e
● D a f fea , a e , a da ce 6 , ca e c ca f ca d e
a e
● S b a ce e a e ed ca c d a e e c ded a d be e
e a ed b a e c d de

T ea e :
● F - e ea e c e-be a a ea e e ea

P c a 499
Get The Step 2 CK Drills Book: usmledrills.com
The USMLE Guys Step 2 CK Crash Course usmleguys.com

Content Re ie Questions:

A 41- ea - d a e e e ec c ac a f ee d ff c . T e a e ae
e a e ad e ce ea e cc ea da f e a 8 .T ee e
a c a e e a a ca ed a e a d e ea e a a ec f fe f
ea da ac e . He f d a d c e. He a e e
c a fee ed e a d ab e. He f d ef a ab ef e bef e bed
a d e ef e ab e fa a ee . W c f e f e a a e ea e
f a e ?

A. E c a a
B. A a
C. Z de
D. Me a

A 25- ea - d fe a e c e ec c ac a fa e .S e ae a f e a
ea ce e ed a e c e a ad a e e fea f e ac e .S e
ae e af a d f ac a a a ca e e e e ec e . W e ef ed,
e a e ded a fe a e b e a e a e e ee ee e e. N ea d
a e d a a d a e e e ed e a e ,b e ea a e
ec fea . W a e e da ?

A. Pa c a ac
B. Pa c d de
C. Ge e a ed a e d de
D. S c a a e d de

A 19- ea - d a e e e e a c e a .T e a e a d f f a
e a a e b a a d ad a e e e f fea a d a e c a ed f 10
e . T e a e e d ed ea , a , a a ,a d e f b ea c a
e ed a e .A ed ca - f ca d a ca e f c e a ae
e a e. T e a e a e e a e e ad a e a .W a e
a aeda ?

A. Ge e a ed a e d de
B. Pa c a ac
C. Pa c d de
D. Sc - e a ed ba

P c a 500
Get The Step 2 CK Drills Book: usmledrills.com
The USMLE Guys Step 2 CK Crash Course usmleguys.com

B a
Manic Episode

DSM C e a:
● Def ed e de ac e ab a d c c a ac e ed b e e a ,
e a , ab c e ab a c ea e f ac e e f 1
ee ( a e d f e f a a f a a e ed) d c 3 f e
f cc a f ca de ee (4 f d ab e) a d a e a ceab e
c a ef a be a :
○ G a d a f ed e f-e ee
○ Le e ed eed f ee c a ed ba e e
○ I c ea ed a a e e e e c e a
○ F f dea ac ( b ec e)
○ D ac b
○ I c ea ed a -d ec ed ac c a a
○ D ae e e ac e a a ea f e a e
c e e ce ( / d ce )
● S c a / cc a a f c a ed d e e ee dd ba ce
a a ece a e e a ef e , f eeae c c
fea e
● T ee de ca ed b b a ce e a e ed ca c d

H a cE de:
● Def ed e de ac e ab a d c c a ac e ed b e e a ,
e a , ab c e ab a c ea e f ac e e 4
c ec e da , e e f e da , ea e e da d c e 3 f e
f cc a f ca de ee (4 f d ab e) a d a e a ceab e
c a ef a be a :
○ G a d a f ed e f-e ee
○ Le e ed eed f ee c a ed ba e e
○ I c ea ed a a e e e e c e a
○ Rac ( b ec e) f f dea
○ D ac b
○ I c ea ed a -d ec ed ac c a a
○ D ae e e ac e a a ea f e a e
c e e ce
● T ee de c a ac e ed b a de ab e c a e f c a ab a f
e d d a e a c, a d e dd ba ce a d c a e
f c a e b e ab e b e
● S c a / cc a a f c e ee a ed d e ee de,
a a ece a , a d e e a e c c fea e
● T ee de ca ed b b a ce e a e ed ca c d

B a ID de :
● C e a a bee e f 1 a ce de
● H a ce de a cc
● Ma de e ee de a cc , b e ed
● M de de be e acc ed f b a c cd de (e . c e a,
c affec e d de , c e f d de , de a d de )

P c a 501
Get The Step 2 CK Drills Book: usmledrills.com
The USMLE Guys Step 2 CK Crash Course usmleguys.com

B a II D de :
● C e a a bee e f 1 a ce de
● C e a a bee e f 1 a de e ee de
● Pa e a c e f a a
● M de de be e acc ed f b a c cd de (e . c e a,
c affec e d de , c e f d de , de a d de )

C c cD de :
● Ma e d a c a d ee c e a f a ce de
a d a e d f de e e c d ee e c e a f a
de e ee de
● S ec f 2c ec e ea a e be f ee f e
a af e e a d e e be f ee f e a c ec e
● S ca e f ca d e c ca a e a e ,a d
ca ed b b a ce e a e ed ca c d
● M d be e acc ed f b a c cd de (e . c e a,
c affec e d de , c e f d de , de a d de )

Treatment of Mania

Se e e Ma a T ea e :
● F - e ea e ( a a e) PLUS a a c c( a e d ,
a a e, e a e, e d e, a a e)

H a a a d M d/M de a e Ma a T ea e :
● F - e ea e e d e a a e

Ma e a ce T e a f B a D de :
● C e e a ed e e ed cce f ea e ac e a a/ a a
e de

L :
● D b f f d a f e a b d ae a de ce cc ea
c ee a e d e
● Ac e c de d a ea, a ea/
● C c c de c f ,a a , a a a, e c a
e c ab
● Se e e c de e e , e ce a a , c e
a e e c
● Se c ce a d be d a a a e ec ed f
a d a ECG b a ed
● T ea e c de ABC , e ace e f a e def c de d a ed a e
c a e, a d d a e e

Med ca I e ac :
● L e e c ea ed NSAID (e ce a ), a de d e c , ACE-I,
e da e, a d e ac c e e
● L e e dec ea ed e ea d a - a d e c e

P c a 502
Get The Step 2 CK Drills Book: usmledrills.com
The USMLE Guys Step 2 CK Crash Course usmleguys.com

Lab Te /S d e W e I a L T ea :
● B d ea e , c ea e, a , e a c e , df c de ,
ca c , a d ECG f ca d ac fac e e

Ad e e Effec :
● Ne e c d abe e d , e , a ea, d a ea, e a d f c , e
a , dd f c , e aa d ,c e a e

Content Re ie Questions:

W c f ef a a f eda cc e af b a II d de ?

A. C e a a bee e f 1 a ce de
B. C e a a bee e f 1 a ce de
C. C e a a bee e f 1 a de e ee de
D. M d e de a e be e acc ed f b a c cd de

W c f ef a a f eda cc e af a a ce de?

A. G a d a f ed e f-e ee
B. Rac f f dea
C. F da f e e a ed d a d c ea ed ac e a a
D. D ac b

W c f ef ed ca a ead a dec ea e e c ce a ?

A. Ib fe
B. Me da e
C. T e e
D. L

P c a 503
Get The Step 2 CK Drills Book: usmledrills.com
The USMLE Guys Step 2 CK Crash Course usmleguys.com

P D
Brief Ps chotic Disorder

DSM C e a:
● 1 f a c a , de ,d a ed eec , d a ed
ca a c be a
● D a f ee de a ea 1 f da b e a 1 e, e
a e e e-e de f c
● N be e e a ed b a e c a cd de , ed ca c d , b a ce e

T ea e :
● A e f e e ce f c da a d c da dea ,a d a e afe
c a c a a fa ae
● Sec d- e e a a c c ed ca (a a e, e d e)
● Se e e a a ea ed -ac be da e e
● P c ea

Schi ophrenia

P eS :
● E a ea e ce f a ce e
○ Ha c a (a d , a , fac , a c)
○ De (b a e a d -b a e): Idea f efe e ce, e a c, a d e,
a a d, c
○ D a a / eec ( a e a ,c c a a , de a e ,
e , d a ad)
○ D a a be a (ca a a)

Ne a eS :
● Lac f dec ea ed e e f a ce e
○ Def c (a a ,a a, ac fe e , f a affec , a ca )

DSM C e a:
● 2 c a ac e c ( a c a , de ,d a ed eec ,
d a ed ca a c be a , e a e ) e e d a e-
e df a f ca f e ( a be e f cce f ea ed)
● 1 a a ea f f c a ed be ba e e e e e
● S fd ba ce f 6 , a ea 1 f c a ac e c
(e f ea ed cce f ) d a e d a f e e a de f
e e
● Sc affec e d de a d dd de c c fea e a e e ca e f
ac e- a e
● S ae d ec ca ed b a b a ce e e a ed ca c d
● If e a e a a cd de a e de e e a d de , c e a
da ed f e de a c a aea e e f e
e, e cce f ea ed

T ea e :
● A e f e e ce f c da a d c da dea ,a d a e afe
c a c a a fa ae

P c a 504
Get The Step 2 CK Drills Book: usmledrills.com
The USMLE Guys Step 2 CK Crash Course usmleguys.com

● Sec d- e e a a c c ed ca e dea c ce
○ F a f a e :A a e e d e
○ F a e a a : Ha e d a a e
○ F a e a: Q e a e
● C e c e e a c c ed ca a effec e, c ea e d e
ea e c a e f a a effec e, e c a a e e d ffe e
a c c f e
● If c da dea e e ece a c c ea e , c a e d ca ed

Schi ophreniform Disorder

DSM C e a:
● Sa e c e a f c e a e ce d a f a e be ee 1 a d
6

Schi oaffective Disorder

DSM C e a:
● U e ed d a f e a dd e ee a a
a c
de e e de de add 2 c a ac e c f a c a ,
de ,d a ed eec , d a ed be a ca a c be a , e a e
, AND a ea e f e e be a c a , de ,
d a ed eec
● Ha c a a d de f 2 ee d c e ee a d
e de, c cc d e e e fe e d a f e e
● C e af a a de de a e e a d cc f f e a d a fb
e ac e a d e d a f ed de
● S ae d ec ca ed b a b a ce e ea ed ca c d

S ec f e :
● De e e e: O a de e ee de ee
● B a e: E de f a aa d b a de e ee

T ea e :
● A c c f c c ( ec d- e e a )
● A de e a f de e e e
● M d ab e f b a e( a ae )

U a Ma De e P c c Fea e :
● Pa e ee d a cc e af a a de e
● D a e de f a a de e de a c a ae e e
● De a d a c a e e cc de a e de f a de e

Delusional Disorder

DSM C e a:
● 1 de ( ) ad a 1
● C e af c e a e
● F c e ee a ed, a d be a c ea b a e, a de ee
e a ed e e ec f e de ( )

P c a 505
Get The Step 2 CK Drills Book: usmledrills.com
The USMLE Guys Step 2 CK Crash Course usmleguys.com

● If a c a de e ee de e e , e a e b ef c a ed ed a
f e de a e d
● S ( )ae be e e a ed b a e c d de , e e a ed ca
c d , b a ce e

S b e :
● Ga d e, e ec ,e a c, ea , a c, ed, ec f ed

T ea e :
● Sec d- e e a a c c (a a e a d e) a d c ea

Content Re ie Questions:

A 30- ea - d a e b e a b b e.T e a e a f e a ee
ad a c ea ed e e f e e a d e e a ed d. T e a e be e e e e e e f
bec ab a ef dea e ac df e a . He a bee ee 1-2
a f e a ee a d e e e ed a e e a ed a a e e a d ac
. He a de ae a e a e a a e a ed. W e
e ed f e , e a e be e e a a e a e e e ab df
ea a d e aec e a , c ea e e a.H
b e ae a e a e a bee a a c a ab a e f e a 6
b e c ed dc e a d e e e a c a e a .T eb e c f
a e a e a ad a c a f a ea 4 ee a a de de. W a
e e da e e e ?

A. B a Id de
B. B a II d de
C. Sc e a
D. Sc affec e d de

A 38- ea - d fe a e b e a b e e.T e a e a ad a de e ed
d e e e f f e a 4 ee . S e a ee a
ac e d e, a d ff c c ce a a d a ab e a e e, 6
d e 4 ee . S e ee f 5 a a d e d e fee f
e e a d a ac f e e . M c ce e a e e a ea a ce f a a
face efe e ee e e e e f. T e a e de e a
a c a de f e a 4 ee . W a e e da ?

A. De a d de
B. Sc e a
C. U a a de e c c fea e
D. Sc affec e d de

W c f ef a f eda cc e af c e a?

A. P e e ce f de
B. P e e ce f a c a
C. P e e ce f a de de
D. P e e ce fd a ed eec

P c a 506
Get The Step 2 CK Drills Book: usmledrills.com
The USMLE Guys Step 2 CK Crash Course usmleguys.com

P a a S D a A S D
Posttraumatic Stress Disorder

Ta a cE e A c a ed PTSD:
Se a a a , ea f/ac a ca (a a , ca acc de ), e e a
e ce (c ba /c f c /d ace e ), ed ca e ,c d d ca a d/
e a ab e

Da c C e a:
● D a f 1
● E e a a a ce e a f ef a :
○ D ec e e e ce e a a c e e
○ W e - e e a a ce e a affec e
○ D c e a a a ce e ed a c e fa e be /f e d
○ Re ea ed e e de a f a a ce e
● Re-e e e c / (a ea 1 e e ):
○ U a ed ec e a e e e ab e a a ce e
○ N ae e a a ce e
○ F a bac e a a ce e
○ Ta a c e de ca e a d e
○ Ta a c e de ca ca eac
● A da ce f a a- e a ed f e a a (a ea 1 e e ):
○ A e a dd e , e e , fee e a ed e
a a
○ A e a de e a e de f e a a ( ace , e e, e e )
● Ne a e a e a c a d d (a ea 2):
○ I ab e e be e de a f e a a
○ E ce e e a e ab ef e d
○ D ed c ead b a e f e f/ e f ca e a a
○ Pe e e a ee a ae
○ I ab e e e ce ee a ae
○ L f ee ac e
○ Fee a ed de ac ed f e
● A ea a a a d eac (a ea 2):
○ I ab e be a a e
○ Rec e e f-de c e be a
○ H e a ce
○ He e ed a e e e
○ D ff c c ce a
○ S ee d ba ce
○ S b a ce e a e ed ca c d a a ca e f e
e c ded, a d ae be e e a ed b a e c d de

D c a e S b e f PTSD:
● I add ee da c c e a f PTSD, a e a a e e
de e a a (fee d c ec ed f b d ) a d/ de ea a ( ea
a d e fa e, e e e ce d a d ea - e a e)

P c a 507
Get The Step 2 CK Drills Book: usmledrills.com
The USMLE Guys Step 2 CK Crash Course usmleguys.com

T ea e :
● F - e ea e a a-f c ed c e be a a ea ( c d
e e)
● Sec d- e ea SSRI e afa e a e a f a a-f c ed
c ea ( c d e e)
● If ee d ba ce e e ( a e ) ea a

Acute Stress Disorder

Da c C e a:
● S cc 3 da 1 af e a a c e e
● E e a a a ce e a f ef a :
○ D ec e e e ce e a a c e e
○ W e e e a a ce e a affec e
○ D c e a a a ce e ed a c e fa e be /f e d
○ Re ea ed e e de a f a a ce e
● A ea 9 f e f :
○ I a , e, ec e , d e e e f e a a ce e
○ Rec e a e /d ea c e e a a ce e
○ F a bac e e e a e fee e ee e ec
○ Ta a c e de a b e/ e e b e e a a c e e a d ca e
e e c ca d e ca eac
○ Pe e ab e e e ce ee a ae
○ A e ed e ce f ea ( e f, e d)
○ L f e e a ed a a a f e a a ce e
○ A e a d , fee , e e a aea c a ed e
a a ce e
○ A e a d e de a b e e ca e d e ,
fee , e e a c a ed e a a ce e
○ S ee d ff c e
○ I ab ,a e
○ H e a ce
○ L f c ce a
○ He e ed a e e e
○ S b a ce e a e ed ca c d a a ca e f e
e c ded, a d ae be e e a ed b a e c d de

T ea e :
● F - e ea e a a-f c ed c e-be a a ea

P c a 508
Get The Step 2 CK Drills Book: usmledrills.com
The USMLE Guys Step 2 CK Crash Course usmleguys.com

Content Re ie Questions:

A a e de c be d ec e e e c e e dea fac ef e d a e ce
acc de . S ce e e e cc ed 20 da a , e a e e d e ed e
e e f e dea , f a bac f e ca c a , a d a e e a bee ab e
e e e ce a a e c e e , e e b ef , ce e e e . Be a
e c e ca e ea ace a d e b ea ac d ea . A e a ec e
ae c e e c a a d e e d e d ff c fa a ee a d a a ee .
A d e ca e a d a e f d a d ef d ef a ae f
e a ce, ab e ea f e e a e , c a ed a f c ce a a
b. W a e e da e e e ?

A. P a a c e d de
B. Ac e e d de
C. Pa c d de
D. Ma de e ed de

W a ed ca ed ea ae a e PTSD?

A. D e da e
B. P a
C. B
D. Na ca

W c f ef a f eda cc e af a a c e d de ?

A. N ae e a a ce e
B. F a bac e a a ce e
C. D ff c c ce a
D. Rac

P c a 509
Get The Step 2 CK Drills Book: usmledrills.com
The USMLE Guys Step 2 CK Crash Course usmleguys.com

A D H a D a Ob -C
D
Attention Deficit H peractivit Disorder

DSM C e a:
● P e e ce f c e a f e e a e a d/ e ac -
● I a e e ac e- e ee bef e a e 12
● I a e e ac e- e e e e a 1 e
● I a e e ac e- e e fe e a e f c
● S be e e a ed b a e c d de , b a ce, e ea
ed ca c d

I a e C e a:
● 6 e f 6 e (5 f 17 a d de )
○ Lac a e de a d ac e
○ D ff c a f c ed ac e
○ Lac fa e e e d ec
○ Lac ff c a df a
○ D ff c a a
○ Re c a ce d e a c a a a ece ae a ed e a
eff
○ L e ac f e eeded f ac e /a
○ Ea d ac ed
○ F ef

H e ac -I C e a:
● 6 e f 6 e (5 f 17 a d de )
○ Fe e fd e ea , a a d fee
○ Fe e ea ea e e a ea ed e
○ R ,c b , e e e e a ae
○ Fe e ca a c ae e e ac e e
○ Ac a f be d e b a a dfe e e be a
○ E ce e a
○ B a e c ee e e e ce
○ D ff c a e
○ I e e c e a ac e

T ea e :
● Ge e a , f e ea e f ADHD a ea e (de e e da e,
e e da e, de a ea e)
● F e ea e f ADHD a da fa b a ce e d de a e e
● F e ea e f ADHD a d c - cc de e b
● F e ea e f ADHD a d c - cc ca e e a ed a e d de
SSRI, e ce a e aec ed a a ea e ea e
● C e be a a ea f a e a d a ac e a

Obsessive-Compulsive Disorder

DSM C e a:
● P e e ce f c e a f e e b e ,c b

P c a 510
Get The Step 2 CK Drills Book: usmledrills.com
The USMLE Guys Step 2 CK Crash Course usmleguys.com

● T e b e c cc 1 e e da ca ed e
a e ff c
● N d e a e c d de , b a ce, ed ca c d

Ob e :
● Rec e , e e , e, a d a ed , a e , e a ca e
a e d e
● T e a e e e e e a e e e a ed , a e ,
e b ef ac e e

C :
● Re e e ca e a be a a e a e fee c e ed ef
e e a b e d e e f- ed e a be d f ed
● T e ca e a be a ae ef ed ed ce e e a e d e
a e e a ee e a f cc . T e e be a ae
ea ca c ec ed a e a e a e e , f e ae
c ec ed e ac ae b e ce e

T ea e :
● F - e ea e f OCD a SSRI ed ca a d/ c e-be a a ea
e ea d e e e e

Content Re ie Questions:

A e e e e a ac f c a a a e fee c e ed ef e e a
e f- ed e f e a be d f ed de ed ce a e
c e a?

A. Reac f a
B. P ec
C. Ob e
D. C

A 18- ea - d a e ee b a c a a d e a e da fa e def c
e ac d de . T e a e a e f d be ab be da e e a a e
16 b a bee be f e 1 ea . W a a ac e a a ae ea
a e f e da ed ADHD?

A. Me e da e
B. B
C. A e e
D. Se a e

P c a 511
Get The Step 2 CK Drills Book: usmledrills.com
The USMLE Guys Step 2 CK Crash Course usmleguys.com

A 33- ea - d fe a e e e e c a ac a fa e .S e ae a f
e a ea e a bee e e e c ea d a ed fc aec a e
de e a e.S e a f d e e f e ea a a e e a d e e e
ead e e e a ed .S e a f d a e e d 2-3
ec e a e a e e a d e da , a e a c ec e a ba e d e f e a
da a e e edd a a a ec c ab e . W a e e da ?

A. Pa cd de
B. Ge e a ed a e d de
C. Ob e e-c ed de
D. Ob e e-c e e a d de

P c a 512
Get The Step 2 CK Drills Book: usmledrills.com
The USMLE Guys Step 2 CK Crash Course usmleguys.com

Ea D a B D D
Bulimia Nervosa

DSM C e a:
● C a ab a a ea ff d ad c e d f e, d c
e e a e e d e be ab e c e ea
● I de e e e a , a aec e a be a ef ed
● T e-f a e f b e ea a d a aec e a be a cc a ea
ce a ee f ee , a ea e
● Pa e e f- e ce f c ed e b d a ea d e
● C d d e cc e c e d e de f a e a e a

T ea e :
● N a e ab a
● C e be a a ea / e e a c ea
● F e ea d f a ae e e, e a e
● B c a d ca ed

Anore ia Nervosa

DSM C e a:
● Pa e e c ca c a e e a e e e e , ca a f ca b d
e
● Fea f bec fa a e e e be a a e e e a ,
e e a a f ca e
● Pe a e ac f ac ed e e f e ed ca a f ec e b d e
a ae e f e' b d e a e e e e ced, d e
f e ce f b d e a e e f-e a a

Re c T e:
● We e e f fa ,de , a d/ e ce ee ec e
● I e 3 , e a e a a c a ed b e ea
be a

B e Ea a dP T e:
● I e 3 , e a e a a c a ed b e ea be a

T ea e :
● N a e ab a
● P c ea
● Oa a e
● M f efeed d e
● B c a d ca ed

Refeeding S ndrome

Ca e:
● Ma ed a e ece e e a e ab a ca ca ef da d
e ec e f ead ed ca c ca c a :
○ Lab ab a e :H a e a, a e a, a e e a

P c a 513
Get The Step 2 CK Drills Book: usmledrills.com
The USMLE Guys Step 2 CK Crash Course usmleguys.com

○ Ca d ac c ca :C e e ea fa e, a a , e e ,
e
○ P a c ca : Re a ff c e c d e dec ea ed
da a a cc ac
○ MSK c ca : R abd , ea e , e a , a a
○ Ne c ca : Pa e e a, e , de ,a d e e

Pe e :
● Re e 1400 1600 ca /da , e ec e a d
a e, ee e ea a ab a

T ea e :
● Red ce a a ea d a d c ec ae a, ae a, a d/
a e e a

Binge Eating Disorder

DSM C e a:
● E de f b e ea ee a e de c be be ab e c e ea
● O a e a e, e de a e cc a ea ce a ee f ee
● T e e e a e f a ae e be a e , e ce e
e e c e, b e e fa a a e ee b a e a
● T eb e ea d e cc e e a e ee c e a f b a e a
a e a e a
● A ea 3 f e f be a a e ee : Ea a d ; ea c f ab
f ; ea a ea ff d e ; ea d e e ba a e
e ad a ea f f d; fee fd , de e , af e e ea

T ea e :
● F - e ea e c e-be a a ea
● If CBT a a ab e be a a e e a , SSRI , a a e,
de a fe a e, a de a be ed
● SSRI a e f - e ad c e ed ca f a e a a e e CBT

Bod D smorphic Disorder

DSM C e a:
● P e cc a e ce ed f a ( ) ca a ea a ce a e ca ee
a ea a fa
● T e a e a ef ed be a e a ac e ea ed beca e f e c ce
f e e ce ed defec
● T e e cc a ca e d e a e f c
● T e e cc a e a ed b d fa e a a e ee e
da c c e a f a ea d de

T ea e :
● M d- de a e ca e : F - e ea e SSRI c e-be a a ea
● Se e e ca e : F - e ea e SSRI c e-be a a ea

P c a 514
Get The Step 2 CK Drills Book: usmledrills.com
The USMLE Guys Step 2 CK Crash Course usmleguys.com

Content Re ie Questions:

A 17- ea - d fe ae b ec cb e ae a c ef c a f e .
T e a e a e a 30 d e a 4 a d a a BMI f 17.6. W e
e e e a e a e ee d e e c ca c a eb a a e c
e e da a c .A e e e d e e f- d ced af e ea e ae .
W e f ed f e BMI, e a e a e be a a e 15 d a d
c e fee e e a c fa e abd e a d .W a e e
da ?

A. B d d cd de
B. B e ea d de
C. A e a e a
D. B a e a

A 20- ea - d a e e ec c ac a f ca f defec . T e a e ae a
a e c e e ab e ca f c e . He e d e ed b
e ea d ae e ea a c e e b ca da d e e ee e
a be ee c a a e e beac . He e d e e e e
a da ee ca f a ea e .O e a e ca e a ea ca a
e a d e e a ac . W a e e da ?

A. Na c c e a d de
B. H c e a d de
C. A da e a d de
D. B d d cd de

W c f ef ab ab a e c ee efeed d e?

A. H ae a
B. H e ae a
C. H e ca ce a
D. H e a e e a

P c a 515
Get The Step 2 CK Drills Book: usmledrills.com
The USMLE Guys Step 2 CK Crash Course usmleguys.com

D a a S a D
Dissociative Identit Disorder

DSM C e a:
● D c a dd f de
● E e ce f >1 d c e a a e ca ad f de , c e
d c b e e f efa d e e fa e c
● T ed c a dd f de e a ea c c e ,
e , e ce ,c , affec , be a , a d/ e - f c
● T e e f >1 d c e a a e ca be e f- e ed b e a e
b e ed b e
● Pa e e ea ed a a eca f e a f a , e e da e e , a d/
a a ce e
● S ca e d e a e f c
● T e ae de acce ed a a c a e ac ce
● S ae d e a e c d de , b a ce, ed ca c d

T ea e :
● P c ea

Dissociative Amnesia

DSM C e a:
● I a f a ab e e f ca be eca ed, ca fa e f
a a cc e , a c e d a f e
● S ca e d e a e f c
● S ae d e a e c d de , b a ce, ed ca c d

S ec f e :
● D ca ef e
○ U a ed b e e ee ef a e, c f ed a de
a c a ed e f de ef

T ea e :
● R e- ed ca ca e fa e a
● P c ea

Depersonali ation/Dereali ation Disorder

DSM C e a:
● Pa e e e b de e a a a d/ de ea a
○ De e a a e a e fee a e ae b e e
, fee , ac , e a a a de b e e OR e e e ce
a e e f ea de ac e e a ed e ef
○ De ea a e a e fee a e e f ea de ac e a
e a e d
● Rea e e a ac d e de f de e a a a d/ de ea a
● S ca e d e a e f c
● S ae d e a e c d de , b a ce, ed ca c d

P c a 516
Get The Step 2 CK Drills Book: usmledrills.com
The USMLE Guys Step 2 CK Crash Course usmleguys.com

T ea e :
● F - e ea e c e-be a a ea f a e
de e a a /de ea a d de
● F a e c - cc de e a a e d de , f - e ea e
c e-be a a ea SSRI
● F a e ed de f ab e de c f c ,f - e ea e
c d a c c ea

Somatic S mptom Disorder

DSM C e a:
● P e e ce f e e a c c e d e c ca
a e
● D a e fee , , be a ea e a c ,
a e e f ef :
○ Pa e f e e ab e e e f e
○ Pa e f e e e e a e ab e
○ Pa e f e e e e a e ab e ea ea
○ E ce e e e c ce e f e ea
ea
● S e a f e a ( a c a e e
e)

T ea e :
● Re a a e a e a ca e c ca c e ac e

● T ea fa e d de de e ed de a de e a

Illness An iet Disorder

DSM C e a:
● C ce a d ac a a e e
● Lac f a c e d a c
● If a ed ca c d e e OR e a e a a fa f a ed ca
c d , e c ce a d de ab e ce e d ae
● T e a e ea a a ed ab e e a ea a , a e e f
a e d e ea
● T e a e a a a ada e a da ce f ea ca e e ef e ce e
ea - e a ed be a
● S a e bee e e f a ea 6 ,b ab c ec f c
e a c a e e e
● N be e e a ed b a e c a cd de

T ea e :
● Re a a e a e a ca e c c a
● C e-be a a ea f e e e e a a e , a
SSRI f a e d e a e c e-be a a ea

P c a 517
Get The Step 2 CK Drills Book: usmledrills.com
The USMLE Guys Step 2 CK Crash Course usmleguys.com

Conversion Disorder

DSM C e a:
● 1 f ab a a e f c
● Ne ca a fe a ae c a be a ed ca c d
● S ca e d e a e f c
● S ( )ae be e e a ed b a e c d de e ea ed ca
c d

T ea e :
● Ed ca ab c e d de
● F a e de a d a d acce eda fc e d de b
d e d ed ca a e, ca e a f a d
c e-be a a ea f e a ae

Factitious Disorder

DSM C e a:
● Ide f ed dece ee a a e fe d ce ca c ca
f d ea e
● T e a e de f e e e e a c ed e
● E e a e ad ae e e e a f e dece e be a
● N be e e a ed b a e c a cd de

T ea e :
● Sa e d d a c c a d e ee ea e
● P c a c ed
● A e be f ca e ea f ed f e fac d de d a
● A e c de a d ace a e c a b e a e e e f-
be a
● Ge e ed ca d de a e ea ed a a e a d ba ed b ec e c ca
f d
● P c ea a be a e ed, b a f a e ef e f
c a c ea e

Malingering

E a ea e a fe e de ece e a e e a be ef .

P c a 518
Get The Step 2 CK Drills Book: usmledrills.com
The USMLE Guys Step 2 CK Crash Course usmleguys.com

Content Re ie Questions:

A 34- ea - d a e e ec c ac a f e .T e a e ae
e e e f a d ca beca e f e e . Ne ca
e de f e ed b f b e a d d e f a e e
d b a d ab a ab a e a e ee f e e a a .W a e
e da ba ed e ef d ?

A. I e a e d de
B. S a c d de
C. Fac d de
D. C e d de

A 41- ea - d a e be e a a ed a f c a ca e .T e a e ae
a e a a fee a e de ac ed f efa d a e e e e ac
e a a d fee a a de . W e a ed f e be e e e ac ef
a a b e e e ae a e de a d e eae a d fee a d e
a b e e, e a e e ce e f. He e b e a
a d a e fee e d e f. W a e e da e e e ?

A. De e a a /de ea a d de
B. I e a e d de
C. D c a e de d de
D. Sc e a

A 42- ea - d a e a e ee e a f a ef a d. He b
e e e e c de a e b a c e.T e a e ca a e ca a d a ece
f ac e a . W e ea e , ec e f a de a a
e a e e a a ed a d a a e e e c e a a d
de a de e e c ea e a e e f- f c e .W a e
e da be a c e e ef d ?

A. Fac d de
B. C e d de
C. Ma e
D. S a c d de

P c a 519
Get The Step 2 CK Drills Book: usmledrills.com
The USMLE Guys Step 2 CK Crash Course usmleguys.com

A S D a C D
Autism Spectrum Disorder

DSM C e a:
● Def c c a -e a ec c
● Def c e ba c ca e be a ed f ca e ac
● D ff c de e , a a , a d de a d ea
● 2 f ef e c ed a d e e e be a , e e , ac e :
○ S eec , e e , e f b ec a e e e ed e e e
○ S ad e e ce e , e ce a e e , a ed e ba
e ba a e f be a
○ H e c ed, f a ed e e a a e e ce e a d/ e a e
○ A e ed e e e a a e e e e a ec f
ee e
● Add a :
○ S ca e a ed f c
○ S ae e e e ea de e e a e d
○ S ae d e e ec a d ab ba de e e a de a

T ea e :
● Be a a a d ed ca a e e

Conduct Disorder

DSM C e a:
● I e a 12 ,a e e a e f be a e e ba c f e
a a e-a ae cea e ae a ed, e e ce f 3 f e e
15 c e a a d a ea ec e a e e e a 6 :
○ T ea e , b e , da e e
○ Sa ca f
○ Ha ed a ea ca ab e f e ca a e
○ P ca c e e e
○ P ca c e a a
○ Ha e b c f a c
○ C ed e a a a
○ Se f e e e f ca e da a e
○ De ed e e ( f e)
○ B e a c e, b d , ca
○ Le a fa , e , a d b a
○ S ea e f a e c f a c
○ Be bef e a e 13, f e a a de e ae a b
○ Ha a a f e e a ea ce e e ae
a da e, ce e f a e e ded e
● S ca e a e f c
● If e a e 18, c e a e f a ca e a d de

T ea e :
● P c ea (c e-be a a e a , fa e a , a d/ ee ea )

P c a 520
Get The Step 2 CK Drills Book: usmledrills.com
The USMLE Guys Step 2 CK Crash Course usmleguys.com

Content Re ie Questions:

A 16- ea - d a e b e c a e a ae e be a .O e e a
ea e a e 3 ca f a d a e e ed f ba c e eb
c e a f . He a bee a e ed f ea a e f
a e a f eb a dc e a a f e f da a a e
be f d. La ee e a c a ed a f e a e f e e e fa
f e ca ae e e e e f be e e ed. G e e e , a e
e da ?

A. C d c d de
B. A ca e a d de
C. P a a
D. Ma e

W c f ef ae c e ada fa ec
d de ?

A. Def c c a -e a ec c
B. D c f a c e ae e ce e f a e
C. Def c e ba c ca e be a ed f ca e ac
D. D ff c de e , a a , a d de a d ea

W c f ef ae c e ada fa ec
d de ?

A. U a a e e e e a ec f ee e
B. S d e f a e e a d ad e e ce e
C. Re e e eec a d e e
D. H e c ed, f a ed e e

P c a 521
Get The Step 2 CK Drills Book: usmledrills.com
The USMLE Guys Step 2 CK Crash Course usmleguys.com

D M a a P a D
P e Defe e Mec a :
● Ac :C b ef a ac c de ed bad a - c a
● C e :P ca a fe c e ea a e e
● De a : Ac a ea a /e e /ac d e e
● D a a:D fee a a e c f ab e, ef
ac ed e e /fee
● P ec : Ta e d d a e a fee ( ade ac , fa ) ab
e e e a d ac e e( )
● Re e :A a a a e a ea e de e e a a e
● Sc d fa a : A d a b a ae e a d
● S :C a e a e a 100% d bad
● U d : Ac b ca a e a acce ab e e e /ac e a a
ca ce d e e ea a

H e -Le e Defe e Mec a :


● A : Ma a ea a e e ce e
● D ace e : T a fe fee f e ac a de ed a e a afe b e
a e
● H :F d ec ca cc e fa ea a a e
e a d c f
● I e ec a a :U e ce e ca d a a a a d ea a fee
● Ra a a :J f be a b be e a bee a a a a e fa e
● Reac f a : Ac e e a e e fee
● Re e :U c c b c f ea a / e e f ec c
d
● S b a :C a e acce ab e e acce ab e be a
● S e :C c dec d ac ed e a d fee ea a
e /

Pe a D de :
● Ac e a e f e e e e ce a d be a a d e e f ca f
e e ec a f e d d a c e. T e d e e e e e ce a d be a ae
f ed a d e a e ac a a , ead f ca d e a d a e
a d a e bee e e ce ad e ce ce 2 f ef a ea affec ed:
○ C
○ Affec
○ I e e a f c
○ I ec

Pa a d Pe a D de :
● C e A e a d de : Odd a d ecce c
● A e e d / c e f e , e ce e e be ae e ,
4 f ef :
○ S c a e aee , a , dece e
○ P e cc a f ed d b ab e f f e d /a cae
○ Re c a ce ae f a e f fea f be ed a a e
○ Read dde ea e / ae e ea f be e a e e
○ C e bea d e
○ F d e ce ed a ac e c a ac e a d eac a

P c a 522
Get The Step 2 CK Drills Book: usmledrills.com
The USMLE Guys Step 2 CK Crash Course usmleguys.com

○ Rec e a a ed c ab f de f e a a e
● D e cc e c e d a e c cd de a d a b ab e
a e ed ca c d

Sc d Pe a D de :
● C e A e a d de : Odd a d ecce c
● A de ead a e f de ac e f ca ea a d e c ed a e f
e e e a e ac 4 f ef :
○ N de e e e fc e ea , c d a a f a fa
○ Nea a a c a ac e
○ Ha ed ee e a e e e ce e
○ Ta e ea e fe / ac e
○ N c ef e d e a 1 de ee e a e
○ U c ce ed/ affec ed b a e c c
○ F a e ed affec , e a c d e , de ac ed de ea
● D e cc e c e d a e c cd de a d a b ab e
a e ed ca c d

Sc a Pe a D de :
● C e A e a d de : Odd a d ecce c
● Ac e a e f ca a d e e a c d c f a d
ed ced ca ac f c e e a a e a c e e ce a d
a d ecce c e f be a 5 f ef :
○ Idea f efe e ce
○ Ma ca / dd be ef a d c e c a
○ U a e ce a e e e ce
○ Odd a d eec
○ S c e a a d dea
○ I a ae c c ed affec
○ Be a a ea a ce c dd ecce c
○ Lac f c e f e d a de f 1 de ee e a e
○ E ce e ca a e c d e d fa a a d
a c a ed a a d fea a e a e f- d e
● D e cc e c e d a e c cd de a d a b ab e
a e ed ca c d

A c a Pe a D de :
● C e B e a d de : D a a c, e e e e a, e a c
be a
● F e a e f 15 ea , a e e a e f a a dd e ad e f
e a e de ced b 3 f e f :
○ Re ea ed c c e c ae d f a e (fa e c
a f be a )
○ Dece f e f ff e f ea e
○ I fa e a a ead
○ I ab a da e e e
○ D e a df afe f ef e
○ C e e b ef a d f a ca b a
○ Lac f e ef a ea e , fe b a a
be d ffe e e e a /d c f

P c a 523
Get The Step 2 CK Drills Book: usmledrills.com
The USMLE Guys Step 2 CK Crash Course usmleguys.com

● T e a e be a ea 18 ea da d ee d be e de ce f c d c
d de e bef e 15 ea d
● T e be a a e c e cc ec e f c e a b a
d de

B de e Pe a D de :
● C e B e a d de : D a a c, e e e e a, e a c
be a
● C e a e f a f e e a ea , e f- a e, a d
a a e f a 5 f ef :
○ De e a e eff a d ea a ed aba d e
○ U ab e a d e e e e e a ea c a ac e ed b a e
be ee dea a a d de a a
○ S f ca a d e e ab e e f- a e
○ I 2 e a ea a ae e a e f-da a
○ Rec e c da be a , ea , e f- a
○ Ma ed eac f d ead ab f a e affec
○ Pe e fee fe e
○ I a a e, e e a e , d ff c a a a e
○ Ta e , e - e a ed a a d dea , e eed ca e

H c Pe a D de :
● C e B e a d de : D a a c, e e e e a, e a c
be a
● Ac e a e fe e ee a aea da e ee e e 5 f
ef a fe a :
○ D c f e e ee e a e e ce e f a e
○ I a ae e a ed c e ca e be a e
○ D a a d f e e fe
○ C e e f ca a ea a ce a ac a e
○ S eec c e ce e f c ed a d ac de a
○ T ea ca a d e a e a ed e e fe
○ Ea f e ced b e
○ C de e a be e ae a e d ac ed e

Na c c Pe a D de :
● C e B e a d de : D a a c, e e e e a, e a c
be a
● C e a d e a e a e f a d , eed f ad a , a d ac f
e a a fe a5 f ef :
○ G a d e e e f e f- a ce
○ F c ed fa a e f ed bea , dea e, cce , e, b a
e ec
○ Be ef a e ae e a d ca be de db a c a ed
- a d d a
○ Re e e ce e ad a
○ S e e fe e e
○ E e f ef a
○ Lac fe a
○ E f e be e e a e aee f e
○ D a a a be a a da de

P c a 524
Get The Step 2 CK Drills Book: usmledrills.com
The USMLE Guys Step 2 CK Crash Course usmleguys.com

A da Pe a D de :
● C e C e a d de : A , fea f be a
● Pa e f e e e a ee a a , ca b , a d fee f
ade ac 4 f ef :
○ A d cc a a de a a dea f ca e e a c ac
d e fea fc c ,d a a, e ec
○ W a d e ed e e e ce a f be ed
○ H d bac ae ea beca e f fea f be a ed d c ed
○ Pe e ab be e ec ed c c ed ca a
○ Fee f ade ac b a c a e e e a a
○ Pa e ee e efa ca e , e a a ea , fe
e
○ He a ce a e e a e a e a e ac e beca e e
a e e ba a

De e de Pe a D de :
● C e C e a d de : A , fea f be a
● A e ce e eed be a e ca e f ed a f fe a ead
b ea dc be a a e a fea f e a a a fe a 5 f e
f :
○ E e da dec a e d ff c e ce ea f ad ce a d
ea a ce f e
○ Re e e a e e b f a a ea f e fe
○ Ha a a d e e ba d a ee e e d e fea f f
a a
○ Ha a a d ed e a e ec
○ T e a e d e ae e a e ba a ce a d f e ,
e e ee d a ae ea a
○ Pa e fee d c f e e e a e d e e a e a ed fea f
be ab e ca e f e e e
○ W e ac e ea e d , a e e ee a e ea a a
ce f ca e a d
○ Pa e ea ca af a d f be ef a e ca e f e e f

Ob e e-C e Pe a D de :
● C e C e a d de : A , fea f be a
● Ac e b e de e , e fec ,a e a e a a d
e e a c a e e e e f f e b , e e , a d eff c e c a fe a
4 f ef :
○ P e cc ed de a , e , , de , a a , c ed e c
a e a f e ac ed
○ Pe fec c de a / e e a c e
○ E ce e de a d d c e c de a e e ac e a d
f e d
○ I ec ce a d fe be e a d a , e ca , a e-ba ed
dec
○ I ab e d f - e b ec (e e e
e e a a e)
○ Re c a ce de e a e a e e e b e a
fd
○ M e a ded f f e ca a e , e d ed (f efa d e )

P c a 525
Get The Step 2 CK Drills Book: usmledrills.com
The USMLE Guys Step 2 CK Crash Course usmleguys.com

○ R d a d bb e e ac a d dec

T ea e f Pe a D de :
● F - e ea e f e a d de c ea , b e ad c e,
-f c ed a ac e a a a be c de ed
● D a ec ca be a a ea fe ed ea e f b de e e a d de

Content Re ie Questions:

A a e c ce ed e e a d ff c e ac e d d a a d
ac e e fee f ade ac ab e ef a e d d a a dca
e a a d e ac e de ae a e f defe e ec a ?

A. P ec
B. De a
C. D a a
D. Ac

A 22- ea - d fe a e a e be e a a ed a a c a c ff ce de a e c e
c ce . S e e d a e -b f e d ace f e e e a ee
e e a e a e e e . S e f c a e be ee de c b e e -b f e d a a
e fec e a da fe d e de e e e a e .D e ed e a e f
e ee , e a e e d e e f- a , a e ea a c e a a d
. S e e d e d ff c a a a e a a e c ed e a d a efa
e ce e beca e e ee a e a e. S e e d e c ea ed
a , ee ee d e e ef e a e e e' c e ,
a e a e e a fe e e e beca e e e e b e a
e ce ed a d c ed e d f e. W a e a d de c e
f de c ?

A. A ca e a d de
B. Pa a d e a d de
C. B de e e a d de
D. De e de e a d de

A 36- ea - d a e a e be e a a ed a a c a c ff ce de a c e
c ce . T e a e ae a e be b e a d a ee e
e d a e ed e a d d c e a e d e . He a e a cce
d e b a e ec a d a e a e e f e ea e be c de ed e f e
ea e e e e e fa e. T e a e b a ab a ad a a e f e ee
a d c e , a d b a ab f a d d a af e ea a e ad c a ed a
e. T e a e e e , e a e bec e a ed e e
e ce e ad a f a ed acc e a d acc e e c a f be
e f cce . W a e a d de c e f de c ?

A. H c e a d de
B. Na c c e a d de
C. B de e e a d de
D. A ca e a d de

P c a 526
Get The Step 2 CK Drills Book: usmledrills.com
The USMLE Guys Step 2 CK Crash Course usmleguys.com

A /S b a U D
Alcohol Use Disorder

DSM C e a:
● Ac e ca a e d e a ea 2 f e 11 f
e e e a 12 :
○ Ac c ed f a e e d a e a a a e ded
○ Pa e cce f a e e de e c d ac e
○ La e a f e e ba , , a d/ ec e f ac
○ Ca , e, de e c eac
○ Re ea ed a c e e ab f f a e b a
d ffe e d a ( c , , e)
○ C eac e e ec e ca e e a be
ca ed e ed b e effec fac
○ I a ac e a e aba d ed ed ced beca e f a c e
○ C ed, ec e e fac ce a eed ca
da e
○ C ed a c ee e ed e f a a ca c ca
be e a e bee ca ed e ed b a c
○ T e e e ce f a c e a ce c ca a fe a e e a eed f e
a fac ac e e e a e effec a a dec ea ed ca
effec c ed e f e a e a fac
○ T e e e ce f a c da a c ca a fe a e e c a ac e c
ac da a d e ac ea e f a c da a
(be d a e e) ed ea a d da a

Alcohol Withdrawal

DSM C e a:
● Red c ce a ac e c a bee c a d ed
● W da af e ce a ed c fac a e, 2 /
be a e ee c ca e d e a e ff c
○ Na ea/ , a, a e ,a c e ac , c ea ed a d
e , a c a , c a a , e e a ed c-c c
e e
● S ae ca ed b a e ed ca c d , e a d de , b a ce

M W d a a:
● 6 36 f a d
● M da e , a a ,da e , e e , a e a e

Se e :
● 6 48 f a d
● S e e e a ed c-c c e e( ) ca e d

Ac c Ha c :
● 12 48 f a d
● N a a a c a a d ac e a

P c a 527
Get The Step 2 CK Drills Book: usmledrills.com
The USMLE Guys Step 2 CK Crash Course usmleguys.com

De Te e :
● 48 96 f a d
● Tac ca d a, e e , fe e , d a e , de ,a a

T ea e :
● F - e ea e be da e e (d a e a a e a )
● If a e ef ac -d e be da e e e e ba b a
● I ba e a e e c ca ece a
● If a e ef ac -d e be d a e e a d e ba b a ea f
● P de e ca e

Substance Use Disorder

DSM C e a:
● Sa e d a cc e aa ac ed de , b a ce a ed f ac
e de c b / / a e f be a

A A "B S "

S /S fI ca :
● A e , ac ca d a, e e a, e e ,da e ,a a , e
be a , d a , e e , a c a , aa a

T ea e :
● Be da e e a d e ca e

S /S fI ca :
● M d a , a, a e a, c ea ed e e ,e a

C ca :
● C a a e a c c , e e , e e a, b f a ,
d ea, c a a , c e cb e

T ea e :
● Be da e e f a a a d e e
● Be a b c e c a d ca ed f e e
● A e f a d ea c ca e-a c a ed ca d a c e aa d ca d a
c e a, b d e be a b c e

S /S fI ca :
● C c a ec ,d , c ea ed a e e, ed eec , a a a, c ea ed
e a a e, e e a c e

T ea e :
● C de ce a e e de e ae e e ca e
● Ad d aced a d ed a a d
ded ea a ce f c ba e ca be e be da e e

P c a 528
Get The Step 2 CK Drills Book: usmledrills.com
The USMLE Guys Step 2 CK Crash Course usmleguys.com

S /S fI ca :
● Dec ea ed e a a e, da e, ea a e, b d e e, a d b e d ,
a e a eda a d

T ea e :
● Ve a e a e
● G e a e, ae e a

P (PCP)

S /S fI ca :
● P c a a , a c a , d ec a a , e e ,
ac ca d a

C ca :
● Ta a c e , abd , e e , e e a

T ea e :
● P ca e a a d be da e e e ba e e

LSD

S /S fI ca :
● Se e ce a e ed/ e e ed, e a, b e d f e e ( e e a),
d ed e e f e, e e f be de e' b d , e e f e a e e
● Pa c, fea , d a, e e f d ead, d b a e
● N a a d e e a d ac ca d a

T ea e :
● P ace a ca , - a e e
● Be da e e f a a a d c c fea e
● Ha e d e f a e c fa e d be da e e

Content Re ie Questions:

H a af e a a e c e a ac c be e a e a e f de
e e e cc a a e e e e c ac d a a?

A. 6 36
B. 6 48
C. 12 48
D. 48 96

P c a 529
Get The Step 2 CK Drills Book: usmledrills.com
The USMLE Guys Step 2 CK Crash Course usmleguys.com

I ca c f ef ec ea a b a ce e d ce
c a e a, e e a, d ed e e f e, e e fe a e e a
e a a b e e e f d ead d b a e ?

A. C ca e
B. LSD
C. O d
D. Ma a a

A 65- ea - d a e a a ed ca fac ab e e e e a 48
ce a ac c be e a e. T e a e a a e HR 118 BP 160/95 T 101.0
SPO2 97%. T e a e da e c e a a de e e a a ed ab
d d a a ac .W c f ef e be a a ac e a f
a e ?

A. Ha e d
B. D a e a
C. Labe a
D. C c be a e

P c a 530
Get The Step 2 CK Drills Book: usmledrills.com
The USMLE Guys Step 2 CK Crash Course usmleguys.com

S D
Central Disorders of H persomnolence: Narcoleps

S :
● Ca a e , da e ee e , a c a c a , ee aa

Lab :
● Oe -A c ce a dec ea ed CSF

S de :
● M e ee a e c e (MSLT) ea ee ae c f 8 e a d
e ee e REM ee e d

Da :
● Da e a e ee da e e b e fee e eed ee cc 3

● L Oe -A c ce a e CSF a d/ ca a e a ea ee ae c f
8 e a d 2 ee e REM ee e d (SOREMP ) MSLT

T ea e :
● Be a a c a e c a e a ee c ed e f ff c e d a , c ed ed
da e a ,a d ed / b a ce a d ce da e ee e a
● T ea ca a e REM ee - e ed ca c a SNRI ( e afa e,
a e e) SSRI (f e e)
● T ea e e e da e ee e daf

Central Disorders of H persomnolence: Idiopathic H persomnia

S :
● Da e eed ee da e fa a ee , c ce e e da e ee e ,
b e d ff c a a e f ee

Da :
● 3 f da e d f eed ee da e fa a ee
● N ca a e
● MSLT e a 2 SOREMP
● E e MSLT a ea ee a e c f 8 e OR e ed a
24- ee e 660 e
● O e ee , ed ca , a d c a cd de a d d e ed a a ca e
a b a ce/ ed ca ca e

T ea e :
● F - e ea e daf , a a c de a daf e e da e

Central Disorders of H persomnolence: Kleine-Levin S ndrome

Da c C e a:
● 2e de f e ce e ee e a d ee d a , e de d a f
be ee 2 da a d 5 ee cc a ea ce e e 18

P c a 531
Get The Step 2 CK Drills Book: usmledrills.com
The USMLE Guys Step 2 CK Crash Course usmleguys.com

● Pa e a a ba e e a e e fae e ,c ef c , be a ,a d d
be ee e de
● D e de 1 f e f e e :C f , a e ed e ce ,
e e a , a e a, e a a
● O e ee , ed ca a d c a cd de a d d e ed a a ca e
a b a ce/ ed ca ca e

T ea e :
● S e a d ed ca a ea e , a f e fe e
e de , a d IV e d f a e e de a e 30 da

C cad a R D de :
● S ee - a e d c e e c c ec e
● Ca ed b c a e e c cad a ee - a e c c e a d e a e
ee - a e c ed e ca a a d/ e ce e ee e
● Re a e d e

Circadian Rh thm Disorders: Jet Lag

Da :
● I a a d/ e ce e ee e da f a e a d ffe e e e

T ea e
● S a e ca - ed e ea d ea e

Circadian Rh thm Disorders: Shift Work Sleep Disorder

Da :
● W c ed e e a a ee e ca e a a d/ e ce e
ee e a ed c a e e
● S e e f 3 a d ca e d e a e f c
● S ee a d ac a d bed ee - a e a e f 14 da
● O e ee , ed ca , a d c a cd de a d d e ed a a ca e
a b a ce/ ed ca ca e

T ea e :
● Be a a d f ca e ee c ed , d ee e e
● P a ac e a -ac be da e e a d ea

Circadian Rh thm Disorders: Advanced Sleep-Wake Phase Disorder

Da :
● Q a a dd a f ee a e a ae
● Ea e a de ed ee e a d a e e e d e a d da e
ee e
● C f ed ee ac a

T ea e :
● L ea ee e
● N a ac ea

P c a 532
Get The Step 2 CK Drills Book: usmledrills.com
The USMLE Guys Step 2 CK Crash Course usmleguys.com

Parasomnias: Sleepwalking

Fea e :
● U a cc f d f ee d a
● Le e a d e a b a c a ea a -d ec ed
● Occ a e ee , c de a e a ac ee
a
● Pa e a a e a d ffe e ca
● If a e d e e a e ca bec ec f ed a d a a ed a e e

Parasomnias: Night/Sleep Terrors

Fea e :
● I e e e , ca a ee
● Occ f d f ee d a (dee a e ee )
● A e a fe e e a a e
● C f a a e
● I /fa ff bed c

Parasomnias: Nightmares

Fea e :
● Occ a d f ee d a (REM ee )
● Pa e e e be e ae e a a e
● Lac f e e a d ca a e a ee

Parasomnias: Rapid E e Movement Sleep Behavior Disorder

Fea e :
● REM ee a a e e ac e f d ea

A c a ed C d :
● Pa d ea e, de e a Le b de , e e

A c a ed Med ca :
● A de e a c d SSRI , SNRI , c c c , a d MAOI

T ea e :
● Ad ee e e e e
● Me a f - e a ac ea , e c a e a

Sleep Related Movement Disorder: Restless Leg S ndrome

S /D a :
● U e e e e e ac e d e a c f ab e fee , c e e ed
b e e a d e ee e /a

E ace ba Med /D :
● D a e ece a a ,a a e , caffe e

P c a 533
Get The Step 2 CK Drills Book: usmledrills.com
The USMLE Guys Step 2 CK Crash Course usmleguys.com

R Fac :
● L e , fa , e a , e ce , Pa d ea e,
e a c , e a

T ea e :
● I f e - a fe e e ,a a e ea e e
def c e c a e a
● D c caffe e, c a e ace ba ed ca a e d ca , f
be
● Pe f e a a e, e e c e , a d ea e a
● F - e a ac e a a a-2-de a ca c c a e a d ( aba e ,
e aba ) d a ea ( a e e)

Content Re ie Questions:

A 23- ea - d a e e e a c ef c a f ee e .T e a e de c be
c c da e ee e c ea b e e ea c a e e a ced e e a
. He a ced a f e e a a e f ee e c c b
c ee ab e ef e e . W e fa a ee e f e ea ce
e c e ec e ae e e . T e e a e bee 2 cca a ee
ee a e a ca c a ed e df ab 1 e e
e a c c .W c f ef de d be ef c f e
e da a e ?

A. CT f e ead
B. MRI f e ead
C. ECG
D. M e ee a e c e

A 78- ea - d a e a ed ca f Pa d ea e e e ec c
fe ac a f ee d ba ce. T e a e fe e a e e
a e a bee c , c ,a d a e a ee . S e e d e e a ca e
e e a a a e ec f e a d ea ab . W a e e
da ba ed a e ' ?

A. Ra d e e e e ee be a d de
B. Na c e
C. S ee e
D. Sc e a

A 58- ea - d a e e e ec c ac a f c f ab e e . T e a e
de c be eac e bed a e e e c ee a a ea d
ca be d d e da . A ab a d ca e a a ea a a d e
da f e e e d e ade. W a a ac e a effec e ea
d de ?

A. Me c a de
B. D e da e
C. Gaba e
D. P e a e

P c a 534
Get The Step 2 CK Drills Book: usmledrills.com
The USMLE Guys Step 2 CK Crash Course usmleguys.com

A P a D
Trichotillomania

DSM C e a:
● Ha e f e ec e f e a e a
● Pa e e d e e ea ed a e dec ea e e a
● T e be a ca be a b ed a e ed ca c d
● T e be a be e e a ed b e fa e e a d de
● T e ca e d e a e a d a ff c

T ea e :
● C e-be a a ea ( ab e e a a )

Skin Picking Disorder

DSM C e a:
● S e e f ec e c
● Pa e e d e e ea ed a e dec ea e c
● T e be a ca be a b ed a e ed ca c d b a ce
● T e be a be e e a ed b e fa e e a d de
● T e ca e d e a e a d a ff c

T ea e :
● Med ca ea e a b c , e , c e c ced e a eeded
● C e-be a a ea
● P a ac e a SSRI a f - e ea e ,a ca a c c ,
N-ace c e e e ec ca e

Kleptomania

DSM C e a:
● Re ea ed fa e e e ea b ec a ae eeded f e
ea a e e a e
● I ed a e bef e c e ef , e a e fee a c ea e e f e
c e e aced b e ef, a f ca , ea e a e e fc e
ef
● T e ea d e a a de a c a a d c ed a e
e ea ce
● T e ea e a ed b a e e a e

T ea e :
● C e-be a a ea

P romania

DSM C e a:
● De be a e/ ef f e e >1 cc e ce
● Fa c a f e a d f e- e a ed a a e a a a d c e e ce
● Te a a e af e
● P ea e, e ef, a f ca e e / e f e

P c a 535
Get The Step 2 CK Drills Book: usmledrills.com
The USMLE Guys Step 2 CK Crash Course usmleguys.com

● F e e f e e a a , e e e, da a e e a e
● N be e e a ed b a e c a cd de b a ce ca

T ea e :
● C e-be a a e a /a e ea
● SSRI , a c , a e e c ed ca , ,a ca a c c ,
a a d e a ca e-b -ca e ba a be e c bed

Tourette S ndrome

Tc :
● Ab , b ef e e e e e ca a

M Tc :
● S e c c de fac a d , ead e ,b ,
● C e c c de c a a, ec a a, f b d f c , c ,

P c Tc :
● S e c c c de c ea , a c ea , , ba
● C e c c c de ec a a, c a a, a d a a a

DSM C e a:
● P e e ce f b c a d a ea e c c be e e a e
a e
● Tc cc e e a da , a e e da , e e a e d f
e a e ea
● Ac a e c ed e e ( ca ,fe e c , e, e e , c e )
● O e f c bef e a e 18
● T c ca be e a ed b a e ed ca c d b a ce e
● Tc be e ed b c c a e e de ec d

T ea e :
● M d -d ab c : Ed ca , e ca e, a d c e
● T c ca f c a be :C e e e be a a e e f c (CBIT)
f - e, e abe a e 2 e a ac e a f CBIT a a ab e

Hoarding Disorder

DSM C e a:
● D ff c a e , e a de f e a e
● Ge d f e d ff c d e e a e fee a e e eed a e
e e a d fee fd e a c a ed d ca d e
● T ab d ca d e ead a acc a f e a
e e cc a ea a d c e e e ded e f e a ea
( e e c e e a ea)
● T e ad ca e d e a d a ed f c c d b a e
e e da e f efa d e
● H a d d e a e ed ca c d a e e a e

P c a 536
Get The Step 2 CK Drills Book: usmledrills.com
The USMLE Guys Step 2 CK Crash Course usmleguys.com

T ea e :
● F add e c a f e, f c a e, fa e acc a ed
e
● C e-be a a ea f - e ea e
● T ea c - cc e a d de

Content Re ie Questions:

A 23- ea - d fe a e e e ec c ac a f a .T e a e ae e
a ced ba d a c e e ead ee e a .U f e e e
ae e a bee e a a d a e ea ed ed cce , c
ca e a a ea fd e . S e de e a c d ca c ,
b , a ead e be a a d e a e ca a e e a ba d a c e a d
e a ea ee e a e a e d a . Ba ed e e ,
a e e da ?

A. Ob e ec ed de
B. Ob e ec e e a d de
C. D ca ea e a
D. T c a a

A 38- ea - d a e e e ec c a f ef . T e a e ae a e
d e ed b eed ea b ec a d e d e e b e e fee . He fee
e a d ae e e e a a ce e, ed e a e
a e e . He de c be fee a c ea e e f e ed a e bef e
ea c e e aced b e ef a e e fc e ef . W a e
e da ?

A. C d c d de
B. B a d de
C. K e a a
D. T ef f e a a ( f )

W c f ef ce a d ee eda cc e af a a?
A. M ee de f de be a e e b ec f e
B. Se f e ea e / ca e e
C. Te a a e af e
D. Fa c a f e, f e- e a ed a a e a a, a d c e e ce

P c a 537
Get The Step 2 CK Drills Book: usmledrills.com
The USMLE Guys Step 2 CK Crash Course usmleguys.com

S S ,N Ma a S ,a Ma a
H a
Serotonin S ndrome

Med ca W c Ma P ec a e Se S d e:
● SSRI , SNRI , c c c a de e a , MAOI , a , , c ca e,
a ea e , ec a

S a dP ca eE a F d :
● Me a a c a e : Re e e ,a e ,d e a ,a da a
● P ca a fe a e e e a, ac ca d a, e e , c a c ,
e ,a a a, e ef e a, d c b e a e c , ce d , d a ed
,d c e b a e , c ea ed b e d , , d a ea, f ed ,
a dda e

Lab :
● P be e c , dec ea ed b ca b, a d c ea ed c ea e a e

H e C e a f Se S d e/D a :
● A 1 f ef e e a a e a a e ad ca e e
d e:
○ S a e c
○ I d cbec a a da e
○ Oc a c a a da e
○ H e ef e a a d e
○ Te e a e ab e 38 C (100.4 F) a d c a d cbec

T ea e :
● S f e ad a f e e cd
● Seda e be da e e( a e a )
● O e a d IVF ,c ca d ac
● Ad e c e ad e f a e a fa ea a a d ab a a
● If fe e e 41.1 C de eda , aa , a d e d ac ea ba
● T ea e e a c ae c ba e

Neuroleptic Malignant S ndrome

Med ca W c Ma P ec a e Ne e c Ma a S d e:
● A c c ed ca (e ec a f e ea ), a e e c ed ca ,
d a ea d da a

S :
● Me a a c a e :A a ed de c f , ca a a, ,
e ce a a ,c a
● Se e e c a d ( ead- e d )a d e ,d e a ,d a a,
d a a
● H e e a
● A c ab ( ac ca d a, ac ea, e e )a dda e

P c a 538
Get The Step 2 CK Drills Book: usmledrills.com
The USMLE Guys Step 2 CK Crash Course usmleguys.com

Lab :
● E e a ed c ea e a e
● Le c
● H a e a e a e a, e ae a, ca ce a, a e e a,
e ab c ac d
● E e a ed a a e a a e, LDH, e a a a e
● L e

Da :
● Made ba ed e e e ce f e f :
○ Pa e a a e ad ca e e e c a a d e
e a 3 da
○ H e e a
○ R d
○ A e ed e a a
○ CK e e 4 e f a
○ 2 e f a e c e e ab (e e a ed BP, f c a
BP, d a e , a c e ce)
○ Hea a e a d e a aeee a
○ Ne a e - f e ca e

T ea e :
● S f e ad a f ca a e a e
● T ea e e
● T ea e e a c ba e
● T ea a a be da e e
● P de e ca e ICU e
● I e e e ca e da e eb c e a a ad e a be ed

Malignant H perthermia

Med ca W c Ma P ec a e Ma a H e e a:
● V a ea e e c a e ( e f a e, de f a e, f a e) a d cc c e

S :
● M ce d
● H e ca b a
● Ma e e ce d
● H e e a
● Tac ca d a/a a
● M b a

Lab :
● H e ae a
● E e a ed c ea e a e
● M b a
● M ed e a ac d a d e ab c ac d
● Ra e : Lab c e DIC

Da :
● Def eda ade e e c e

P c a 539
Get The Step 2 CK Drills Book: usmledrills.com
The USMLE Guys Step 2 CK Crash Course usmleguys.com

● Pe eda ba ed a d ab d ed a e
ea e

T ea e :
● M b e ec ca a db a a e e a ca
● N f e , e ASAP
● Ad e a a d e a a a ae
● D c e e a e ( c f a e e c a f )
● G e da e e
● C ec e ec e ab a e
● T ea e e /a a (a d ca c c a e b c e )
● C a e
● M e

Content Re ie Questions:

W c f ef ed ca c a d ca ed f e e b d e e a
a e a a e e a a ece ed da e e?

A. Ve a a
B. Labe a
C. N de
D. C d e

W c f ef ed ca ca e Ne e c Ma a S d e?

A. A a e
B. O a a e
C. O e a e
D. Me c a de

A 33- ea - d fe a e a a ed ca f de e e e e a
fe e . T e a e d e ed a d a a ed, b e ae e a d e e
be e e e a e a a e e d ed e , e e a ed ca
e a e a e f e de e .O e a e a e a ab d e e f 155/93 HR
119 SPO2 99% Te 102.8. T e a e e e e da e ca d e ca e a
d cbec e e ,a e a e ef e a f dee e d ef e e a d e .W a
e e da ?

A. E e d e ed e
B. Se d e
C. Ne e c a a d e
D. Ma a e e a

P c a 540
Get The Step 2 CK Drills Book: usmledrills.com
The USMLE Guys Step 2 CK Crash Course usmleguys.com

P a M a S E a Sa C
Antips chotic Medications

F Ge e a A c c :
● Ha e d ,f e a e, e e a e, de, c a e, da e

Sec d Ge e a A c c :
● R e d e, a a e, a d e, a a e, e a e, c a e

S de Effec :
● Ta d e d e a
○ Face, , e, e e e , c e a e d e e
○ Ca be e e b e a d fe e e af e ca a e ed
● E a a da
○ B ad e a, a a a, e , d , ac e d c eac
● QT e a a
○ Ma ead ade de e
● A a c
○ Le e a, e e a
● Me ab c d e
○ We a ,d de a, d abe e , d abe c e ac d , a d ca d a c a
d ea e
● A ca c effec
○ C a ,d , a ee , b ed
● Ne e c a a d e
○ M ce d , fe e , e a a c a e ,a c ab / ab
○ Of e e e a ed CK a d abd
● E e a ed ac
○ Ga ac ea, fe , e a e a e , f bd / e a d f c

Selective Serotonin Reuptake Inhibitors (SSRIs)

SSRI D :
● Se a e, f e e, a e e, f a e, c a a ,e c a a
● I b CYP450

S de Effec :
● Se a d f c , e a , GI e,d e a, d e ,d
, QTc a

Tric clic Medications

T c c cD :
● N e, a e, c a e, a e, a a e, de a e, d e ,
e, a e

S de Effec :
● Hea b c , e c a a a , a c e ,a a c effec ,
a c e c effec , dec ea ed e e e d, e , e a d f c

P c a 541
Get The Step 2 CK Drills Book: usmledrills.com
The USMLE Guys Step 2 CK Crash Course usmleguys.com

Monoamine O idase Inhibitors (MAOIs)

MAOI D :
● Se e e, ca b a d, e e e, a d a c e

Ad e e Effec :
● Se d e:
○ Ma e f d -d e ac be ee MAOI a d e e e c
ed ca f MAOI e a e
● H e e ec :
○ D -d e ac ( a )
○ D -f d e ac ( a e)
■ A ed, d ed/c ed, ed c ee e; ea ; ; f
■ Fe e ed f d

S de Effec :
● C a , a ea, e a , eadac e, a c e ,d ,
b ed , a, c , e e a ede a, eda , e a d f c ,
a e a c

Content Re ie Questions:

W c f ef c ca e be ee a a e a a e a
ed e fa e a c de a e ?

A. Ve c a a a
B. Ne e c a a d e
C. Ne e c d abe e d
D. Ta d e d e a

W c f ef ed ca ea a c a ed e de e e f ad e
d e a?

A. Me c a de
B. L
C. F e a e
D. Ha e d

W c ca f ed ca a c a ed e de e e f e e ec af e ea
c ed ea a ed c ee e a e?

A. F e ea a c c
B. Sec d e e a a c c
C. T c c c ed ca
D. M a e da e b

P c a 542
Get The Step 2 CK Drills Book: usmledrills.com
The USMLE Guys Step 2 CK Crash Course usmleguys.com

Pulmonolog
P a E b
P a a I c ea ed R f PE:
● Ma a c
● Pe a c
● Ac e e
● H a ed a e
● Ne c d e
● T a a a e ace e
● Ac e a a c a c d
● Ma a a
● I e ed b cd de
● C e / ece fec
● D (c ace e , HRT)
● I b a
● S

V c ' T ad:
● Ve a
● Va c a e d e a da a e/
● H e c a ab f e ed ac ed ae

We C e a f Dee Ve T b (DVT):
● Pa a , ae , ece ed c ca fa e e e (1 )
● Rece bed dde f e a ee da a e e a f
ee (1 )
● L ca ed e de e e dee e e (1 )
● S e f a e e e (1 )
● Ca f e 3c ea e a e e e , ea ed 10 c be e b a be
(1 )
● P ede a ea e e a c e (1 )
● C aea - a c e e fca e (1 )
● Ac e ca ce ca ce ea ed (1 )
● A e a eda e e a DVT (e . Ba e ' c , ce , c e da a e,
eb c d e, a ade a ,e e a e c e )
( 2 )

We Sc e:
● 0 e L bab
● 1 2 M de a e bab
● 3 8 H bab

I e a P e e ce f DVT:
● If de a e bab We c e, e f - e D-d e e . If
a, eed f f e e . If e e a ed ceed c e a a
● If We c e, ceed d ec c e a a ( eed e
D-d e )

P 543
Get The Step 2 CK Drills Book: usmledrills.com
The USMLE Guys Step 2 CK Crash Course usmleguys.com

● If DVT ee c e a a , ea a c a a f
c a d ca ed

Dee Ve T b (DVT) T ea e :
● P a DVT = ea , fe a, ac e (a a eed a c a a e
c a d ca ed)
● F e a e a ab ec a d ca a c a a ea , fe
e a ca a (IVC) f e a be ed

P a E b :
● M c c de d ea, c e a ( e c), c
● ECG a eS a e ead I, Q a e III, a d e ed T a e ead III
(S1Q3T3) c a f ac e c a e, b sinus tach cardia e
c f d ECG f a e PE

We C e a f PE:
● C ca ec ed DVT (3 )
● A e a eda e e a PE (3 )
● Tac ca d a (1.5 )
● I b a ( 3 d) OR e e f ee (1.5 )
● H f DVT PE (1.5 )
● He (1 )
● Ma a c (1 )

We Sc e f PE:
● Sc e >6: H
● Sc e 2-6: M de a e
● Sc e <2: L

D-D e:
● F a e de a e ba ed We c e, a a D-d e
effec e e c de PE a d f e e eeded
● If D-d e e e a ed, ceed -

PE E a a :
● He d a ca ab e a e ec ed f PE: A e We c e f PE
● L :O ec e da c a a fda ce a a a e e a
24
● M de a e :O ec e da c a a fda ce a a a e e
a 4
● H :G ea c a a e a f e e f eda ce a a
● CT a a f -c ce d a c a
● If d a ce a a PE, d c ea a c a a ea f a
a ed
● If PE c f ed, d e e PE e e b ?
Ab e d ca e e ce f e e a ed PE
P e a d ca c de: Pa e e ee e c a d f c ,
2/2 PE, e e e e a, a e be dec e a e, a e
e e e c b de
● T b ( Pa) f c a d ca , ca ca e e e b ec f
c a d ca e e

P 544
Get The Step 2 CK Drills Book: usmledrills.com
The USMLE Guys Step 2 CK Crash Course usmleguys.com

PE a d He d a c S ab :
● He d a ca ab e a e : S ab e e d a c (IVF,
a e )a d e a ( e , a e a e ec a ca
e a )
● O ce ab ed f e e e e f ab e a e be e a a ed f PE
c d We c e a de e e e ea c a a ea
● F a e e a e d a ca ab e a d e c ca c
de a e, e a ac a c a a d be e a e a f a e ae
e d a ca ab e
● If e e a ab e, e a a e ef ca a ac c
ec ca d a , f e de ce f RV e ad. If e e , f e ca e
be de PE
● If RV a e e , ef b ( Pa) f c a d ca ed. If c a d ca ed,
ef e b ec

PE I a :
● CT a a (f -c ce d a c a )
○ Ma ee ed e a ed fa c ( fe e e )
○ PE a ea a a a a e f defec
● V/Q ca ed e f ca e a a e c a -d e a e
○ A ea f e be e a ed b ade a e ef ed
○ Ve a ef a c ead A-a ad e

T eA e a a e a ad e (A-a ad e ):
PAO2 = (F O2 [Pa PH2O]) (PaCO2 0.8)

Pa e a a ea e e (ab ef a f ed f c ca e):
PAO2 = 150 (PaCO2 0.8)

E ec ed A-a ad e = 2.5 + [ a e a e ∗ 0.21]

PE T ea e :
● L ec a e e a (e a a ), bc a e f da a , e a
fac Xa b a aba a aba
● F e e e e e a fa e, e d a c ab , a e fe a DVT,
f a e ee a e d f eed ac e d c e
e e ea c a a , IV f ac a ed e a

Fa E b :
● M c b e (fe ) a d e c f ac e a d ec
e f ac e . A be c / ec ,b e a a a ,
b , fa af ced e ( a a ca d ce fa b e e
c c a )
● Def ed b e e e ce f fa b e e a c c a
● S a ea 24-72 af e e
● C a c ad: H e a, e c e def c , a d e ec a a
● C ca d a :S ec a e fac (e . b ef )a d f
e a fa e
● I a : CXR c e CT. CT MRI f e b a f e e c

P 545
Get The Step 2 CK Drills Book: usmledrills.com
The USMLE Guys Step 2 CK Crash Course usmleguys.com

● Pe f e ab c d CBC (a e a a d b c e a) a d c a a
d e (ab a ). S e e ca de ec d a e de ,b
e c ec ed a e ec ed f fa e b
● T ea e e ca e f d , e ,a d e ece a ec a ca
e a ( ce a c a a )

A cF dE b (AFE):
● C e a f AFE (a be e e ):
○ S dde e f ca d e a a e OR e e de ce f
e a c e
○ O e DIC
○ C ca e d ab 30 e f ace a de e
○ Ab e ce f fe e d ab

A E b :
● C ca e : S e , a a, a c a e e , a d ba a af
ec a ca e a a d c ba d
● S ec a e b e e f e ab e c ca e ( c a a e
ca e e e ) d ec ecede dde e a d e ( e a e b )
a d/ a e ca e e (a e a e b )
● If enous a e b a , ace e ef a e a dec b ; f arterial a
e b , ace e e
● P de e ca e, e ba c e ed e ec ca e

Content Re ie Questions:

A a e e a a ed a e ED f ef e a f ac c f .T e a e
e d e ca ed e de e , ede a e a c e e e
c aae a e ,a d e f e e e ef e . W a e e be e de e e
da ?

A. D-d e
B. C e a a
C. X- a ef e
D. Lac c ac d e e

A 33- ea - d fe a e a e e e e a e cc e a a d e f
b ea a d f d be e d a ca ab e. S e de e ed be f PE
acc d e We c e a a d e a c a a e a a e f e
da ce a a .S e a c a d ca a e / ed ca .W a e e
be e e da ce a a ?

A. D-d e
B. CT a a
C. C e a a
D. Pa ad a

P 546
Get The Step 2 CK Drills Book: usmledrills.com
The USMLE Guys Step 2 CK Crash Course usmleguys.com

A a e a ca e e be aced a a e f ea e e dde c a f
ef ded ea e . Y ec a a e b eba .W a d e a e
be aced e ff e e b a ?

A. Lef a e a dec b
B. R a e a dec b
C. T e de e b
D. S e

P 547
Get The Step 2 CK Drills Book: usmledrills.com
The USMLE Guys Step 2 CK Crash Course usmleguys.com

L Ca
Solitar Pulmonar Nodule

P a N d e:
● 30 a e
● S ded c ee b a ae c a
● We -def ed e
● L a >30 a e e ec ed beca e e a e e e be a a
● N d e 8 ( d c e ed )ae ed e a CT ca

Cause E amples

L ca ce S a ce ca c a, a e ce ca c a, a
ce ca c a, ade ca c a, a d a
ca c d
Ca ce ea a c e a da B ea , ea a, ead a d ec , c , d e , e
e e f e ce , a c a, e a da a
Ga a ( fec ) T be c ,a ca c bac e a, a ,
c cc d d c ,c c cc ,ba c
I fec Bac e a ab ce , e c ec , a e
I e a d ea e Sa c d , a a a ,
e a d d e
Be e a Ha a a, f b a, a, a a

R Fac f Ma a c S a P a N d e:
● A e
● Se (fe a e e )
● H f
● Fa f ca ce
● E e a
● P a a c
● A be e e
● La e d e e
● L ca f e d e e e be
● Pa - d d e e, c a

I a fS a P a N d e :
● S a a d e be a de ec ed a be f a da e
(CXR, PET, MRI)
● If e a d e, b a a e
○ G d e e e be a a a d e eb , ab e d e
(f 2 ea ) e e be eed f f e -
● CT c e c a d e ad a a d - ec e c ca
e best a e af e de ec a a a d e

P 548
Get The Step 2 CK Drills Book: usmledrills.com
The USMLE Guys Step 2 CK Crash Course usmleguys.com

S a P a N d eW -U :
Solid Nodule >8 mm Work-Up

L bab f a a c Se a CT ca (f c a ed, e e ;
f ,b e d e)
I e ed a e bab f a a c PET/CT a d/ a b (FDG a d d be
b ed e c ed)
H bab f a a c B e c
<6 d d e N f e - eeded (ca c de
e ea CT f e ed a e bab f
a a c )
6 8 d d e Se a CT ca (f c a ed, e e ;
f ,b e d e)

Primar Lung Cancer Screening

W S d Rece e Yea Sc ee ?
● 30 ac / ea ea e
● C e e e e a 15 ea a
● A e 55-80
● O e e d ea

Sc ee Occ :
● Yea
● W d e CT c e (DO NOT c ee f ca ce c e ad a )

Primar Lung Cancer

Ge e a S :
● C
● We
● He
● C e a

R Fac :
● S
● Rad a ea
● E e a

Non-Small Cell Lung Cancer (NSCLC)

Ade ca c a:
● M c f f ca ce (50%)
● Pe ea ca ed
● I d d a ed ea e e a ed, e e-d ec ed ea fe e
cce f c a ed a da d c e ea

P 549
Get The Step 2 CK Drills Book: usmledrills.com
The USMLE Guys Step 2 CK Crash Course usmleguys.com

S a Ce Ca c a:
● U a ce a ca ed
● T a a e ce a ec ca a
● H e ca ce a (f ec e f aa d e- e a ed e (PTH P))

La e Ce Ca c a:
● La e e ea a e ec
● Da fe c
● E a

S a Ce L Ca ce :
● Ce a ca ed
● SIADH ( ec e f ADH, ca e a e a)
● C d e( ec e f ACTH; ca e c e ea e , e e ,
, e , ae ca a , a d/ e ce a)
● La be -Ea d e( e e a e e e ea e ,
e e ce e e)

Add a L Ca ce Pa a e a c S d e :
● O e e ca d de c d : Ce ebe a a a a, e e a ,
e a
● He a cd de c d : A e a, e c , b c ,
e c a ab e d de (T ea d e, DVT a d b e b , DIC,
b ca d - b c c a a )
● H e c e a a (c bb , e ea fe a f e b a b e ,
e ca a d a f a a )
● De a a d ( a ce ea e )

L Ca ce Me a a e :
● B e( a a f a d c e e eb a b d e , e e a ed e
a a e a a e, a d e e a ed e ca c )
● B a ( eadac e, , a fed , c a a e e def c , e ae , e e )
● L e ( a a a c, ca e e e e ab a e )
● Ad e a a d ( a a a c)

Content Re ie Questions:

A a a d e de ec ed c e X- a a d c e CT c a
d e ad a a d - ec e c ca .T e d e f d be 10 e.
Beca e f e a e a d e c a ed c a ac e f e d e,
dee ed a ea bab f a a c .W a e e be e a a e e ?

A. D
B. Se a CT ca
C. Se a PET ca
D. B e c

P 550
Get The Step 2 CK Drills Book: usmledrills.com
The USMLE Guys Step 2 CK Crash Course usmleguys.com

W c f ef e c f f ca ce ?

A. Ade ca c a
B. S a ce ca c a
C. La e ce ca c a
D. S a ce ca c a

W c f ef aa e a c d e c e a c a ed a ce
ca c a?

A. SIADH
B. PTH P
C. C d e
D. La be -Ea d e

P 551
Get The Step 2 CK Drills Book: usmledrills.com
The USMLE Guys Step 2 CK Crash Course usmleguys.com

L a M a a T
Med a a Ge Ce T (GCT):
● Med a a e ce (GCT ) ca be e f f e f : Ma e
ea a , a e ea a , e a ,a d e a GCT
● M fe e e a ada ca :A e ed a

Mediastinal Germ Cell Characteristics


Tumor (GCT)

Ma e ea a -M c ed a a e ce
-S f c e ca ed b ea a: C e a ,d ea,
c ,b c a b c
-T ea e : S ca e a
I a e ea a -Ma e c e f a ee e a a e , ed a e e
-Of e c de c c a e c ae e a c a d ec c
-T ea e : C e a d ca e a
Med a a e a -Y e (be ee 20 a d 40)
-Pe f e c a a a a d a d c ec f a
-N a e a a-fe e (AFP); e be a b f a
c c ad (be a- CG) a e e a ed
-S :C e a ,d ea, c , e , SVC d e
-T ea e : C e a de - e d ea e-f ee a
Med a a e a -Y e (be ee 20 a d 40)
-Pe f e c a a a a d a d c ec f a
-N a e a a-fe e (AFP); e be a b f a
c c ad (be a- CG) a e e a ed
-S :C e a ,d ea, c , e , SVC d e
-T ea e : C e a de - e d ea e-f ee a

Ne e d c eT :
● Ne e d c e (NET) a a ed e e ac ( c )

● L NET : C ce a ca ed fc , e ,
ee , ec e - b c e e a
● Ca c d d e: M c d a ea a d f .A a ca e
- ded a a ea d ea ea db c a
● Da c e : 24- a e ce f 5- d d eace c ac d (5-HIAA) a d CT
ca f e c e a d/ abd e a d e
● L NET c f ed b

S e P a S c (Pa c a ) T :
● S de a
● C8 a d T1 e e a
○ Wea e /a f c ce f e a d
○ Pa / a e e a f a dff d a d e ed a a a d f ea ,a a
● H e S d e: P , ,a d ,e a
○ Pa a e eb a a e c c a a d fe ce ca a
● E a ed ac a c a de

P 552
Get The Step 2 CK Drills Book: usmledrills.com
The USMLE Guys Step 2 CK Crash Course usmleguys.com

● We
● P e c ec e a ea e e a
● S e e a ca a (SVC) d e

Content Re ie Questions:

A 24- ea - d a e e e e ED c e a ,d ea, a d fe e a d f d a e
a a e ed a a a a . Lab f d c de a e e a ed AFP a d e e a ed
be a- CG. W c f e f e ce e e e ef d ?

A. Ma e e a a
B. I a e ea a
C. Med a a e a e ce
D. Med a a e a

A a e c a ffe e f a d d a ea a a e e e e e
e a 2 .W c f ef e be a da c e f ca c d
d e?

A. 24- a e ce f 5- d d eace c ac d (5-HIAA)


B. 24 e c ea e
C. Se 5- d d eace c ac d (5-HIAA)
D. Se c ea e

A a e e e e ED de a , e ,c ,d ea, a d a e e .
O a e a e ed a ea e a c (Pa c a ) .T
e e f c f ef e e e e a e e ?

A. Pa a e eb a a e cc a
B. Rec e a ea e e
C. I fe ce ca a
D. I e c a e e

P 553
Get The Step 2 CK Drills Book: usmledrills.com
The USMLE Guys Step 2 CK Crash Course usmleguys.com

I a L D a
I a ed I a cD :
● S ca e
○ S c
○ A be
○ Ta c
○ Be
● Ca b
○ C a d (c a e e c )
● Me a

S c :
● E e: Sa db a , , a , a d ce a c a fac
● Ac e c :
○ Wee ea
○ C , e , e c a , fa e
○ CXR b a e a , d ff e d a ac e e a ba a e
○ BAL (b c a e a a a e) d ce c , eff e
○ C f a ac a e a d e e ace ae a a
e PAS ea e

S c T ea e :
● A df e e ea d e ca e

A be :
● E e: W ce e , f c ae a , a , b d , e a
a be -c a b d
● De e af e 20-30 ea f e e
● P e e b ea e e e e
● CT pleural plaques; subpleural ea de e ; ba a a d d a
ae c a fibrosis, e b c a, a b a,a d e b a e a fibrosis;
c a e ae c a ba d c e pleura, c a e e c b
● T ea e : S e ca e, ce a ,a df e e e, e c cca
a d f e a acc a , e e a e a eeded

A be E e:
● A be a fac f a a e e a( )
○ S :D ea, c ,c e a , ea
○ Ve a e f f ca ce , a be a a fac f ec
ca ce (b c e c)
● R f ca ce ea a f ed b e e b bacc e a d a be

Ta c :
● E e: I d a d c f ce a c , a e , a c, bbe , a , c e c
● CT a ce b a a d b e a nodules, conglomerate masses, focal
ground-glass opacities, a d e a e a f ba e

P 554
Get The Step 2 CK Drills Book: usmledrills.com
The USMLE Guys Step 2 CK Crash Course usmleguys.com

Be :
● E e: A ee e ab e be ea a .T c de ae ace,
a e, ce a c , c e , defe e, e ec c, c c , ad ea
ec c d e
● S :D ea a d - d c ec
● CXR: H a ade a e c a a d d a ac e
● Lab : Be c e fe a e (BeLPT)
● Da :E e be , e BeLPT, a d ca ea a a a d/
c ea ce f ae a f b e
● T ea e : Ce a f be e e, ce a , e c cc a d
f e a acc a , e e a e ( f eeded), a e ab a (f
eeded), a d e c e d (f a cc d ea)

C a W e P e c :
● E e: C a d f
● Ca e e b c a acc a f e a d e c f be ead a e
a b ac a e a cca a a ea f ed ce e
● CXR: R ded, d a ac e , e a 1c da ee, e e be .
O e e e e ac e ca c b e ead e e a efb

E e Occ a a Re a ed:
● I a ed a cd (ca e f e e e )
○ T e cf (fa e )
○ T ef (a e )
○ Bac e a
○ A a e (b d fa c e ' d ea e)

H e e P e (HP):
● A c eac d ced b a e e a d a e e a
ae c a
● E e: A a e e ( e , fa e ), a a d f ce , /c ac
e a e
● Ac e: 4-6 af e e e f fe e , c , a a e, a ea, c ,
c e e ,a dd ea ee .S e e 12
e e a da e e e a e a ed, a d a c ca a d ad ca f d
ca c ee a e
● C c: C ,d ea, fa e, a d e , d a c bb , a fb .
O a a e e a fe e
● O e CT d- - e e ed a ce f ce b a d- a
d a ac e fa - a
● H f HP: P -f ed granulomata c ea ed giant cells ca ed ea
e a e a b c e ,c c ce a b c ,a dc c ce a
e a c l mphoplasmac tic infiltration
● Da e fe e a e , e CT f d c a be HP, a d
e e a e a da ce

P a D ea e A c a ed ILD:
● Sa c d
● Ga a a (We e e )
● E c a a a (C -S a )

P 555
Get The Step 2 CK Drills Book: usmledrills.com
The USMLE Guys Step 2 CK Crash Course usmleguys.com

Sa c d :
● De a c :Y ad a d b ac a e efe e affec ed
● S :C ,d ea, c e a ,e e e , fa e, a a e,
fe e , a d e
● CXR: Bilateral hilar adenopath , a e c a d ff e e c a d a
ac e , c c ca , d a c da
● H a : Noncaseating granulomas (b f d a c ca
a d ad a c a fe a ed a eda )
● Lab ab a e : H e ca c a/ e ca ce a, e e a ed ACE a d ADA e e

S d e a d Ma fe a f Sa c d :
● L f e d e:
○ Fe e , e e a d , a ade a , a a a a
● Oc a a fe a :
○ U e ,d e e , c e , d fa a , b a d ea e
● Ca d ac a fe a :
○ Hea b c , a a , ea fa e, a a d f c , e ca d a d ea e,
ca d a
● N d e ca e ea e e a e a d ca e d f c

Sa c d T ea e :
● If a a c, ea e ; a a e a e e e e
d ea e
● O a c c c d f a c a e

Ga a P a :
● De a c : O de , e ad
● ANCA-a c a ed a c de
● S : Fe e , a a e, a e a, a d e
● CXR: Hilar adenopath , nodules, a c d ff e ac e , a d a f ae
● ENT a fe a :S , ed a, ea ac e, ea, e /b d a a
d c a e, a a d/ a a ce , a d c d
● P a a fe a :H a e e ,c ,d ea, d , ee , e ,
e c a
● Re a a fe a :G e e ,a a c e a a, e a
● Da b a da e e f ANCA
● T ea c c c d e e ab, c c a de, e e ae
de e d

E cGa a P a
-S a ): (C
● S :A a, , e ea e a ( e e ),
a ab e a, bc a e d e , a d ac a a a e e a a e
● Lab : Pe ea b de a, MPO-ANCA P-ANCA e
● H e CT: Pa c ae c a c da d a ac f ca ,
d e
● Da :A a, ,a de aa d eb
● T ea e : S e c c c c d a d de e d e e a e e
a e

P 556
Get The Step 2 CK Drills Book: usmledrills.com
The USMLE Guys Step 2 CK Crash Course usmleguys.com

Id a cP a Fb (IPF)/U a I e a P e a (UIP):
● De a c : A e 60 a d de ( e fa a a fb ), e > e ,H
f bacc e
● S : G ad a e d ea e e a d d c ec
● H Re CT (HRCT) e e a , b ba a e c a ac e
a c ec a d , hone comb changes
● H a :S b e a d b f a , e a fa a ,fb ba
f c,a d e c bc a e
● Da :E c f e ca e f ILD a d def e fea e f IPF HRCT
HRCT a d b fea e f IPF
● T ea e : S e ca e, fe d e eda b, a a

Content Re ie Questions:

A 22- ea - d b ac a e c e e ED c ,d ea, a d c e a . C e X- a
b aea a ade a a d d a ac e e ae c a. B fa
e ca ea a a . W a ab f d e
a e ?

A. Dec ea ed ACE e e
B. H e ca c a
C. Dec ea ed ADA e e
D. MPO-ANCA e

A 55- ea - d a e e e e ED fe e , c ,c ,a dd ea. S be a
4 af e a e ba a ce a .H e CT
d- - e e f ce b a d a ac e fa - a .B f
e f ed a a a ca ed ea e a b c e .W a e
e da ?

A. G a a a
B. E c a a a
C. H e e e
D. Be

A 72- ea - d a e e e a e e b ea e e e e . He
a e ed c c e a e a ed d a e e . CT e a
a e , b e a ea de e , ba a a d d a ae c a fb .W a e
e da ?

A. S c
B. Be
C. Ta c
D. A be

P 557
Get The Step 2 CK Drills Book: usmledrills.com
The USMLE Guys Step 2 CK Crash Course usmleguys.com

S
Mea e e :
● FVC (f ced a ca ac ) e e fa a ca f c b e a ed af e f
a
● FEV1 (f ced e a e 1 ec d) e e fa a ca f c b be
e a ed ef 1 ec d, af e f a

Ob c eL D ea e:
● L FEV1/FVC a = b c e d ea e

S e Bef e a d Af e Ad a faB c d a :
● I a a, e-b c d a e a a f ed c e e a d af e
ad a fab c d a , ed c ee be e e b e a ea ed b
e f ced e a e e ec d (FEV1)
● I COPD, e-b c d a e a a f ed c e e a d af e
ad a fab c d a , ed c c ee e e bea ea ed
b e f ced e a e e ec d (FEV1)

F -V eL :

● N a f - e :E a a a a d e e ea f a e, e
a a c ee ea fa f .T e a c e e ca a d
add e- a ed

P 558
Get The Step 2 CK Drills Book: usmledrills.com
The USMLE Guys Step 2 CK Crash Course usmleguys.com

● L e a a b c :E a a a c ca e ad a efe e
de c bed a " c ed- " a e
● Re c e d ea e: Dec ea e a ca ac , a e e a f ,a da a
" c ' a"a ea a ce a ee de ce d b

D ff Ca ac (DLCO):
● L DLCO = E e a, e a d ea e ( a fb ), f a e
d ea e , a e a
● N a DLCO = A a, e c a d ea e
● I c ea ed DLCO = Obe , c e a, a e a e

Content Re ie Questions:

T a ca ac e a e f c f ef c e ?

A. I a ca ac a de a e e e e
B. T da ea d a ca ac
C. Re d a ea d da e
D. V a ca ac a d e d a e

W c f ef c a ac e c c e a a e da ed
a ac d ?

A. N a FEV1/FVC a a d dec ea ed DLCO


B. N a FEV1/FVC a a d c ea ed DLCO
C. L FEV1/FVC a a d dec ea ed DLCO
D. L FEV1/FVC a a d c ea ed DLCO

W c f ef c e a DLCO a d FEV1/FVC a ?

A. A a
B. COPD
C. A e a
D. Obe

P 559
Get The Step 2 CK Drills Book: usmledrills.com
The USMLE Guys Step 2 CK Crash Course usmleguys.com

COPD
C cB c :
● C c d c ec f ee eac f cce e ea ee e
ca e f c cc a e bee e c ded

E e a:
● De c f ea ace a a e a a ca a d e a e e a ed a
ace d a e e a b c e ( b fb e e )

R Fac :
● Fa
● S (d a , ac ea , f ac e e)
● E e a / cc a a e e
● A a

S :
● D ea
● C
● S d c

COPD D a :
● COPD d a c f ed e e de a a FEV1/FVC a
e a 0.7 (a f b c )a d c ee e e b fa f b c af e
a ed b c d a e
● I a de e e d d a a e ca e f COPD beca ee a f
e e ca ed ce d ea e e .

AAT Def c e c a a Ca e f COPD:


● A a-1 a a ea e b a e e da a e f e e c a
ea a e
● S c de e e a, e c ,a d a c
● AAT def c e c a a c a ac e c CXR f d fb c a e e e a
e ba e a a e a ce
● I a ae b a AAT e e e a d AAT e f a a e
COPD, b e ec a f a e ,e e a a e , CXR
e e a c a e a ba e , fa fe e a a d/ e d ea e,
c e a c

COPD Ma a e e :
● A da ce f fac ( ,e e a)
● Re a e e /c ec f a e ec e
○ L -ac ca c a a (LAMA) a a OR c b a
LABA ( -ac be a a ) PLUS LAMA de e d e e a d SABA
(a b e ) a eeded f e ef ( e d ca e SABA
ece a a e a ce ed )
○ C b a c c c d-LABA ae fa a/COPD e a
● I f e aa d e c cca acc a
● P a e ab a
● L -e e ea fc c e a

P 560
Get The Step 2 CK Drills Book: usmledrills.com
The USMLE Guys Step 2 CK Crash Course usmleguys.com

H eO e T ea :
● L -e e e a (LTOT) c ea e a f e c a e COPD
● I d ca f LTOT f c c d ea e c a COPD c de:
○ Re PaO2 55 H S O2 88% a
○ PaO2 59 H S O2 89% f e e e de ce f c a e, ea
fa e, e c
○ PaO2 59 H S O2 88% d e ec e ee a be ef f
e e a e d e ec f c e

Se e e L fe-T ea e COPD E ace ba :


● COPD e ace ba ae fe ec a ed b URI ( a bac e a )
● Ma e c ae c d JVD, e e a ede a

C ca Da e S :
● U e f acce ce
● B ef eec e ed b b ea
● I ab e e
● P f dda e
● A a
● S fa e a ea e
● D ff e ee
● D a b ea d
● Tac ea, ac ca d a

Da c S d e COPD E ace ba :
● C e e
● ABG
● CXR
● CBC, BMP
● I f e a e
● ECG

Ma a e e COPD E ace ba :
● S e e a O2 a e a S O2 f 88% 92% PaO2 f 60 70 H
● Med ca a a e e c d : A b e (be a a )a d a (a c e c)
eb e , IV e ed e ( c c c d), a b c e a , a d f f e a
ec ed a a ea
● N a e e- e e e a f fa ed c a d ca ed, e ba

Content Re ie Questions:

A COPD d a c f ed c f ef e d ?

A. L b a e a fb e e
B. S e e de a a FEV1/FVC a e a 0.7 e ac e
c a e e
C. S e e de a a FEV1/FVC a e a 0.7 a d c ee
e e b fa f b c af e a ed b c d a e
D. S e e de a a FEV1/FVC a e a 0.7 a d c ee
e e b fa f b c af e a ed b c d a e

P 561
Get The Step 2 CK Drills Book: usmledrills.com
The USMLE Guys Step 2 CK Crash Course usmleguys.com

W c f ef ae ec ec d ca f -e e ea a a e
COPD?

A. Re PaO2 55 H S O2 88% a
B. Re PaO2 60 H S O2 90% a
C. Re PaO2 59 H S O2 88% a
D. Re PaO2 59 H S O2 90% a

A 40- ea - d ae f d a e ea e e e aa e a e
c a d e f d a e a a a-1 a def c e c . W c f e
f CXR f d c e da ?

A. B c a e e e a e ba e a a e a ce
B. B c a e e e a e a ce a a e ba e
C. T e e e ce f a de eda a
D. I f a e ac f ca e e e f e

P 562
Get The Step 2 CK Drills Book: usmledrills.com
The USMLE Guys Step 2 CK Crash Course usmleguys.com

A a
A aDa :
● P a f c e (PFT ) c d :
○ S e bef e a d af e ad a fab c d a
■ P e-b c d a e a a f ed c e e a d af e
ad a fb c d a , ed c ee be e e bea
ea ed b e f ced e a e e ec d (FEV1)
○ B c ca e ( e ac e)
■ He e ed e ee a a c a e
○ PEF ( ea e a f ) ( efe e ed e
da a e e fa a a e)
■ Va ab f e a 20% e ec e a a

A a Se e :
● I e e
● M d e e
● M de a e e e
● Se e e e e

I e e A a:
● Da ea a ce e e ee
● 2 fe e c a a a e e
● U e f -ac be a a f e ef f da fe e e ee
● A a d e fe e a ac e be ee e ace ba
● FEV1 be ee e ace ba 80 e ce f ed c ed ( a a e)
● FEV1/FVC a be ee e ace ba a a
● O e fe e e ace ba e a c c c d e ea
● E e c e- d ced a a (e e f e e c e da e ee a d a SABA
e e e a ce a ee )

I e e A a T ea e :
● S -ac be a a (a b e )a eeded, da a e a ce ed eeded

M d Pe e A a:
● S e a ce ee (b e e da )
● 3-4 a a e a ed c a a a e e (b e e ee )
● U e f SABA e e e e a da f e ee (b da )
● A a ca e e fe e ce a ac e
● FEV1 ea e e 80 e ce f ed c ed ( a a e)

M d Pe e A a T ea e :
● S ac be a a (a b e ) a eeded PLUS da d e a ed
c c e d f a e a ce ( efe ed) OR da e e e ece a a

M de a e Pe e A a:
● Da a a
● A a e a ed c a a a e e a ce e ee
● Da eed f SABA f a a e ef
● S e a a ac
● FEV1 60 a d <80 e ce f ed c ed a d FEV1/FVC be a

P 563
Get The Step 2 CK Drills Book: usmledrills.com
The USMLE Guys Step 2 CK Crash Course usmleguys.com

M de a e Pe e A a T ea e :
● S ac be a a (a b e ) a eeded PLUS d e a ed c c e d
(ICS) a d ac be a a (LABA) c b a a e OR da ed d e
ICS OR da d e ICS e e e ece a a

Se e e Pe e A a:
● S a da
● N c a a a e eac
● Need f SABA (a b e ) e e a da
● E e e ed ac d e a a
● FEV1 <60% ed c ed a e
● E ace ba 2 e e e ea

Se e e Pe e A a T ea e :
● I a ed : Da ed d e ICS-LABA c b ed ae ; b e add f LTRA,
e
● NEXT e : Da d e ICS-LABA c b ed ae,c de add f
a ab f a e a e e
● NEXT e : Da d e ICS-LABA c b ed ae, c e f a
c c c d ,c de add f a ab f a e a e e

S fA a E ace ba :
● B ea e e a e ,a a ,d
● U ab e ea f e e ce
● RR >30 b ea /
● HR >120 b
● SPO2 <90% a a

Se e e L fe-T ea e A a E ace ba :C ca Da e S
● U e f acce ce
● B ef eec e ed b b ea
● I ab e e
● P f dda e
● A a /de e ed e a a
● S fa e a ea e
● C a

T ea e f Se e e L fe-T ea e A a E ace ba :
● Ab e eb e e e 20 e f 3d e , e e e 1-4 a eeded
● S e e a e a a SP02 a 92% e
● I a b de eb e e e 20 f 3d e
● S e c c c c d IV
● Ma e fa e 2 e 20 e
● P e a e e e eb a e a d/ ba

S de Effec fA a Med ca :
● SABA a d LABA Tac ca d a, e e , e , ae a
● ICS Oa , ad e a e

P 564
Get The Step 2 CK Drills Book: usmledrills.com
The USMLE Guys Step 2 CK Crash Course usmleguys.com

T e e:
● W a ab c d a ,a - fa a , d a ,a d
b c ec e ed ca
● N ec e df e ea e fa a e ace ba
● N ec e ded f e ea a -e c ed ca ( d a
e a a e a e add- )
● T e e c c a ac e ed b , e e ,a a, e
ea ac a ed c a c a a d e d a

Content Re ie Questions:

A a e c e e a ca e d c a da fa a. T e e
a a- e a ed c a a a e ce e ee a d e f a b e e e e
3 da f e ee . W a e e acc a e de c be a e c e c d ?

A. I e e a a
B. M d e e a a
C. M de a e e e a a
D. Se e e e e a a

A 18- ea - d a e a da fc d da a e e e a
ca e d c a e fa a ee e ee 3a a- e a ed
c a a a e e . He e ab e e e e 3 da f e
ee . W a e a a e ed ca ea f a e ?

A. A b e a
eeded
B. A b e a
eeded a d da d e a ed c c e d
C. A b e a
eeded a d d e a ed c c e d a d ac be a
a c b a ae
D. A b e a eeded a d ed d e a ed c c e d a d ac be a
a c b a ae

A a e de b c ca e de de f a e da f
a a. W c f ef da ca e c ed f e?

A. E ed e
B. P e e e
C. T e e
D. Me ac e

P 565
Get The Step 2 CK Drills Book: usmledrills.com
The USMLE Guys Step 2 CK Crash Course usmleguys.com

A R a D S (ARDS) a M a a
V a
Be Def f ARDS:
● C ca 1 ee f e a e f e a
( e e )
● Bilateral ac e ee CXR CT e a ed b e a
● Re a fa e must not be d e ca d ac fa e f d e ad
● H e a be e e a def ed b e PaO2/F O2
○ M d ARDS = PaO2/F O2 201-300 H , e a PEEP CPAP 5 c
H 2O
○ M de a e ARDS = PaO2/F O2 101-200 H , e a PEEP 5 c
H 2O
○ Se e e ARDS = PaO2/F O2 100 H e a PEEP 5 c H2O

ARDS C R Fac :
● Se
● P e a
● A a e
● Se e e a a
● P a c
● S e/ a / a e (d )
● B d d c a f
● Pa c ea
● Ia e c (c e , a da e, ad a )

Mec a ca Ve a Se :
● T da e=a fa a e f e eac e a c ce
( a 6 / f dea b d e )
● M e e a = da e e a ae
● P e e d-e a e e (PEEP) = e e e a e a e
a a a ee d f e e a c c e (e d f e a a )
ec a ca e a ed a e
● F ac f ed e (F O2 ) = e ce a e f e a a e ae (
0.21 a ea e e )

A e a B d Ga (ABG) a d Mec a ca Ve a :
● Pa a e e f ca b d de (PaCO2) = ea e e f CO2 e e a e a
b d, b a ed f a ABG
○ A e ed b e a a e a d da e
● Pa a e e f e (PaO2) = ea e e f e e e a e a b d,
b a ed f a ABG
○ A e ed b PEEP a d F O2

Ve a M de :
● V e- ed a c e a
○ C a da e a a ac e e a e e a
● Pe e- ed a c e a
○ L e ea a a e e , e e a a e a ed e beca e
e e ed, be e a e - e a c , ea e e a f
ec a ca e a c a ed e- ed e a

P 566
Get The Step 2 CK Drills Book: usmledrills.com
The USMLE Guys Step 2 CK Crash Course usmleguys.com

ARDS Ma a e e :
● S e ca e ( eda , DVT a , GI a , a , e
acce )
● MECHANICAL VENTILATION (G a PaO2 55 80 H S O2 88 95 e ce b
ad F O2 a d PEEP)

N a e Ve a (NIV):
● P e e e e a de e ed a a e ea ( a a a , face
a ) e ba
● C de f ef :
○ Acute h percapnic respirator failure for acute e acerbation of COPD
(AECOPD)
○ Acute cardiogenic pulmonar edema
○ H e c - e ca c e a fa e
○ Ac e e a fa e d e a a a e ace ba af e a
b c d a ea c e a e e ee
○ Pre ent e tubation failure

N a e Ve a (NIV) C a d ca :
● I e c e ca d ac e a a e
● U ab e ca d ac a a e d a c ab
● Fac a e a a
● I ab c eae
● H fa a
● U e a a b c

Content Re ie Questions:

W c f ef f d d e ec ee c e X- a a a e ac e
e a d e d e?

A. A ea ad ac f a d a e ad
B. B a e a ac e
C. H e f a ed ,a c e , a d a f a e ed d a a
D. N d a f ba e H a ade a

Pa a e e f e (PaO2) affec ed b c f ef aa ee ?

A. Re a a e a d da e
B. PEEP a d b e a ae
C. PEEP a d F O2
D. T da e a d F O2

W c f ef ac a d ca a e ec a ca e a ?

A. H fa a
B. U e a a b c
C. Ac e ca d e c a ede a
D. U ab e ca d ac a a

P 567
Get The Step 2 CK Drills Book: usmledrills.com
The USMLE Guys Step 2 CK Crash Course usmleguys.com

S A a
Ob c e S ee A ea (OSA):
● D ee a f dec ea ed ( ea), c ee b c ed b f e
(a ea), a d/ e a e e e e ce e a eff e a ed a a
● R fac : Obe , c a fac a a d/ e a a ab a e , ad a ced a e,
a e e de
● S : Da e ee e , / a f a d ee ,
eadac e , e c a c

W e U de Da c Te f OSA:
● E ce e da e ee e (EDS) da a d f ef
/ f OSA
○ L d cc a e a ba
○ W e ed a ea/ a /c d ee b bed a e
○ H e e

G d S a da d D a c Te f OSA: P a (PSG)
● Da c f ed e e e:
○ 15 e a ea , ea , e a eff - e a ed a a e f
ee a a a c a e
○ 5 e b c e a ea , b c e ea , RERA e f
ee a a c a e

Se erit and Apnea-H popnea S mptoms


Inde (AHI)

M d OSA, AHI 5-14 e a -A a c a e da e ee e


e e e -S ee a e a d a e ee e e ed

M de a e OSA, AHI 15-30 -Ab e c e e da ac e b a e be a


e a e e e a d fa a ee a a ae e
-S ee f a e a , ee a c ec e a e ed b a
c a e e e OSA
Se e e OSA, AHI 30+ e a -Da e ee e e fe e a da ac e
e e e -Of e fa a ee d da ( )
-R f acc de a fa a ee da e
c c a ce (d )
-I c ea ed f a -ca e a a d e e ,
a e e ,c a a e d ea e, a d
a a

Ob c e S ee A ea (OSA) T ea e :
● OSA dee ed cce f ea ed e :S a d e e; ee a ,
a ea- ea de , a d e b a a ea e e a a e
● Be a a d f ca e OSA: We f e e a e ,
E OH ce a , e e c e, c a ee f a b c
c b OSA
● T ea e f d- e e e OSA: CPAP ( ea a e e)

P 568
Get The Step 2 CK Drills Book: usmledrills.com
The USMLE Guys Step 2 CK Crash Course usmleguys.com

Ce a S ee A ea (CSA):
● Re e e ce a dec ea e f b a f a d entilator effort d ee
● CSA ca be a ec da ( .e. de e e a e , ed ca e, ea
fa e)
● S e a OSA
● Pa e da e ee e a d fac f ec da CSA e CSA
d a e a (PSG)

Obesit H poventilation S ndrome

Def ed a a a e a f ef :
1) BMI >30 / 2 ( be )
2) A a e a e a e a (PaCO2 >45 H )
3) H e a ca be a b ed a e c d

Lab F d :
● E e a ed e b ca b a e (>27 E /L)
● PaCO2 >45 H
● PaO2 <70 H
● P c e a

W E c de A e a e C d :
● Se c e e a d e ec e
● C eeb dc
● T df c e
● P a f c e
● C e ad a

C ca :
● H e e a OSA (90%), a e e (66%)
● H e e , CHF, e a ce

T ea e :
● P ea a e ea (CPAP)
● We (e e c e, d e , ba a c e )

Content Re ie Questions:

W a e be a ea f a a e da ed e ee b c e ee a ea?

A. Ace a a de
B. N c a ea a e e
C. Me e da e
D. Ba a c e

P 569
Get The Step 2 CK Drills Book: usmledrills.com
The USMLE Guys Step 2 CK Crash Course usmleguys.com

A a e a f CVA a d e e e ec c e ce e da e
ee e da , c ce a ,a d a e e e ed a ea e e
a e ee .W a e be a e e a ae e a e ?

A. P a f c e
B. C e ad a
C. ABG
D. P a

W c f ef ab f d c e be e a d e?

A. E e a ed e b ca b ae
B. E e a ed PaCO2
C. Dec ea ed PaO2
D. A e a

P 570
Get The Step 2 CK Drills Book: usmledrills.com
The USMLE Guys Step 2 CK Crash Course usmleguys.com

P a E
T ace e :
● I d ca ed f d a e e e ee a e f d f a e a eff
○ E ce c de e a a f e a f da d a ead a e e
da ( a e )a d e e eff c ea a b ab e ea
fa e
● I d ca ed f e a e c e ef e a e a c (d ea), e e eff
c ca ed ( c a ed, e e ce f bac e a), e ac d a e a
c e a d b e e e c f e e ( be c e a )
● M c c ca : Pneumothora , fec , e ee c e, b eed
● C a d ca :I ff c e e a f d, fec a eed e e e,
f b eed

P e a F d C a ac e a :
● Ta da e e a eff :
○ Ca ed b d ffe e ce d a ca d c c e e
○ C ca e : CHF, ab e a, e c d e
● E da e e a eff :
○ Ca ed b c ea ed ca a e eab d e e a a d fa a
( c ea ed e ff da d e ) dec ea ed a cda a e f e
e a ace (dec ea ed e a f e a f da d e )
○ C ca e : I fec , a a c, a a c , fa a d de ,
c ec e e d ea e, a c ab a e , e e f df
abd e e a ace

L C e a R e:
● If e e f e e ee c e a e, ef d def ed a a e da e:
○ Pe a f d e / e e a >0.5
○ Pe a f d LDH/ e LDH a >0.6
○ Pe a f d LDH ea e a - d e e f a e LDH

R e P e a F d Te :
● P e (L c e a e)
● Lac a e de d e a e (LDH) (L c e a e)
● Ce c a d ce d ffe e a
● H
○ N a H f e a f d = 7.60
○ Ta da e e a f d H = 7.40 7.55
○ E da e e a f d H = 7.30 7.45
● G c e

E da e P e a Eff :
Disease/Condition Positi e Findings in Pleural Fluid

Ma a c C
T be c AFB a ,c e
E e a C e, a ea a ce ( ,f e )
C a T ce de >110 /dL

P 571
Get The Step 2 CK Drills Book: usmledrills.com
The USMLE Guys Step 2 CK Crash Course usmleguys.com

He a Ra f e a f d b d e a c >0.5
E a ea e Sa a a a e, H, f dfa e
F a - e a ed eff P ef a a ,c e
Pa a e- e a ed eff P e e ce f a a e

Content Re ie Questions:

A 60- ea - d fe a e a e a ed ca e e ee e e c
de a e d ea. S e e
d a ca ab e a d c e X- a e a e
ed a e a a e ef ded e a eff .W a e be e e e a a e
a e ?

A. D e
B. I ba a d ec a ca e a
C. CT c e
D. T ace e

A a e e e e a d ea a d f d a ea e e a eff .
Af e e f ace e e a e f d a e a e a f d LDH ea e a
- d e e f a e LDH. W c f e f c d e
e ca e f a e e a eff ?

A. H ab e a
B. Ne c d e
C. C e e ea fa e
D. S e c e e a

W c f ef ab f d eeded de eL c e a e de e e
f a e a eff a da e e da e?

A. P e a f d e
B. P e a f d LDH
C. P e a f d c e e e
D. Se e

P 572
Get The Step 2 CK Drills Book: usmledrills.com
The USMLE Guys Step 2 CK Crash Course usmleguys.com

P a H
S /S :
● I a :E e a d ea a d fa e
● La e c e ea fa e: E e a ed JVP, ede a, a c e ,
e a e a , e a eff ,e e a c e a , c e
● Hea d : I c ea ed e f P2 c e

G 1: P a A e a H e e
● Re e ea ca e e a (RHC) a eda
● A e be e c ded bef e a b a e e
a a e a e e
● Ca e c de: Id a c, e ab e e e c defec , d a d , e c
d de ec ec e ed de a d HIV
● T ea e : P ac c a a a ,e d e ece a a ,
de e a e5 b

G 2: D e Lef Hea D ea e
● Ca be d a ed c ca e de ce f ef ea d ea e ec ca d a
da ed ea ca e e a (RHC)
● T ea e a e ef ea d ea e: Med (d e c e a , a e e
b c e , a d be a b c e ), de ce e a e ca e e (f a e defec
f c e c ea d ea e), a d/ ad a ced ca e LVAD ca d ac a a

G 3: D e C cL D ea e a d/ H e a
● Da ade e e ea ca e e a (RHC) ec ca d a AND
e de ce f de a e e ee d f c a d/ e a AND a
e e ca be a b ed a e ca e
● Ca e : COPD, ILD, OSA, e a d e
● T ea e : T ea de d ea e a d e f e e a

G 4: D e P a A e Ob c
● Da ade ad c de e a
- ef ca
● Ca e : C c b e b c a e e ( a f ca e ),
b c a a e ,c e a a a e e ,a e
a c a ed c ec e ed de , a a e
● T ea e : P a b e da e ec (1 c ce) ba a
a a , def e a c a a f c c b e b c a
e e a d ea e f de c d f e ca e

G 5: D e M fac a Mec a
● Da ade ea ca e e a
● Ca e : He a cd de (c c e c a e a, e fe a ed de ),
e cd de ( a c d , a La e a c X,
e fb a ,c c d e d ea e), a d e ab c d de ( c e a e
d ea e)
● T ea e d ec ed a d de c d

P 573
Get The Step 2 CK Drills Book: usmledrills.com
The USMLE Guys Step 2 CK Crash Course usmleguys.com

C P a e:
● C ca f a e e ee c ea ed a a e a e e
a d e a ce ca e e d aa f e e ce c ca ead
ded ea fa e
● Da :R ea ca e e a
● ECG f d :R b deb a c b c , e c a e ,a d a a
e a e e
● Ec f d :P a e e , d a ed RV, a d c d e a

Content Re ie Questions:

W c f ef ea d c e a c a ed af d f a
e e ?

A. I c ea ed e f A2 c e
B. I c ea ed e f A2 a c e
C. Dec ea ed e f P2 c e
D. I c ea ed e f P2 c e

A a e a f COPD f d a e a e e c
c f ed ea ca e e a .W c f ef acc a e
ca e e a e ?

A. G 1 a e e
B. G 2 a e e
C. G 3 a e e
D. G 4 a e e

W c f ef e acc a e e c f eda fc a e?

A. R ea ca e e a
B. Lef ea ca e e a
C. Ec ca d a
D. ECG

P 574
Get The Step 2 CK Drills Book: usmledrills.com
The USMLE Guys Step 2 CK Crash Course usmleguys.com

P a
C Ac ed P e a (CAP):
● R Fac : A e 65 ea , c bdc d , ece c c e e a a
fec , ab ec a a , ,ac ab e
● M c a e :S ,H ,
M
● S :C , d c fe e , a d/ d ea
● Da :S a d CXR c e e a ( ba c da ,
e a f ae ,a d b e a eff )

CURB-65 C e a De e e Le e f Ca e:
● C e a:
○ C f
○ U ea >20 /dL
○ Re a a e 30
○ B d e e c <90 H da c 60 H
○ 65 ea de
● Sc e:
○ 0= a e
○ 1-2 = a e
○ 3-5 = a e ,c de ICU

CAP T ea e :
● O a e de 65: A c e e ac de (a c ) d c c e
● O a e a c bd e / fac (d abe e , e , e c.) a d/ ece
a b c e: A c -c a a a e e e ac de (a c ) d c c e
● I a e : Be a- ac a (cef a e) ac de OR f e ( e f ac )
ea
● If MRSA ec ed add a c c e d
● ICU: Be a- ac a (cef a e) ac de OR be a- ac a a e a
f e

A a P e a:
● R fac f a a e a: Red ced c c e ,d a a, e GI
ac d de
● Occ e a e a ed ced a a ec e f c a d aa f a f
ae a
○ Ha f a e a ca ca e 1) d ec ca da a e ( a c c e ); 2)
d ce bac e a ca a fa a e e; 3) b c ea a d e
e f a ed a e a

C e ca P e :
● A a ed a c c e ca e e a d e a dc a a d (e a 2
af e a a )
● CXR f ae e b e be
● C ca e e 24-36 ithout a b c

A a P e a - Bac e a :
● A ae b c bac e a c a a e a
● R fac : Pe d a d ea e

P 575
Get The Step 2 CK Drills Book: usmledrills.com
The USMLE Guys Step 2 CK Crash Course usmleguys.com

● Ma ead ab ce e e a
● Bac e a a a e a ec f c :P e f d ea e f e ea
da ee af e a a , a d d
● P e a e e be e a e a ae a d e e
e e f e e be e e e f e e be e a e
a ae e
● T ea e : A c - bac a

A a P e a - Ob c :
● F e b d a a de e d e f ea a e a d ca
● M e e e: T ac ea a e a d e ,a a, a d dea
● A a f a e a ce a e a f e a a ead a f cef a e
c a d bac e a e fec a ae b c a f e a a
● T ea e e a f ef e b ec , He c a e e f ca ed a ,a d
f be c db c c f e b c /b c e

Le a e ' D ea e:
● Occ e c a a ed a e c c a e b d ( a , e ,a a e ,
e )
● S f e a a d add a :
○ GI ( a ea, , d a ea)
○ H a e a
○ E e a ed e a c a a a e a d C- eac e e e e
● Pa e ea ed be a- ac a e a f CAP e f e a e
Le a e d ea e
● T da e Le a e ' d ea e L PCR a e e a ac a e
( BAL) efe ed b ea e e a ef ed
● T ea e : Le f ac a c

Content Re ie Questions:

W c f ef c d a c a ed a c ea ed fa a
e a?

A. Ge e a a e e a
B. D a af CVA
C. U ec f ed e ed de
D. S e c e e a

W c f ef ab f d c e eda f Le a e ' d ea e?

A. H a e a
B. E e a ed c ac d e e
C. E e a ed e a c a a a e
D. E e a ed C- eac e e e e

P 576
Get The Step 2 CK Drills Book: usmledrills.com
The USMLE Guys Step 2 CK Crash Course usmleguys.com

A ac e a e ad ed e ICU e c c ec da e a. W c f
ef a b c e e d be a ae ea d d a?

A. Be a- ac a ac de
B. A c -c a a a e ac de
C. F e ea
D. A c -c a a a e d c c e

P 577
Get The Step 2 CK Drills Book: usmledrills.com
The USMLE Guys Step 2 CK Crash Course usmleguys.com

A R a C
A -E ace ba ed Re a D ea e (AERD):
● Pa e a aa dc c a a , e e e ce
a a / c a /a a f COX-1 b NSAID e
● Reac cc 30 3 af e e
● L f a e AERD ca ed b e a d e e e (
I E ed a ed)
● T ea e : Ma a e e fa aa dc c a a ,
a da ce f NSAIDS, de e a ea a f d ca c a
a e c e c ea / a c a d ea e e ea aee e a , e e e
b a d e e e ece b c e

A e cR :
● S a be c d d/a a ad
● Ca be e e ec f c e e ( e ), ea a ( e ), e e (d
e )
● A c a ed d d a a e ec e a a a
● S c de a e ea, nasal itching/c e ,a d ee ,
a e e a a c ea e, a e c e
● Na a c a ede a a d ae
● C c e a e cc c c
● C ca d a

N a e cR :
● O e a a ae a e
● Ma : Na a c e , a a d a a e, a d a e e e (a ea ,
a c a ed ea a c a e )
● U a ac a a a d c a c e ee
● De e e e e, e a , e f e , c ea d c , e ea e
c a e
● Da fe c ( e e ca e f )

B c ec a :
● T c d be e f a a e de e b c ec a
1) I fec
2) I a ed d a a e, a a b c , a ed e e e
● Neutrophils a a a e e b e ea e a a e, ca fa a , e
da a e, a d e e a a de c
● Ma fea e: C da d c a f ea
● A a a e ,d ea, e , e
● O PE ee , c ac e , d a c bb

C d W c Ma C b e e De e e fB c ec a :
● A a
● COPD
● A a b c
● C cfb
● P a c a d f c
● I e / def c e c
● A a-1 a def c e c

P 578
Get The Step 2 CK Drills Book: usmledrills.com
The USMLE Guys Step 2 CK Crash Course usmleguys.com

● R e a c d ea e (RA a d S e )
● A e cb c a a e

B c ec a W -U :
● CBC d ffe e a a d c e/ ea
● I b a a
● Te f c cfb
● M de ec c ed a (MDCT) ( efe ed be a f e c ce)
e c ed a (HRCT)
○ B c a a c e a d ac f a e ,a a d aa " e- "

● CXR f d (c b a ed, NOT DIAGNOSTIC) f ea a e ec a , d a ed


a d/ c e ed a a

Ac e B c :
● U a ca ed b fec e a e , cca a bac e a
● S c de e e d c e d c ec a a
e f- e ed 1-3 ee
● Da ade c ca , ab a ece a f e da be
c de ed a e
● T ea e c de e ca e, de e a , a d/ a fe e PRN f
c

P e a :
● Ga e e a ace a fe a c a ed ac e d ea a d e cc e
a
● Ca be a a cc e ce ( ec ae ae e )
a c a ed d ea e (COPD, CF, a a c ), a a c, a e c
● He d a ca ab e e ee e a d e : U e bed de a d
a e
● S ab e a e ca a e bed de c e ad a ( a ) c e CT (
acc a e)
● Ma a e e de e d e , ea d a e ab a f b e a ,
e e a O2 ea e a a a a eed e ca e e / be
ac

Content Re ie Questions:

W c f ef e be e f a eda fb c ec a ?

A. Ve a - ef ca
B. C e ad a
C. M de ec c ed a
D. S e

P 579
Get The Step 2 CK Drills Book: usmledrills.com
The USMLE Guys Step 2 CK Crash Course usmleguys.com

W c f ef e ce ca ed e de e e fb c ec a ?

A. E
B. Ne
C. Ma ce
D. Mac a e

W c f ef a a f d a c a ed a -e ace ba ed e a
d ea e?

A. Re e c f aa a a e
B. Na a
C. Pe f a ed a a e
D. De a ed a a e

P 580
Get The Step 2 CK Drills Book: usmledrills.com
The USMLE Guys Step 2 CK Crash Course usmleguys.com

Surger

S E a D ba T a R a
T e fS c :
● Va d a (d b e)
○ Se c
○ S e c fa a e e d e (SIRS)
○ A a ac c
○ Ne e c
○ E d c e
○ D a d - d ced
● H e c
○ He a c
○ N - e a c
● Ca d e c
○ A a
○ Ca d ac ec e
○ Mec a ca
● Ob c e

Shock and Hemod namics

Va d a (D b e) S c :
● P e ad (PCWP) decreased
● Ca d ac increased
● Af e ad ( e c a c a e a ce) decreased

H e cS c :
● P e ad (PCWP) decreased
● Ca d ac decreased
● Af e ad ( e c a c a e a ce) increased

Ca d e cS c :
● P e ad (PCWP) increased
● Ca d ac decreased
● Af e ad ( e c a c a e a ce) increased

Ob c e: Pe ca d a Ta ade
● P e ad (PCWP) increased
● Ca d ac decreased
● Af e ad ( e c a c a e a ce) increased

S e 581
Get The Step 2 CK Drills Book: usmledrills.com
The USMLE Guys Step 2 CK Crash Course usmleguys.com

Ob c e: Te P e a ,P a E b ,P a H e e
● P e ad (PCWP) decreased
● Ca d ac decreased
● Af e ad ( e c a c a e a ce) increased

T ea e fS c :
● I a a e ec e e a a ba , ec a ca e a ( e eeded),
a d/ e e a e
● Ob a IV acce ad e IV fluids a d pressors
● Pe f ec f c e e de e d ca e f c
○ C ea da b c f e c c
○ E e e, d e da e, e d f a a ac c c
○ H d c e f ad e a c
○ 2L cc a d e b d d c f e a c( e c) c
ca e e f a a ed b eed
○ S e / ced e ( a e e a , a c d ec e a , e a c a a ,
ca d e , ace a e ace e , a-a c ba ) f ca d e c
c
○ Pe ca d ce e f e ca d a a ade ca b c e c
○ C e be/ eed e a a f e e a ca b c e
c
○ T b f PE ca b c e c

H pothermia

C ca Ma fe a :
● M d e a (32-35 C): Tac ca d a, ac ea, e ,c dd e ,c f ,
d a a, a a a, b eed d a e , DIC, e
● M de a e e a (28-32 C): B ad ca d a, e , e a ,
a a (a-f b, b ad ca d a), e a , f a ef e e , dec ea ed e ,
ce d
● Se e e e a (<28 C): A ea, a ede a, e c a fb a ,a e,
ca d a c a c a e, c a, a ef e a, fa c a ef e e , a, a c ea

Lab :
● CBC dec ea ed WBC a d a e e , b e ab a a e a c
● P ed PT a d PTT
● G c e dec ea ed, c ea ed, a
● P b e e e a ed a e ( f a c ea e e )

ECG:
● P b a-f b b ad ca d a

S e 582
Get The Step 2 CK Drills Book: usmledrills.com
The USMLE Guys Step 2 CK Crash Course usmleguys.com

C e X- a :
● P be a ede a

Da :
● C e e ea e be 35 C

T ea e :
● M d e a ea ed a ee e a e a b ac a e a a
, e e c ,c e ba e (e d c ea e a ea
0.5 C e )
● M de a e e a ea ed ac e e e a e a b a f ced
a ed a a d a ed b a e ( a a ec ef )
● Se e e e a ea ed ac e e e a ( ee ab e) a d e a e a b
ad e a ed c f d, a ed d f ed e , a f e a a d
e ea ca e a c f d, a d/ ECMO
● C e e c a a ea e a e a bee a ed

H perthermia

Ca e :
● Hea e, d / ed ca , e ab cd de

T ea e :
● U e fb d c (fa a d ae ed c b d )a dc e e a IV
f d , f e a e f d de e ed

Content Re ie Questions:

W c f ef d f c a c a ed dec ea ed e c a c a
e a ce?

A. D b e
B. H e c
C. Ob c e
D. Ca d e c

S e 583
Get The Step 2 CK Drills Book: usmledrills.com
The USMLE Guys Step 2 CK Crash Course usmleguys.com

W a e d a c ae ca ee a a e b c e c ca ed b e ca d a
a ade?

A. I c ea ed PCWP, dec ea ed ca d ac , c ea ed e c a c a
e a ce
B. Dec ea ed PCWP, dec ea ed ca d ac ,c ea ed e c a c a e a ce
C. I c ea ed PCWP, c ea ed ca d ac , c ea ed e c a c a e a ce
D. I c ea ed PCWP, dec ea ed ca d ac , dec ea ed e c a c a e a ce

W c f ef c ca a fe a a c a ed e ee e a a
c eb d e ea e f 26 C?

A. L f c a ef e e
B. Ve c a f b a
C. Tac ea
D. Pa c ea

S e 584
Get The Step 2 CK Drills Book: usmledrills.com
The USMLE Guys Step 2 CK Crash Course usmleguys.com

Ta a
Primar Surve

ABCDE M e c:
● A a
● B ea
● C c a
● D ab / e ca e a a
● E ea de e a c

P a S e :A a
● A e :
○ A "W a a e?
○ Ob e e f f e a d e
○ I ec f e a ea ca
○ I ec a d a a f ea e ec
○ Sec e e a a

P a S e : B ea
● A e e a a d e a
● I ec f f acce ce e a ad ca e e (f a c e )
● A c a e b ea d a d a aef ce /b e b
● P ab e c e ad a f ab e a e ( ed a e ea e f c a ac e c
c ca e e a d de a d ca e bd / a )
● Ra d ea e f e a , e a , a d ca d ac a ade

P a S e :C c a
● Pa a e ca d fe a e
● P ace 2 a e b e e e a IV a d d a b d
● Ma a e e a e( / b eed , f d a d b d d c e c a )
● Re e e a c a a
● E a aef e a c ca e f c

P a S e : D ab a d Ne cE a a
● Pe f Ga c a ca e
● A e e a a df c a d e a d eac

Glasgow Coma Scale

E eO e Sc e:
● S a e :4
● T eec : 3
● T a :2
● N e e e :1

S e 585
Get The Step 2 CK Drills Book: usmledrills.com
The USMLE Guys Step 2 CK Crash Course usmleguys.com

Be Ve ba Re e Sc e:
● O e ed e , ace, a d e: 5
● C f ed: 4
● I a ae d :3
● U e be d :2
● N e e: 1

Be M Re e Sc e:
● Obe c a d :6
● L ca ed e e a :5
● W da a e e a :4
● Fe e e a :3
● E e e e a :2
● N e e a :1

T a Sc e:
● Sc e 8: Se e e b a
● Sc e 9-12: M de a e b a
● Sc e 13: M d b a

P a S e :E ea dE e a C
● C ee de a e a d ec e eb d f f
● T ea e a f e e

I a :
● Sc ee a ad a d be b a ed f e a e a ce ca e, c e , a d
e ( e e a e b a ed af e be b e ea ab e a e )
● F c ed a e e a f a a (FAST) ef ed d e
a e f ab e a e a dd e ec da e f ab e a e
● Ta e ed e e e c CT ( f a e ab e e )

Secondar Surve

C e f e Sec da S e :
● Pa e
● P ca e a a
● I d ca ed d a c a

Flail Chest

Ca e:
● 3 ad ace b ae d d a f ac ed 2 ca

S e 586
Get The Step 2 CK Drills Book: usmledrills.com
The USMLE Guys Step 2 CK Crash Course usmleguys.com

Ma a e e :
● M e e a d ca a
● P de e e a e a d ec a ca e a ( f eeded)

Content Re ie Questions:

A 29- ea - d fe a e a e e e f a e cec a .T e a e e e
e e eec , e a ae d a e e ,a d a a da a e e
a . Acc d e ef d , a e a e Ga c a c e?

A. 9
B. 10
C. 11
D. 12

W e ef c ed a e e a f a a (FAST) e f ed
e d a ca ab e a e ?

A. D e a e
B. D e ec da e
C. D e e a e
D. Ne e

W c f ef a e aea f de e fa c e ?

A. A e b f ac ed 3 e ca
B. 3 c b f ac ed 2 ca
C. 2 b f ac ed b a e a
D. F ac ed e

S e 587
Get The Step 2 CK Drills Book: usmledrills.com
The USMLE Guys Step 2 CK Crash Course usmleguys.com

S I ,H a I ,P a C ,B Ca a
I
Splenic Injur

C ca Ma fe a :
● P e Ke ' ; a e ef e abd e , ef de , a d/ ef c e a ;
e ea

I a :
● FAST e a ec c ace d ee , f d M ' c ,
a e ea f d ea e ee
● CT ca f d e e e ea e ee , e a a a fc a ,
a ea f de e e c ae c a

T ea e :
● He d a ca ab e a e e e ed a e e ec
● He d a ca ab e a e ca e a e eb eed e e ed a e
e ec
● He d a ca ab e a e ca e a e eb eed a d a e - ade
e c e ca be a a ed e a abd a e a , e a ab , a d
a a ce b a , f eeded

Hepatic Injur

C ca Ma fe a :
● Pa e e abd e , de , a d/ c e a ; e ea

I a :
● FAST e a ec c ace d e,f d M ' c ,
a e ea f d ea e e
● CT IV c a a d e e

T ea e :
● He d
a ca ab e a e a e ea ed e ca e a d b e a
○ He a c e b a ed e a e fa b e a a a a e e
● He d a ca ab e a e e e ed a e e a a a a d
e a ec
○ Tec e c de a a c e , a ca , e e a c ac ,
ca e a c ed ca , a f a ae c a e e , a d/
d ec e
○ H ade e a be ea ed e a ca e a OR e a ec
a d e a a

S e 588
Get The Step 2 CK Drills Book: usmledrills.com
The USMLE Guys Step 2 CK Crash Course usmleguys.com

Pulmonar Contusion

C ca Ma fe a :
● C e a ,d ea, c , ee e , a e , e

I a :
● C e ad a e a c da a d - b a ac ac f ca f
e ae c a

T ea e :
● Ma a e e a (a d e ce ef d e c a ), a c , a e

Blunt Cardiac Injur

C ca :
● Ca d ac d f c ,a a, e a , a a , ca d a fa c ,
c a be e

W -U :
● S ab e e a e a d e e a ea e ca e f e
● ECG, ec ca d a ,

Ma a e e :
● T ea d d a c ca

Content Re ie Questions:

A 24- ea - d a e a e c ec e b ac ba d a a a . He e e
e a ef da af e e c e e e e ce c e a a dd ea. O
e a e e c ea b ac ec e a a dc e ad a - b a
ac ac f ca f e ae c a. W c f ef da e
c e e ef d ?

A. P e a
B. P a c
C. P e a
D. He a

S e 589
Get The Step 2 CK Drills Book: usmledrills.com
The USMLE Guys Step 2 CK Crash Course usmleguys.com

A 45- ea - d fe a e a e c e acc de a d e e e a c c .
T e a e a e e a a e BP 84/60 HR 125 SPO2 94% T 36.8 C. O FAST e a ,
e a e de f ed a a a ec c ace d e ee
a e ea f d ca ed ea e ee . W c f e f e a ae e
e a a e e f e ee e a e c ca c e?

A. Ob e a
B. N ea e e ca e
C. Vacc a a a e ca a ed bac e a
D. S e ec

W c f ef c ca ae a c a ed b ca d ac ?

A. M a
B. M ca d a fa c
C. Ve c a e
D. A a f b a

S e 590
Get The Step 2 CK Drills Book: usmledrills.com
The USMLE Guys Step 2 CK Crash Course usmleguys.com

B a W Ca
Burn Injur

S e fca B :
● D , ed, a f , b a c e e e, d e b e
● O e de a a e ed
● Hea 6 da ca

S e f c a Pa a T c e B :
● Red, a f , ee ,ba c e e e, f b e
● E de a a d e f c a de a a e ed
● Hea 7-21 da ca b fe e c a e

Dee Pa a T c e B :
● M ed c ( e ed) a e a a ea a ce, a f e e,
d e ba c e e, f b e
● E de a a d dee de a a e ed
● Ha f c e da a ed
● Hea 2-9 ee e c ca

F -T c e B :
● D a d ea c a a e, ea e a , c a ed a d b ac
a ea a ce ac f e a ; ba c e e; b e
● B e c a f a
● A a e f e de ed ( b bc a e ea )
● C e e ea e b e, e e e ca a dc ac e
de e

Dee T eI e e (4 De ee):
● B c e e d e c e a d/ b e

E a T a Pe ce a e f B d S face A ea (R e f 9 ):
● T e ead, ef a , a d a eac e e e 9% f a e ce a e f b d face
a ea
● T e ef e a d e eac e e e 18% f a e ce a e f b d face a ea
● T ea e a d e eac e e e 18% f a e ce a e f b d face
a ea

Ma a e e :
● Se e e b ( e fca b c
e 20% f b d ) e ef d e c a ,
ca d a c a a d e a , a c ,e c , a d af
● S e fca a a c e b a e ea ed deb de e , ca e ,a d
de c a e

S e 591
Get The Step 2 CK Drills Book: usmledrills.com
The USMLE Guys Step 2 CK Crash Course usmleguys.com

● Dee a a c e ,f c e , a d dee eb e eb d
e c a d af

Wound Care

C e fW d Ca e:
● Deb de e :
○ I a , a e c
● W d ac
● Ne a e e e d ea
● T ca a e c
● W dde
● W dc e

Content Re ie Questions:

W c f ef e fb c a ac e ed b ef a fab e c a?

A. S e fca b
B. S e fca a a c e b
C. Dee a a c e b
D. F - c e b

A ae a a e ce a e f b d face a ea ed b c e e
ead, e ef a ,a d e e ?

A. 20%
B. 27%
C. 36%
D. 45%

A ae a a e ce a e f b d face a ea ed b c e e
e e a db e ?

A. 45%
B. 54%
C. 63%
D. 72%

S e 592
Get The Step 2 CK Drills Book: usmledrills.com
The USMLE Guys Step 2 CK Crash Course usmleguys.com

P a A D a ,A L bI a, a C a
S
Peripheral Arter Disease

R Fac :
● H e e , , d abe e , e de a, de a e, a e e , b ac ace,
e a fa fa e ce , c e e a

Sc ee :
● Sc ee a e 70 a d e, e dec ea ed/ab e eda e ,a d e a ed
50-69 d abe e f
● Mea e a e-b ac a de : Va e f 0.90 d a c f e e a a e d ea e

C ca Ma fe a :
● Ma a a c; e e e c a d ca , a a e , ce a ,
a e e

Da :
● P e e ce f b a a e , - ea ce a , a e e ff c e a e e
da c ca
● If e e c a d ca a ca a e e , e a e-b ac a de (ABI)
○ Va e f 0.90 d a c f e e a a e d ea e
○ If ABI 0.91-1.3, e f e ec e e .A -e e c e ABI dec ea ed b
20% d a c f e e a a e d ea e

Ma a e e :
● Dec ea f d ea e e ( ce a ,a c d e, a
ea , ce c a d b d e ec , a a a ae e de a d
e ed e e c e)
● Pa e a a e ,d a e e, - ea ce a d a e
e a c a a ( ec a e e e a d/ ca b a )
● Pa e e a e e d a e d deb de e f ed b
e a c a a ( ec a e e e a d/ ca b a )

Acute Limb Ischemia

A e a Ca e :
● A e ce c a e
● E b
● D ec
● A e
● Ve e c e
● Ta a(a e c )

S e 593
Get The Step 2 CK Drills Book: usmledrills.com
The USMLE Guys Step 2 CK Crash Course usmleguys.com

● H ec a ab

Ve Ca e:
● P e a a ce ea d e

C ca Ma fe a :
● S P c c de a , a e e a, a , aa , e a, a d
ee e

G ad f Ac e L b I c e a:
● N ab e ac e b c e a: N ca a ef , a dbea e a e d e,
c ee aa ,c ee f e a , a ef a e ee a
● I ed a e ea e ed ac e b c e a: De a ed ca a ef , a dbea e a
d e,a dbe e d e, a a defec ( ea e ), e e ce f e
e defec a e , e ee a
● Ma a ea e ed ac e b c e a: De a ed ca a ef , a dbea e a
d e,a dbe e d e, defec , e ( e ) e
defec , de a e a
● V ab e ac e b c e a: I ac ca a ef , a e a a d e d e a d b e,
e defec , d a

I a :
● Pa e ab e a a ea e ed ac e b c e a d de e e
CT a a ca e e -ba ed a e a
● V a a fa e a a a :
○ E b a ea a a a c ff a ded e e e e c
○ T b a ea a a a - ded c ff a e ed c ff

T ea e :
● Re a c a a ca e e -d ec ed b ca e a c a a
● A a f ab e ac e b c e a

Compartment S ndrome

Ca e :
● L b e f ac e , b (c c fe e a ), e e a e ,a e e ,c
e ,a a b e ec e , c e da a e ( abd ,
, ec ), ed e e c e a e ef , a ef d
e c a ,e a a a f f d/d /c a de f e e e e e,
bc e ( e , a e ed a e ), e a ae ( e c
d e), a e b eed (b eed d de , a c a a )

S e 594
Get The Step 2 CK Drills Book: usmledrills.com
The USMLE Guys Step 2 CK Crash Course usmleguys.com

C ca Ma fe a :
● Ra d e e a f e , a ec c a e
c e , a e e a, ea e , ed ced/ab e e, f /e a ed
c a e

C a e Pe e Mea e e :
● Ha d e d a ee e eed e a ee e ed
● N a a e 0-8 H , a de e a 20-30 H , ca a b d f
c ed e e e 25 30 H f ea a e a e e, a d
c e a cc e e e e a ea a e a e e
● De a e e c a e d e ca c a ed b b ac ec a e
e ef eda c e e, a d f 30 H e c a e d e
da ed

Lab :
● E e a ed e c ea e a e, b a

Da :
● Ca be ade c ca ba ed a d ca e a , f e ed b
ea e e fc a e e e

T ea e :
● Fa c

Content Re ie Questions:

A 33- ea - d fe a e a e e e e a a a f a fa f a
adde . I a a f ac e f e ad a d a. O e a e a e a
e a d a e ee a e f ea c e e e . R ad a
a e e ed ced, a d f ea ce a ef a d e e a a .W c f e
f e e be e a a e e ?

A. M c e b f e f ea
B. CT a f e a
C. Mea e c a e e e f ea a d ed a ee
D. Ca e a

S e 595
Get The Step 2 CK Drills Book: usmledrills.com
The USMLE Guys Step 2 CK Crash Course usmleguys.com

A 71- ea - d a e a f d abe e a d e e e e ecc a


c a f ef e a . T e a e de c be a ac a a e e e ef ca f
c b b e e c e a d e e ed e . He de e a a a. W a e e
be e e a a e ca e f a e e a ?

A. Rea a ce
B. Mea e e a e-b ac a de
C. X- a f e ef e e
D. Lef e e b

W c f ef c ca f d ca a c a ed ac e b c e a?

A. Lac f e
B. P e a
C. Se defec
D. H ae a

S e 596
Get The Step 2 CK Drills Book: usmledrills.com
The USMLE Guys Step 2 CK Crash Course usmleguys.com

V a a G H a
Groin Hernias

R Fac :
● Ma e e , fa f a e a, f a ec , a e be ee 0-5
ea 75 80 ea

C e a I a He a:
● Ca ed b ce a a fa c e

Ac ed He a:
● Ca ed b ea e ed d ed f b c a e

A a c C a f ca :
● I d ec a e a de e e a a , aea e fe
e a c e e
● D ec a e a de He e bac a e (abd a a ), ed a
e fe e a c e e
○ He e bac a eb da e a e ed a - a e a b de f e ec
abd c e, a e a - fe e a c e e , a d fe - a
a e
● Fe a e a de e fe a , c ed a e fe a e ,
aea e ac a a e , a d fe e a a e
● T e a f a fe a e a a e e fe a e aea e a a e
a e a d e ac a a e ed a

C ca Ma fe a :
● B e ca ed e / c , a / ea e /d c f ade e
a a d/ a d
● U Va a a, a f a fe ee a e f e e aced e
e e a ae

C ca :
● I ca ce a / a a ead c e a a d ec a fe a e e a,
a a a , fe e , a ea, , abd a f e

I a :
● US, CT, MRI ca be ed de f e e e ce f a e a, a d d ffe e ae
a a c c a f ca ba ed ca

Da :
● C ca a d ca e a a c f a a

S e 597
Get The Step 2 CK Drills Book: usmledrills.com
The USMLE Guys Step 2 CK Crash Course usmleguys.com

T ea e :
● Wa c f a f a e a / d
● S ca e a f a fe a e a a df a e a de a e- e e e

Ve a He a :
● E a c e a: He a ca ed be ee e b c a d e d ce e
abd a d e d e ea a ba
● U b ca e a: He a ca ed e ce e f e b ca
● I c a e a: He a ca ed a e e f ca c
● A a c e a e a ca be ed, a c e a e e e ec e
ca e a , a d ac e ca ce a ed/ a a ed e a e ee e e ca
e a

Content Re ie Questions:

C e a a e a a e ca ed b e fa ed c e f c f ef ?

A. L ea a ba
B. A e f ee e a b e
C. T a e a fa c a
D. P ce a a

W a ae eb da e f He e bac a e c a d ec a e a
de ?

A. Med a b da : a e a b de f e ec abd c e; a e a b da :
fe e a c e e ; fe b da : a a e
B. Med a b da : fe e a c e e ; aea b da : a a e ;
fe b da : e b de f e ec abd ce
C. Med a b da : a e a b de f e ec abd c e; a e a b da :
fe e a c e e ; fe b da : a e a fa c a
D. Med a b da : fe a e ; aea b da : ac a a e ; fe b da :
a a e

W c f ef e f e a ca ed a e a e fe e a c e e ?

A. Fe a e a
B. D ec a e a
C. I d ec a e a
D. U b ca e a

S e 598
Get The Step 2 CK Drills Book: usmledrills.com
The USMLE Guys Step 2 CK Crash Course usmleguys.com

P a I ,S a B Ob ,A
Postoperative Ileus

S /S ( e e e da 4 a e ):
● Lac ffa e a 24 , ca eaea a de e a 24
, a ea/ , abd a d e , a a d/ dec ea ed/ab e b e
d abd a a c a , d e de e a a e
abd e

Lab :
● CBC, CMP, a e ,a a e, a e

I a :
● Abd a ad a d a ed f a b e a d e e ce f c c a

Da :
● C ca d a ade ba ed e e da 4 ae a d
a c e ea e e a d a b e b c

T ea e :
● S e ca e (b e e /dec e (f a a ed), IVF e ace e )
● Se a abd a e a
● T ea a e e b e ca e (e ec e ab a e ,d c e ca a e
ed ca , ea fec )

Small Bowel Obstruction

Ca e :
● Ad e , ,c ca ed e a , , ce ,C d ea e

R Fac :
● H f abd a e c e ad a , ca ce , C d ea e, f e
b d e , e e ce f a e a (abd a a )

S /S :
● Ca abd a a , ac f f a (c ee b c ), abd a
d e , a ea/ , f e a, c ed/ e a c d
abd a a c a , e e a ce ec f e abd e ( e
ec e f d f ed b e e d e ea d)
● Reb d e de e a d a d ae f b e c e a, ec , a d/
ef a

Lab :
● CBC, CMP, ABG, e ac a e e e , ca c ,b dc e

S e 599
Get The Step 2 CK Drills Book: usmledrills.com
The USMLE Guys Step 2 CK Crash Course usmleguys.com

I a :
● Abd a ad a e e ce f a -f d e e a d d a ed a b e ,
e a ec
● CT ca d a ed, f d-f ed f a b e a a (d a ed
b e a e , c a ed b e d a ), e e f e b c ( a a,
c e e), b e c e a ( ac f b e a e a ce e ), a d b e ca e
( a e , e a)

Da :
● Abd a a ( ca a a f f ef a )c f eda

T ea e :
● Ma e a e NPO
● IV f d e c a / a e a ce a d c ec e ec e ba a ce
● Ide f f ed a e e a a ed (a d e f f ece a )
○ F e a e ca e NG be aced f dec e f abd a
c e
○ F ad e e ca e e ed a e ca e e , e
a af
● M f f b e c e a/ ec / ef a ( ed a e e f e e )
f 3-5 da , e ceed e f e e

Appendicitis

S /S :
● Pe b ca a c e e e ad a , a ea/ ,a e a,
fe e
● P be e e a d d a ea, a fe e c a dd a
● McB e ' e de e , e a , eR ' , e
b a , eb d e de e e e ad a
Lab :
● E e a ed WBC ( a e e d f e c ea e a ea e d bec e a e
e f a ed)

I a :
● CT f e abd e c a e a e d cea fa a d , a e d cea
a c e a d e a ce e , a e d cea d a e e e 6 , cc ded
a e d cea e , feca ea e d
● U a d a e d cea d a e e f e 6
● Ma e c e a ce a ( ed d e a c ) a e d cea d a e e f
e 7

Da :
● C f ed def e f ca ec e

S e 600
Get The Step 2 CK Drills Book: usmledrills.com
The USMLE Guys Step 2 CK Crash Course usmleguys.com

T ea e :
● A e dec 12 fda f ac e e f a ed a e d c
● Pe f a ed a e dc e d a c ab ea ed e e e
a e dec , a a dda a e f e e ea ca , a d b e e ec (f
eeded)
● Pe f a ed a e dc a e ae e d a ca ab e a d ca ed
ca be ea ed ed a e a e dec OR a ea e
a a e e IV a b c a d f a a e ab ce e e , ec a e
d a a e, a d e a e dec e ee

Content Re ie Questions:

W c f ef a fac f de e a b e b c ?

A. Abd a a e a
B. I ab e b e d e
C. C ' d ea e
D. H f abd a e

W c f ef a f d abd a ad a a d ca ceed
d ec e ea f a e f a ed b e a a e f a b e
b c ?

A. P e e ce f a -f d e e e e e
B. A de e d a a
C. D a ed a b e
D. N a de f ed ec

W c f ef ca a c a ed ac e a e dc ?

A. M
B. Ob a
C. P a
D. R '

S e 601
Get The Step 2 CK Drills Book: usmledrills.com
The USMLE Guys Step 2 CK Crash Course usmleguys.com

P a C a
I ed a e Pe /P e a e Fe e - Ca e :
● I fec e e e
● I fa a eda /ca ed d e
● Ma a e e a
● Med ca / a f eac

Ea P e a e Fe e (Da 0-3) - Ca e :
● I fec e e e
● I fa a eda /ca ed d e
● U a ac fec (e ec a f ca e e e e )
● I fec f e ca e(f A e c cc C ae e
ca a e a )
● P e a
● A e ec a
● Med ca / a f eac

La e P ea e Fe e (O e 72 H ) - Ca e :
● UTI
● DVT/PE
● I fec f e ca e
● S e - ec f c ca e
● Med ca / a f eac

T e5W M e c:
● W de d :A d (a b c , e a , H2-b c e ) (a e)
● W d: A e ec a ( - da 0-2), e a( - da 2-3)
● Wa e : UTI ( - da 3-4)
● Wa : DVT, PE ( - da 4-5)
● W d: I fec f e ca e( - 4+ da )

Postoperative Urinar Retention

R Fac :
● O e a e fac : O e 750 f d ad e ed, a c e c ed ca ed,
e a e 2 , c e ce ad ca e c e , e a a e e a
● Pa e fac : A e e 50, e-e a ee , f e c e

Ca e :
● Se def c a c a ed b adde f , def c a c a ed b adde
c ac , e a b c

S e 602
Get The Step 2 CK Drills Book: usmledrills.com
The USMLE Guys Step 2 CK Crash Course usmleguys.com

S /S :
● Se a f c e e b adde e , eed aea a ed a e af e
d
, a d, ea e ea , -de e de d , a bc
a / e e

Da :
● O e 100 f e d e d a ea ed a b adde ca e e a
a d

Ma a e e :
● R e b c a e ca e, c ea e e ca e e a ef ed
e e e e b adde ed e

Postoperative Nausea/Vomiting

R Fac :
● H f c e ea e a ea/ , fe a e e ,
, eed f ea e d

P ac c Med ca :
● T a de a c a e, IV de a e a e, IV da e

Content Re ie Questions:

T e a f ea e fe e ca ed b ca e d fec e e
c f ef e a e da (POD)?

A. POD 0-1
B. POD 1-2
C. POD 4-10
D. POD 35-45

W c f ef c d a c a ed a a e e a f ea e
fe e ?

A. U a ac fec
B. U a e e d e e a a e e a
C. Dee e b
D. Med ca eac

S e 603
Get The Step 2 CK Drills Book: usmledrills.com
The USMLE Guys Step 2 CK Crash Course usmleguys.com

W c f ef ed ca ed a a e e ea e a ea
a d ?

A. IV da e
B. T a de a c a e
C. IV de a e a e
D. IV fe a

S e 604
Get The Step 2 CK Drills Book: usmledrills.com
The USMLE Guys Step 2 CK Crash Course usmleguys.com

T a P
Carbon Mono ide (CO) Poisoning

C S ce f CO:
● F e, a e c e , f e /c a c a e ed d ea e , e

C ca Ma fe a :
● Headac e, d e , b ed , a e ed e a a , e e , c a, c e ed
, a ea, ac c ac d , a ede a, ca d a c e a, a a
( e c a)
● De a ed e c a c d e ca de e 20 da f a d a a
ea e fc e def c , e e d de , f ca def c ,
a d e a c a e a be
● P e e acc a e ca e f CO beca e ec e
ca b e b a e b

Lab :
● E e a ed ca b e b e e ABG
● P b e ac c ac d

I a :
● ECG a f c e a e c a a a

Da :
● C ca a de a c e CO , e e a ed
ca b e b e e ABG

T ea e :
● H -f e a face a ba a d 100% e e a e e a
f ab e ec a a
● H e ba c e ed f e a a c a e / c c a e , ECG
c e c c a e , ABG H <7.1, ca b e b e e f e 20%
e a a e e 25% a e a e
● P c a ca e e f a e e a CO

Organophosphate Poisoning

Ca e:
● E e ec c de b c e ca ea

Mec a f Ac :
● B d f ace c e e a e, a e e f c a

S e 605
Get The Step 2 CK Drills Book: usmledrills.com
The USMLE Guys Step 2 CK Crash Course usmleguys.com

C ca Ma fe a :
● M ca c e e c SLUDGEBBB: Sa a , ac a , a ,
defeca , a c e e , b ad ca d a, b c a ,b c ea
● Nc c c de c e ea e , fa c c a ,a d aa

I e ed a e S d e:
● De e 24-96 af e e e
● Dec ea ed dee e d ef e e , c a a e e ab a e , a c e a d ec
fe ea e , e a c e
● La 2-3 ee

Lab :
● L RBC ace c e e a e ac

Da :
● P e e ce f c ca

T ea e :
● Re a (100% e a eb ea e face a , ea ba f
f e a c e a e ed e a a )
● A e ed f a c e c effec (f c a ca c ece )
● Pa d e ed eac a e ace c e e a e (f c a b ca ca d
c c ece )
● Ca ef e e a d d ca d c (c a a )
● Ac a ed c a c a e f e a 1 ce e
● Da e a ed ea e e

Methemoglobinemia

Ca e :
● Ge e c ca e (c c e b5 ed c a e def c e c , c c e b5 def c e c ,
e b M d ea e)
● T ca a e e cd (be ca e, d ca e, ca e)
● I a ed c de
● Da e
● A aa a d (c e, d c e)
● N ae a d e

C ca Ma fe a :
● Headac e, eaded e , c a ,d ea, e a d e , e e ,c a
● S d e e e a e
● P e e ca de ec e e b , d a 85%

S e 606
Get The Step 2 CK Drills Book: usmledrills.com
The USMLE Guys Step 2 CK Crash Course usmleguys.com

Lab :
● C c a e-c ed b d
● E e a ed e e b e e ABG a a e a e e e (PaO2)
● G6PD ac d be e ed a e de ea e a a e e

Da :
● S f e e b e a e e a ed e e b ABG

T ea e :
● Ce a f ca a e ed ca
● S e ca e ( e e a e , ba , IV f d e c a ,a - e e
ed )
● Me e b e e >30% 20-30% e e ea e
○ F - e ed ca e e e b e (c a d ca ed G6PD def c e c a d
e a e e cd )
○ Pa e G6PD def c e c a d/ a e e cd ea ed
a c b c ac d

Arsenic Poisoning

Ca e :
● C a a ed d a e , a e c-c a e c de , e e- ea ed d

Ac e C ca Ma fe a (La e D e):
● M e af e ac e a a c d f e b ea a d , a ea,
, d a ea, abd a a , e , AMS, e e , c a, QTc
a ead ade de e a a de e
● Le a 1 ee af e ac e a c e aa d e a a de e
● 1-3 ee af e ac e ad c ,a a f e ea e a
a ce d e ab a e a d aa , a ec a, ce , a d/ a d ff e
c ac a a ca de e
● Af e 1 Mee e ( a e e e a ) a f

C c C ca Ma fe a (L D e):
● S c a e (c a e e a , e ea / ca )
● Ne a ( a e a d a d fee e f a c / e
d b e ca ea e ; dec ea ed a , c , e e a e, a d dee
e d ef e e )
● Ca ce (b adde , d e , , e, )

Lab :
● E e a ed e a a e ca e a a e c e ab e MMA a d DMA

S e 607
Get The Step 2 CK Drills Book: usmledrills.com
The USMLE Guys Step 2 CK Crash Course usmleguys.com

I a :
● ECG a ed QTc e a a d/ ac a a
● Abd a ad a a b ad a e a e a (a e c) e GI ac
af e e

Da :
● C ca f d a d e a a e cf d (e ec a e ab e )

T ea e f Ac e P :
● S e ca e (e ac ea e b a e e a ,e c a e a f , a d/
e da ( e e e ca e ), IV f d , a d ca d ac )
● Dec a a
○ If c ac e ec a d a a d a , de eed a d
d ec c ac
○ If e e ac a ed c a c a , f ad a e a e a ee abd a
ad a , e-b e a ae a e be a
● C ea d e ca DMSA, cc e

Content Re ie Questions:

A 37- ea - d fe a e a e e e e a d ea. T e d ea be a 2
a ,a d ea e d e eadac e a d eaded e . S e ece be a a e
a aa a d d c e. He e e e ead 85%. He b d a ea
c c a e-c ed e da a d e f ab . Ba ed e ef d , a e
e da ?

A. Ca b de
B. A e c
C. Me e b e a
D. O a ae

W c f ef c e a ae ?

A. C a
B. Sa a
C. B ad ca d a
D. B c a

S e 608
Get The Step 2 CK Drills Book: usmledrills.com
The USMLE Guys Step 2 CK Crash Course usmleguys.com

W c f ef e a f e e e a a e ec ed f
ca b de ?

A. D a a ABG a e f ca b e b e e
B. P ac e a e -f e a face a
C. Rad a f ec e
D. Ad e a e

S e 609
Get The Step 2 CK Drills Book: usmledrills.com

Potrebbero piacerti anche